HESI Fall 2013

Réussis tes devoirs et examens dès maintenant avec Quizwiz!

Nursing consideration for Alpha andrenergic blockers

Orthostasis

Nursing considerations for Central Acting Inhibitors

Orthostasis

diverticulosis

Outpouching or herniations of the intestinal mucosa that can occur in any part of the intestine but is most common in the sigmoid colon.

Chest Physiotherapy and Interventions

Vibration/Clapping and suctioning Postural drainage Aerosol nebulizer & IPPB Incentive spirometer Breathing exercises - diaphragmatic, purse-lip, exercise flutter valve device

Major Nutrients in Grains Group

Vitamin B complex, Iron, Incomplete proteins, Fiber

Osteomalacia

Vitamin D deficiency in adults - softening of bones - pelvis, spine, lower extremities

Rickets

Vitamin D deficiency in children - soft fragile bones - bow legs, knock knees, misshapen skulls

Benign prostatic hypertrophy (benign prostatic hyperplasia; BPH)

a slow enlargement of the prostate gland, with hypertrophy and hyperplasia of normal tissue. Enlargement compresses the urethra, resulting in partial or complete obstruction. Usually occurs in men older than 50 years.

Vitamin A (function)

maintains integrity of epithelial tissue - dark adaptation

Imitation

Has flavor of food but not nutrients

Respiratory Physical Exam

...

Rhoncus

...

Complications of prostate surgery

#1 hemorrhage infection bladder spasms urinary retention erectile dysfunction

Secondary Hypertension

...

Mediastinum

...

Pulmonary Angiogram

...

Normal Digoxin level

0.5 - 2

Each liter of body fluid weighs

1 kg

How to calculate heart rate from EKG

count the number of R waves in a six second strip and multiply by 10

Hypernatremia lab value

> 145 mEq/L

Albuterol may be given as often as...

30 - 60 minutes

Signs and Symptoms of Hypercalcemia

muscle weakness, constipation, anorexia, nausea/vomiting, cardiac dysrhythmias

Fat Soluble Vitamins

A, D, E, & K - can be stored in the body

Grey Turner Spots

associated with pancreatitis - bluish discoloration of the flanks. Both signs are indicative of pancreatitis.

Cullen's sign

associated with pancreatitis - the discoloration of the abdomen and periumbilical area.

Atrial fibrillation

atria quiver at a rate of 300 - 600 BPM - irregular ventricular response - risk of mural thrombi

GI assessment findings for FVD

decreased motility and diminished bowel sounds, constipation, thirst, decreased body weight

Renal assessment findings for FVD

decreased urine output

Complications of diarrhea

dehydration electrolyte imbalance

Circumstances that cause hypercoagulability

dehydration, birth control pills, smoking, blood dyscrasias, infection

Pulmonary Perfusion

Blood flow the pulmonary vasculature Vascular component of respiration.

Nursing considerations for Thiazide diuretics

Check for allergies b/c sulfa based, monior BP, I&O, K, dehydration

Diagnostics for peritonitis

CBC serum electrolytes x-ray, CT, ultrasound paracentesis

6 Nutrients

Carbohydrates, Fats, Protein, Vitamins, Minerals, Water

Riboflavin Deficiency

Cheilosis

Sources of PE

DVT, fat embolus from long bone fracture, a fib, amniotic embolism

Hypoxia

Decrease in oxygen supply to the tissues and cells Signs are restlessness, SOB, confusion, tachypnea, tachycardia, cyanosis

Hypoxemia

Decrease is arterial oxygen tension in the blood, measured by ABGs, PaO2 < 80

Action of Loop Diuretics

Decreases sodium reabsorption in the loop of henle, promotes K excretion

Metabolic alkalosis is most commonly due to

GI suction/vomiting

Hypoglycemia lab value

glucose < 50 mg/dl

Antifungal Agents GI

Nystatin swish and swallow

Interventions for Pneumonia

O2, antibiotics, hydration, chest physiotherapy, pneumonia vaccine, CDB

Care for PE

O2, mechanical intubation, head elevation, thrombolytics, anticaogulants, PT/PTT, INR

Treatment for hypertensive crisis

Potent and direct IV vasodilators, ie Nipride and nitroglycerine (most common)

Vitamin B9/Folate (function)

RBC production, DNA synthesis

Vitamin D deficiency

Rickets (children) - Osteomalacia (adults)

Conduction system of heart

SA node --> AV node --> Bundle of His --> Purkinje fibers

Cardiac conduction

SA node --> through right atrium --> Bachman's bundle --> through left atrium --> AV node --> bundle of his --> through R & L ventricles

STEMI

ST elevation on EKG - indicates MI

Individuals who are tuberculin skin test converters should

begin a 6-month regimen of an antitubercular drug such as INH, and they should never have another skin test

Afterload

The force against which the heart has to pump to eject blood from the left ventricle.

Contractility

The inherent ability of the myocardium to alter contractile force and velocity. Decreased contractility ---> decreased stroke volume ---> decreased cardiac output

Endocarditis - diagnostic tests

WBC/sed rate (elevated), blood cultures (+), TEE

Type 2 diabetes mellitus

a relative lack of insulin or resistance to the action of insulin; usually, insulin is sufficient to stabilize fat and protein metabolism but not carbohydrate metabolism - genetically determined defects in insulin receptors - obesity decreases insulin receptor sites - leads to beta cell secretory exhaustion

What criteria should the nurse use to determine normal sinus rhythm for a client on a cardiac monitor? Check all that apply. a. The RR intervals are relatively consistent b. One P wave precedes each QRS complex c. Four to eight complexes occur in a 6-second strip d. The ST segment is higher than the PR interval e. The QRS complex ranges from 0.12 to 0.2 seconds

a and b. (a) The consistency of the RR interval indicates a regular rhythm. (b) A normal P wave before each complex indicates the impulse originated in the SA node. (c) The number of complexes in a 6-second strip is multiplied by 10 to approximate the heart rate; normal sinus rhythm is 60 to 100. (d) Elevation of the ST segment is a sign of cardiac ischemia and is unrelated to the rhythm. (e) The QRS duration should be less than 0.12 seconds; the PR interval should be 0.12 to 0.2 seconds.

A client with chronic obstructive pulmonary disease is admitted to an acute care facility because of an acute respiratory infection. When assessing the client's respiratory rate, the nurse notes an abnormal inspiratory-expiratory (I:E) ratio of 1:4. What is a normal I:E ratio? a) 1:2 b) 2:1 c) 1:1 d) 2:2

a) 1:2

The nurse is teaching a client with emphysema how to perform pursed-lip breathing. The client asks the nurse to explain the purpose of this breathing technique. Which explanation should the nurse provide? a) It helps prevent early airway collapse. b) It increases inspiratory muscle strength. c) It decreases use of accessory breathing muscles. d) It prolongs the inspiratory phase of respiration.

a) It helps prevent early airway collapse.

A trauma victim in the intensive care unit has a tension pneumothorax. Which signs or symptoms are associated with a tension pneumothorax? a) Decreased cardiac output b) Flattened neck veins c) Tracheal deviation to the affected side d) Hypotension e) Tracheal deviation to the opposite side f) Bradypnea

a) Decreased cardiac output d) Hypotension e) Tracheal deviation to the opposite side

A 29-year-old client with severe shortness of breath comes to the emergency department. He tells the emergency department staff that he recently traveled to China for business. Based on his travel history and presentation, the staff suspects severe acute respiratory syndrome (SARS). Which isolation precautions should the staff institute? a) Droplet precautions b) Airborne and contact precautions c) Contact and droplet precautions d) Contact precautions

b) Airborne and contact precautions

When caring for a group of pts, the nurse realizes that which of the following health problems increases the risk for metabolic alkalosis? a. COPD b bulimia c. dialysis d. venous stasis ulcer

b bulimia

When a client's ventilation is impaired, the body retains which substance? a) Sodium bicarbonate b) Carbon dioxide c) Nitrous oxide d) Oxygen

b) Carbon dioxide

During inspiration, which of the following occurs? a) Lungs recoil. b) Diaphragm descends. c) Alveolar pressure is positive. d) Inspiratory muscles relax.

b) Diaphragm descends.

Insensible fluid loss is

unnoticeable and immeasurable, ie sweat and diaphoresis

Vitamin K (function)

benefits clotting

A client admitted to the facility for treatment for tuberculosis receives instructions about the disease. Which statement made by the client indicates the need for further instruction? a) "I will have to take the medication for up to a year." b) "This disease may come back later if I am under stress." c) "I will stay in isolation for at least 6 weeks." d) "I will always have a positive test for tuberculosis."

c) "I will stay in isolation for at least 6 weeks."

An elderly pt does not complain of thirst. What should the nurse do to assess that this pt is not dehydrated? a) Ask the physician for an order to begin intravenous fluid replacement. b) Ask the physician to order a chest x-ray. c) Assess the urine for osmolality. d) Ask the physician for an order for a brain scan.

c) Assess the urine for osmolality.

A postoperative pt with a fluid volume deficit is prescribed progressive ambulation yet is weak from an inadequate fluid status. What can the nurse do to help this pt? a) Assist the pt to maintain a standing position for several minutes. b) This pt should be on bed rest. c) Assist the pt to move into different positions in stages. d) Contact physical therapy to provide a walker.

c) Assist the pt to move into different positions in stages.

In order to be effective, percutaneous transluminal coronary angioplasty (PTCA) must be performed within what time frame, beginning with arrival at the emergency department after diagnosis of myocardial infarction? a. 6 to 12 months b. 9 days c. 60 minutes d. 30 minutes

c. 60 minutes

After cardiac surgery, a client's blood pressure measures 126/80. The nurse determines that the mean arterial pressure (MAP) is which of the following? a. 46 mm Hg b. 80 mm Hg c. 95 mm Hg d. 90 mm Hg

c. 95 mm Hg

Which task can be safely delegated to a licensed practical nurse (LPN)? a) Teaching a newly diagnosed diabetic about insulin administration. b) Admitting a client who underwent a thoracotomy to the nursing unit from the postanesthesia care unit. c) Changing the dressing of a client who underwent surgery two days ago. d) Administering an I.V. bolus of morphine sulfate to a client experiencing incisional pain

c) Changing the dressing of a client who underwent surgery two days ago.

Drugs that can cause upper GI bleed

corticosteroids, aspirin, NSAIDs

Blood pH is maintained by:

chemical buffers kidneys lungs

Circumstances that cause blood vessel injury

chemotherapy, trauma, central vein catheterization, sepsis

A nurse caring for a client with deep vein thrombosis must be especially alert for complications such as pulmonary embolism. Which findings suggest pulmonary embolism? a) Nonproductive cough and abdominal pain b) Hypertension and lack of fever c) Bradypnea and bradycardia d) Chest pain and dyspnea

d) Chest pain and dyspnea

A client is undergoing a complete physical examination as a requirement for college. When checking the client's respiratory status, the nurse observes respiratory excursion to help assess: a) lung vibrations b) vocal sounds c) breath sounds d) chest movements

d) chest movements

Generic of Lasix

furosemide

location of central line

inferior or superior vena cava

Vitamin B1/Thiamine (function)

necessary for normal growth

Anecdote for heparin

protamine sulfate

Aprida

rapid acting insulin

Embolectomy

surgical removal of emboli which are blocking blood circulation

Earliest sign of hypovolemia is

thirst.

Premature ventricular contraction

wide and distorted QRS complex on EKG - irritable focus in ventricle - result of increased automaticity of ventricular muscle cells - warning sign for sick heart

Anorexia Nervosa

∙ Self imposed weight loss ∙ Distorted psychological attitude toward weight and eating

Interventions for constipation

laxatives (use cautiously) enemas (oil retention - softens feces, hypotonic, phosphate) high fiber food increased fluids (at least 3,000mL/day)

How is 3rd degree heart block treated?

pace maker

pace maker - capture

pacemaker provides adequate stimulus to produce ventricular contraction

Pericarditis - signs and symptoms

pain radiating to neck or back, increased pain on inspiration, increased pain when supine and decrease when leaning forward - friction rub on auscultation near lower left sternal border, elevated WBC and sed rate, fever, + blood culture, STEMI or inverted T wave, echo showing pericardial effusion, narrow pulse pressure

Local allergic reactions to insulin therapy

redness, swelling, tenderness, and induration or a wheal at the site of injection may occur 1 to 2 hours after administration - reactions usually occur during the early stages of insulin therapy

Beta blockers (CAD/Angina)

reduces BP - prevents adrenaline from causing tachycardia and vasoconstriction - desired heart rate 50-60 to enhane cardiac output

Heart murmur

reflects turbulent blood flow through valve - valve can be normal or abnormal

Plasma proteins

regulate fluid balance - plasma colloidal osmotic pressure

Antidiuretic hormone controls ECFV by

regulating the amount of water reabsorbed by kidneys

Symptoms of CAD occur when...

the coronary artery is occluded to the point that inadequate blood supply to the muscle occurs, causing ischemia

Bilirubin

the end product of RBC breakdown

Urge incontinence

the involuntary loss of urine associated with a strong urge to void that cannot be suppressed. The patient is aware of the need to void but is unable to reach a toilet in time

Iatrogenic incontinence

the involuntary loss of urine due to extrinsic medical factors, predominantly medications

Stress incontinence

the involuntary loss of urine through an intact urethra as a result of sneezing, coughing, or changing position

Post Op for arterial bypass procedures

pulse checks with every vital signs check(hypotensive vein could collapse, hypertensive graft can blow), assess for return of blood flow(itching/throbbing), pain meds, OOB-do not bend graft site, ROM, assess suture line

Low Residue Diet

purpose is to decrease colonic contents - limits milk, seeds, nuts, raw/dried fruits & vegetables used for severe diarrhea, partial bowel obstruction, inflmmatory bowel disease

When administered I.V., I.M., or S.C, ephedrine sulfate's onset of action is...

rapid

Humalog

rapid acting insulin

Novolog

rapid acting insulin

Sinus bradycardia

rate < 60 BPM regular rhythm P wave before each QRS PR interval .12 to .20 QRS < .12

Sinus tachycardia

rate > 100 BPM regular rhythm P wave before each QRS PR interval .12 to .20 QRS < .12 usually related to SNS stimulation

medicamentosa

rebound nasal congestion commonly associated with overuse of otc nasal decongestants

Avoid thrombolytics in patients who have had:

recent long term CPR recent aneurism recent open surgery

Oselr's nodes

reddish tender lesions on pads of fingers, hands, and toes - associated with endocarditis

ventilation-perfusion ratio

the ratio between ventilation and perfusion in the lung - matching optimizes gas exchange

Intermittent claudication and pain occur when:

there is > 60% occlusion of artery

Metabolic alkalosis occurs when...

there is excess loss of acid causing a deficit, ie K of Cl. Also occurs with excessive intake of alkaline substances.

Signs and Symptoms of Hypernatremia

thirst, elevated temp, dry mouth, irritability, seizures, hallucinations

Cardiac assessment finding for FVD

thready pulse, decreased BP and orthostatic hypotension, flat neck and hand veins, diminished peripheral pulses, decreased CVP, dysrhythmias

Treatment of DVT

thrombolytics, thrombectomy, Heparin IV/SQ, coumadin, lower extremity elevation, scd's, vena cava filter

Causes of Hyperkalemia

tissue trauma, acidosis, renal failure

Causes of hypoglycemia

too much insulin too little food unusual amounts of exercise delayed eating

TEP

transesophageal puncture, technique for alaryngeal speech,

Hepatitis E

transmitted by fecal-oral route by contaminated water - usually occurs in developing countries - full recovery is expected

Interventions for hernias

truss scrotal support avoid coughing no heavy lifting surgery

electrophysiology studies

try to reproduce dysrhytmia

Minnesota/Sengstaken Blakemore Tube

tubes used to create a balloon tamponade and compress esophageal varices to prevent bleeding

Gastric ulcer

ulceration of the mucosal lining that extends to the submucosal layer of the stomach

Nitrogen Balance

when intake and output of nitrogen is equal

Positive Nitrogen Balance

when intake exceeds output - required for growth, normal pregnancy, and wound healing

Negative Nitrogen Balance

when output exceeds intake - can occur with infection, burns, fever, starvation, and injury

salt poor albumin

used to treat ascites - increases colloidal oncotic pressure and maintains intravascular volume and urinary output

Signs and symptoms of hypoglycemia

cold sweats - weakness - trembling - nervousness - irritability - pallor - increased heart rate - confusion - altered LOC and irrational behavior (cerebral glucose defecit)

Mitral stenosis - signs and symptoms

dyspnea on exertion, orthopnea, paroxsymal nocturnal dyspnea, pulmonary edema/hemoptysis, JVD, peripheral edema, right ventricular failure and associated signs and symptoms

Symptoms of PE

dyspnea, chest pain, tachycardia, apprehension, cough, hemoptysis

Prevention of DVT

early ambulation, adequate hydration, scd's, anticoagulants

Soft Diet

easier to chew - for difficulty swallowing

Sources of Vitamin B12

eggs, meat, poultry, shellfish, milk

Wich population is ephedrine not recommended for?

elderly

EKG

electrocardiogram, uses leads to show heart's rhythym - can show ischemic changes to heart

Interventions for Ascites

elevate HOB I&O daily weights and abdominal girth sodium restriction - no beer, baking soda/powder, chips, canned goods, carbonation administer k+ sparing diuretics administer salt poor albumin paracentesis (empty bladder)

Causes for sinus tachycardia

fever acute blood loss anemia shock exercise CHF pain anxiety drugs

Endocarditis - signs and symptoms

fever, anorexia, weight loss, fatigue, embolic complications, petechiae, splinter hemmorhages, Osler's nodes, Janeway lesions

BNP

hormone released in response to atrial and ventricular stretch - marker for CHF > 100

a tracheostomy tube is cleaned with

hydrogen peroxide and sterile saline

Movement of fluid through capillary walls depends on

hydrostatic pressure and osmotic pressure

Percussion over the lungs of a patient with emphysema would produce which type of sound?

hyperresonance

DVT

formation of blood clot in the deep veins of the calves - blood clot can dislodge and travel via venous circulation and lodge is pulmonary vasculature - can result in death

Sources of Vitamin B1/Thiamine

fortified breads and cereals, fish, lean meats, milk

Transesophogeal echocardiogram

gives a better view of the left atrium, mitral valve, and aortic arch

Interventions for hepatic encephalopathy

goal: decrease ammonia levels protein restriction administer neomycin sulfate - reduces bacterial action of bowel proteins administer lactulose - decreases colon pH producing acid, decreases bacterial growth and ammonia formation administer levdopa - decreases dopamine and norepinephrine

Signs of Digoxin toxicity

green/yellow vision, bradycardia, nausea

Refined Grains

have been milled - removes bran and germ which contain most of nutrients - removes fiber, iron, and b vitamins - products can be enriched to put nutrients back in

Normal sinus rhythm

heart rate 60 - 100 BPM regular rhythm P wave before each QRS PR interval .12 to .20 QRS < .12

Complications of gastric ulcers

hemorrhage perforation pyloric obstruction

Clients with COPD and respiratory distress should be placed in which position?

high Fowler's

Diet for fecal incontinence

high fiber increased fluid

Wheezing

high pitched - indicative of bronchioconstriction or airway narrowing expiratory (asthma) inspiratory (bronchitis)

Plumpy Nut

high protein, high energy, peanut based paste used for the treatment of severely malnourished children

Obstructive Jaundice

impaired flow of bile through the liver or biliary tract - occurs with hepatitis, cirrhosis, hepatic and pancreatic CA, and biliary stones

Increased amylase and lipase levels

indicates pancreatitis

Bilirubin

increase indicates liver damage or biliary obstruction

Hyperresonant sounds on percussion over lungs in an adult indicate...

increased air in the lungs or pleural space

LABS FVD

increased hematocrit, increased BUN, increased sodium level, increased urine specific community

GI assessment findings for FVE

increased motility, diarrhea, increased body weight, liver enlargement, ascites

Respiratory assessment findings for FVD

increased rate and depth of respirations, dyspnea

Renal assessment findings for FVE

increased urine output if kidneys can compensate, decreased urine output if kidney damage is cause

ST depression

indicated ventricles are having difficulty returning to resting state

Risk factors for duodenal ulcers

infection with H. pylori; alcohol intake; smoking; stress; caffeine; the use of aspirin, corticosteroids, and NSAIDs

Pericarditis

inflammation of pericardium - inflammation causes layers to rub and can result in accumulation of fluid in sac

Endocarditis

inflammation of the inner lining of the heart and valves - occurs primarily in IV drug users, valve replacements disease, and invasive procedures (portal of entry) - eroded areas in endocardium attract sediment which attracts bacteria - lesions grow and destroy endocardium and valves -eventually leads to obstruction of blood flow through valves

Hypercalcemia lab value

> 10.5 mEq/L

Hypermagnesemia lab value

> 2.5 mEq/L

Hyperkalemia lab value

> 5 mEq/L

Intra-pleural Pressure is...

airtight and negative

Unmodifiable risk factors CAD

age, gender, race, family hx

Sources of Vitamin E

corn, nuts, olives, green leafy vegetables, vegetable oils, wheat germ

Osmotic pressure

exerted by proteins in plasma

Diagnostic tests for pneumonia

chest x-ray, abg's, pulse ox, sputum c&s, cbc

Aspirin and MI

chew 325mg - antiplatelet

Complications of peritonitis

hypovolemic shock septicemia paralytic ileus organ failure

Complications of bowel obstruction

hypovolemic shock - due to necrosis and rupture of bowel - fluid shifts into peritoneum fluid/electrolyte and acid/base imbalance - metabolic alkalosis high in bowel - metabolic acidosis lower in bowel dehydration - from vomiting strangulation and gangrene of bowel

Surfactant

Protein responsible for surface tension and maintaining alveolar patency.

Pyloroplasty

Surgical intervention for gastric ulcers Enlargement of the pylorus to prevent or decrease pyloric obstruction, thereby enhancing gastric emptying

Circumstances that cause stasis of blood flow

immobility, surgery, pregnancy, obesity, varicose veins, artrial fibbrillation, heart failure

Treatment for varicose veins

ligation and stripping, thermal ablation, scleotherapy

Lantus

long acting insulin onset: 2 - 4 hours

Splinter hemmorhages

longitudinal black lines in nail beds - associated with endocarditis

Lasix

loop diuretic

Third spacing

loss of ECF into space that does not contribute to equilibrium

Insulin lipodystrophy

loss of subcutaneous fat and appears as slight dimpling or more serious pitting of subcutaneous fat; the use of human insulin helps prevent this complication

Medical management of fluid volume excess

restrict fluids and sodium, administer diuretics

Prinzmetal's angina

results from coronary artery spasm - may occur at rest - changes in ST segment on EKG - treat with calcium channel blockers

Dawn phenomenon

results from reduced tissue sensitivity to insulin that usually develops between 5 and 8 AM (prebreakfast hyperglycemia occurs); it may be caused by nocturnal release of growth hormone Treatment includes administering an evening dose (or increasing the amount of a current dose) of intermediate-acting insulin at about 10 PM.

The acidic condition of the blood shifts the oxyhemoglobin dissociation curve to the...

right - enables oxygen molecules to unload more easily from the hemoglobin

Treatment for PAD

risk factor modification, exercise (increases collateral circulation), extremity protection, medications (anticoagulants, antilipemics, pain meds)

The ideal oxygen source is...

room air F IO 2 0.18 to 0.21

Multifocal PVCs

runs of PVCs - danger of decreased cardiac output - risk of V-tach or V-fib

Complications of cholecystitis

subphrenic abscess pancreatitis biliary cirrhosis peritonitis from rupture of the gallbladder bile duct obstruction

Sources of Vitamin D

sun exposure, cheese, butter, margarine, fortified milk, fish, fortified cereals

Billroth II

Surgical intervention for gastric ulcers Partial gastrectomy, with the remaining segment anastomosed to the jejunum; also called gastrojejunostomy

Treatment for polycystic kidney disease

nephrectomy dialysis

Potassium plays a role in

nerve conduction, muscle function, ABB, osmotic pressure - controls rate and force of cardiac contraction

Clinical manifestations of hypertension

often nothing/silent killer, headache, dizziness, angina, blurred vision, epistaxis

Medications used for Esophageal Varices

Vasopressin Nitroglycerin Beta blockers

Which organ experiences the most serious effects of dehydration?

the brain

Signs of Bowel Perforation and Peritonitis

Guarding of the abdomen, Increased fever and chills, Pallor, Progressive abdominal distention and abdominal pain, Restlessness, Tachycardia and tachypnea

Noscomial Pneumonia (HAP)

onset of symptoms more than 48 hours after hospital admission

Rapid acting insulin

onset: 15 minutes peak: 1 hour duration: 2 - 4 hours *do not delay eating

Sources of Vitamin C

Citrus fruits, green peppers, strawberries, tomatoes, broccoli, potatoes

A client with chronic renal failure (CRF) is receiving a hemodialysis treatment. After hemodialysis, the nurse knows that the client is most likely to experience: 1. hematuria. 2. weight loss. 3. increased urine output. 4. increased blood pressure.

2. weight loss. Because CRF causes loss of renal function, the client with this disorder retains fluid. Hemodialysis removes this fluid, causing weight loss. Hematuria is unlikely to follow hemodialysis because the client with CRF usually forms little or no urine. Hemodialysis doesn't increase urine output because it doesn't correct the loss of kidney function, which severely decreases urine production in this disorder. By removing fluids, hemodialysis decreases rather than increases the blood pressure.

SVR

systemic vascular resistance

During preparation for bowel surgery, a client receives an antibiotic to reduce intestinal bacteria. Antibiotic therapy may interfere with synthesis of which vitamin and may lead to hypoprothrombinemia? 1. vitamin A 2. vitamin D 3. vitamin E 4. vitamin K

4. vitamin K Intestinal bacteria synthesize such nutritional substances as vitamin K, thiamine, riboflavin, vitamin B12, folic acid, biotin, and nicotinic acid. Therefore, antibiotic therapy may interfere with synthesis of these substances, including vitamin K. Intestinal bacteria don't synthesize vitamins A, D, or E.

Normal blood pH

7.35 - 7.45

normal pH range for arterial blood

7.35 to 7.45

Pulmonary Function Test

Assess respiratory function and determine the extent of dysfunction, non invasive

Stook Softeners

Colace, Peri Colace

Portal systemic encephalopathy

End -stage hepatic failure characterized by altered level of consciousness, neurological symptoms, impaired thinking, and neuromuscular disturbances; caused by failure of the diseased liver to detoxify neurotoxic agents such as ammonia. S&S: euphoria, depression, apathy, irritability, aggitation, slow/slurred speech, + babinski reflex, memory loss, disorientation, asterixis, fine motor impairment, apraxia, fector hepaticus

Three layers of the heart

Epicardium - outermost layer Myocardium - middle layer - muscle of heart Endocardium - innermost layer - lines inner chambers of heart and valves

Antimicrobials for H. Pylori

Flagyl, Amoxil, Biaxin, Achromycin

Bulimia

Frequent binge eating and self induced vomiting

Interventions for gastric ulcers

Monitor vital signs. Instruct the client about a bland diet, with small frequent meals. Provide for adequate rest. Encourage the cessation of smoking. Instruct the client to avoid alcohol intake, caffeine, the use of aspirin, corticosteroids, and NSAIDs. fAdminister medications to treat H. pylori and antacids to neutralize acid secretions as prescribed. Administer H2-receptor antagonists or proton pump inhibitors as prescribed to block the secretion of acid.

Pain management for MI

Morphine - increases oxygen to myocardium - 2-5mg IV push - decreases preload

Interventions for intestinal obstruction

NPO NG tube (irrigate with normal saline) Intestinal tube enema IV fluids (NS with K+) TPN surgery (preop neomycin enema)

Proctoscopy and Sigmoidoscopy

Requires the use of a flexible scope to examine the rectum and sigmoid colon; the client is placed on the left side with the right leg bent and placed anteriorly. - Biopsies and polypectomies can be performed. Preprocedure: Enemas are administered to cleanse the bowel. Postprocedure: Monitor for rectal bleeding and signs of perforation and peritonitis

Home Care for Hepatitis

Rest No close contact Avoid alcohol and hepatotoxic medications Antiemetics - Tigan - no compazine High carb, high protein, low fat diet - max intake at breakfast and small frequent feedings Vitamin B complex and Vitamin K supplements Fluids - 2,500 - 3,000 mL/day

Automaticity

The ability of cardiac cells to initiate impulse spontaneously and repetitively without external neurohormonal control.

Conductivity

The ability of the heart muscle fibers to propagate electrical impulses along and across cell membranes.

Enrichment

The addition of nutrients previously present in a food but removed during food processing or lost during storage

Cardiac output

Total volume of blood pumped through the heart in 1 minute. Stroke volume x heart rate.

Dumping Syndrome

The rapid emptying of the gastric contents into the small intestine that occurs following gastric resection Symptoms occur 30 minutes after eating: Nausea and vomiting, Feelings of abdominal fullness and abdominal cramping, Diarrhea, Palpitations and tachycardia, Perspiration, Weakness and dizziness, Borborygmi (loud gurgles indicating hyperperistalsis)

Stents

a small mesh tube that's used to treat narrow or weak arteries

Post Op Concerns for Laryngectomy

airway, nutrition, body image, speech rehab (esophageal vs TEP)

Diagnostic tests for pancreatitis

amylase lipase glucose triglycerides calcium 24hr urine (renal amylase and creatinine clearance) ERCP X-Ray/Ultrasound/CT

Manifestations of fluid volume excess

edema, distended neck veins, abnormal lung sounds, tachycardia, increased BP pulse presure and CVP, increased weight, increased urinary output, SOB, wheezing

Nitrates and MI

given IV dilates coronary arteries decreases SVR/afterload decreases preload

Risk factors for fluid volume excess

heart failure, renal failure, cirhosis of liver

Ventricular tachycardia

heart rate > 100 bpm regular rhythm no p wave QRS > .12

Chvostek's sign is seen with which electrolyte imbalance?

hypocalcemia

Trouseau's sign is seen with which electrolyte imbalance?

hypocalcemia

P wave

represents atrial depolarization/contraction

QRS complex

represents ventricular depolarization/contraction - normal is < .12 seconds

T wave

represents ventricular repolarization/relaxation - should appear upright

Alpha - Glucosidase Inhibitors

slow sugar absorption in the gut - blocks enzymes that digest starches Taken with meals Side effects: gas, bloating, diarrhea ie. Precose, Glyset

Digoxin - how does it work?

slows heart rate by increasing block at AV node and increases force of myocardial contraction

Diarrhea, seizures, tremors, and twitching together indicate which electrolyte imbalance?

Hypernatremia

Causes of Hypercalcemia

Hyperparathyroidism, malignant bone disease, immobilization, excess supplementation

Before weaning a client from mechanical ventilation, it's most important to have...

baseline ABG levels

Atelectasis

closure or collapse of alveoli

Hepatitis D

coinfects with Hepatitis B - causes more severe disease and greater risk of fulminant hepatitis

Symptoms of varicose veins

lower extremity ache or fatigue, edema, cosmetic disfigurment

Consolidation

lung tissue that becomes more solid in nature due to collapse of alveoli or infectious process

Signs and Symptoms of hypocalcemia

seizures, numbness, hyperactive DTRs, Trousseau's sign, Chvostek's sign/tetany

Major Nutrients in Dairy Group

Complete proteins, Calcium, Phosphorus, Potassium, Vitamin B (Riboflavin & B12), Vitamin D

Assessment findings for ulcerative colitis

1. Anorexia 2. Weight loss 3. Malaise 4. Abdominal tenderness and cramping 5. Severe diarrhea that may contain blood and mucus 6. Malnutrition, dehydration, and electrolyte imbalances 7. Anemia 8. Vitamin K deficiency

Cirrhosis and Portal Hypertension

A persistent increase in pressure in the portal vein that develops as a result of obstruction to flow - collateral circulation develops along esophagus, abdominal, and rectal areas.

Factors Affecting Respiration

CNS - controls rate and depth Lung compliance Lung elasticity/recoil Alveolar surfactant Chemoreceptors

Three requirements are necessary for informed decision-making:

-the decision must be given voluntarily -the client making the decision must have the capacity and competence to understand -the client must be given adequate information to make the decision

Pulse Oximetry

...

Respiratory Health Assessment Interview

...

Sputum Collection

...

Respiratory acidosis occurs when...

...alveolar underventilation is present. An indication the lungs are unable to elimintate CO2 as rapidly as it is produced. When PaCO2 increases and PO2 decreases with room air.

Respiratory alkalosis occurs when....

...there is alveolar hyperventalation. The lungs are eliminating CO2 in excress of production.

Metabolic acidosis occurs when...

...there is excess production of acids and loss of base OR when the body fails to excrete acids.

Small box on EKG =

0.04 seconds

Big box on EKG =

0.20 seconds

A client with a peptic ulcer reports epigastric pain that frequently awakens her during the night. The nurse should instruct the client to do which activities? Select all that apply. 1. Obtain adequate rest to reduce stimulation. 2. Eat small, frequent meals throughout the day. 3. Take all medications on time as ordered. 4. Sit up for one hour when awakened at night. 5. Stay away from crowded areas.

1, 2, 3, 4. The nurse should encourage the client to reduce stimulation that may enhance gastric secretion. The nurse can also advise the client to utilize health practices that will prevent recurrences of ulcer pain, such as avoiding fatigue and elimination of smoking. Eating small, frequent meals helps to prevent gastric distention if not actively bleeding and decreases distension and release of gastrin. Medications should be administered promptly to maintain optimum levels. After awakening during the night, the client should eat a small snack and return to bed, keeping the head of the bed elevated for an hour after eating. It is not necessary to stay away from crowded areas.

Which instruction about insulin administration should the nurse give to a client? 1. "Always follow the same order when drawing the different insulins into the syringe." 2. "Shake the vials before withdrawing the insulin." 3. "Store unopened vials of insulin in the freezer at temperatures well below freezing." 4. "Discard the intermediate-acting insulin if it appears cloudy."

1. "Always follow the same order when drawing the different insulins into the syringe." The client should be instructed always to follow the same order when drawing the different insulins into the syringe. Insulin should never be shaken because the resulting froth prevents withdrawal of an accurate dose and may damage the insulin protein molecules. Insulin also should never be frozen because the insulin protein molecules may be damaged. Intermediate-acting insulin is normally cloudy.

One hour before a client is to undergo abdominal surgery, the physician orders atropine, 0.6 mg I.M. The client asks the nurse why this drug must be administered. How should the nurse respond? 1. "Atropine decreases salivation and gastric secretions." 2. "Atropine controls the heart rate and blood pressure." 3. "Atropine improves ventilation by increasing the respiratory rate." 4. "Atropine enhances the effect of anesthetic agents."

1. "Atropine decreases salivation and gastric secretions." When used as preanesthesia medications, atropine and other cholinergic blocking agents reduce salivation and gastric secretions, thus helping to prevent aspiration of secretions during surgery. Atropine increases the heart rate and cardiac contractility, decreases bronchial secretions, and causes bronchodilation. No evidence indicates that the drug enhances the effect of anesthetic agents.

The physician prescribes glipizide (Glucotrol), an oral antidiabetic agent, for a client with type 2 diabetes mellitus who has been having trouble controlling the blood glucose level through diet and exercise. Which medication instruction should the nurse provide? 1. "Be sure to take glipizide 30 minutes before meals." 2. "Glipizide may cause a low serum sodium level, so make sure you have your sodium level checked monthly." 3. "You won't need to check your blood glucose level after you start taking glipizide." 4. "Take glipizide after a meal to prevent heartburn."

1. "Be sure to take glipizide 30 minutes before meals." The client should take glipizide twice per day, 30 minutes before a meal, because food decreases its absorption. The drug doesn't cause hyponatremia and therefore doesn't necessitate monthly serum sodium measurement. The client must continue to monitor blood glucose levels during glipizide therapy.

A client asks a nurse how soon after bowel surgery normal bowel function will return. The best response by the nurse would be: 1. "By 72 hours you should start to pass gas." 2. Around 48 hours, if there are no complications." 3. "Some function will return by 12 hours." 4. "You'll pass gas by 24 hours."

1. "By 72 hours you should start to pass gas." The bowel should be functioning by 72 hours. There may be faint bowel sounds at 48 hours but the bowel will not be fully functional. At 12 hours the effects of general anesthesia on the bowel are still present.

A client admitted for treatment of a gastric ulcer is being prepared for discharge on antacid therapy. Discharge teaching should include which instruction? 1. "Continue to take antacids even if your symptoms subside." 2. "You may take antacids with other medications." 3. "Avoid taking magnesium-containing antacids if you develop a heart problem." 4. "Be sure to take antacids with meals."

1. "Continue to take antacids even if your symptoms subside." Antacids decrease gastric acidity and should be continued even if the client's symptoms subside. Because other medications may interfere with antacid action, the client should avoid taking antacids concomitantly with other drugs. If cardiac problems arise, the client should avoid antacids containing sodium, not magnesium. For optimal results, the client should take an antacid 1 hour before or 2 hours after meals.

A 55-year-old diabetic client is admitted with hypoglycemia. Which information should the nurse include in her client teaching? 1. "Hypoglycemia can result from excessive alcohol consumption." 2. "Skipping meals can cause hypoglycemia." 3. "Symptoms of hypoglycemia include thirst and excessive urinary output." 4. "Strenuous activity may result in hypoglycemia." 5. "Symptoms of hypoglycemia include shakiness, confusion, and headache." 6. "Hypoglycemia is a relatively harmless situation."

1. "Hypoglycemia can result from excessive alcohol consumption." 2. "Skipping meals can cause hypoglycemia." 4. "Strenuous activity may result in hypoglycemia." 5. "Symptoms of hypoglycemia include shakiness, confusion, and headache." Alcohol consumption, missed meals, and strenuous activity may lead to hypoglycemia. Symptoms of hypoglycemia include shakiness, confusion, headache, sweating, and tingling sensations around the mouth. Thirst and excessive urination are symptoms of hyperglycemia. Hypoglycemia can become a life-threatening disorder involving seizures and death to brain cells; the client shouldn't be told that the condition is relatively harmless.

A client diagnosed with hyperosmolar hyperglycemic nonketotic syndrome (HHNS) is stabilized and prepared for discharge. When preparing the client for discharge and home management, which of the following statements indicates that the client understands her condition and how to control it? 1. "I can avoid getting sick by not becoming dehydrated and by paying attention to my need to urinate, drink, or eat more than usual." 2. "If I experience trembling, weakness, and headache, I should drink a glass of soda that contains sugar." 3. "I will have to monitor my blood glucose level closely and notify the physician if it's constantly elevated." 4. "If I begin to feel especially hungry and thirsty, I'll eat a snack high in carbohydrates."

1. "I can avoid getting sick by not becoming dehydrated and by paying attention to my need to urinate, drink, or eat more than usual." Inadequate fluid intake during hyperglycemic episodes often leads to HHNS. By recognizing the signs of hyperglycemia (polyuria, polydipsia, and polyphagia) and increasing fluid intake, the client may prevent HHNS. Drinking a glass of nondiet soda would be appropriate for hypoglycemia. A client whose diabetes is controlled with oral antidiabetic agents usually doesn't need to monitor blood glucose levels. A high-carbohydrate diet would exacerbate the client's condition, particularly if fluid intake is low.

A home health nurse visits a client with a diagnosis of type 1 diabetes mellitus. The client relates a history of vomiting and diarrhea and tells the nurse that no food has been consumed for the last 24 hours. Which additional statement by the client indicates a need for further teaching? 1. "I need to stop my insulin." 2. "I need to increase my fluid intake." 3. "I need to monitor my blood glucose every 3 to 4 hours." 4. "I need to call the physician because of these symptoms."

1. "I need to stop my insulin." Rationale: When a client with diabetes mellitus is unable to eat normally because of illness, the client still should take the prescribed insulin or oral medication. The client should consume additional fluids and should notify the physician. The client should monitor the blood glucose level every 3 to 4 hours. The client should also monitor the urine for ketones.

After undergoing a total cystectomy and urinary diversion, a client has a Kock pouch (continent internal reservoir). Which statement by the client indicates a need for further teaching? 1. "I'll have to wear an external collection pouch for the rest of my life." 2. "I should eat foods from all the food groups." 3. "I'll need to drink at least eight glasses of water a day." 4. "I'll have to catheterize my pouch every 2 hours."

1. "I'll have to wear an external collection pouch for the rest of my life." An internal collection pouch, such as the Kock pouch, allows the client to perform self-catheterization for ileal drainage. This pouch is an internal reservoir, eliminating the need for an external collection pouch. A well-balanced diet is essential for healing; the client need not include or exclude particular foods. The client should drink at least eight glasses of fluid daily to prevent calculi formation and urinary tract infection (UTI). Intervals between pouch drainings should be increased gradually until the pouch is emptied two to four times daily.

A client is evaluated for type 1 diabetes mellitus. Which client comment correlates best with this disorder? 1. "I'm thirsty all the time. I just can't get enough to drink." 2. "It seems like I have no appetite. I have to make myself eat." 3. "I have a cough and cold that just won't go away." 4. "I notice pain when I urinate."

1. "I'm thirsty all the time. I just can't get enough to drink." Classic signs and symptoms of diabetes mellitus are polydipsia (excessive thirst), polyuria (excessive urination), and polyphagia (excessive appetite). Decreased appetite, lingering cough and cold, and pain on urination aren't related to diabetes. Decreased appetite reflects a GI disorder; cough and cold indicate an upper respiratory problem; and pain on urination suggests a urinary tract infection (UTI).

Acarbose (Precose), an alpha-glucosidase inhibitor, is prescribed for a client with type 2 diabetes mellitus. During discharge planning, the nurse would be aware of the client's need for additional teaching when the client states: 1. "If I have hypoglycemia, I should eat some sugar, not dextrose." 2. "The drug makes my pancreas release more insulin." 3. "I should never take insulin while I'm taking this drug." 4. "It's best if I take the drug with the first bite of a meal."

1. "If I have hypoglycemia, I should eat some sugar, not dextrose." Acarbose delays glucose absorption, so the client should take an oral form of dextrose rather than a product containing table sugar when treating hypoglycemia. The alpha-glucosidase inhibitors work by delaying the carbohydrate digestion and glucose absorption. It's safe to be on a regimen that includes insulin and an alpha-glucosidase inhibitor. The client should take the drug at the start of a meal, not 30 minutes to an hour before.

While obtaining a client's medication history, the nurse learns that the client takes ranitidine (Zantac), as prescribed, to treat a peptic ulcer. The nurse continues gathering medication history data to assess for potential drug interactions. The client should avoid taking a drug from which class with ranitidine? 1. Antacids 2. Antibiotics 3. Antipsychotics 4. Antiarrhythmics

1. Antacids Because antacids can interact with ranitidine and interfere with its absorption, the client shouldn't take these drugs together. Ranitidine doesn't interact with antibiotic, antipsychotic, or antiarrhythmic agents.

A client is scheduled for a renal clearance test. The nurse should explain that this test is done to assess the kidneys' ability to remove a substance from the plasma in: 1. 1 minute. 2. 30 minutes. 3. 1 hour. 4. 24 hours.

1. 1 minute. The renal clearance test determines the kidneys' ability to remove a substance from the plasma in 1 minute. It doesn't measure the kidneys' ability to remove a substance over a longer period.

Because diet and exercise have failed to control a 63-year-old client's blood glucose level, the client is prescribed glipizide (Glucotrol). After oral administration, the onset of action is: 1. 15 to 30 minutes 2. 30 to 60 minutes 3. 1 to 1½ hours 4. 2 to 3 hours

1. 15 to 30 minutes Glipizide begins to act in 15 to 30 minutes. The other options are incorrect.

After a 3-month trial of dietary therapy, a client with type 2 diabetes mellitus still has blood glucose levels above 180 mg/dl. The physician adds glyburide (DiaBeta), 2.5 mg P.O. daily, to the treatment regimen. The nurse should instruct the client to take the glyburide: 1. 30 minutes before breakfast. 2. in the midmorning. 3. 30 minutes after dinner. 4. at bedtime.

1. 30 minutes before breakfast. Like other oral antidiabetic agents prescribed in a single daily dose, glyburide should be taken with breakfast or 30 minutes before breakfast. If the client takes glyburide later, such as in the midmorning, after dinner, or at bedtime, the drug won't provide adequate coverage for all meals consumed during the day.

A client with type 1 diabetes is scheduled to receive 30 U of 70/30 insulin. There is no 70/30 insulin available. As a substitution, the nurse may give the client: 1. 9 U regular insulin and 21 U neutral protamine Hagedorn (NPH). 2. 21 U regular insulin and 9 U NPH. 3. 10 U regular insulin and 20 U NPH. 4. 20 U regular insulin and 10 U NPH.

1. 9 U regular insulin and 21 U neutral protamine Hagedorn (NPH). A 70/30 insulin preparation is 70% NPH and 30% regular insulin. Therefore, a correct substitution requires mixing 21 U of NPH and 9 U of regular insulin. The other choices are incorrect dosages for the prescribed insulin.

A client has developed hepatitis A after eating contaminated oysters. The nurse assesses the client for which of the following? 1. Malaise 2. Dark stools 3. Weight gain 4. Left upper quadrant discomfort

1. Malaise Hepatitis causes gastrointestinal symptoms such as anorexia, nausea, right upper quadrant discomfort , and weight loss. Fatigue and malaise are common. Stools will be light- or clay-colored if conjugated bilirubin is unable to flow out of the liver because of inflammation or obstruction of the bile ducts.

Treatment for Gastritis

1. Acute gastritis: Food and fluids may be withheld until symptoms subside; afterward, ice chips can be given, followed by clear liquids, and then solid food. 2. Monitor for signs of hemorrhagic gastritis such as hematemesis, tachycardia, and hypotension, and notify the physician if these signs occur. 3. Instruct the client to avoid irritating foods, fluids, and other substances, such as spicy and highly seasoned foods, caffeine, alcohol, and nicotine. 4. Instruct the client in the use of prescribed medications, such as antibiotics and antacids. 5. Provide the client with information about the importance of vitamin B12 injections if a deficiency is present.

A client comes to the emergency department complaining of severe pain in the right flank, nausea, and vomiting. The physician tentatively diagnoses right ureterolithiasis (renal calculi). When planning this client's care, the nurse should assign highest priority to which nursing diagnosis? 1. Acute pain 2. Risk for infection 3. Impaired urinary elimination 4. Imbalanced nutrition: Less than body requirements

1. Acute pain Ureterolithiasis typically causes such acute, severe pain that the client can't rest and becomes increasingly anxious. Therefore, the nursing diagnosis of Acute pain takes highest priority. Diagnoses of Risk for infection and Impaired urinary elimination are appropriate when the client's pain is controlled. A diagnosis of Imbalanced nutrition: Less than body requirements isn't pertinent at this time.

A client is evaluated for severe pain in the right upper abdominal quadrant, which is accompanied by nausea and vomiting. The physician diagnoses acute cholecystitis and cholelithiasis. For this client, which nursing diagnosis takes top priority? 1. Acute pain related to biliary spasms 2. Deficient knowledge related to prevention of disease recurrence 3. Anxiety related to unknown outcome of hospitalization 4. Imbalanced nutrition: Less than body requirements related to biliary inflammation

1. Acute pain related to biliary spasms The chief symptom of cholecystitis is abdominal pain or biliary colic. Typically, the pain is so severe that the client is restless and changes positions frequently to find relief. Therefore, the nursing diagnosis of Acute pain related to biliary spasms takes highest priority. Until the acute pain is relieved, the client can't learn about prevention, may continue to experience anxiety, and can't address nutritional concerns.

Interventions for ulcerative colitis

1. Acute phase: Maintain NPO status and administer fluids and electrolytes intravenously or via parenteral nutrition as prescribed. 2. Restrict the client's activity to reduce intestinal activity. 3. Monitor bowel sounds and for abdominal tenderness and cramping. 4. Monitor stools, noting color, consistency, and the presence or absence of blood. 5. Monitor for bowel perforation, peritonitis, and hemorrhage. 6. Following the acute phase, the diet progresses from clear liquids to a low-fiber diet as tolerated. 7. Instruct the client about diet; usually a low-fiber, high-protein diet with vitamins and iron supplements are prescribed. 8. Instruct the client to avoid gas-forming foods, milk products, and foods such as whole wheat grains, nuts, raw fruits and vegetables, pepper, alcohol, and caffeine-containing products. 9. Instruct the client to avoid smoking. 10. Administer medications as prescribed , which may include a combination of medications such as salicylate compounds , corticosteroids, immunosuppressants, and antidiarrheals.

Which of the following statements about fluid replacement is accurate for a client with hyperosmolar hyperglycemic nonketotic syndrome (HHNS)? 1. Administer 2 to 3 L of I.V. fluid rapidly. 2. Administer 6 L of I.V. fluid over the first 24 hours. 3. Administer a dextrose solution containing normal saline solution. 4. Administer I.V. fluid slowly to prevent circulatory overload and collapse.

1. Administer 2 to 3 L of I.V. fluid rapidly. Regardless of the client's medical history, rapid fluid resuscitation is critical for maintaining cardiovascular integrity. Profound intravascular depletion requires aggressive fluid replacement. A typical fluid resuscitation protocol is 6 L of fluid over the first 12 hours, with more fluid to follow over the next 24 hours. Various fluids can be used, depending on the degree of hypovolemia. Commonly prescribed fluids include dextran (in cases of hypovolemic shock), isotonic normal saline solution and, when the client is stabilized, hypotonic half-normal saline solution.

When preparing a client for a hemorrhoidectomy, the nurse should take which action? 1. Administer an enema as ordered. 2. Administer oral antibiotics as prescribed. 3. Administer topical antibiotics as prescribed. 4. Administer analgesics as prescribed.

1. Administer an enema as ordered. When preparing a client for a hemorrhoidectomy, the nurse should administer an enema, as ordered, and record the results. After surgery, the client may require antibiotics and analgesics.

Atherosclerosis

an accumulation of lipid-containing plaque in the arteries

A nurse is reviewing the prescription for a client admitted to the hospital with a diagnosis of acute pancreatitis. Which of the following interventions would the nurse expect to be prescribed for the client? Select all that apply. 1. Administer antacids as prescribed. 2. Encourage coughing and deep breathing. 3. Administer anticholinergics as prescribed. 4. Give small, frequent high-calorie feedings. 5. Maintain the client in a supine and flat position. 6. Give Meperidine (Demerol) as prescribed for pain.

1. Administer antacids as prescribed. 2. Encourage coughing and deep breathing. 3. Administer anticholinergics as prescribed. 6. Give Meperidine (Demerol) as prescribed for pain. The client with acute pancreatitis normally is placed on NPO status to rest the pancreas and suppress gastrointestinal secretions. Because abdominal pain is a prominent symptom of pancreatitis, pain medication such as meperidine is prescribed. Some clients experience lessened pain by assuming positions that flex the trunk, with the knees drawn up to the chest. A side-lying position with the head elevated 45 degrees decreases tension on the abdomen and may help ease the pain. The client is susceptible to respiratory infections because the retroperitoneal fluid raises the diaphragm, which causes the client to take shallow, guarded abdominal breaths. Therefore measures such as turning, coughing, and deep breathing are instituted. Antacids and anticholinergics may be prescribed to suppress gastrointestinal secretions.

A client with type 1 diabetes must undergo bowel resection in the morning. How should the nurse proceed while caring for him on the morning of surgery? 1. Administer half of the client's typical morning insulin dose. 2. Administer an oral antidiabetic agent. 3. Administer an I.V. insulin infusion. 4. Administer the client's normal daily dose of insulin.

1. Administer half of the client's typical morning insulin dose. If the nurse administers the client's normal daily dose of insulin while he's on nothing-by-mouth status before surgery, he'll become hypoglycemic; half the daily insulin dose will provide all that's needed. Oral antidiabetic agents aren't effective for type 1 diabetes. I.V. insulin infusions aren't necessary to manage blood glucose levels in clients undergoing routine surgery.

The nurse is teaching a client about the risk factors associated with colorectal cancer. The nurse determines that further teaching related to colorectal cancer is necessary if the client identifies which of the following as an associated risk factor? 1. Age younger than 50 years 2. History of colorectal polyps 3. Family history of colorectal cancer 4. Chronic inflammatory bowel disease

1. Age younger than 50 years Colorectal cancer risk factors include age older than 50 years, a family history of the disease, colorectal polyps, and chronic inflammatory bowel disease.

Interventions for hemorrhoids

1. Apply cold packs to the anal-rectal area followed by sitz baths as prescribed. 2. Apply witch hazel soaks and topical anesthetics as prescribed. 3. Encourage a high-fiber diet and fluids to promote bowel movements without straining. 4. Administer stool softeners as prescribed.

After undergoing renal arteriogram, in which the left groin was accessed, the client complains of left calf pain. Which intervention should the nurse perform first? 1. Assess peripheral pulses in the left leg. 2. Place cool compresses on the calf. 3. Exercise the leg and foot. 4. Assess for anaphylaxis.

1. Assess peripheral pulses in the left leg. The nurse should begin by assessing peripheral pulses in the left leg to determine if blood flow was interrupted by the procedure. The client may also have thrombophlebitis. Cool compresses aren't used to relieve pain and inflammation in thrombophlebitis. The leg should remain straight after the procedure. Calf pain isn't a symptom of anaphylaxis.

PostOp interventions for rectal surgery

1. Assist the client to a prone or side-lying position to prevent bleeding. 2. Maintain ice packs over the dressing as prescribed until the packing is removed by the physician. 3. Monitor for urinary retention. 4. Administer stool softeners as prescribed. 5. Instruct the client to increase fluids and high-fiber foods. 6. Instruct the client to limit sitting to short periods of time. 7. Instruct the client in the use of sitz baths three or four times a day as prescribed.

A client, age 82, is admitted to an acute care facility for treatment of an acute flare-up of a chronic GI condition. In addition to assessing the client for complications of the current illness, the nurse monitors for age-related changes in the GI tract. Which age-related change increases the risk of anemia? 1. Atrophy of the gastric mucosa 2. Decrease in intestinal flora 3. Increase in bile secretion 4. Dulling of nerve impulses

1. Atrophy of the gastric mucosa Atrophy of the gastric mucosa reduces hydrochloric acid secretion; this, in turn, impairs absorption of iron and vitamin B12, increasing the risk of anemia as a person ages. A decrease in hydrochloric acid increases, not decreases, intestinal flora; as a result, the client is at increased risk for infection, not anemia. A reduction, not increase, in bile secretion may lead to malabsorption of fats and fat-soluble vitamins. Dulling of nerve impulses associated with aging increases the risk of constipation, not anemia.

Assessment findings for BPH

1. Diminished size and force of urinary stream (early sign of BPH) 2. Urinary urgency and frequency 3. Nocturia 4. Inability to start (hesitancy) or continue a urinary stream 5. Feelings of incomplete bladder emptying 6. Postvoid dribbling from overflow incontinence (later sign) 7. Urinary retention and bladder distention 8. Hematuria 9. Urinary stasis 10. Dysuria and bladder pain 11. UTIs

Interventions for UTIs

1. Before administering prescribed antibiotics, obtain a urine specimen for culture and sensitivity, if prescribed, to identify bacterial growth. 2. Encourage the client to increase fluids up to 3000 mL/ day, especially if the client is taking a sulfonamide; sulfonamides can form crystals in concentrated urine. 3. Administer prescribed medications , which may include analgesics, antiseptics, antispasmodics, antibiotics, and antimicrobials. 4. Maintain an acid urine pH (5.5); instruct the client about foods to consume to maintain acidic urine. 5. Provide heat to the abdomen or sitz baths for complaints of discomfort. 6. Note that if the client is prescribed an aminoglycoside, sulfonamide, or nitrofurantoin (Macrodantin), the actions of these medications are decreased by acidic urine. 7. Use sterile technique when inserting a urinary catheter. 8. Maintain closed urinary drainage systems for the client with an indwelling catheter and avoid elevating the urinary drainage bag above the level of the bladder. 9. Provide meticulous perineal care for the client with an indwelling catheter. 10. Discourage caffeine products such as coffee, tea, and cola. 11. Client education a. Avoid alcohol. b. Take medications as prescribed. c. Take antibiotics on schedule and complete the entire course of medications as prescribed, which may be 10 to 14 days. d. Repeat the urine culture following treatment. e. Prevent recurrence of cystitis

Assessment findings for colon cancer

1. Blood in stool (most common manifestation) 2. Anorexia, vomiting, and weight loss 3. Anemia 4. Abnormal stools a. Ascending colon tumor: Diarrhea b. Descending colon tumor: Constipation or some diarrhea, or flat, ribbon-like stool caused by a partial obstruction c. Rectal tumor: Alternating constipation and diarrhea 5. Guarding or abdominal distention, abdominal mass (late sign) 6. Cachexia (late sign) 7. Masses noted on barium enema, colonoscopy, CT scan, sigmoidoscopy

Modes of Transmission for Hepatitis B

1. Blood or body fluid contact 2. Infected blood products 3. Infected saliva or semen 4. Contaminated needles 5. Sexual contact 6. Parenteral 7. Perinatal period 8. Blood or body fluids contact at birth

A client with inflammatory bowel disease undergoes an ileostomy. On the first day after surgery, the nurse notes that the client's stoma appears dusky. How should the nurse interpret this finding? 1. Blood supply to the stoma has been interrupted. 2. This is a normal finding 1 day after surgery. 3. The ostomy bag should be adjusted. 4. An intestinal obstruction has occurred.

1. Blood supply to the stoma has been interrupted. An ileostomy stoma forms as the ileum is brought through the abdominal wall to the surface skin, creating an artificial opening for waste elimination. The stoma should appear cherry red, indicating adequate arterial perfusion. A dusky stoma suggests decreased perfusion, which may result from interruption of the stoma's blood supply and may lead to tissue damage or necrosis. A dusky stoma isn't a normal finding. Adjusting the ostomy bag wouldn't affect stoma color, which depends on blood supply to the area. An intestinal obstruction also wouldn't change stoma color.

A 75-year-old client with renal insufficiency is admitted to the hospital with pneumonia. He's being treated with gentamicin (Garamycin), which can be nephrotoxic. Which laboratory value should be closely monitored? 1. Blood urea nitrogen 2. Sodium level 3. Alkaline phosphatase 4. White blood cell (WBC) count

1. Blood urea nitrogen Blood urea nitrogen and creatinine levels should be closely monitored to detect elevations caused by nephrotoxicity. Sodium level should be routinely monitored in all hospitalized clients. Alkaline phosphatase helps evaluate liver function. The WBC count should be monitored to evaluate the effectiveness of the antibiotic; it doesn't help evaluate kidney function.

Assessment findings for hemorrhoids

1. Bright red bleeding with defecation 2. Rectal pain 3. Rectal itching

Which food should the nurse advise a client with a colostomy to avoid? 1. Carbonated drinks. 2. Fresh-cooked green beans. 3. Liver and bacon. 4. Cooked cereals.

1. Carbonated drinks. Carbonated drinks, cabbage, sauerkraut, and nuts tend to increase flatulence, and most clients feel uncomfortable passing flatus into the colostomy bag, because it causes the bag to inflate. Onions, cheese, and fish may cause odorous drainage.

The nurse is teaching the client about insulin infusion pump use. What intervention should the nurse include to prevent infection at the injection site? 1. Change the needle every 3 days. 2. Wear sterile gloves when inserting the needle. 3. Take the prescribed antibiotics before initiating treatment. 4. Use clean technique when changing the needle.

1. Change the needle every 3 days. The nurse should teach the client to change the needle every 3 days to prevent infection. The client doesn't need to wear gloves when changing the needle. Antibiotic therapy isn't necessary before the initiation of treatment. Sterile technique, not clean technique, is needed when changing the needle.

Complications of Hepatitis C

1. Chronic liver disease 2. Cirrhosis 3. Primary hepatocellular carcinoma

A client's blood glucose level is 45 mg/dl. The nurse should be alert for which signs and symptoms? 1. Coma, anxiety, confusion, headache, and cool, moist skin. 2. Kussmaul's respirations, dry skin, hypotension, and bradycardia. 3. Polyuria, polydipsia, hypotension, and hypernatremia. 4. Polyuria, polydipsia, polyphagia, and weight loss.

1. Coma, anxiety, confusion, headache, and cool, moist skin. Signs and symptoms of hypoglycemia include anxiety, restlessness, headache, irritability, confusion, diaphoresis, cool skin, tremors, coma, and seizures. Kussmaul's respirations, dry skin, hypotension, and bradycardia are signs of diabetic ketoacidosis. Excessive thirst, hunger, hypotension, and hypernatremia are symptoms of diabetes insipidus. Polyuria, polydipsia, polyphagia, and weight loss are classic signs and symptoms of diabetes mellitus.

What is the primary nursing diagnosis for a client with a bowel obstruction? 1. Deficient fluid volume 2. Deficient knowledge 3. Acute pain 4. Ineffective tissue perfusion

1. Deficient fluid volume Feces, fluid, and gas accumulate above a bowel obstruction. Then the absorption of fluids decreases and gastric secretions increase. This process leads to a loss of fluids and electrolytes in circulation. Options 2, 3, and 4 are applicable but not the primary nursing diagnosis.

A client is admitted with a serum glucose level of 618 mg/dl. The client is awake and oriented, with hot, dry skin; a temperature of 100.6° F (38.1° C); a heart rate of 116 beats/min; and a blood pressure of 108/70 mm Hg. Based on these findings, which nursing diagnosis takes highest priority? 1. Deficient fluid volume related to osmotic diuresis 2. Decreased cardiac output related to increased heart rate 3. Imbalanced nutrition: Less than body requirements related to insulin deficiency 4. Ineffective thermoregulation related to dehydration

1. Deficient fluid volume related to osmotic diuresis A serum glucose level of 618 mg/dl indicates hyperglycemia, which causes polyuria and fluid volume deficit. In this client, tachycardia is more likely to result from a fluid volume deficit than from decreased cardiac output because the blood pressure is normal. Although the client's serum glucose level is elevated, food isn't a priority because fluids and insulin should be administered to lower the serum glucose level. Therefore, Imbalanced nutrition: Less than body requirements isn't an appropriate nursing diagnosis. A temperature of 100.6° F (38.1° C) isn't life-threatening, eliminating Ineffective thermoregulation as the top priority.

A client with a serum glucose level of 618 mg/dl is admitted to the facility. He's awake and oriented, has hot dry skin, and has the following vital signs: temperature of 100.6° F (38.1° C), heart rate of 116 beats/minute, and blood pressure of 108/70 mm Hg. Based on these assessment findings, which nursing diagnosis takes highest priority? 1. Deficient fluid volume related to osmotic diuresis 2. Decreased cardiac output related to elevated heart rate 3. Imbalanced nutrition: Less than body requirements related to insulin deficiency 4. Ineffective thermoregulation related to dehydration

1. Deficient fluid volume related to osmotic diuresis A serum glucose level of 618 mg/dl indicates hyperglycemia, which causes polyuria and fluid volume deficit. In this client, tachycardia is more likely to result from fluid volume deficit than from decreased cardiac output because his blood pressure is normal. Although the client's serum glucose is elevated, food isn't a priority because fluids and insulin should be administered to lower the serum glucose level. Therefore, a diagnosis of Imbalanced nutrition: Less than body requirements isn't appropriate. A temperature of 100.6° F isn't life-threatening, eliminating ineffective thermoregulation as the top priority.

The nurse is reviewing the record of a client with Crohn's disease. Which stool characteristic should the nurse expect to note documented in the client's record? 1. Diarrhea 2. Chronic constipation 3. Constipation alternating with diarrhea 4. Stool constantly oozing from the rectum

1. Diarrhea Crohn's disease is characterized by nonbloody diarrhea of usually not more than four to five stools daily. Over time, the diarrhea episodes increase in frequency, duration, and severity. Options 2, 3, and 4 are not characteristics of Crohn's disease.

A client with mild diarrhea, fever, and abdominal discomfort is being evaluated for inflammatory bowel disease (IBD). Which statement about IBD is true? 1. Diarrhea is the most common sign of IBD. 2. Transmural inflammation with fistula formation occurs in ulcerative colitis, one form of IBD. 3. Abscesses may occur in IBD as poor nutrition causes breakdown of cells in the GI tract. 4. Bowel cancer is common in clients with a history of Crohn's disease, one form of IBD.

1. Diarrhea is the most common sign of IBD. IBD is a collective term for several GI inflammatory diseases with unknown causes. The most prominent sign of IBD is mild diarrhea, which sometimes is accompanied by fever and abdominal discomfort. The pathophysiology of ulcerative colitis involves vascular congestion, hemorrhage, and edema — usually affecting the rectum and left colon. Although abscesses may occur in IBD, they result from buildup of lymphocytes and cellular debris in crypts, which may serve as abscess sites. Only about 3% of clients with a long history of Crohn's disease develop bowel cancer.

A client returns to the medical-surgical unit after coronary artery bypass graft surgery, which was complicated by prolonged cardiopulmonary bypass and hypotension. After 3 hours in the unit, the client's condition stabilizes. However, the urine output has decreased despite adequate filling pressures. The nurse expects the physician to add which drug, at which flow rate, to the client's regimen? 1. Dopamine (Intropin), 3 mcg/kg/min 2. Epinephrine, 2 mcg/kg/min 3. Dopamine (Intropin), 8 mcg/kg/min 4. Epinephrine, 4 mcg/kg/min

1. Dopamine (Intropin), 3 mcg/kg/min This client is at high risk for acute prerenal failure secondary to decreased renal perfusion during surgery. To dilate the renal arteries and help prevent renal shutdown, the physician is likely to prescribe dopamine at a low flow rate (2 to 5 mcg/kg/min). Although this drug has mixed dopaminergic and beta activity when given at 5 to 10 mcg/kg/min, the client is stabilized and thus doesn't need the beta effects from the higher flow rate — or the sympathomimetic effects of epinephrine. The dopaminergic effects of dopamine increase renal perfusion, contractility, and vasodilation. Stimulation of beta receptors causes beta effects — namely, increases in the heart rate, myocardial contraction force, and cardiac conduction.

The nurse is caring for a client who had a stroke. Which nursing intervention promotes urinary continence? 1. Encouraging intake of at least 2 L of fluid daily 2. Giving the client a glass of soda before bedtime 3. Taking the client to the bathroom twice per day 4. Consulting with a dietitian

1. Encouraging intake of at least 2 L of fluid daily By encouraging a daily fluid intake of at least 2 L, the nurse helps fill the client's bladder, thereby promoting bladder retraining by stimulating the urge to void. The nurse shouldn't give the client soda before bedtime; soda acts as a diuretic and may make the client incontinent. The nurse should take the client to the bathroom or offer the bedpan at least every 2 hours throughout the day; twice per day is insufficient. Consultation with a dietitian won't address the problem of urinary incontinence.

The client is prescribed continuous bladder irrigation at a rate of 60 gtt/minute. The nurse hangs a 2 L bag of sterile solution with tubing on a three-legged I.V. pole. She then attaches the tubing to the client's three-way urinary catheter, adjusts the flow rate, and leaves the room. Which important procedural step did the nurse fail to follow? 1. Evaluating patency of the drainage lumen 2. Counter-balancing the I.V. pole 3. Attaching the infusion set to an infusion pump 4. Collecting a urine specimen before beginning irrigation

1. Evaluating patency of the drainage lumen The nurse should evaluate patency of the drainage tubing before leaving the client's room. If the lumen is obstructed, the solution infuses into the bladder but isn't eliminated through the drainage tubing, a situation that may cause client injury. Balancing the pole is important; however, the nurse would have had to address this issue immediately after hanging the 2 L bag. Using an I.V. pump isn't necessary for continuous bladder irrigation. Unless specifically ordered, obtaining a urine specimen before beginning continuous bladder irrigation isn't necessary.

4 Primary Functions of Respiratory System

1. Exchange of O2 and CO2 2. Delivery of O2 to tissues 3. Elimination of CO2 4. Maintain ABB (acid base balance)

Risk factors for nephrolithiasis

1. Family history of stone formation 2. Diet high in calcium, vitamin D, protein, oxalate, purines, or alkali 3. Obstruction and urinary stasis 4. Dehydration 5. Use of diuretics, which can cause volume depletion 6. Urinary tract infections and prolonged urinary catheterization 7. Immobilization 8. Hypercalcemia and hyperparathyroidism 9. Elevated uric acid level, such as in gout

Transmission of Hepatitis A

1. Fecal-oral route 2. Person-to-person contact 3. Parenteral 4. Contaminated fruits, vegetables, or uncooked shellfish 5. Contaminated water or milk 6. Poorly washed utensils

Assessment findings for Chron's disease

1. Fever 2. Cramp-like and colicky pain after meals 3. Diarrhea (semisolid), which may contain mucus and pus (non-bloody) 4. Abdominal distention 5. Anorexia, nausea, and vomiting 6. Weight loss 7. Anemia 8. Dehydration 9. Electrolyte imbalances 10. Malnutrition (may be worse than that seen in ulcerative colitis)

Assessment findings for UTIs

1. Frequency and urgency 2. Burning on urination 3. Voiding in small amounts 4. Inability to void 5. Incomplete emptying of the bladder 6. Lower abdominal discomfort or back discomfort 7. Cloudy, dark, foul-smelling urine 8. Hematuria 9. Bladder spasms 10. Malaise, chills, fever 11. Nausea and vomiting 12. WBC count greater than 100,000 cells/mm3 on urinalysis 13. An elevated specific gravity and pH may be noted on urinalysis.

Complications of Hepatitis B

1. Fulminant hepatitis 2. Chronic liver disease 3. Cirrhosis 4. Primary hepatocellular carcinoma

Which infection control equipment is necessary for the client diagnosed with Clostridium difficile diarrhea? 1. Gloves 2. Mask 3. Face shield 4. N-95 respirator

1. Gloves The client diagnosed with Clostridium difficile diarrhea requires contact isolation. Contact isolation precautions require the use of glove and a gown if soiling is likely. A mask, face shield, and N-95 respirator aren't necessary to maintain contact isolation.

Assessment findings for glomerulonephritis

1. Gross hematuria 2. Dark, smoky, cola-colored or red-brown urine 3. Proteinuria that produces a persistent and excessive foam in the urine 4. Urinary debris 5. Moderately elevated to high urine specific gravity 6. Low urinary pH 7. Urinalysis shows large numbers of erythrocytes 8. Oliguria or anuria 9. Headache 10. Chills and fever 11. Fatigue and weakness 12. Anorexia, nausea, and vomiting 13. Pallor 14. Edema in the face, periorbital area, feet, or generalized 15. Shortness of breath, ascites, pleural effusion, and CHF 16. Abdominal or flank pain 17. Hypertension 18. Reduced visual acuity 19. Increased blood urea nitrogen and serum creatinine levels 20. Increased antistreptolysin O titer (used to diagnose disorders caused by streptococcal infections)

The nurse is teaching a client how to irrigate his stoma. Which action indicates that the client needs more teaching? 1. Hanging the irrigation bag 24″ to 36″ (60 to 90 cm) above the stoma 2. Filling the irrigation bag with 500 to 1,000 ml of lukewarm water 3. Stopping irrigation for cramps and clamping the tubing until cramps pass 4. Washing hands with soap and water when finished

1. Hanging the irrigation bag 24″ to 36″ (60 to 90 cm) above the stoma An irrigation bag should be elevated 18″ to 24″ (40 to 60 cm) above the stoma. Typically, adults use 500 to 1,000 ml of water at a temperature no higher than 105° F (41° C) to irrigate a colostomy. If cramping occurs during irrigation, irrigation should be stopped and the client should take deep breaths until the cramping stops. Irrigation can then be resumed. Hand washing reduces the spread of microorganisms.

A client with a retroperitoneal abscess is receiving gentamicin (Garamycin). Which signs should the nurse monitor? 1. Hearing 2. Urine output 3. Hematocrit (HCT) 4. Blood urea nitrogen (BUN) and creatinine levels 5. Serum calcium level

1. Hearing 2. Urine output 4. Blood urea nitrogen (BUN) and creatinine levels Adverse reactions to gentamicin include ototoxicity and nephrotoxicity. The nurse must monitor the client's hearing and instruct him to report any hearing loss or tinnitus. Signs of nephrotoxicity include decreased urine output and elevated BUN and creatinine levels. Gentamicin doesn't affect the serum calcium level or HCT.

The physician has determined that the client with hepatitis has contracted the infection from contaminated food. The nurse understands that this client is most likely experiencing what type of hepatitis? 1. Hepatitis A 2. Hepatitis B 3. Hepatitis C 4. Hepatitis D

1. Hepatitis A Hepatitis A is transmitted by the fecal-oral route via contaminated food or infected food handlers. Hepatitis B, C, and D are transmitted most commonly via infected blood or body fluids.

What does a positive Chvostek's sign indicate? 1. Hypocalcemia 2. Hyponatremia 3. Hypokalemia 4. Hypermagnesemia

1. Hypocalcemia Chvostek's sign is elicited by tapping the client's face lightly over the facial nerve, just below the temple. If the client's facial muscles twitch, it indicates hypocalcemia. Hyponatremia is indicated by weight loss, abdominal cramping, muscle weakness, headache, and postural hypotension. Hypokalemia causes paralytic ileus and muscle weakness. Clients with hypermagnesemia exhibit a loss of deep tendon reflexes, coma, or cardiac arrest.

Signs of biliary obstruction

1. Jaundice 2. Dark orange and foamy urine 3. Steatorrhea and clay-colored feces 4. Pruritus

Populations at Risk for Hepatitis B

1. IV drug users 2. Clients undergoing long-term hemodialysis 3. Health care personnel

If a bowel obstruction occurs from inflammatory bowel disease at the transverse colon, a nurse will initially hear bowel sounds that are: 1. Increased RLQ; decreased LLQ. 2. Decreased RLQ; decreased LLQ. 3. Absent RLQ; absent LLQ. 4. Decreased RLQ; increased LLQ.

1. Increased RLQ; decreased LLQ. Initially peristalsis will increase in the ascending colon (RLQ) in an attempt to clear the blockage. There will be no peristalsis distal to the obstruction.

A client with type 1 diabetes mellitus is admitted to an acute care facility with diabetic ketoacidosis. To correct this acute diabetic emergency, which measure should the health care team take first? 1. Initiate fluid replacement therapy. 2. Administer insulin. 3. Correct diabetic ketoacidosis. 4. Determine the cause of diabetic ketoacidosis.

1. Initiate fluid replacement therapy. The health care team first initiates fluid replacement therapy to prevent or treat circulatory collapse caused by severe dehydration. Although diabetic ketoacidosis results from insulin deficiency, the client must have an adequate fluid volume before insulin can be administered; otherwise, the drug won't circulate throughout the body effectively. Therefore, insulin administration follows fluid replacement therapy. Determining and correcting the cause of diabetic ketoacidosis are important steps, but the client's condition must be stabilized first to prevent life-threatening complications.

A client, age 23, is diagnosed with diabetes mellitus. The physician prescribes 15 U of U-100 regular insulin and 35 U of U-100 isophane insulin suspension (NPH) to be taken before breakfast. The nurse checks the medication order, assembles equipment, washes hands, rotates the NPH insulin vial, puts on disposable gloves, and cleans the stoppers. To draw the two insulin doses into the single U-100 insulin syringe, which sequence should the nurse use? 1. Inject 35 U air into NPH vial; inject 15 U air into regular insulin vial, withdraw 15 U regular insulin; withdraw 35 U NPH. 2. Inject 15 U air into regular insulin vial; inject 35 U air into NPH vial, withdraw 35 U of NPH; withdraw 15 U regular insulin. 3. Inject 15 U air into regular insulin vial, withdraw 15 units of regular insulin; inject 35 U air into NPH vial and withdraw 35 U NPH. 4. Inject 35 U air into NPH vial; inject 15 U air into regular insulin vial; withdraw 35 U NPH; withdraw 15 U regular insulin.

1. Inject 35 U air into NPH vial; inject 15 U air into regular insulin vial, withdraw 15 U regular insulin; withdraw 35 U NPH. To avoid creating a vacuum, the nurse must inject exactly the same amount of air into a multidose vial to replace the amount of medication to be withdrawn. Follow these steps: (1) Inject air into the vial from which the second insulin dose will be withdrawn (isophane insulin). (2) Inject air into the vial from which insulin will be withdrawn first (regular insulin). (3) With the needle inserted into the regular insulin vial, withdraw the correct amount. (4) With 15 U of regular insulin in the syringe, carefully withdraw 35 U of NPH, for a total of 50 U in the syringe. Options 2 and 4 are incorrect because regular insulin must be withdrawn first. Option 3 is incorrect because the nurse must not insert air into a multiple-dose vial with a syringe containing medication.

Following perineal surgery, a client is at risk for a wound infection related to incontinence. The correct management of this problem is to: 1. Insert a continuous indwelling catheter per order. 2. Assist to the toilet and protect the skin with cream. 3. Limit oral fluid intake. 4. Give a loop diuretic, such as furosemide, as ordered.

1. Insert a continuous indwelling catheter per order. An indwelling catheter diverts urine away from the operative site, reducing risk of wound infection.

Treatments for GERD

1. Instruct the client to avoid factors that decrease lower esophageal sphincter pressure or cause esophageal irritation such as peppermint, chocolate, coffee, fried or fatty foods, carbonated beverages, alcoholic beverages, and cigarette smoking. 2. Instruct the client to eat a low-fat, high-fiber diet and to avoid eating and drinking 2 hours before bedtime, and wearing tight clothes; also, elevate the head of the bed on 6- to 8-inch blocks. 3. Avoid the use of anticholinergics, which delay stomach emptying; also, nonsteroidal anti- inflammatory medications and other medications that contain acetylsalicylic acid need to be avoided. 4. Instruct the client regarding prescribed medications, such as antacids, H2-receptor antagonists, or proton pump inhibitors. 5. Instruct the client regarding the administration of prokinetic medications, if prescribed, which accelerate gastric emptying. 6. If medical management is unsuccessful, surgery may be required; this involves a fundoplication (wrapping a portion of the gastric fundus around the sphincter area of the esophagus); surgery may be performed by laparoscopy.

While a client is being prepared for discharge, the nasogastric (NG) feeding tube becomes clogged. To remedy this problem and teach the client's family how to deal with it at home, what should the nurse do? 1. Irrigate the tube with cola. 2. Advance the tube into the intestine. 3. Apply intermittent suction to the tube. 4. Withdraw the obstruction with a 30-ml syringe.

1. Irrigate the tube with cola. The nurse should irrigate the tube with cola because its effervescence and acidity are suited to the purpose, it's inexpensive, and it's readily available in most homes. Advancing the NG tube is inappropriate because the tube is designed to stay in the stomach and isn't long enough to reach the intestines. Applying intermittent suction or using a syringe for aspiration is unlikely to dislodge the material clogging the tube but may create excess pressure. Intermittent suction may even collapse the tube.

A client comes to the emergency department complaining of sudden onset of sharp, severe pain in the lumbar region, which radiates around the side and toward the bladder. The client also reports nausea and vomiting and appears pale, diaphoretic, and anxious. The physician tentatively diagnoses renal calculi and orders flat-plate abdominal X-rays. Renal calculi can form anywhere in the urinary tract. What is their most common formation site? 1. Kidney 2. Ureter 3. Bladder 4. Urethra

1. Kidney The most common site of renal calculi formation is the kidney. Calculi may travel down the urinary tract with or without causing damage and may lodge anywhere along the tract or may stay within the kidney. The ureter, bladder, and urethra are less common sites of renal calculi formation.

Assessment findings for diverticulitis

1. Left lower quadrant abdominal pain that increases with coughing, straining, or lifting 2. Elevated temperature 3. Nausea and vomiting 4. Flatulence 5. Cramp-like pain 6. Abdominal distention and tenderness 7. Palpable, tender rectal mass may be present. 8. Blood in the stools

The nurse is reviewing the laboratory results in a client with cirrhosis and notes that the ammonia level is elevated. Which diet does the nurse anticipate to be prescribed for this client? 1. Low-protein diet 2. High-protein diet 3. Moderate-fat diet 4. High-carbohydrate diet

1. Low-protein diet Cirrhosis is a chronic, progressive disease of the liver characterized by diffuse degeneration and destruction of hepatocytes. Most of the ammonia in the body is found in the gastrointestinal tract. Protein provided by the diet is transported to the liver by the portal vein. The liver breaks down protein, which results in the formation of ammonia. If the client has hepatic encephalopathy, a low-protein diet would be prescribed.

Interventions for cholecystitis

1. Maintain NPO status during nausea and vomiting episodes. 2. Maintain nasogastric decompression as prescribed for severe vomiting. 3. Administer antiemetics as prescribed for nausea and vomiting. 4. Administer analgesics as prescribed to relieve pain and reduce spasm. 5. Administer antispasmodics (anticholinergics ) as prescribed to relax smooth muscle. 6. Instruct the client with chronic cholecystitis to eat small, low-fat meals. 7. Instruct the client to avoid gas-forming foods. 8. Prepare the client for nonsurgical and surgical procedures as prescribed.

Functions of the kidney

1. Maintain acid-base balance 2. Excrete end products of body metabolism 3. Control fluid and electrolyte balance 4. Excrete bacterial toxins, water-soluble drugs, and drug metabolites 5. Secrete renin to regulate the blood pressure and erythropoietin to stimulate the bone marrow to produce red blood cells. 6. Synthesize vitamin D for calcium absorption and regulation of the parathyroid hormones.

Post Op interventions after cholecystecomy

1. Monitor for respiratory complications caused by pain at the incisional site. 2. Encourage coughing and deep breathing. 3. Encourage early ambulation. 4. Instruct the client about splinting the abdomen to prevent discomfort during coughing. 5. Administer antiemetics as prescribed for nausea and vomiting. 6. Administer analgesics as prescribed for pain relief. 7. Maintain NPO status and nasogastric tube suction as prescribed. 8. Advance diet from clear liquids to solids when prescribed and as tolerated by the client. 9. Maintain and monitor drainage from the T-tube, if present

Interventions for pyelonephritis

1. Monitor vital signs, especially for elevated temperature. 2. Encourage fluid intake up to 3000 mL /day to reduce fever and prevent dehydration. 3. Monitor intake and output (ensure that output is a minimum of 1500 mL/ 24 hr). 4. Monitor weight. 5. Encourage adequate rest. 6. Instruct the client in a high-calorie, low-protein diet. 7. Provide warm, moist compresses to the flank area to help relieve pain. 8. Encourage the client to take warm baths for pain relief. 9. Administer analgesics, antipyretics, antibiotics, urinary antiseptics, and antiemetics as prescribed.

Interventions for glomerulonephritis

1. Monitor vital signs, especially for hypertension and temperature elevations. 2. Monitor intake and output and urine characteristics closely. 3. Monitor daily weight. 4. Monitor for edema. 5. Monitor for fluid overload, ascites, pulmonary edema, and CHF. 6. Restrict fluid intake as prescribed. 7. Provide a high-calorie, low-protein, low-sodium, and low-potassium diet to prevent worsening azotemia, fluid retention, and hyperkalemia. 8. Provide bed rest and limit activity. 9. Administer diuretics, antihypertensives, and antibiotics as prescribed. 10. Monitor for signs of renal failure, cardiac failure, and hypertensive encephalopathy. 11. Instruct the client to report signs of bloody urine, headache, or edema. 12. Instruct the client to obtain treatment for infections, especially sore throats, skin lesions, and upper respiratory infections.

Interventions for nephrolithiasis

1. Monitor vital signs, especially the temperature, for signs of infection. 2. Monitor intake and output. 3. Assess for fever, chills, and infection. 4. Monitor for nausea, vomiting, and diarrhea. 5. Encourage fluid intake up to 3000 mL/ day, unless contraindicated, to facilitate the passage of the stone and prevent infection. 6. Administer fluids intravenously as prescribed if unable to take fluids orally or in adequate amounts to increase the flow of urine and facilitate passage of the stone. 7. Provide warm baths and heat to the flank area (massage therapy should be avoided). 8. Administer analgesics at regularly scheduled intervals as prescribed to relieve pain. 9. Assess the client's response to pain medication. 10. Assist the client in performing relaxation techniques to assist in relieving pain. 11. Encourage client ambulation, if stable, to promote the passage of the stone. 12. Turn and reposition the immobilized client to promote passage of the stone. 13. Instruct the client in the diet restrictions specific to the stone composition if prescribed. 14. Prepare the client for surgical procedures if prescribed

PostOp Interventions for urinary diversion

1. Monitor vital signs. 2. Assess incision site. 3. Assess stoma (should be red and moist) every hour for the first 24 hours. 4. Monitor for edema in the stoma, which may be present in the immediate postoperative period. 5. Notify the physician if the stoma appears dark and dusky (indicates necrosis). 6. Monitor for prolapse or retraction of the stoma. 7. Assess bowel function; monitor for expected return of peristalsis in 3 to 4 days. 8. Maintain NPO status as prescribed until bowel sounds return. 9. Monitor continuous urine flow (30 to 60 mL/ hr). 10. Notify the physician if the urine output is less than 30 mL/ hr or if no urine output occurs for more than 15 minutes. 11. Ureteral stents or catheters, if present, may be in place for 2 to 3 weeks or until healing occurs; maintain stability with catheters to prevent dislodgment. 12. Monitor for hematuria. 13. Monitor for signs of peritonitis. 14. Monitor for bladder distention following a partial cystectomy. 15. Monitor for shock, hemorrhage, thrombophlebitis, and lower extremity lymphedema after a radical cystectomy. 16. Monitor the urinary drainage pouch for leaks, and check skin integrity. 17. Monitor the pH of the urine (do not place the dipstick in the stoma) because strongly alkaline urine can cause skin irritation and facilitate crystal formation. 18. Instruct the client regarding the potential for urinary tract infection or the development of calculi. 19. Instruct the client to assess the skin for irritation, monitor the urinary drainage pouch, and report any leakage. 20. Encourage the client to express feelings about changes in body image, embarrassment, and sexual dysfunction.

Interventions of esophageal varices

1. Monitor vital signs. 2. Elevate the head of the bed. 3. Monitor for orthostatic hypotension. 4. Monitor lung sounds and for the presence of respiratory distress. 5. Administer oxygen as prescribed to prevent tissue hypoxia. 6. Monitor level of consciousness. 7. Maintain NPO status. 8. Administer fluids intravenously as prescribed to restore fluid volume and electrolyte imbalances; monitor intake and output. 9. Monitor hemoglobin and hematocrit values and coagulation factors. 10. Administer blood transfusions or clotting factors as prescribed. 11. Assist in inserting a nasogastric tube or a balloon tamponade as prescribed; balloon tamponade is not used frequently because it is very uncomfortable for the client and its use is associated with complications. 12. Prepare to assist with administering medications to induce vasoconstriction and reduce bleeding. 13. Instruct the client to avoid activities that will initiate vasovagal responses. 14. Prepare the client for endoscopic procedures or surgical procedures as prescribed.

PostOp prostate surgery

1. Monitor vital signs. 2. Monitor urinary output and urine for hemorrhage or clots. 3. Increase fluids to 2400 to 3000 mL/ day, unless contraindicated. 4. Monitor for arterial bleeding as evidenced by bright red urine with numerous clots; if it occurs, increase CBI and notify the physician immediately. 5. Monitor for venous bleeding as evidenced by burgundy-colored urine output; if it occurs, inform the physician, who may apply traction on the catheter. 6. Monitor hemoglobin and hematocrit levels. 7. Expect red to light pink urine for 24 hours, turning to amber in 3 days. 8. Ambulate the client as early as possible and as soon as urine begins to clear in color. 9. Inform the client that a continuous feeling of an urge to void is normal. 10. Instruct the client to avoid attempts to void around the catheter because this will cause bladder spasms. 11. Administer antibiotics, analgesics, stool softeners, and antispasmodics as prescribed. 12. Monitor the three-way Foley catheter, which usually has a 30- to 45-mL retention balloon. 13. Maintain CBI with sterile bladder irrigation solution as prescribed to keep the catheter free of obstruction and maintain the urine pink in color.

S&S of Cholecystitis

1. Nausea and vomiting 2. Indigestion 3. Belching 4. Flatulence 5. Epigastric pain that radiates to the scapula 2 to 4 hours after eating fatty foods and may persist for 4 to 6 hours 6. Pain localized in right upper quadrant 7. Guarding, rigidity, and rebound tenderness 8. Mass palpated in the right upper quadrant 9. Murphy's sign ( cannot take a deep breath when the examiner's fingers are passed below the hepatic margin because of pain) 10. Elevated temperature 11. Tachycardia 12. Signs of dehydration

A client undergoes total gastrectomy. Several hours after surgery, the nurse notes that the client's nasogastric (NG) tube has stopped draining. How should the nurse respond? 1. Notify the physician 2. Reposition the tube 3. Irrigate the tube 4. Increase the suction level

1. Notify the physician An NG tube that fails to drain during the postoperative period should be reported to the physician immediately. It may be clogged, which could increase pressure on the suture site because fluid isn't draining adequately. Repositioning or irrigating an NG tube in a client who has undergone gastric surgery can disrupt the anastomosis. Increasing the level of suction may cause trauma to GI mucosa or the suture line.

The nurse is monitoring a client admitted to the hospital with a diagnosis of appendicitis who is scheduled for surgery in 2 hours. The client begins to complain of increased abdominal pain and begins to vomit. On assessment, the nurse notes that the abdomen is distended and bowel sounds are diminished. Which is the appropriate nursing intervention? 1. Notify the physician. 2. Administer the prescribed pain medication. 3. Call and ask the operating room team to perform the surgery as soon as possible. 4. Reposition the client and apply a heating pad on warm setting to the client's abdomen.

1. Notify the physician. Based on the signs and symptoms presented in the question, the nurse should suspect peritonitis and notify the physician. Administering pain medication is not an appropriate intervention. Heat should never be applied to the abdomen of a client with suspected appendicitis because of the risk of rupture. Scheduling surgical time is not within the scope of nursing practice, although the physician probably would perform the surgery earlier than the prescheduled time.

The female client is admitted to the emergency department following a fall from a horse and the physician prescribes insertion of a Foley catheter. While preparing for the procedure, the nurse notes blood at the urinary meatus. The nurse should: 1. Notify the physician. 2. Use a small-sized of catheter. 3. Administer pain medication before inserting the catheter. 4. Use extra povidone-iodine solution in cleansing the meatus.

1. Notify the physician. The presence of blood at the urinary meatus may indicate urethral trauma or disruption . The nurse notifies the physician, knowing that the client should not be catheterized until the cause of the bleeding is determined by diagnostic testing. Therefore options 2, 3, and 4 are incorrect.

A client with a history of severe abdominal cramping arrives at the emergency department vomiting a moderate amount of blood. The nurse's primary concern should be: 1. Observing for signs and symptoms of shock. 2. Immediately paging the physician on call. 3. Filling out the appropriate assessment tool. 4. Immediately administering CPR.

1. Observing for signs and symptoms of shock. The client has experienced some gastric bleeding; the client should be observed closely for symptoms of shock. Paging the physician is secondary to the close monitoring of the client's vital signs for symptoms of shock. Completing the appropriate assessment tool may be done later, when the client is under less stress. CPR is not indicated, the client is not experiencing cardiac or respiratory failure.

A client with a peptic ulcer reports epigastric pain that frequently awakens her during the night. The nurse should instruct the client to do which activities? Select all that apply. 1. Obtain adequate rest to reduce stimulation. 2. Eat small, frequent meals throughout the day. 3. Take all medications on time as ordered. 4. Sit up for one hour when awakened at night. 5. Stay away from crowded areas.

1. Obtain adequate rest to reduce stimulation. 2. Eat small, frequent meals throughout the day. 3. Take all medications on time as ordered. 4. Sit up for one hour when awakened at night. The nurse should encourage the client to reduce stimulation that may enhance gastric secretion. The nurse can also advise the client to utilize health practices that will prevent recurrences of ulcer pain, such as avoiding fatigue and elimination of smoking. Eating small, frequent meals helps to prevent gastric distention if not actively bleeding and decreases distension and release of gastrin. Medications should be administered promptly to maintain optimum levels. After awakening during the night, the client should eat a small snack and return to bed, keeping the head of the bed elevated for an hour after eating. It is not necessary to stay away from crowded areas.

A group of nursing assistants hired for the medical-surgical floors are attending hospital orientation. Which topic should the educator cover when teaching the group about caring for clients with diabetes? 1. Obtaining, reporting, and documenting fingerstick glucose levels 2. Treating a hypoglycemic reaction 3. Teaching the client dietary changes necessary with diabetes mellitus 4. Assessing the client experiencing a hypoglycemic reaction

1. Obtaining, reporting, and documenting fingerstick glucose levels The educator should teach the nursing assistants how to obtain and document a fingerstick glucose level. She should also teach them normal and abnormal results and the importance of reporting them to the registered nurse caring for the client. Options 2, 3, and 4 are outside the scope of practice for a nursing assistant. They are the responsibility of the registered nurse.

Assessment findings for polycystic kidney disease

1. Often asymptomatic until the ages of 30 to 40 years 2. Flank, lumbar , or abdominal pain that worsens with activity and is relieved when lying 3. Fever and chills 4. Recurrent urinary tract infections 5. Hematuria, proteinuria, pyuria 6. Calculi 7. Hypertension 8. Palpable abdominal masses and enlarged kidneys

A nurse administers glucagon to her diabetic client, then monitors the client for adverse drug reactions and interactions. Which type of drug interacts adversely with glucagon? 1. Oral anticoagulants 2. Anabolic steroids 3. Beta-adrenergic blockers 4. Thiazide diuretics

1. Oral anticoagulants As a normal body protein, glucagon only interacts adversely with oral anticoagulants, increasing the anticoagulant effects. It doesn't interact adversely with anabolic steroids, beta-adrenergic blockers, or thiazide diuretics.

A client had a positive fecal occult blood test during a health screening. The nursing assessment would include: Select all the apply. 1. Orthostatic blood pressure and pulse. 2. STAT request for hemoglobin and hematocrit. 3. Description of stool volume, color, and consistency. 4. Diet history of raw meat consumption. 5. Regular use of aspirin or NSAIDs. 6. Vitamin C, 250-mg tablets, before the test.

1. Orthostatic blood pressure and pulse. 3. Description of stool volume, color, and consistency. 4. Diet history of raw meat consumption. 5. Regular use of aspirin or NSAIDs. 6. Vitamin C, 250-mg tablets, before the test. Inadequate volume from a significant blood loss will result in a drop of the systolic blood pressure greater than 25 mm Hg and the diastolic value greater than 20 mm Hg, and an increase in pulse rate of 30 bpm when the client goes from flat to stitting/standing. Ordering lab tests is not part of nursing assessment. Nursing assessment would include a description of the characteristics of the stool. Eating red meat before the test may result in a false positive finding. It should be known if the client has a history of frequent and high doses of drugs known to cause GI irritation and bleeding. Taking Vitamin C within 48 hours of a fecal occult blood test will cause a false positive reading.

Assessment findings for appendicitis

1. Pain in the periumbilical area that descends to the right lower quadrant 2. Abdominal pain that is most intense at McBurney's point 3. Rebound tenderness and abdominal rigidity 4. Low-grade fever 5. Elevated white blood cell count 6. Anorexia, nausea, and vomiting 7. Client in side-lying position, with abdominal guarding and legs flexed 8. Constipation or diarrhea

Populations at Risk for Hepatitis C

1. Parenteral drug users 2. Clients receiving frequent transfusions 3. Health care personnel

Two weeks after being diagnosed with a streptococcal infection, a client develops fatigue, a low-grade fever, and shortness of breath. The nurse auscultates bilateral crackles and observes neck vein distention. Urinalysis reveals red and white blood cells and protein. After the physician diagnoses poststreptococcal glomerulonephritis, the client is admitted to the medical-surgical unit. Which immediate action should the nurse take? 1. Place the client on bed rest. 2. Provide a high-protein, fluid-restricted diet. 3. Prepare to assist with insertion of a Tenckhoff catheter for hemodialysis. 4. Place the client on a sheepskin, and monitor for increasing edema.

1. Place the client on bed rest. The nurse immediately must enforce bed rest for a client with glomerulonephritis to ensure a complete recovery and help prevent complications. Depending on disease severity, the client may require fluid, sodium, potassium, and protein restrictions. Because of the risk of altered urinary elimination related to oliguria, this client may require hemodialysis or plasmapheresis for several weeks until renal function improves; however, a Tenckhoff catheter is used in peritoneal dialysis, not hemodialysis. Although comfort measures such as placing the client on a sheepskin are important, they don't take precedence.

A nurse is monitoring a client newly diagnosed with diabetes mellitus for signs of complications. Which of the following, if exhibited in the client, would indicate hyperglycemia and warrant physician notification? 1. Polyuria 2. Diaphoresis 3. Hypertension 4. Increased pulse rate

1. Polyuria Rationale: Classic symptoms of hyperglycemia include polydipsia, polyuria, and polyphagia. Options 2, 3, and 4 are not signs of hyperglycemia.

The nurse is caring for a client who has just had an upper GI endoscopy. The client's vital signs must be taken every 30 minutes for 2 hours after the procedure. The nurse assigns an unlicensed nursing personnel (UAP) to take the vital signs. One hour later, the UAP reports the client, who was previously afebrile, has developed a temperature of 101.8 ° F (38.8 ° C). What should the nurse do in response to this reported assessment data? 1. Promptly assess the client for potential perforation. 2. Tell the assistant to change thermometers and retake the temperature. 3. Plan to give the client acetaminophen (Tylenol) to lower the temperature. 4. Ask the assistant to bathe the client with tepid water.

1. Promptly assess the client for potential perforation. A sudden spike in temperature following an endoscopic procedure may indicate perforation of the GI tract. The nurse should promptly conduct a further assessment of the client, looking for further indicators of perforation, such as a sudden onset of acute upper abdominal pain; a rigid, boardlike abdomen; and developing signs of shock. Telling the assistant to change thermometers is not an appropriate action and only further delays the appropriate action of assessing the client. The nurse would not administer acetaminophen without further assessment of the client or without a physician's order; a suspected perforation would require that the client be placed on nothing-by-mouth status. Asking the assistant to bathe the client before any assessment by the nurse is inappropriate.

The nurse is assessing for stoma prolapse in a client with a colostomy. What should the nurse observe if stoma prolapse occurs? 1. Protruding stoma 2. Sunken and hidden stoma 3. Narrowed and flattened stoma 4. Dark- and bluish-colored stoma

1. Protruding stoma A prolapsed stoma is one in which the bowel protrudes through the stoma. A stoma retraction is characterized by sinking of the stoma. Ischemia of the stoma would be associated with a dusky or bluish color. A stoma with a narrowed opening at the level of the skin or fascia is said to be stenosed.

Interventions for diverticulitis

1. Provide bedrest during the acute phase. 2. Maintain NPO status or provide clear liquids during the acute phase as prescribed. 3. Introduce a fiber-containing diet gradually, when the inflammation has resolved. 4. Administer antibiotics, analgesics, and anticholinergics to reduce bowel spasms as prescribed. 5. Instruct the client to refrain from lifting, straining, coughing, or bending to avoid increased intra-abdominal pressure. 6. Monitor for perforation, hemorrhage, fistulas, and abscesses. 7. Instruct the client to increase fluid intake to 2500 to 3000 mL daily, unless contraindicated. 8. Instruct the client to eat soft high-fiber foods, such as whole grains; the client should avoid high-fiber foods when inflammation occurs because these foods will irritate the mucosa further. 9. Instruct the client to avoid gas-forming foods or foods containing indigestible roughage, seeds, or nuts because these food substances become trapped in diverticula and cause inflammation. 10. Instruct the client to consume a small amount of bran daily and to take bulk-forming laxatives as prescribed to increase stool mass.

A client diagnosed with acute pancreatitis is being transferred to another facility. The nurse caring for the client completes the transfer summary, which includes information about the client's drinking history and other assessment findings. Which assessment findings confirm his diagnosis? 1. Recent weight loss and temperature elevation 2. Presence of blood in the client's stool and recent hypertension 3. Presence of easy bruising and bradycardia 4. Adventitious breath sounds and hypertension

1. Recent weight loss and temperature elevation Assessment findings associated with pancreatitis include recent weight loss and temperature elevation. Inflammation of the pancreas causes a response that elevates temperature and leads to abdominal pain that typically occurs with eating. Nausea and vomiting may occur as a result of pancreatic tissue damage that's caused by the activation of pancreatic enzymes. The client may experience weight loss because of the lost desire to eat. The assessment findings in option 2 aren't associated with pancreatitis; fatty diarrhea and hypotension are usually present. Option 3 findings aren't found with pancreatitis; the client typically experiences tachycardia, not bradycardia. Option 4 findings aren't associated with pancreatitis.

A client is admitted to the health care facility with abdominal pain, a low-grade fever, abdominal distention, and weight loss. The physician diagnoses acute pancreatitis. What is the primary goal of nursing care for this client? 1. Relieving abdominal pain 2. Preventing fluid volume overload 3. Maintaining adequate nutritional status 4. Teaching about the disease and its treatment

1. Relieving abdominal pain The predominant clinical feature of acute pancreatitis is abdominal pain, which usually reaches peak intensity several hours after onset of the illness. Therefore, relieving abdominal pain is the nurse's primary goal. Because acute pancreatitis causes nausea and vomiting, the nurse should try to prevent fluid volume deficit, not overload. The nurse can't help the client achieve adequate nutrition or understand the disease and its treatment until the client is comfortable and no longer in pain.

Assessment findings for nephrolithiasis

1. Renal colic, which originates in the lumbar region and radiates around the side and down to the testicles in men and to the bladder in women 2. Ureteral colic, which radiates toward the genitalia and thighs 3. Sharp, severe pain of sudden onset 4. Dull, aching pain in the kidney 5. Nausea and vomiting, pallor, and diaphoresis during acute pain 6. Urinary frequency, with alternating retention 7. Signs of a urinary tract infection 8. Low-grade fever 9. High numbers of red blood cells, white blood cells, and bacteria noted in the urinalysis report 10. Gross hematuria

The nurse is developing a discharge plan for a client with a new ileostomy. The client is 32 years old, mentally competent, lives with her husband of 3 years, and works outside her home. Which nursing diagnoses are important to include in the discharge plan? Select all that apply. 1. Risk for fluid volume deficit. 2. Risk for fluid volume excess. 3. Risk for constipation. 4. Risk for impaired skin integrity. 5. Altered nutrition: more than body requirements. 6. Altered sexuality patterns.

1. Risk for fluid volume deficit. 4. Risk for impaired skin integrity. 6. Altered sexuality patterns. An ileostomy produced liquid stool. The client's daily fluid intake should be 2 to 3 liters to prevent dehydration. Fecal drainage or a poorly fitting ostomy appliance can irritate and erode the stoma and surrounding the skin. A client with an ileostomy is more likely at risk for nutrition less than body requirements. The client with an ileostomy is often concerned about loss of sexual appeal or leakage of fecal material during sexual activity.

The nurse would position a client with ruptured appendix in: 1. Semi-Fowler's. 2. Trendelenburg. 3. Left Sims'. 4. Dorsal recumbent.

1. Semi-Fowler's. The client is placed in a semi-Fowler's position to promote the flow of drainage to the pelvic region, where a localized abscess can be frained or resolved by the body's normal defenses. The elevated position also keeps the infection from spreading upward in the peritoneal cavity.

After being sick for 3 days, a client with a history of diabetes mellitus is admitted to the hospital with diabetic ketoacidosis (DKA). The nurse should evaluate which diagnostic test results to prevent arrhythmias. 1. Serum potassium level 2. Serum calcium level 3. Serum sodium level 4. Serum chloride level

1. Serum potassium level During periods of acidosis, potassium leaves the cell causing hyperkalemia. As blood glucose levels normalize with treatment, potassium reenters the cell causing hypokalemia if levels aren't monitored closely. Hypokalemia places the client at risk for cardiac arrhythmias such as ventricular tachycardia. DKA has a lesser affect on serum calcium, sodium, and chloride levels. Changes in these levels don't typically cause cardiac arrhythmias.

Why low flow O2 in COPD patients?

COPD patient have CO2 retention and body becomes used to increased level of CO2 CO2 is primary stimulus for breathing Low O2 level becomes stimulus for breathing in these patients If on too much O2 breathing mechanism may not be stimulated

Treatment and Prevention of Hepatitis A

1. Strict handwashing 2. Stool precautions 3. Treatment of municipal water supplies 4. Serological screening of food handlers 5. Hepatitis A vaccine (Havrix VAQTA) 6. Immune globulin: For individuals exposed to HAV who have never received the hepatitis A vaccine; administer immune globulin during the period of incubation and within 2 weeks of exposure. 7. Immune globulin and hepatitis A vaccine are recommended for household members and sexual contacts of individuals with hepatitis A. 8. Preexposure prophylaxis with immunoglobulin is recommended to individuals traveling to countries with poor or uncertain sanitation conditions.

Prevention and Treatment for Hepatitis B

1. Strict handwashing 2. Screening blood donors 3. Testing of all pregnant women 4. Needle precautions 5. Avoiding intimate sexual contact if test for hepatitis B surface antigen (HBsAg) is positive. 6. Hepatitis B vaccine : Engerix-B (adult), Recombivax HB (pediatric); there is also an adult vaccine that protects against hepatitis A and B known as Twinrix. 7. Hepatitis B immune globulin is for individuals exposed to HBV through sexual contact or through the percutaneous or transmucosal routes who have never had hepatitis B and have never received hepatitis B vaccine.

A 46-year-old client with status asthmaticus requires endotracheal intubation and mechanical ventilation. Twenty-four hours after intubation, the client is started on the insulin infusion protocol. The nurse must monitor the client's blood glucose levels hourly and watch for which early signs and symptoms associated with hypoglycemia? 1. Sweating, tremors, and tachycardia 2. Dry skin, bradycardia, and somnolence 3. Bradycardia, thirst, and anxiety 4. Polyuria, polydipsia, and polyphagia

1. Sweating, tremors, and tachycardia Sweating, tremors, and tachycardia are early signs of hypoglycemia. Dry skin, bradycardia, and somnolence are signs and symptoms associated with hypothyroidism. In option 3, thirst and anxiety are signs of hypoglycemia, but not bradycardia. Polyuria, polydipsia, and polyphagia are signs and symptoms of diabetes mellitus.

The nurse suspects wound dehiscence, and lifts the edges of the client's dressings. The nurse notes that the wound edges are entirely separated. What is the next nursing action? 1. Tell the client to remain quiet and not to cough. 2. Offer the client a warm drink to promote relaxation. 3. Position the cient in a chair with the feet elevated. 4. Apply a Scultetus bandage.

1. Tell the client to remain quiet and not to cough. The client should remain quiet in a low Fowler's or horizontal position. They should be cautioned not to cough so as note to extrutde any intestines by increasing intra-abdominal pressures. The physician should be notified next. Remain with the client, reassuring them, monitoring vital signs, and having others bring equipment such as IV setup, nasogastric tube, and suction equipment. The client should be kept NPO. The dressing should be left in place to prevent evisceration.

The graduate nurse and her preceptor are establishing priorities for their morning assessments. Which client should they assess first? 1. The newly admitted client with acute abdominal pain 2. The client who underwent surgery three days ago and who now requires a dressing change 3. The client receiving continuous tube feedings who needs the tube-feeding residual checked 4. The sleeping client who received pain medication 1 hour ago

1. The newly admitted client with acute abdominal pain The graduate nurse and her preceptor should assess the new admission with acute abdominal pain first because he just arrived on the floor and might be unstable. Next, they should change the abdominal dressing for the postoperative client or measure feeding tube residual in the client with continuous tube feedings. These tasks are of equal importance. They should assess the sleeping client who received pain medication 1 hour ago last because he just received relief from his pain and is able to sleep.

The graduate nurse and her preceptor are establishing priorities for their morning assessments. Which client should they assess first? 1. The newly admitted client with acute abdominal pain 2. The client who underwent surgery three days ago and who now requires a dressing change 3. The client receiving continuous tube feedings who needs the tube-feeding residual checked 4. The sleeping client who received pain medication 1 hour ago

1. The newly admitted client with acute abdominal pain The graduate nurse and her preceptor should assess the new admission with acute abdominal pain first because he just arrived on the floor and might be unstable. Next, they should change the abdominal dressing for the postoperative client or measure feeding tube residual in the client with continuous tube feedings. These tasks are of equal importance. They should assess the sleeping client who received pain medication 1 hour ago last because he just received relief from his pain and is able to sleep.

The client had a new colostomy created 2 days earlier and is beginning to pass malodorous flatus from the stoma. What is the correct interpretation by the nurse? 1. This is a normal, expected event. 2. The client is experiencing early signs of ischemic bowel. 3. The client should not have the nasogastric tube removed. 4. This indicates inadequate preoperative bowel preparation.

1. This is a normal, expected event. As peristalsis returns following creation of a colostomy, the client begins to pass malodorous flatus. This indicates returning bowel function and is an expected event. Within 72 hours of surgery, the client should begin passing stool via the colostomy. Options 2, 3, and 4 are incorrect.

Why are antacids administered regularly, rather than as needed, to treat peptic ulcer disease? 1. To keep gastric pH at 3.0 to 3.5 2. To promote client compliance 3. To maintain a regular bowel pattern 4. To increase pepsin activity

1. To keep gastric pH at 3.0 to 3.5 To maintain a gastric pH of 3.0 to 3.5 throughout each 24-hour period, regular (not as needed) doses of an antacid are needed to treat peptic ulcer disease. Frequent administration of an antacid tends to decrease client compliance. Antacids don't regulate bowel patterns, and they decrease pepsin activity.

A client with benign prostatic hyperplasia doesn't respond to medical treatment and is admitted to the facility for prostate gland removal. Before providing preoperative and postoperative instructions to the client, the nurse asks the surgeon which prostatectomy procedure will be done. What is the most widely used procedure for prostate gland removal? 1. Transurethral resection of the prostate (TURP) 2. Suprapubic prostatectomy 3. Retropubic prostatectomy 4. Transurethral laser incision of the prostate

1. Transurethral resection of the prostate (TURP) TURP is the most widely used procedure for prostate gland removal. Because it requires no incision, TURP is especially suitable for men with relatively minor prostatic enlargements and for those who are poor surgical risks. Suprapubic prostatectomy, retropubic prostatectomy, and transurethral laser incision of the prostate are less common procedures; they all require an incision.

A nurse is monitoring a client who was diagnosed with type 1 diabetes mellitus and is being treated with NPH and regular insulin. Which client complaint( s) would alert the nurse to the presence of a possible hypoglycemic reaction? Select all that apply. 1. Tremors 2. Anorexia 3. Irritability 4. Nervousness 5. Hot, dry skin 6. Muscle cramps

1. Tremors 3. Irritability 4. Nervousness Rationale: Decreased blood glucose levels produce autonomic nervous system symptoms, which are manifested classically as nervousness, irritability, and tremors. Option 5 is more likely to occur with hyperglycemia. Options 2 and 6 are unrelated to the signs of hypoglycemia.

The nurse is caring for a client with chronic renal failure. The laboratory results indicate hypocalcemia and hyperphosphatemia. When assessing the client, the nurse should be alert for which of the following? 1. Trousseau's sign 2. Cardiac arrhythmias 3. Constipation 4. Decreased clotting time 5. Drowsiness and lethargy 6. Fractures

1. Trousseau's sign 2. Cardiac arrhythmias 6. Fractures Hypocalcemia is a calcium deficit that causes nerve fiber irritability and repetitive muscle spasms. Signs and symptoms of hypocalcemia include Trousseau's sign, cardiac arrhythmias, diarrhea, increased clotting times, anxiety, and irritability. The calcium-phosphorus imbalance leads to brittle bones and pathologic fractures.

A client has a serum calcium level of 7.2 mg/dl. During the physical examination, the nurse expects to assess: 1. Trousseau's sign. 2. Homans' sign. 3. Hegar's sign. 4. Goodell's sign.

1. Trousseau's sign. This client's serum calcium level indicates hypocalcemia, an electrolyte imbalance that causes Trousseau's sign (carpopedal spasm induced by inflating the blood pressure cuff above systolic pressure). Homans' sign (pain on dorsiflexion of the foot) indicates deep vein thrombosis. Hegar's sign (softening of the uterine isthmus) and Goodell's sign (cervical softening) are probable signs of pregnancy.

Which nursing action is most appropriate for a client hospitalized with acute pancreatitis? 1. Withholding all oral intake, as ordered, to decrease pancreatic secretions 2. Administering morphine, as prescribed, to relieve severe pain 3. Limiting I.V. fluids, as ordered, to decrease cardiac workload 4. Keeping the client supine to increase comfort

1. Withholding all oral intake, as ordered, to decrease pancreatic secretions The nurse should withhold all oral intake to suppress pancreatic secretions, which may worsen pancreatitis. Typically, this client requires a nasogastric tube to decompress the stomach and GI tract. Although pancreatitis may cause considerable pain, it's treated with I.M. meperidine (Demerol), not morphine, which may worsen pain by inducing spasms of the pancreatic and biliary ducts. Pancreatitis places the client at risk for fluid volume deficit from fluid loss caused by increased capillary permeability. Therefore, this client needs fluid resuscitation, not fluid restriction. A client with pancreatitis is most comfortable lying on the side with knees flexed.

When caring for a client with acute renal failure (ARF), the nurse expects to adjust the dosage or dosing schedule of certain drugs. Which of the following drugs would not require such adjustment? 1. acetaminophen (Tylenol) 2. gentamicin sulfate (Garamycin) 3. cyclosporine (Sandimmune) 4. ticarcillin disodium (Ticar)

1. acetaminophen (Tylenol) Because acetaminophen is metabolized in the liver, its dosage and dosing schedule need not be adjusted for a client with ARF. In contrast, the dosages and schedules for gentamicin and ticarcillin, which are metabolized and excreted by the kidney, should be adjusted. Because cyclosporine may cause nephrotoxicity, the nurse must monitor both the dosage and blood drug level in a client receiving this drug.

A client had a nephrectomy 2 days ago and is now complaining of abdominal pressure and nausea. The first nursing action should be to: 1. auscultate bowel sounds. 2. palpate the abdomen. 3. change the client's position. 4. insert a rectal tube.

1. auscultate bowel sounds. If abdominal distention is accompanied by nausea, the nurse must first auscultate bowel sounds. If bowel sounds are absent, the nurse should suspect gastric or small intestine dilation and these findings must be reported to the physician. Palpation should be avoided postoperatively with abdominal distention. If peristalsis is absent, changing positions and inserting a rectal tube won't relieve the client's discomfort.

The nurse is caring for a client who's hypoglycemic. This client will have a blood glucose level: 1. below 70 mg/dl. 2. between 70 and 120 mg/dl. 3. between 120 and 180 mg/dl. 4. over 180 mg/dl.

1. below 70 mg/dl. A blood glucose level under 70 mg/dl is considered hypoglycemic. A normal blood glucose level is between 70 and 120 mg/dl. Over 120 mg/dl indicates hyperglycemia.

The nurse must provide total parenteral nutrition (TPN) to a client through a triple-lumen central line. To prevent complications of TPN, the nurse should: 1. cover the catheter insertion site with an occlusive dressing. 2. use clean technique when changing the dressing. 3. insert an indwelling urinary catheter. 4. keep the client on complete bed rest.

1. cover the catheter insertion site with an occlusive dressing. TPN increases the client's risk of infection because the catheter insertion site creates a port of entry for bacteria. To reduce the risk of infection, the nurse should cover the insertion site with an occlusive dressing, which is airtight. Because the insertion site is an open wound, the nurse should use sterile technique when changing the dressing. TPN doesn't necessitate placement of an indwelling urinary catheter or bed rest.

A client develops acute renal failure (ARF) after receiving I.V. therapy with a nephrotoxic antibiotic. Because the client's 24-hour urine output totals 240 ml, the nurse suspects that the client is at risk for: 1. cardiac arrhythmia. 2. paresthesia. 3. dehydration. 4. pruritus.

1. cardiac arrhythmia. As urine output decreases, the serum potassium level rises; if it rises sufficiently, hyperkalemia may occur, possibly triggering a cardiac arrhythmia. Hyperkalemia doesn't cause paresthesia (sensations of numbness and tingling). Dehydration doesn't occur during this oliguric phase of ARF, although typically it does arise during the diuretic phase. In a client with ARF, pruritus results from increased phosphates and isn't associated with hyperkalemia.

A client with acute renal failure is undergoing dialysis for the first time. The nurse monitors the client closely for dialysis equilibrium syndrome, a complication that is most common during the first few dialysis sessions. Typically, dialysis equilibrium syndrome causes: 1. confusion, headache, and seizures. 2. acute bone pain and confusion. 3. weakness, tingling, and cardiac arrhythmias. 4. hypotension, tachycardia, and tachypnea.

1. confusion, headache, and seizures. Dialysis equilibrium syndrome causes confusion, a decreasing level of consciousness, headache, and seizures. These findings, which may last several days, probably result from a relative excess of interstitial or intracellular solutes caused by rapid solute removal from the blood. The resultant organ swelling interferes with normal physiological functions. To prevent this syndrome, many dialysis centers keep first-time sessions short and use a reduced blood flow rate. Acute bone pain and confusion are associated with aluminum intoxication, another potential complication of dialysis. Weakness, tingling, and cardiac arrhythmias suggest hyperkalemia, which is associated with renal failure. Hypotension, tachycardia, and tachypnea signal hemorrhage, another dialysis complication.

A triple-lumen indwelling urinary catheter is inserted for continuous bladder irrigation following a transurethral resection of the prostate. In addition to balloon inflation, the functions of the three lumens include: 1. continuous inflow and outflow of irrigation solution. 2. intermittent inflow and continuous outflow of irrigation solution. 3. continuous inflow and intermittent outflow of irrigation solution. 4. intermittent flow of irrigation solution and prevention of hemorrhage.

1. continuous inflow and outflow of irrigation solution. When preparing for continuous bladder irrigation, a triple-lumen indwelling urinary catheter is inserted. The three lumens provide for balloon inflation and continuous inflow and outflow of irrigation solution.

The nurse must provide total parenteral nutrition (TPN) to a client through a triple-lumen central line. To prevent complications of TPN, the nurse should: 1. cover the catheter insertion site with an occlusive dressing. 2. use clean technique when changing the dressing. 3. insert an indwelling urinary catheter. 4. keep the client on complete bed rest.

1. cover the catheter insertion site with an occlusive dressing. TPN increases the client's risk of infection because the catheter insertion site creates a port of entry for bacteria. To reduce the risk of infection, the nurse should cover the insertion site with an occlusive dressing, which is airtight. Because the insertion site is an open wound, the nurse should use sterile technique when changing the dressing. TPN doesn't necessitate placement of an indwelling urinary catheter or bed rest.

The nurse is caring for a client with a colostomy. The client tells the nurse that he makes small pin holes in the drainage bag to help relieve gas. The nurse should teach him that this action: 1. destroys the odor-proof seal. 2. won't affect the colostomy system. 3. is appropriate for relieving the gas in a colostomy system. 4. destroys the moisture-barrier seal.

1. destroys the odor-proof seal. Any hole, no matter how small, will destroy the odor-proof seal of a drainage bag. Removing the bag or unclamping it is the only appropriate method for relieving gas.

A client is admitted for treatment of glomerulonephritis. On initial assessment, the nurse detects one of the classic signs of acute glomerulonephritis of sudden onset. Such signs include: 1. generalized edema, especially of the face and periorbital area. 2. green-tinged urine. 3. moderate to severe hypotension. 4. polyuria.

1. generalized edema, especially of the face and periorbital area. Generalized edema, especially of the face and periorbital area, is a classic sign of acute glomerulonephritis of sudden onset. Other classic signs and symptoms of this disorder include hematuria (not green-tinged urine), proteinuria, fever, chills, weakness, pallor, anorexia, nausea, and vomiting. The client also may have moderate to severe hypertension (not hypotension), oliguria or anuria (not polyuria), headache, reduced visual acuity, and abdominal or flank pain.

A client with severe inflammatory bowel disease is receiving total parenteral nutrition (TPN). When administering TPN, the nurse must take care to maintain the prescribed flow rate because giving TPN too rapidly may cause: 1. hyperglycemia. 2. air embolism. 3. constipation. 4. dumping syndrome.

1. hyperglycemia. Hyperglycemia may occur if TPN is administered too rapidly, exceeding the client's glucose metabolism rate. With hyperglycemia, the renal threshold for glucose reabsorption is exceeded and osmotic diuresis occurs, leading to dehydration and electrolyte depletion. Although air embolism may occur during TPN administration, this problem results from faulty catheter insertion, not overly rapid administration. TPN may cause diarrhea, not constipation, especially if administered too rapidly. Dumping syndrome results from food moving through the GI tract too quickly; because TPN is given I.V., it can't cause dumping syndrome.

While preparing a client for cholecystectomy, the nurse explains that incentive spirometry will be used after surgery primarily to: 1. increase respiratory effectiveness. 2. eliminate the need for nasogastric intubation. 3. improve nutritional status during recovery. 4. decrease the amount of postoperative analgesia needed.

1. increase respiratory effectiveness. The high abdominal incision used in a cholecystectomy interferes with respirations postoperatively, increasing the risk of atelectasis. Therefore, incentive spirometry is used to promote lung expansion, increase alveolar inflation, and strengthen respiratory muscles. Incentive spirometry has no effect on intubation, nutrition, or analgesia.

A client is recovering from an ileostomy that was performed to treat inflammatory bowel disease. During discharge teaching, the nurse should stress the importance of: 1. increasing fluid intake to prevent dehydration. 2. wearing an appliance pouch only at bedtime. 3. consuming a low-protein, high-fiber diet. 4. taking only enteric-coated medications.

1. increasing fluid intake to prevent dehydration. Because stool forms in the large intestine, an ileostomy typically drains liquid waste. To avoid fluid loss through ileostomy drainage, the nurse should instruct the client to increase fluid intake. The nurse should teach the client to wear a collection appliance at all times because ileostomy drainage is incontinent, to avoid high-fiber foods because they may irritate the intestines, and to avoid enteric-coated medications because the body can't absorb them after an ileostomy.

A client with amebiasis, an intestinal infection, is prescribed metronidazole (Flagyl). When teaching the client about adverse reactions to this drug, the nurse should mention: 1. metallic taste. 2. tinnitus. 3. blurred vision. 4. loss of smell.

1. metallic taste. Metronidazole commonly causes a metallic taste. Other adverse reactions include nausea, anorexia, headache, and dry mouth. The drug isn't associated with tinnitus, blurred vision, or loss of smell.

The nurse is planning to administer a sodium polystyrene sulfonate (Kayexalate) enema to a client with a potassium level of 6.2 mEq/L. Correct administration and the effects of this enema would include having the client: 1. retain the enema for 30 minutes to allow for sodium exchange; afterward, the client should have diarrhea. 2. retain the enema for 30 minutes to allow for glucose exchange; afterward, the client should have diarrhea. 3. retain the enema for 60 minutes to allow for sodium exchange; diarrhea isn't necessary to reduce the potassium level. 4. retain the enema for 60 minutes to allow for glucose exchange; diarrhea isn't necessary to reduce the potassium level.

1. retain the enema for 30 minutes to allow for sodium exchange; afterward, the client should have diarrhea. Kayexalate is a sodium-exchange resin. Thus, the client will gain sodium as potassium is lost in the bowel. For the exchange to occur, Kayexalate must be in contact with the bowel for at least 30 minutes. Sorbitol in the Kayexalate enema causes diarrhea, which increases potassium loss and decreases the potential for Kayexalate retention.

When caring for a client with a history of hypoglycemia, the nurse should avoid administering a drug that may potentiate hypoglycemia. Which drug fits this description? 1. sulfisoxazole (Gantrisin) 2. mexiletine (Mexitil) 3. prednisone (Orasone) 4. lithium carbonate (Lithobid)

1. sulfisoxazole (Gantrisin) Sulfisoxazole and other sulfonamides are chemically related to oral antidiabetic agents and may precipitate hypoglycemia. Mexiletine, an antiarrhythmic, is used to treat refractory ventricular arrhythmias; it doesn't cause hypoglycemia. Prednisone, a corticosteroid, is associated with hyperglycemia. Lithium may cause transient hyperglycemia, not hypoglycemia.

A 32-year-old male client with appendicitis is experiencing excruciating abdominal pain. An abdominal X-ray film reveals intraperitoneal air. The nurse should prepare the client for: 1. surgery. 2. colonoscopy. 3. nasogastric tube insertion. 4. barium enema.

1. surgery. The client should be prepared for surgery because his signs and symptoms indicate bowel perforation. Appendicitis is the most common cause of bowel perforation in the United States. Because perforation can lead to peritonitis and sepsis, surgery wouldn't be delayed to perform any other intervention. Also, none of the other procedures are necessary at this point.

A client is admitted for treatment of chronic renal failure (CRF). The nurse knows that this disorder increases the client's risk of: 1. water and sodium retention secondary to a severe decrease in the glomerular filtration rate. 2. a decreased serum phosphate level secondary to kidney failure. 3. an increased serum calcium level secondary to kidney failure. 4. metabolic alkalosis secondary to retention of hydrogen ions.

1. water and sodium retention secondary to a severe decrease in the glomerular filtration rate. A client with CRF is at risk for fluid imbalance — dehydration if the kidneys fail to concentrate urine, or fluid retention if the kidneys fail to produce urine. Electrolyte imbalances associated with this disorder result from the kidneys' inability to excrete phosphorus; such imbalances may lead to hyperphosphatemia with reciprocal hypocalcemia. CRF may cause metabolic acidosis, not metabolic alkalosis, secondary to inability of the kidneys to excrete hydrogen ions.

A client with cholelithiasis has a gallstone lodged in the common bile duct. When assessing this client, the nurse expects to note: 1. yellow sclerae. 2. light amber urine. 3. circumoral pallor. 4. black, tarry stools.

1. yellow sclerae. Yellow sclerae may be the first sign of jaundice, which occurs when the common bile duct is obstructed. Urine normally is light amber. Circumoral pallor and black, tarry stools don't occur in common bile duct obstruction; they are signs of hypoxia and GI bleeding, respectively.

A therapeutic theophylline level is

10 to 20 mcg/ml

Essential Minerals

100mg or more needed/day - calcium, magnesium, phosphorus

Which laboratory test is the most accurate indicator of a client's renal function? 1. Blood urea nitrogen 2. Creatinine clearance 3. Serum creatinine 4. Urinalysis

2. Creatinine clearance Creatinine clearance is the most accurate indicator of a client's renal function because it closely correlates with the kidney's glomerular filtration rate and tubular excretion ability. Results from the other options may be influenced by various conditions and aren't specific to renal disease.

Hypertension =

140/90

The normal I:E ratio is...

1:2

Normal ratio of carbonic acid to bicarbonate

1:20

Treatment for Hiatal Hernia

1Medical and surgical management are similar to those for gastroesophageal reflux disease. Provide small frequent meals and limit the amount of liquids taken with meals. Advise the client not to recline for 1 hour after eating. Avoid anticholinergics, which delay stomach emptying.

Nursing considerations for Ace inhibitors

1st line therapy , used to prevent heart failure and for renal protection in diabetics. Can lead to dry cough.

How many lobes does the left lung have?

2

INR NL value for anticoagulant therapy

2 - 3

How many weeks does a patient have to be on antitubercular therapyto not be considered contagious?

2 weeks

Causes of respiratory acidosis

COPD, atelectasis, drug overdose, severe pneumonia, mechanical ventilation, respiratory muscle weakness

A client with peptic ulcer disease reports that he has been nauseated most of the day and is now feeling light-headed and dizzy. Based upon these findings, which nursing actions would be most appropriate for the nurse to take? Select all that apply. 1. Administering an antacid hourly until nausea subsides. 2. Monitoring the client's vital signs. 3. Notifying the physician of the client's symptoms. 4. Initiating oxygen therapy. 5. Reassessing the client in an hour.

2, 3. The symptoms of nausea and dizziness in a client with peptic ulcer disease may be indicative of hemorrhage and should not be ignored. The appropriate nursing actions at this time are for the nurse to monitor the client's vital signs and notify the physician of the client's symptoms. To administer an antacid hourly or to wait 1 hour to reassess the client would be inappropriate; prompt intervention is essential in a client who is potentially experiencing a gastrointestinal hemorrhage. The nurse would notify the physician of assessment findings and then initiate oxygen therapy if ordered by the physician.

The nurse is caring for a client who has had a gastroscopy. Which of the following signs and symptoms may indicate that the client is developing a complication related to the procedure? Select all that apply. 1. The client has a sore throat. 2. The client has a temperature of 100 ° F (37.8 ° C). 3. The client appears drowsy following the procedure. 4. The client has epigastric pain. 5. The client experiences hematemesis.

2, 4, 5. Following a gastroscopy, the nurse should monitor the client for complications, which include perforation and the potential for aspiration. An elevated temperature, complaints of epigastric pain, or the vomiting of blood (hematemesis) are all indications of a possible perforation and should be reported promptly. A sore throat is a common occurrence following a gastroscopy. Clients are usually sedated to decrease anxiety and the nurse would anticipate that the client will be drowsy following the procedure.

Daily Water Requirement

2,500 cc -- 1,000 - 1,500 cc are fluids - the rest comes from food - water is produced when carbs, fat, and protein are metabolized

A client is diagnosed with Crohn's disease after undergoing two weeks of testing. The client's boss calls the medical-surgical floor requesting to speak with the nurse manager. He expresses concern over the client and explains that he must know the client's diagnosis for insurance purposes. Which response by the nurse is best? 1. "Sure, I understand how demanding insurance companies can be." 2. "I appreciate your concern, but I can't give out any information." 3. "Why don't you come in, and we can further discuss this issue." 4. "He has been diagnosed with Crohn's Disease."

2. "I appreciate your concern, but I can't give out any information." The nurse may not release any confidential information to unauthorized individuals, such as the client's boss. Options 1, 3, and 4 breech client confidentiality.

The client understands post-ileostomy care when the client states: 1. "I empty my bag when it gets too heavy." 2. "I empty my bag before it is half full." 3. "I empty my bag before it is one-third full." 4. I don't need to wear a bag all the time."

2. "I empty my bag before it is half full." The bag should be emptied before it gets "too heavy." Emptying the bag when it is about half full will prevent problems with the bag falling off from the weight of the effluent. Emptying the bag when it is only one-third full could be expensive because they will need to buy supplies more frequently. A client with an ileostomy needs to wear a bag all the time.

The nurse is teaching an elderly client about good bowel habits. Which statement by the client would indicate to the nurse that additional teaching is required? 1. "I should eat a fiber-rich diet with raw, leafy vegetables, unpeeled fruit, and whole grain bread." 2. "I need to use laxatives regularly to prevent constipation." 3. "I need to drink 2 to 3 liters of fluid every day." 4. "I should exercise four times per week."

2. "I need to use laxatives regularly to prevent constipation." The elderly client should be taught to gradually eliminate the use of laxatives. Point out that using laxatives to promote regular bowel movements may have the opposite effect. A high-fiber diet, ample amounts of fluids, and regular exercise promote good bowel health.

A client undergoes extracorporeal shock wave lithotripsy. Before discharge, the nurse should provide which instruction? 1. "Take your temperature every 4 hours." 2. "Increase your fluid intake to 2 to 3 L per day." 3. "Apply an antibacterial dressing to the incision daily." 4. "Be aware that your urine will be cherry-red for 5 to 7 days."

2. "Increase your fluid intake to 2 to 3 L per day." Increasing fluid intake flushes the renal calculi fragments through — and prevents obstruction of — the urinary system. Measuring temperature every 4 hours isn't needed. Lithotripsy doesn't require an incision. Hematuria may occur for a few hours after lithotripsy but should then disappear.

A client with diabetes mellitus must learn how to self-administer insulin. The physician has prescribed 10 U of U-100 regular insulin and 35 U of U-100 isophane insulin suspension (NPH) to be taken before breakfast. When teaching the client how to select and rotate insulin injection sites, the nurse should provide which instruction? 1. "Inject insulin into healthy tissue with large blood vessels and nerves." 2. "Rotate injection sites within the same anatomic region, not among different regions." 3. "Administer insulin into areas of scar tissue or hypotrophy whenever possible." 4. "Administer insulin into sites above muscles that you plan to exercise heavily later that day."

2. "Rotate injection sites within the same anatomic region, not among different regions." The nurse should instruct the client to rotate injection sites within the same anatomic region. Rotating sites among different regions may cause excessive day-to-day variations in the blood glucose level; also, insulin absorption differs from one region to the next. Insulin should be injected only into healthy tissue lacking large blood vessels, nerves, or scar tissue or other deviations. Injecting insulin into areas of hypertrophy may delay absorption. The client shouldn't inject insulin into areas of lipodystrophy (such as hypertrophy or atrophy); to prevent lipodystrophy, the client should rotate injection sites systematically. Exercise speeds drug absorption, so the client shouldn't inject insulin into sites above muscles that will be exercised heavily.

A client has just been diagnosed with type 1 diabetes mellitus. When teaching the client and family how diet and exercise affect insulin requirements, the nurse should include which guideline? 1. "You'll need more insulin when you exercise or increase your food intake." 2. "You'll need less insulin when you exercise or reduce your food intake." 3. "You'll need less insulin when you increase your food intake." 4. "You'll need more insulin when you exercise or decrease your food intake."

2. "You'll need less insulin when you exercise or reduce your food intake." Exercise, reduced food intake, hypothyroidism, and certain medications decrease insulin requirements. Growth, pregnancy, greater food intake, stress, surgery, infection, illness, increased insulin antibodies, and certain medications increase insulin requirements.

An agitated, confused client arrives in the emergency department. The client's history includes type 1 diabetes mellitus, hypertension, and angina pectoris. Assessment reveals pallor, diaphoresis, headache, and intense hunger. A stat blood glucose sample measures 42 mg/dl, and the client is treated for an acute hypoglycemic reaction. After recovery, the nurse teaches the client to treat hypoglycemia by ingesting: 1. 2 to 5 g of a simple carbohydrate. 2. 10 to 15 g of a simple carbohydrate. 3. 18 to 20 g of a simple carbohydrate. 4. 25 to 30 g of a simple carbohydrate.

2. 10 to 15 g of a simple carbohydrate. To reverse hypoglycemia, the American Diabetes Association recommends ingesting 10 to 15 g of a simple carbohydrate, such as three to five pieces of hard candy, two to three packets of sugar (4 to 6 tsp), or 4 oz of fruit juice. If necessary, this treatment can be repeated in 15 minutes. Ingesting only 2 to 5 g of a simple carbohydrate may not raise the blood glucose level sufficiently. Ingesting more than 15 g may raise it above normal, causing hyperglycemia.

A client is recovering from a small-bowel resection. To relieve pain, the physician prescribes meperidine (Demerol), 75 mg I.M. every 4 hours. How soon after administration should meperidine's onset of action occur? 1. 5 to 10 minutes 2. 10 to 15 minutes 3. 30 to 60 minutes 4. 2 to 4 hours

2. 10 to 15 minutes Meperidine's onset of action is 10 to 15 minutes. It peaks between 30 and 60 minutes and has a duration of action of 2 to 4 hours.

As a client recovers from gastric resection, the nurse monitors closely for complications. When the client resumes oral feedings, the nurse observes for early manifestations of dumping syndrome. The vasomotor disturbances associated with this syndrome usually occur how soon after eating? 1. Immediately 2. 15 to 30 minutes 3. 1 to 2 hours 4. 2 to 4 hours

2. 15 to 30 minutes Early manifestations of dumping syndrome occur 15 to 30 minutes after eating. Signs and symptoms of this syndrome include vertigo, tachycardia, syncope, sweating, pallor, palpitations, diarrhea, nausea, and the desire to lie down. Manifestations of dumping syndrome don't occur immediately because food takes a few minutes to reach the jejunum. Early manifestations of vasomotor disturbances usually arise within 45 minutes.

After admission for acute appendicitis, a client undergoes an appendectomy. He complains of moderate postsurgical pain for which the physician prescribes pentazocine (Talwin), 50 mg by mouth every 4 hours. How soon after administration of this drug can the nurse expect the client to feel relief? 1. Less than 15 minutes 2. 15 to 30 minutes 3. 30 to 60 minutes 4. 1 to 2 hours

2. 15 to 30 minutes Orally administered pentazocine has an onset of action of 15 to 30 minutes, reaches peak concentration in less than 1 hour, and has a duration of 3 to 4 hours.

The nurse knows the colostomy begins functioning: 1. Immediately. 2. 2 to 3 days postoperatively. 3. 1 week postoperatively. 4. 2 weeks postoperatively.

2. 2 to 3 days postoperatively. Ileostomies begin to drain immediately. The stoma will begin to secrete mucus within 48 hours, and the proximal loop should begin to drain fecal material within 72 hours.

The nurse must administer an enema to an adult client. The appropriate depth for inserting an enema into an average-sized adult is: 1. 1″ to 2″. 2. 3″ to 4″. 3. 4″ to 6″. 4. 6″ to 8″.

2. 3″ to 4″. The appropriate depth for inserting an enema into an average-sized adult is 3″ to 4″.

A 72-year-old client seeks help for chronic constipation. This is a common problem for elderly clients due to several factors related to aging. Which is one such factor? 1. Increased intestinal motility 2. Decreased abdominal strength 3. Increased intestinal bacteria 4. Decreased production of hydrochloric acid

2. Decreased abdominal strength Decreased abdominal strength, muscle tone of the intestinal wall, and motility all contribute to chronic constipation in the elderly. A decrease in hydrochloric acid causes a decrease in absorption of iron and B12, whereas an increase in intestinal bacteria actually causes diarrhea.

The nurse finds a client who has been diagnosed with a peptic ulcer surrounded by papers from his briefcase and arguing on the telephone with a coworker. The nurse's response to observing these actions should be based on knowledge that: 1. Involvement with his job will keep the client from becoming bored. 2. A relaxed environment will promote ulcer healing. 3. Not keeping up with his job will increase the client's stress level. 4. Setting limits on the client's behavior is an important nursing responsibility.

2. A relaxed environment will promote ulcer healing. A relaxed environment is an essential component of ulcer healing. Nurses can help clients understand the importance of relaxation and explore with them ways to balance work and family demands to promote healing. Being involved with his work may prevent boredom; however, this client is upset and argumentative. Not keeping up with his job will probably increase the client's stress level, but the nurse's response is best if it is based on the fact that a relaxed environment is an essential component of ulcer healing. Nurses cannot set limits on a client's behavior; clients must make the decision to make lifestyle changes.

The nurse finds a client who has been diagnosed with a peptic ulcer surrounded by papers from his briefcase and arguing on the telephone with a coworker. The nurse's response to observing these actions should be based on knowledge that: 1. Involvement with his job will keep the client from becoming bored. 2. A relaxed environment will promote ulcer healing. 3. Not keeping up with his job will increase the client's stress level. 4. Setting limits on the client's behavior is an important nursing responsibility.

2. A relaxed environment will promote ulcer healing. A relaxed environment is an essential component of ulcer healing. Nurses can help clients understand the importance of relaxation and explore with them ways to balance work and family demands to promote healing. Being involved with his work may prevent boredom; however, this client is upset and argumentative. Not keeping up with his job will probably increase the client's stress level, but the nurse's response is best if it is based on the fact that a relaxed environment is an essential component of ulcer healing. Nurses cannot set limits on a client's behavior; clients must make the decision to make lifestyle changes.

The nurse is reviewing the history of a client with bladder cancer. The nurse expects to note documentation of which most common symptom of this type of cancer? 1. Dysuria 2. Hematuria 3. Urgency on urination 4. Frequency of urination

2. Hematuria The most common symptom in clients with cancer of the bladder is hematuria. The client also may experience irritative voiding symptoms such as frequency, urgency, and dysuria, and these symptoms often are associated with carcinoma in situ.

The nurse is monitoring a client receiving paregoric to treat diarrhea for drug interactions. Which drugs can produce additive constipation when given with an opium preparation? 1. Antiarrhythmic drugs 2. Anticholinergic drugs 3. Anticoagulant drugs 4. Antihypertensive drugs

2. Anticholinergic drugs Paregoric has an additive effect of constipation when used with anticholinergic drugs. Antiarrhythmics, anticoagulants, and antihypertensives aren't known to interact with paregoric.

Clinical Manifestations of Bronchiectasis

Decreased chest expansion, decreased breath sounds, adventitious lung sounds in lower lobes, thick secretions, fatigue, weight loss

Which task can the nurse delegate to a nursing assistant? 1. Irrigating a nasogastric (NG) tube 2. Assisting a client who had surgery three days ago walk down the hallway 3. Helping a client who just returned from surgery to the bathroom 4. Administering an antacid to a client complaining of heartburn

2. Assisting a client who had surgery three days ago walk down the hallway Because the client had surgery three days ago, the nurse can safely delegate the task of helping the client walk down the hallway. Irrigating an NG tube, administering medications, and assisting a client who just returned from surgery are tasks that must be performed by licensed nursing personnel.

The client with a new ileostomy has received discharge teaching. Which dinner menu selected by the client indicates understanding of an appropriate diet for a new ileostomy? 1. Baked chicken, whole-grain biscuit, corn on the cob, canned peaches. 2. Baked chicken, mashed potato, cooked carrots, angel food cake. 3. Ham, mashed potato, salad with raw carrots, canned peaches. 4. Roast beef, pasta with butter and garlic, split pea soup, 2 chocolate chip cookies with walnuts.

2. Baked chicken, mashed potato, cooked carrots, angel food cake. Whole grains, corn, peas, nuts, and raw vegetables are considered high-fiber foods and may cause blockages. Garlic may produce odor.

A client with diabetes mellitus demonstrates acute anxiety when first admitted for the treatment of hyperglycemia. The appropriate intervention to decrease the client's anxiety is to: 1. Administer a sedative. 2. Convey empathy, trust, and respect toward the client. 3. Ignore the signs and symptoms of anxiety so that they will soon disappear. 4. Make sure that the client knows all the correct medical terms to understand what is happening.

2. Convey empathy, trust, and respect toward the client. Rationale: The appropriate intervention is to address the client's feelings related to the anxiety. Administering a sedative is not the most appropriate intervention. The nurse should not ignore the client's anxious feelings. A client will not relate to medical terms, particularly when anxiety exists.

A nurse is preparing a plan of care for a client with diabetes mellitus who has hyperglycemia. The priority nursing diagnosis would be: 1. Deficient knowledge 2. Deficient fluid volume 3. Compromised family coping 4. Imbalanced nutrition, less than body requirements

2. Deficient fluid volume Rationale: An increased blood glucose level will cause the kidneys to excrete the glucose in the urine. This glucose is accompanied by fluids and electrolytes, causing an osmotic diuresis leading to dehydration. This fluid loss must be replaced when it becomes severe. Options 1, 3, and 4 are not related specifically to the subject of the question.

A client with peptic ulcer disease tells the nurse that he has black stools, which he has not reported to his physician. Based on this information, which nursing diagnosis would be appropriate for this client? 1. Ineffective coping related to fear of diagnosis of chronic illness. 2. Deficient knowledge related to unfamiliarity with significant signs and symptoms. 3. Constipation related to decreased gastric motility. 4. Imbalanced nutrition: Less than body requirements related to gastric bleeding.

2. Deficient knowledge related to unfamiliarity with significant signs and symptoms. Black, tarry stools are an important warning sign of bleeding in peptic ulcer disease. Digested blood in the stool causes it to be black. The odor of the stool is very offensive. Clients with peptic ulcer disease should be instructed to report the incidence of black stools promptly to their primary health care provider. The data do not support the other diagnoses.

A client with peptic ulcer disease tells the nurse that he has black stools, which he has not reported to his physician. Based on this information, which nursing diagnosis would be appropriate for this client? 1. Ineffective coping related to fear of diagnosis of chronic illness. 2. Deficient knowledge related to unfamiliarity with significant signs and symptoms. 3. Constipation related to decreased gastric motility. 4. Imbalanced nutrition: Less than body requirements related to gastric bleeding.

2. Deficient knowledge related to unfamiliarity with significant signs and symptoms. Black, tarry stools are an important warning sign of bleeding in peptic ulcer disease. Digested blood in the stool causes it to be black. The odor of the stool is very offensive. Clients with peptic ulcer disease should be instructed to report the incidence of black stools promptly to their primary health care provider. The data do not support the other diagnoses.

An obese Hispanic client, age 65, is diagnosed with type 2 diabetes mellitus. Which statement about diabetes mellitus is true? 1. Nearly two-thirds of clients with diabetes mellitus are older than age 60. 2. Diabetes mellitus is more common in Hispanics and Blacks than in Whites. 3. Type 2 diabetes mellitus is less common than type 1 diabetes mellitus. 4. Approximately one-half of the clients diagnosed with type 2 are obese.

2. Diabetes mellitus is more common in Hispanics and Blacks than in Whites. Diabetes mellitus is more common in Hispanics and Blacks than in Whites. Only about one-third of clients with diabetes mellitus are older than age 60 and 85% to 90% have type 2. At least 80% of clients diagnosed with type 2 diabetes mellitus are obese.

The nurse is reviewing the client's record and notes that the physician has documented that the client has a renal disorder. On review of the laboratory results, the nurse most likely would expect to note which of the following? 1. Decreased hemoglobin level 2. Elevated creatinine level 3. Decreased red blood cell count 4. Decreased white blood cell count

2. Elevated creatinine level Measuring the creatinine level is a frequently used laboratory test to determine renal function. The creatinine level increases when at least 50% of renal function is lost. A decreased hemoglobin level and red blood cell count may be noted if bleeding from the urinary tract occurs or if erythropoietic function by the kidney is impaired. An increased white blood cell count is most likely to be noted in renal disease.

Which laboratory finding is the primary diagnostic indicator for pancreatitis? 1. Elevated blood urea nitrogen (BUN) 2. Elevated serum lipase 3. Elevated aspartate aminotransferase (AST) 4. Increased lactate dehydrogenase (LD)

2. Elevated serum lipase Elevation of serum lipase is the most reliable indicator of pancreatitis because this enzyme is produced solely by the pancreas. A client's BUN is typically elevated in relation to renal dysfunction; the AST, in relation to liver dysfunction; and LD, in relation to damaged cardiac muscle.

The nurse asks a client who had abdominal surgery 3 days ago if he has moved his bowels since surgery. The client states, "I haven't moved my bowels, but I am passing gas." How should the nurse intervene? 1. Apply moist heat to the client's abdomen. 2. Encourage the client to ambulate at least three times per day. 3. Administer a tap water enema. 4. Notify the physician.

2. Encourage the client to ambulate at least three times per day. The nurse should encourage the client to ambulate at least three times per day. Ambulating stimulates peristalsis, which helps the bowels to move. It isn't appropriate to apply heat to a surgical wound. Moreover, heat application can't be initiated without a physician order. A tap water enema is typically administered as a last resort after other methods fail. A physician's order is needed with a tap water enema as well. Notifying the physician isn't necessary at this point because the client is exhibiting bowel function by passing flatus.

Which of the following would be an expected outcome for a client with peptic ulcer disease? The client will: 1. Demonstrate appropriate use of analgesics to control pain. 2. Explain the rationale for eliminating alcohol from the diet. 3. Verbalize the importance of monitoring hemoglobin and hematocrit every 3 months. 4. Eliminate contact sports from his or her lifestyle.

2. Explain the rationale for eliminating alcohol from the diet. Alcohol is a gastric irritant that should be eliminated from the intake of the client with peptic ulcer disease. Analgesics are not used to control ulcer pain; many analgesics are gastric irritants. The client's hemoglobin and hematocrit typically do not need to be monitored every 3 months, unless gastrointestinal bleeding is suspected. The client can maintain an active lifestyle and does not need to eliminate contact sports as long as they are not stress-inducing.

Which of the following would be an expected outcome for a client with peptic ulcer disease? The client will: 1. Demonstrate appropriate use of analgesics to control pain. 2. Explain the rationale for eliminating alcohol from the diet. 3. Verbalize the importance of monitoring hemoglobin and hematocrit every 3 months. 4. Eliminate contact sports from his or her lifestyle.

2. Explain the rationale for eliminating alcohol from the diet. Alcohol is a gastric irritant that should be eliminated from the intake of the client with peptic ulcer disease. Analgesics are not used to control ulcer pain; many analgesics are gastric irritants. The client's hemoglobin and hematocrit typically do not need to be monitored every 3 months, unless gastrointestinal bleeding is suspected. The client can maintain an active lifestyle and does not need to eliminate contact sports as long as they are not stress-inducing.

The nurse is developing a care plan for a client with hepatitis A. What is the main route of transmission of this hepatitis virus? 1. Sputum 2. Feces 3. Blood 4. Urine

2. Feces The hepatitis A virus is transmitted by the fecal-oral route, primarily through ingestion of contaminated food or liquids. It isn't transmitted by way of sputum, blood, or urine. However, the hepatitis B virus is transmitted primarily through contact with contaminated blood, human secretions, and feces.

The nurse is reviewing a client's fluid intake and output record. Fluid intake and urine output should relate in which way? 1. Fluid intake should be double the urine output. 2. Fluid intake should be approximately equal to the urine output. 3. Fluid intake should be half the urine output. 4. Fluid intake should be inversely proportional to the urine output.

2. Fluid intake should be approximately equal to the urine output. Normally, fluid intake is approximately equal to the urine output. Any other relationship signals an abnormality. For example, fluid intake that is double the urine output indicates fluid retention; fluid intake that is half the urine output indicates dehydration. Normally, fluid intake isn't inversely proportional to the urine output.

The nurse is teaching a client with type 1 diabetes mellitus how to treat adverse reactions to insulin. To reverse a hypoglycemic reaction, the client ideally should ingest an oral carbohydrate. However, this treatment isn't always possible or safe. Therefore, the nurse should advise the client to keep which alternate treatment on hand? 1. Epinephrine 2. Glucagon 3. 50% dextrose 4. Hydrocortisone

2. Glucagon During a hypoglycemic reaction, a layperson may administer glucagon, an antihypoglycemic agent, to raise the blood glucose level quickly in a client who can't ingest an oral carbohydrate. Epinephrine isn't a treatment for hypoglycemia. Although 50% dextrose is used to treat hypoglycemia, it must be administered I.V. by a skilled health care professional. Hydrocortisone takes a relatively long time to raise the blood glucose level and therefore isn't effective in reversing hypoglycemia.

A client with diabetes mellitus has a prescription for 5 U of U-100 regular insulin and 25 U of U-100 isophane insulin suspension (NPH) to be taken before breakfast. At about 4:30 p.m., the client experiences headache, sweating, tremor, pallor, and nervousness. What is the most probable cause of these signs and symptoms? 1. Hyperglycemia 2. Hypoglycemia 3. Hyperuricemia 4. Hypochondria

2. Hypoglycemia Headache, sweating, tremor, pallor, and nervousness typically result from hypoglycemia, an insulin reaction. Hypoglycemia may occur 4 to 18 hours after administration of isophane insulin suspension or insulin zinc suspension (Lente), which are intermediate-acting insulins. Although hypoglycemia may occur at any time, it usually precedes meals. Hyperglycemia, in contrast, causes such early manifestations as fatigue, malaise, drowsiness, polyuria, and polydipsia. Hyperuricemia refers to an abnormally large amount of uric acid in the blood. Hypochondria is abnormal anxiety about one's health, with a false belief that one has a disease.

Following an ileostomy, when should the drainage appliance be applied to the stoma? 1. 24 hours later, when edema has subsided. 2. In the operating room. 3. After the ileostomy begins to function. 4. When the client is able to begin self-care procedures.

2. In the operating room. Drainage from the ileostomy contains secretions that are rich in digestive enzymes and highly irritating to the skin. Protection of the skin from the effects of these enzymes is begun at once. Skin exposed to these enzymes even for a short time becomes reddened, painful, and excoriated.

The client is admitted to the hospital with viral hepatitis, complaining of "no appetite" and "losing my taste for food." What instruction should the nurse give the client to provide adequate nutrition? 1. Select foods high in fat. 2. Increase intake of fluids, including juices. 3. Eat a good supper when anorexia is not as severe. 4. Eat less often, preferably only three large meals daily.

2. Increase intake of fluids, including juices. Although no special diet is required to treat viral hepatitis, it is generally recommended that clients consume a low-fat diet because fat may be tolerated poorly because of decreased bile production. Small frequent meals are preferable and may even prevent nausea. Frequently , appetite is better in the morning, so it is easier to eat a good breakfast. An adequate fluid intake of 2500 to 3000 mL/ day that includes nutritional juices is also important.

The nurse is assessing a client who is receiving total parenteral nutrition (TPN). Which finding suggests that the client has developed hyperglycemia? 1. Cheyne-Stokes respirations 2. Increased urine output 3. Decreased appetite 4. Diaphoresis

2. Increased urine output Glucose supplies most of the calories in TPN; if the glucose infusion rate exceeds the client's rate of glucose metabolism, hyperglycemia arises. When the renal threshold for glucose reabsorption is exceeded, osmotic diuresis occurs, causing an increased urine output. A decreased appetite and diaphoresis suggest hypoglycemia. Cheyne-Stokes respirations are characterized by a period of apnea lasting 10 to 60 seconds, followed by gradually increasing depth and frequency of respirations. Cheyne-Stokes respirations typically occur with cerebral depression or heart failure.

The nurse is assessing a client who is receiving total parenteral nutrition (TPN). Which finding suggests that the client has developed hyperglycemia? 1. Cheyne-Stokes respirations 2. Increased urine output 3. Decreased appetite 4. Diaphoresis

2. Increased urine output Glucose supplies most of the calories in TPN; if the glucose infusion rate exceeds the client's rate of glucose metabolism, hyperglycemia arises. When the renal threshold for glucose reabsorption is exceeded, osmotic diuresis occurs, causing an increased urine output. A decreased appetite and diaphoresis suggest hypoglycemia. Cheyne-Stokes respirations are characterized by a period of apnea lasting 10 to 60 seconds, followed by gradually increasing depth and frequency of respirations. Cheyne-Stokes respirations typically occur with cerebral depression or heart failure.

The nurse is caring for a client with acute pyelonephritis. Which nursing intervention is most important? 1. Administering a sitz bath twice per day 2. Increasing fluid intake to 3 L/day 3. Using an indwelling urinary catheter to measure urine output accurately 4. Encouraging the client to drink cranberry juice to acidify the urine

2. Increasing fluid intake to 3 L/day Acute pyelonephritis is a sudden inflammation of the interstitial tissue and renal pelvis of one or both kidneys. Infecting bacteria are normal intestinal and fecal flora that grow readily in urine. Pyelonephritis may result from procedures that involve the use of instruments (such as catheterization, cystoscopy, and urologic surgery) or from hematogenic infection. The most important nursing intervention is to increase fluid intake to 3 L/day. This helps empty the bladder of contaminated urine and prevents calculus formation. Administering a sitz bath would increase the likelihood of fecal contamination. Using an indwelling urinary catheter could cause further contamination. Encouraging the client to drink cranberry juice to acidify urine is helpful but isn't the most important interaction.

The nurse explains to the client that a flecible sigmoidoscopy involves: 1. Instillation of a radiopaque contrast medium into the lower GI tract. 2. Insertion of a fiberoptic scope that allows for direct visual examination of the anal canal, rectum, and sigmoid colon. 3. Insertion of a fiberoptic scope that allows for direct visualization of the sigmoid colon, transverse colon and ileocecal valve. 4. Surgical removal of polyps and biopsy of suspicious GI mucosa.

2. Insertion of a fiberoptic scope that allows for direct visual examination of the anal canal, rectum, and sigmoid colon.

The nurse is assessing a client who complains of abdominal pain, nausea, and diarrhea. When examining the client's abdomen, which sequence should the nurse use? 1. Inspection, palpation, percussion, and auscultation 2. Inspection, auscultation, percussion, and palpation 3. Auscultation, inspection, percussion, and palpation 4. Palpation, auscultation, percussion, and inspection

2. Inspection, auscultation, percussion, and palpation The correct sequence for abdominal examination is inspection, auscultation, percussion, and palpation. This sequence differs from that used for other body regions (inspection, palpation, percussion, and auscultation) because palpation and percussion increase intestinal activity, altering bowel sounds. Therefore, the nurse shouldn't palpate or percuss the abdomen before auscultating. Assessment of any body system or region starts with inspection; therefore, auscultating or palpating the abdomen first would be incorrect.

The client has a NG tube connected to low continuous suction for abdominal decompression. The nurse notes that gastric fluid in the suction tubing is not moving and the client's abdomen is becoming distended. The nurse's best action is to: 1. Pull out the NG tube and insert a new one. 2. Irrigate the tube with 30 mL of water. 3. Tell the client to take a few deep breaths. 4. Turn the suction higher.

2. Irrigate the tube with 30 mL of water. Problem solving should be done first. Inserting a new NG tube would cause the client unnecessary discomfort. The mos likely cause of the problem is that the NG tube is plugged with gastric contents of has adhered to the gastric mucosa and is no longer draining. Irrigating the tube should clear any obstructions and free the tube from the gastric mucosa. Having the client take a few deep breaths will not solve the problem. Turning the suction higher may cause additional trauma to the gastric mucosa.

A client takes 30 ml of magnesium hydroxide and aluminum hydroxide with simethicone (Maalox TC) P.O. 1 hour and 3 hours after each meal and at bedtime for treatment of a duodenal ulcer. Why does the client take this antacid so frequently? 1. It has a slow onset of action. 2. It has a short duration of action. 3. It has a prolonged half-life. 4. It's highly metabolized.

2. It has a short duration of action. Because of the short duration of action, frequent doses of antacids are needed. Antacids usually provide a rapid to immediate onset of action, don't have prolonged half-lives, and aren't highly metabolized.

A nurse is observing a client for possible complications of postoperative peritonitis. Which manifestations are most indicative of peritonitis? Select all that apply. 1. Hyperactive bowel sounds. 2. Localized or diffuse pain. 3. Abdominal rigidity. 4. Shallow respirations. 5. Temperature over 102.

2. Localized or diffuse pain. 3. Abdominal rigidity. 4. Shallow respirations. Peristalsis is diminished and paralytic ileus may develop. Pain is often diffuse initially and then localized. The abdominal muscles are rigid and the abdomen is tender and distended. With abdominal tenderness and distension, the client will not breathe as deeply. Temperature is usually 100 - 101.

The physician prescribes spironolactone (Aldactone), 50 mg by mouth four times daily, for a client with fluid retention caused by cirrhosis. Which finding indicates that the drug is producing a therapeutic effect? 1. Serum potassium level of 3.5 mEq/L 2. Loss of 2 lb in 24 hours 3. Serum sodium level of 135 mEq/L 4. Blood pH of 7.25

2. Loss of 2 lb in 24 hours Daily weight measurement is the most accurate indicator of fluid status; a loss of 2.2 lb (1 kg) indicates loss of 1 L of fluid. Because spironolactone is a diuretic, weight loss is the best indicator of its effectiveness. This client's serum potassium and sodium levels are normal. A blood pH of 7.25 indicates acidosis, an adverse reaction to spironolactone.

The most important postoperative nursing concern for a client following an ileostomy is to: 1. Check for the presence of diarrhea. 2. Maintain skin-protective barrier. 3. Allow the client to observe irrigation. 4. Change the stoma pouch daily when full.

2. Maintain skin-protective barrier. The drainage from an ileostomy will always be liquid. The first concern is the stoma and the condition of the skin around the stoma. The drainage from the stoma is made up of digestive enzymes. The pouch opening should be no more than 1/8 inch larger than the stoma. Irrigation is not done with an ileostomy. The pouch is empties, not changed, when it is 1/3 full. It is changed every 5-7 days, usually before eating, when the stoma is least active.

When administering spironolactone (Aldactone) to a client who has had a unilateral adrenalectomy, the nurse should instruct the client about which possible adverse effect of the drug? 1. Constipation 2. Menstrual irregularities 3. Hypokalemia 4. Hypernatremia

2. Menstrual irregularities Spironolactone can cause menstrual irregularities and decreased libido. Men may also experience gynecomastia and impotence. Diarrhea, hyponatremia, and hyperkalemia are also adverse effects of spirolactone.

A client with peptic ulcer disease reports that he has been nauseated most of the day and is now feeling light-headed and dizzy. Based upon these findings, which nursing actions would be most appropriate for the nurse to take? Select all that apply. 1. Administering an antacid hourly until nausea subsides. 2. Monitoring the client's vital signs. 3. Notifying the physician of the client's symptoms. 4. Initiating oxygen therapy. 5. Reassessing the client in an hour.

2. Monitoring the client's vital signs. 3. Notifying the physician of the client's symptoms. The symptoms of nausea and dizziness in a client with peptic ulcer disease may be indicative of hemorrhage and should not be ignored. The appropriate nursing actions at this time are for the nurse to monitor the client's vital signs and notify the physician of the client's symptoms. To administer an antacid hourly or to wait 1 hour to reassess the client would be inappropriate; prompt intervention is essential in a client who is potentially experiencing a gastrointestinal hemorrhage. The nurse would notify the physician of assessment findings and then initiate oxygen therapy if ordered by the physician.

Which condition is most likely to have a nursing diagnosis of fluid volume deficit? 1. Appendicitis 2. Pancreatitis 3. Cholecystitis 4. Gastric ulcer

2. Pancreatitis Hypovolemic shock from fluid shifts is a major factor in acute pancreatitis. The other conditions are less likely to exhibit fluid volume deficit.

Every morning a client with type 1 diabetes receives 15 units of Humulin 70/30. What does this type of insulin contain? 1. 70 units of neutral protamine Hagedorn (NPH) insulin and 30 units of regular insulin 2. 70 units of regular insulin and 30 units of NPH insulin 3. 70% NPH insulin and 30% regular insulin 4. 70% regular insulin and 30% NPH insulin

3. 70% NPH insulin and 30% regular insulin Humulin 70/30 insulin is a combination of 70% NPH insulin and 30% regular insulin.

A client with severe abdominal pain is being evaluated for appendicitis. What is the most common cause of appendicitis? 1. Rupture of the appendix 2. Obstruction of the appendix 3. A high-fat diet 4. A duodenal ulcer

2. Obstruction of the appendix Appendicitis most commonly results from obstruction of the appendix, which may lead to rupture. A high-fat diet or duodenal ulcer doesn't cause appendicitis; however, a client may require dietary restrictions after an appendectomy.

When preparing a client, age 50, for surgery to treat appendicitis, the nurse formulates a nursing diagnosis of Risk for infection related to inflammation, perforation, and surgery. What is the rationale for choosing this nursing diagnosis? 1. Obstruction of the appendix may increase venous drainage and cause the appendix to rupture. 2. Obstruction of the appendix reduces arterial flow, leading to ischemia, inflammation, and rupture of the appendix. 3. The appendix may develop gangrene and rupture, especially in a middle-aged client. 4. Infection of the appendix diminishes necrotic arterial blood flow and increases venous drainage.

2. Obstruction of the appendix reduces arterial flow, leading to ischemia, inflammation, and rupture of the appendix. A client with appendicitis is at risk for infection related to inflammation, perforation, and surgery because obstruction of the appendix causes mucus fluid to build up, increasing pressure in the appendix and compressing venous outflow drainage. The pressure continues to rise with venous obstruction; arterial blood flow then decreases, leading to ischemia from lack of perfusion. Inflammation and bacterial growth follow, and swelling continues to raise pressure within the appendix, resulting in gangrene and rupture. Geriatric, not middle-aged, clients are especially susceptible to appendix rupture.

The nurse is caring for a client who is 2 days postoperative following an abdominal hysterectomy. The client has a history of diabetes mellitus and has been receiving regular insulin according to capillary blood glucose testing four times a day. A carbohydrate-controlled diet has been prescribed but the client has been complaining of nausea and is not eating . On entering the client's room, the nurse finds the client to be confused and diaphoretic. Which action is appropriate at this time? 1. Call a code to obtain needed assistance immediately. 2. Obtain a capillary blood glucose level and perform a focused assessment. 3. Stay with the client and ask the nursing assistant to call the physician for a prescription for intravenous 50% dextrose. 4. Ask the nursing assistant to stay with the client while obtaining 15 to 30 g of a carbohydrate snack for the client to eat.

2. Obtain a capillary blood glucose level and perform a focused assessment. Rationale: Diaphoresis and confusion are signs of moderate hypoglycemia. A likely cause of the client's change in condition could be related to the administration of insulin without the client eating enough food. However, an assessment is necessary to confirm the presence of hypoglycemia. The nurse would obtain a capillary blood glucose level to confirm the hypoglycemia and perform a focused assessment to determine the extent and cause of the client's condition. Once hypoglycemia is confirmed, the nurse stays with the client and asks the nursing assistant to obtain the appropriate carbohydrate snack. A code is called if the client is not breathing or if the heart is not beating.

The client returns to the postanesthesia care unit following GI surgery. Postoperative laboratory values include: O2 saturation 99%, WBC 11,500, Hct 35, Na+ 139, K+ 3.9, glucose 51. The first priority of the nurse would be to: 1. Assess the postoperative wound for infection. 2. Obtain an order for dextrose solution IV. 3. Increase nasal O2 to improve oxygen delivery. 4. Decrease amount of K+ in IV to avoid arrhythmias.

2. Obtain an order for dextrose solution IV. It is too soon for any indication of wound infection. The slight elevation in the WBC count is most likely related to the reason for the GI surgery. The client's serum glucose level is dangerously low and will interfere with accurately assessing the level of consciousness as the client is waking up from anesthesia. Glucose is needed for normal brain functioning. The lowest normal for serum glucose is 70 mg/dL. The blood is well saturated with oxygen. The potassium level is low normal.

What nursing action best facilitates the passage of the NG tube from the stomach though the pylorous and into the duodenum? 1. Gently advancing the tube 1 to 4 inches at the regular time intervals. 2. Positioning the client on the right side for 2 hours after insertion. 3. Maintaining strict bedrest and avoiding all unnecessary movement. 4. Positioning the client in a flat supine position.

2. Positioning the client on the right side for 2 hours after insertion.

When caring for a client with acute pancreatitis, the nurse should use which comfort measure? 1. Administering an analgesic once per shift, as prescribed, to prevent drug addiction 2. Positioning the client on the side with the knees flexed 3. Encouraging frequent visits from family and friends 4. Administering frequent oral feedings

2. Positioning the client on the side with the knees flexed The nurse should place the client with acute pancreatitis in a side-lying position with knees flexed; this position promotes comfort by decreasing pressure on the abdominal muscles. The nurse should administer an analgesic, as needed and prescribed, before pain becomes severe, rather than once each shift. Because the client needs a quiet, restful environment during the acute disease stage, the nurse should discourage frequent visits from family and friends. Frequent oral feedings are contraindicated during the acute stage to allow the pancreas to rest.

Following surgery for a ruptured appendix, the nurse should place the client in a semi-Fowler's position primarily to: 1. Fully aerate the lungs. 2. Promote drainage and prevent subdiaphragmatic abscesses. 3. Splint the wound. 4. Facilitate movement and reduce complications from immobility.

2. Promote drainage and prevent subdiaphragmatic abscesses. Although this position will help aerate the lungs by allowing for maximum expansion, it is not he primary reason for this position. This position will no help splint the incision. This position will not help facilitate movement or necessarily reduce complications.

A client who was diagnosed with type 1 diabetes mellitus 14 years ago is admitted to the medical-surgical unit with abdominal pain. On admission, the client's blood glucose level is 470 mg/dl. Which finding is most likely to accompany this blood glucose level? 1. Cool, moist skin 2. Rapid, thready pulse 3. Arm and leg trembling 4. Slow, shallow respirations

2. Rapid, thready pulse This client's abnormally high blood glucose level indicates hyperglycemia, which typically causes polyuria, polyphagia, and polydipsia. Because polyuria leads to fluid loss, the nurse should expect to assess signs of deficient fluid volume, such as a rapid, thready pulse; decreased blood pressure; and rapid respirations. Cool, moist skin and arm and leg trembling are associated with hypoglycemia. Rapid respirations — not slow, shallow ones — are associated with hyperglycemia.

One day after surgery for intestinal resection, a client has no bowel sounds. Which action should a nurse take? 1. Take the vital signs and notify the physician. 2. Record this expected finding. 3. Check rectally for impacted stool. 4. Perform abdominal massage.

2. Record this expected finding. Paralytic ileus and the absence of bowel sounds is expected on the first postop day.

A client with ulcerative colitis is receiving methaneline bromide (Banthine). This nurse knows the primary reason this client is receiving methaneline bromide is to: 1. Supress inflammation of the bowel. 2. Reduce peristaltic activity. 3. Neutralize acid in the gastrointestinal tract. 4, Increase bowel tone.

2. Reduce peristaltic activity.

A client is scheduled for bowel resection with anastomosis involving the large intestine. Because of the surgical site, the nurse formulates the nursing diagnosis of Risk for infection. To complete the nursing diagnosis statement, the nurse should add which "related-to" phrase? 1. Related to major surgery required by bowel resection 2. Related to the presence of bacteria at the surgical site 3. Related to malnutrition secondary to bowel resection with anastomosis 4. Related to the presence of a nasogastric (NG) tube postoperatively

2. Related to the presence of bacteria at the surgical site The large intestine normally contains bacteria because its alkaline environment permits growth of organisms that putrefy and break down remaining proteins and indigestible residue. These organisms include Escherichia coli, Aerobacter aerogenes, Clostridium perfringens, and Lactobacillus. Although bowel resection with anastomosis is considered major surgery, it poses no greater risk of infection than any other type of major surgery. Malnutrition seldom follows bowel resection with anastomosis because nutritional absorption (except for some water, sodium, and chloride) is completed in the small intestine. An NG tube is placed through a natural opening, not a wound, and therefore doesn't increase the client's risk of infection.

A client is admitted to the medical intensive care unit with a diagnosis of pancreatitis. Which nursing intervention is most appropriate? 1. Providing generous servings at mealtime 2. Reserving an antecubital site for a peripherally inserted central catheter (PICC) 3. Providing the client with plenty of P.O. fluids 4. Limiting I.V. fluid intake according to the physician's order

2. Reserving an antecubital site for a peripherally inserted central catheter (PICC) Pancreatitis treatment typically involves resting the GI tract by maintaining nothing-by-mouth status. The nurse should reserve the antecubital site for a PICC, which enables the client to receive long-term total parenteral nutrition. Clients in the acute stages of pancreatitis also require large volumes of I.V. fluids to compensate for fluid loss.

A client with type 1 diabetes mellitus takes 15 U of Humulin N insulin before breakfast and 8 U before dinner. During a follow-up visit, the nurse reevaluates the client's knowledge about insulin therapy and self-administration skills and learns that the client is unaware that certain over-the-counter (OTC) preparations and other medications may interact with insulin. The nurse should advise the client to avoid which OTC preparations? 1. Antacids 2. Salicylate-containing preparations 3. Vitamins with iron 4. Acetaminophen-containing preparations

2. Salicylate-containing preparations Salicylates may interact with insulin, causing hypoglycemia. Antacids, vitamins with iron, and acetaminophen aren't known to interact with insulin.

The day after colostomy surgery an adult says to the nurse, "I know the doctor did not really do a colostomy." The nurse understands that he is in an early stage of adjustment to his diagnosis and surgery. What nursing action is indicated at this time? 1. Agree with him until he is ready to accept his colostomy. 2. Say "It must be difficult to have this kind of surgery." 3. Force him to look at his colostomy. 4. Ask the surgeon to explain the surgery to the client.

2. Say "It must be difficult to have this kind of surgery."

The client has been admitted to the hospital with a diagnosis of acute pancreatitis and the nurse is assessing the client's pain. What type of pain is consistent with this diagnosis? 1. Burning and aching, located in the left lower quadrant and radiating to the hip 2. Severe and unrelenting, located in the epigastric area and radiating to the back 3. Burning and aching, located in the epigastric area and radiating to the umbilicus 4. Severe and unrelenting, located in the left lower quadrant and radiating to the groin

2. Severe and unrelenting, located in the epigastric area and radiating to the back Rationale: The pain associated with acute pancreatitis is often severe and unrelenting, is located in the epigastric region, and radiates to the back. The other options are incorrect.

A nurse teaches a client with diabetes mellitus about differentiating between hypoglycemia and ketoacidosis. The client demonstrates an understanding of the teaching by stating that a form of glucose should be taken if which of the following symptoms develops? 1. Polyuria 2. Shakiness 3. Blurred vision 4. Fruity breath odor

2. Shakiness Rationale: Shakiness is a sign of hypoglycemia and would indicate the need for food or glucose. A fruity breath odor, blurred vision, and polyuria are signs of hyperglycemia.

The client is admitted to the emergency department following a motor vehicle accident. The client was wearing a lap seat belt when the accident occurred and now the client has hematuria and lower abdominal pain. To assess further whether the pain is caused by bladder trauma, the nurse asks the client if the pain is referred to which of the following areas? 1. Hip 2. Shoulder 3. Umbilicus 4. Costovertebral angle

2. Shoulder Bladder trauma or injury is characterized by lower abdominal pain that may radiate to one of the shoulders due to phrenic nerve irritation. Bladder injury pain does not radiate to the umbilicus, costovertebral angle, or hip.

The mouth care measure that should be used with caution by the nurse when a client has a nasogastric tube is: 1. Regularly brushing teeth and tongue with soft brush. 2. Sucking on ice chips to relieve dryness. 3. Occasionally rinsing mouth with a nonastringent substance and massaging gums. 4. Application of lemon juice and glycerine swabs to the lips.

2. Sucking on ice chips to relieve dryness. The client should be cautioned to limit the number of ice chips he or she sucks on. The NG tube will remove not only the increased water ingested from the melted chips but also essential electrolytes.

After taking glipizide (Glucotrol) for 9 months, a client experiences secondary failure. What would the nurse expect the physician to do? 1. Initiate insulin therapy. 2. Switch the client to a different oral antidiabetic agent. 3. Prescribe an additional oral antidiabetic agent. 4. Restrict carbohydrate intake to less than 30% of the total caloric intake.

2. Switch the client to a different oral antidiabetic agent. Many clients (25% to 60%) with secondary failure respond to a different oral antidiabetic agent. Therefore, it wouldn't be appropriate to initiate insulin therapy at this time. However, if a new oral antidiabetic agent is unsuccessful in keeping glucose levels at an acceptable level, insulin may be used in addition to the antidiabetic agent.

The nurse can quickly assess volume depletion in a client with ulcerative colitits by: 1. Measuring the quantity and specific gravity of the client's urine output. 2. Taking the client's blood pressure first supine, then sitting, noting any changes. 3. Comparing the client's present weight with the weight on a previous admission. 4. Administering the oral water test.

2. Taking the client's blood pressure first supine, then sitting, noting any changes. Urine output and specific gravity are better measures of the adequacy of fluid volume replacement than of fluid volume depletion. Postural blood pressure readings are an excellent mode for assessing volume depletion. If the systolic blood pressure decreases more than 10 mm Hg and there is a concurrent increase in pulse rate, a volume depletion problem is indicated.

The nurse and the nursing assistant are caring for a paraplegic client admitted with peptic ulcer disease. Which intervention should be performed by the registered nurse? 1. Repositioning the client 2. Teaching the client about consuming a bland diet 3. Measuring the client's blood pressure 4. Measuring urinary output

2. Teaching the client about consuming a bland diet The registered nurse should teach the client about consuming a bland diet. Unlicensed assistive personnel aren't educated to teach clients about specialty diets. Repositioning the client, measuring blood pressure, and measuring urinary output are all tasks that can be performed by a nursing assistant.

While preparing a client for an upper GI endoscopy (esophagogastroduodenoscopy), the nurse should implement which interventions? 1. Administer a preparation to cleanse the GI tract, such as Golytely or Fleets Phospha-Soda. 2. Tell the client he shouldn't eat or drink for 6 to 12 hours before the procedure. 3. Tell the client he must be on a clear liquid diet for 24 hours before the procedure. 4. Inform the client that he'll receive a sedative before the procedure. 5. Tell the client that he may eat and drink immediately after the procedure.

2. Tell the client he shouldn't eat or drink for 6 to 12 hours before the procedure. 4. Inform the client that he'll receive a sedative before the procedure. The client shouldn't eat or drink for 6 to 12 hours before the procedure to assure that his upper GI tract is clear for viewing. The client will receive a sedative before the endoscope is inserted that will help him relax, but allow him to remain conscious. GI tract cleansing and a clear liquid diet are interventions for a client having a lower GI tract procedure, such as a colonoscopy. Food and fluids must be withheld until the gag reflex returns.

While preparing a client for an upper GI endoscopy (esophagogastroduodenoscopy), the nurse should implement which interventions? 1. Administer a preparation to cleanse the GI tract, such as Golytely or Fleets Phospha-Soda. 2. Tell the client he shouldn't eat or drink for 6 to 12 hours before the procedure. 3. Tell the client he must be on a clear liquid diet for 24 hours before the procedure. 4. Inform the client that he'll receive a sedative before the procedure. 5. Tell the client that he may eat and drink immediately after the procedure.

2. Tell the client he shouldn't eat or drink for 6 to 12 hours before the procedure. 4. Inform the client that he'll receive a sedative before the procedure. The client shouldn't eat or drink for 6 to 12 hours before the procedure to assure that his upper GI tract is clear for viewing. The client will receive a sedative before the endoscope is inserted that will help him relax, but allow him to remain conscious. GI tract cleansing and a clear liquid diet are interventions for a client having a lower GI tract procedure, such as a colonoscopy. Food and fluids must be withheld until the gag reflex returns.

When a client with an indwelling urinary catheter insists on walking to the hospital lobby to visit with family members, the nurse teaches him how to do this without compromising the catheter. Which client action indicates an accurate understanding of this information? 1. The client sets the drainage bag on the floor while sitting down. 2. The client keeps the drainage bag below the bladder at all times. 3. The client clamps the catheter drainage tubing while visiting with the family. 4. The client loops the drainage tubing below its point of entry into the drainage bag.

2. The client keeps the drainage bag below the bladder at all times. To maintain effective drainage, the client should keep the drainage bag below the bladder; this allows the urine to flow by gravity from the bladder to the drainage bag. The client shouldn't lay the drainage bag on the floor because it could become grossly contaminated. The client shouldn't clamp the catheter drainage tubing because this impedes the flow of urine. To promote drainage, the client may loop the drainage tubing above — not below — its point of entry into the drainage bag.

After having transurethral resection of the prostate (TURP), a client returns to the unit with a three-way indwelling urinary catheter and continuous closed bladder irrigation. Which finding suggests that the client's catheter is occluded? 1. The urine in the drainage bag appears red to pink. 2. The client reports bladder spasms and the urge to void. 3. The normal saline irrigant is infusing at a rate of 50 drops/min. 4. About 1,000 ml of irrigant have been instilled; 1,200 ml of drainage have been returned.

2. The client reports bladder spasms and the urge to void. Reports of bladder spasms and the urge to void suggest that a blood clot may be occluding the catheter. After TURP, urine normally appears red to pink, and normal saline irrigant usually is infused at a rate of 40 to 60 drops/min or according to facility protocol. The amount of returned fluid (1,200 ml) should correspond to the amount of instilled fluid, plus the client's urine output (1,000 ml + 200 ml), which reflects catheter patency.

The nurse is assessing the colostomy of a client who has had an abdominal perineal resection for a bowel tumor. Which of the following assessment findings indicates that the colostomy is beginning to function? 1. Absent bowel sounds 2. The passage of flatus 3. The client's ability to tolerate food 4. Bloody drainage from the colostomy

2. The passage of flatus Following abdominal perineal resection, the nurse would expect the colostomy to begin to function within 72 hours after surgery, although it may take up to 5 days . The nurse should assess for a return of peristalsis, listen for bowel sounds, and check for the passage of flatus . Absent bowel sounds would not indicate the return of peristalsis. The client would remain NPO until bowel sounds return and the colostomy is functioning. Bloody drainage is not expected from a colostomy.

A client with bladder cancer has had the bladder removed and an ileal conduit created for urine diversion. While changing this client's pouch, the nurse observes that the area around the stoma is red, weeping, and painful. What should the nurse conclude? 1. The skin wasn't lubricated before the pouch was applied. 2. The pouch faceplate doesn't fit the stoma. 3. A skin barrier was applied properly. 4. Stoma dilation wasn't performed.

2. The pouch faceplate doesn't fit the stoma. If the pouch faceplate doesn't fit the stoma properly, the skin around the stoma will be exposed to continuous urine flow from the stoma, causing excoriation and red, weeping, and painful skin. A lubricant shouldn't be used because it would prevent the pouch from adhering to the skin. When properly applied, a skin barrier prevents skin excoriation. Stoma dilation isn't performed with an ileal conduit, although it may be done with a colostomy if ordered.

During rectal examination, which finding would be further evidence of a urethral injury? 1. A low-riding prostate 2. The presence of a boggy mass 3. Absent sphincter tone 4. A positive Hemoccult

2. The presence of a boggy mass When the urethra is ruptured, a hematoma or collection of blood separates the two sections of urethra. This may feel like a boggy mass on rectal examination. Because of the rupture and hematoma, the prostate becomes high riding. A palpable prostate gland usually indicates a nonurethral injury. Absent sphincter tone would refer to a spinal cord injury. The presence of blood would probably correlate with GI bleeding or a colon injury.

A client has type 1 diabetes. Her husband finds her unconscious at home and administers glucagon, 0.5 mg subcutaneously. She awakens in 5 minutes. Why should her husband offer a complex carbohydrate snack to her as soon as possible? 1. To decrease the possibility of nausea and vomiting 2. To restore liver glycogen and prevent secondary hypoglycemia 3. To stimulate her appetite 4. To decrease the amount of glycogen in her system

2. To restore liver glycogen and prevent secondary hypoglycemia A client with type 1 diabetes who requires glucagon should be given a complex carbohydrate snack as soon as possible to restore the liver glycogen and prevent secondary hypoglycemia. A complex carbohydrate snack doesn't decrease the possibility of nausea and vomiting or stimulate the appetite, and it increases the amount of glycogen in the system.

Which conditions are functions of antidiuretic hormone (ADH)? 1. Sodium absorption and potassium excretion 2. Water reabsorption and urine concentration 3. Water reabsorption and urine dilution 4. Sodium reabsorption and potassium retention

2. Water reabsorption and urine concentration ADH stimulates the renal tubules to reabsorb water, thereby concentrating urine. Aldosterone is responsible for sodium reabsorption and potassium excretion by the kidneys.

Which clinical finding would the nurse look for in a client with chronic renal failure? 1. Hypotension 2. Uremia 3. Metabolic alkalosis 4. Polycythemia

2. Uremia Uremia is the buildup of nitrogenous wastes in the blood, evidenced by an elevated blood urea nitrogen and creatine levels. Uremia, anemia, and acidosis are consistent clinical manifestations of chronic renal failure. Metabolic acidosis results from the inability to excrete hydrogen ions. Anemia results from a lack of erythropoietin. Hypertension (from fluid overload) may or may not be present in chronic renal failure. Hypotension, metabolic alkalosis, and polycythemia aren't present in renal failure.

Which statement describing urinary incontinence in the elderly is true? 1. Urinary incontinence is a normal part of aging. 2. Urinary incontinence isn't a disease. 3. Urinary incontinence in the elderly can't be treated. 4. Urinary incontinence is a disease.

2. Urinary incontinence isn't a disease. Urinary incontinence isn't a normal part of aging nor is it a disease. It may be caused by confusion, dehydration, fecal impaction, restricted mobility, or other causes. Certain medications, including diuretics, hypnotics, sedatives, anticholinergics, and antihypertensives, may trigger urinary incontinence. Most clients with urinary incontinence can be treated; some can be cured.

The nurse is reviewing the report of a client's routine urinalysis. Which value should the nurse consider abnormal? 1. Specific gravity of 1.03 2. Urine pH of 3.0 3. Absence of protein 4. Absence of glucose

2. Urine pH of 3.0 Normal urine pH is 4.5 to 8; therefore, a urine pH of 3.0 is abnormal. Urine specific gravity normally ranges from 1.002 to 1.035, making this client's value normal. Normally, urine contains no protein, glucose, ketones, bilirubin, bacteria, casts, or crystals. Red blood cells should measure 0 to 3 per high-power field; white blood cells, 0 to 4 per high-power field. Urine should be clear, with color ranging from pale yellow to deep amber.

An adult male is admitted to the emergency department with a strangulated inguinal hernia. Emergency surgery that involves reduction of the hernia, resection of a portion of the bowel, and repair of the abdominal wall is performed. What is most likely to be included in the immediate postoperative care? 1. Encouraging him to cough and deep breathe every 2 hours. 2. Using a scrotal support. 3. Frequently offering him oral fluids. 4. Inserting an indwelling urinary catheter.

2. Using a scrotal support. due to scrotal edema

The nurse explains to a client that the colostomy appliance for a double-barrel colostomy should be changed: 1. Every day. 2. When drainage leaks through the seal. 3. Once a week. 4. At a time selected by the visiting nurse.

2. When drainage leaks through the seal. Every day is too often and can damage the skin from pulling the appliance off so often. The appliance should be changes every 2-3 days or as soon as there is leakage. Drainage can excoriate the skin; therefore a new appliance needs to be applied as soon as leakage appears. 1 week is too long for the skin to go without being examined and cleansed.

A client with complaints of right, lower quadrant pain is admitted to the emergency department. Blood specimens are drawn and sent to the laboratory. Which laboratory finding should be reported to the physician immediately? 1. Hematocrit 42% 2. White blood cell (WBC) count 22.8/mm3 3. Serum potassium 4.2 mEq/L 4. Serum sodium 135 mEq/L

2. White blood cell (WBC) count 22.8/mm3 The nurse should report the elevated WBC count, which is evident in option 2. This finding, which is a sign of infection, indicates that the client's appendix might have ruptured. Hematocrit of 42%, serum potassium of 4.2 mEq/L, and serum sodium of 135 mEq/L are within normal limits. Alterations in these levels aren't indicative of appendicitis.

Because of difficulties with hemodialysis, peritoneal dialysis is initiated to treat a client's uremia. Which finding signals a significant problem during this procedure? 1. Blood glucose level of 200 mg/dl 2. White blood cell (WBC) count of 20,000/mm3 3. Potassium level of 3.5 mEq/L 4. Hematocrit (HCT) of 35%

2. White blood cell (WBC) count of 20,000/mm3 An increased WBC count indicates infection, probably resulting from peritonitis, which may have been caused by insertion of the peritoneal catheter into the peritoneal cavity. Peritonitis can cause the peritoneal membrane to lose its ability to filter solutes; therefore, peritoneal dialysis would no longer be a treatment option for this client. Hyperglycemia occurs during peritoneal dialysis because of the high glucose content of the dialysate; it's readily treatable with sliding-scale insulin. A potassium level of 3.5 mEq/L can be treated by adding potassium to the dialysate solution. An HCT of 35% is lower than normal. However, in this client, the value isn't abnormally low because of the daily blood samplings. A lower HCT is common in clients with chronic renal failure because of the lack of erythropoietin.

A client has just been diagnosed with hepatitis A. On assessment, the nurse expects to note: 1. severe abdominal pain radiating to the shoulder. 2. anorexia, nausea, and vomiting. 3. eructation and constipation. 4. abdominal ascites.

2. anorexia, nausea, and vomiting. Hallmark signs and symptoms of hepatitis A include anorexia, nausea, vomiting, fatigue, and weakness. Abdominal pain may occur but doesn't radiate to the shoulder. Eructation and constipation are common in gallbladder disease, not hepatitis A. Abdominal ascites is a sign of advanced hepatic disease, not an early sign of hepatitis A.

The nurse is teaching the client about glipizide (Glucotrol) therapy. The nurse warns the client that glipizide commonly causes hypoglycemia when combined with which over-the-counter preparation? 1. acetaminophen (Tylenol) 2. aspirin 3. St. Johns Wort 4. multivitamins

2. aspirin When taken in combination with aspirin, glipizide commonly causes hypoglycemia. Acetaminophen, St. John's Wort, and multivitamins may be taken with glipizide without increasing the risk of hypoglycemia.

The nurse is explaining the action of insulin to a newly diagnosed diabetic client. During the teaching, the nurse reviews the process of insulin secretion in the body. The nurse is correct when stating that insulin is secreted from the: 1. adenohypophysis. 2. beta cells of the pancreas. 3. alpha cells of the pancreas. 4. parafollicular cells of the thyroid.

2. beta cells of the pancreas. The beta cells of the pancreas secrete insulin. The adenohypophysis or anterior pituitary gland secretes many hormones, such as growth hormone, prolactin, thyroid-stimulating hormone, corticotropin, follicle-stimulating hormone, and luteinizing hormone, but not insulin. The alpha cells of the pancreas secrete glucagon, which raises the blood glucose level. The parafollicular cells of the thyroid secrete the hormone calcitonin, which plays a role in calcium metabolism.

The nurse is inserting a urinary catheter into a client who is extremely anxious about the procedure. The nurse can facilitate the insertion by asking the client to: 1. initiate a stream of urine. 2. breathe deeply. 3. turn to the side. 4. hold the labia or shaft of the penis.

2. breathe deeply. When inserting a urinary catheter, facilitate insertion by asking the client to breathe deeply. Doing this will relax the urinary sphincter. Initiating a stream of urine isn't recommended during catheter insertion. Turning to the side or holding the labia or penis won't ease insertion, and doing so may contaminate the sterile field.

For a client who must undergo colon surgery, the physician orders preoperative cleansing enemas and neomycin sulfate (Mycifradin). The rationale for neomycin use in this client is to: 1. control postoperative nausea and vomiting. 2. decrease the intestinal bacteria count. 3. increase the intestinal bacteria count. 4. prevent the development of megacolon.

2. decrease the intestinal bacteria count. The antibiotic neomycin sulfate (Mycifradin) is prescribed to decrease the bacterial count and reduce the risk of fecal contamination during surgery. After surgery, the physician may prescribe an antiemetic — not an antibiotic — to control postoperative nausea and vomiting. Antibiotics decrease the intestinal bacteria count, not increase it. They have no relation to megacolon development. To prevent this complication, the client should avoid opioid analgesics, such as morphine (Duramorph), which can decrease intestinal motility and contribute to megacolon.

When evaluating a client for complications of acute pancreatitis, the nurse would observe for: 1. increased intracranial pressure. 2. decreased urine output. 3. bradycardia. 4. hypertension.

2. decreased urine output.Acute pancreatitis can cause decreased urine output, which results from the renal failure that sometimes accompanies this condition. Intracranial pressure neither increases nor decreases in a client with pancreatitis. Tachycardia, not bradycardia, usually is associated with pulmonary or hypovolemic complications of pancreatitis. Hypotension can be caused by a hypovolemic complication, but hypertension usually isn't related to acute pancreatitis.

For a client with cirrhosis, deterioration of hepatic function is best indicated by: 1. fatigue and muscle weakness. 2. difficulty in arousal. 3. nausea and anorexia. 4. weight gain.

2. difficulty in arousal. Hepatic encephalopathy, a major complication of advanced cirrhosis, occurs when the liver no longer can convert ammonia (a by-product of protein breakdown) into glutamine. This leads to an increased blood level of ammonia — a central nervous system toxin — which causes a decrease in the level of consciousness. Fatigue, muscle weakness, nausea, anorexia, and weight gain occur during the early stages of cirrhosis.

A client is admitted with a diagnosis of acute renal failure. The nurse should monitor closely for: 1. enuresis. 2. drug toxicity. 3. lethargy. 4. insomnia.

2. drug toxicity. Acute renal failure is characterized by oliguria and rapid accumulation of nitrogen waste in the blood. Kidneys excrete medications, so the nurse should monitor the client closely for drug toxicity. With decreased urinary output or no output, enuresis shouldn't occur. The client will most likely feel lethargic, but this isn't as serious a problem as drug toxicity. The client isn't likely to have insomnia, but, may instead want to sleep most of the time.

A client with chronic renal failure who receives hemodialysis three times weekly has a hemoglobin (Hb) level of 7 g/dl. The most therapeutic pharmacologic intervention would be to administer: 1. ferrous sulfate (Feratab). 2. epoetin alfa (Epogen) 3. filgrastim (Neupogen) 4. enoxaparin (Lovenox)

2. epoetin alfa (Epogen) Chronic renal failure diminishes the production of erythropoietin by the kidneys and leads to a subnormal Hb level. (Normal Hb level is 13 to 18 g/dl in men and 12 to 16 g/dl in women.) An effective pharmacologic treatment for this is epoetin alfa, a recombinant erythropoietin. Because the client's anemia is caused by a deficiency of erythropoietin and not a deficiency of iron, administering ferrous sulfate would be ineffective. Neither filgrastim, a drug used to stimulate neutrophils, nor enoxaparin (low-molecular-weight heparin) will raise the client's Hb level.

A client with type 1 diabetes mellitus asks the nurse about taking an oral antidiabetic agent. The nurse explains that these medications are only effective if the client: 1. prefers to take insulin orally. 2. has type 2 diabetes. 3. has type 1 diabetes. 4. is pregnant and has type 2 diabetes.

2. has type 2 diabetes. Oral antidiabetic agents are only effective in adult clients with type 2 diabetes. Oral antidiabetic agents aren't effective in type 1 diabetes. Pregnant and lactating women aren't prescribed oral antidiabetic agents because the effect on the fetus or breast-fed infant is uncertain.

A client is scheduled to undergo surgical creation of an ileal conduit. The primary nurse educates the client about surgery and the postoperative period. The nurse informs the client that many members of the health care team (including a mental health practitioner) will see him. A mental health practitioner should be involved in the client's care to: 1. assess whether the client is a good candidate for surgery. 2. help the client cope with the anxiety associated with changes in body image. 3. assess suicidal risk postoperatively. 4. evaluate the client's need for mental health intervention.

2. help the client cope with the anxiety associated with changes in body image. Many clients who undergo surgery for creation of an ileal conduit experience anxiety associated with changes in body image. The mental health practitioner can help with client cope these feelings of anxiety. Mental health practitioners don't evaluate whether the client is a surgical candidate. None of the evidence suggests that urinary diversion surgery, such as creation of an ileal conduit, places the client at risk for suicide. Although evaluating the need for mental health intervention is always important, this client displays no behavioral changes that suggest intervention is necessary at this time.

The nurse is performing an assessment on a client who has developed a paralytic ileus. The client's bowel sounds will be: 1. hyperactive. 2. hypoactive. 3. high-pitched. 4. blowing.

2. hypoactive. If a paralytic ileus occurs, bowel sounds will be hypoactive or absent. Hyperactive bowel sounds may signify hunger, intestinal obstruction, or diarrhea. High-pitched sounds may signify a dilated bowel. A blowing sound may be a bruit from a partially obstructed abdominal aorta.

The physician enters a computer order for the nurse to irrigate a client's nephrostomy tube every four hours to maintain patency. The nurse irrigates the tube using sterile technique. After irrigating the tube, the nurse decides that she can safely use the same irrigation set for her 8-hour shift if she covers the set with a paper, sterile drape. This action by the nurse is: 1. appropriate because the irrigation just checks for patency. 2. inappropriate because irrigation requires strict sterile technique. 3. appropriate because the irrigation set will only be used during an 8-hour period. 4. inappropriate because the sterile drape must be cloth, not paper.

2. inappropriate because irrigation requires strict sterile technique. Irritating a nephrostomy tube requires strict sterile technique; therefore, reusing the irrigation set (even if covered by a sterile drape) is inappropriate. Bacteria can proliferate inside the syringe and irrigation container. Although this procedure checks patency, it requires sterile technique to prevent the introduction of bacteria into the kidney. The material in which the sterile drape is made is irrelevant because a sterile drape doesn't deter bacterial growth in the irrigation equipment.

A client with pancreatitis complains of pain. The nurse expects the physician to prescribe meperidine (Demerol) instead of morphine to relieve pain because: 1. meperidine provides a better, more prolonged analgesic effect. 2. morphine may cause spasms of Oddi's sphincter. 3. meperidine is less addictive than morphine. 4. morphine may cause hepatic dysfunction.

2. morphine may cause spasms of Oddi's sphincter. For a client with pancreatitis, the physician will probably avoid prescribing morphine because this drug may trigger spasms of the sphincter of Oddi (a sphincter at the end of the pancreatic duct), causing irritation of the pancreas. Meperidine has a somewhat shorter duration of action than morphine. The two drugs are equally addictive. Morphine isn't associated with hepatic dysfunction.

A client is admitted with suspected cirrhosis. During assessment, the nurse is most likely to detect: 1. an increase in chest hair. 2. muscle wasting. 3. testicular hypertrophy. 4. an increased clotting tendency.

2. muscle wasting. Cirrhosis causes muscle wasting, a decrease in chest and axillary hair, testicular atrophy, and an increased bleeding tendency.

A client with advanced cirrhosis has a prothrombin time (PT) of 15 seconds, compared with a control time of 11 seconds. The nurse expects to administer: 1. spironolactone (Aldactone). 2. phytonadione (Mephyton). 3. furosemide (Lasix). 4. warfarin (Coumadin).

2. phytonadione (Mephyton). Prothrombin synthesis in the liver requires vitamin K. In cirrhosis, vitamin K is lacking, precluding prothrombin synthesis and, in turn, increasing the client's PT. An increased PT, which indicates clotting time, increases the risk of bleeding. Therefore, the nurse should expect to administer phytonadione (vitamin K1) to promote prothrombin synthesis. Spironolactone and furosemide are diuretics and have no effect on bleeding or clotting time. Warfarin is an anticoagulant that prolongs PT.

A client has undergone a colon resection. While turning him, wound dehiscence with evisceration occurs. The nurse's first response is to: 1. call the physician. 2. place saline-soaked sterile dressings on the wound. 3. take a blood pressure and pulse. 4. pull the dehiscence closed.

2. place saline-soaked sterile dressings on the wound. The nurse should first place saline-soaked sterile dressings on the open wound to prevent tissue drying and possible infection. Then the nurse should call the physician and take the client's vital signs. The dehiscence needs to be surgically closed, so the nurse should never try to close it.

A client with renal dysfunction of acute onset comes to the emergency department complaining of fatigue, oliguria, and coffee-colored urine. When obtaining the client's history to check for significant findings, the nurse should ask about: 1. chronic, excessive acetaminophen use. 2. recent streptococcal infection. 3. childhood asthma. 4. family history of pernicious anemia.

2. recent streptococcal infection. A skin or upper respiratory infection of streptococcal origin may lead to acute glomerulonephritis. Other infections that may be linked to renal dysfunction include infectious mononucleosis, mumps, measles, and cytomegalovirus. Chronic, excessive acetaminophen use isn't nephrotoxic, although it may be hepatotoxic. Childhood asthma and a family history of pernicious anemia aren't significant history findings for a client with renal dysfunction.

The nurse is teaching a client with malabsorption syndrome about the disorder and its treatment. The client asks which part of the GI tract absorbs food. The nurse tells the client that products of digestion are absorbed mainly in the: 1. stomach. 2. small intestine. 3. large intestine. 4. rectum.

2. small intestine. The small intestine absorbs products of digestion, completes food digestion, and secretes hormones that help control the secretion of bile, pancreatic juice, and intestinal secretions. The stomach stores, mixes, and liquefies the food bolus into chyme and controls food passage into the duodenum; it doesn't absorb products of digestion. Although the large intestine completes the absorption of water, chloride, and sodium, it plays no part in absorbing food. The rectum is the portion of the large intestine that forms and expels feces from the body; its functions don't include absorption.

A client receiving total parental nutrition is prescribed a 24-hour urine test. When initiating a 24-hour urine specimen, the collection time should: 1. start with the first voiding. 2. start after a known voiding. 3. always be with the first morning urine. 4. always be the evening's last void as the last sample.

2. start after a known voiding. When initiating a 24-hour urine specimen, have the client void, then start the timing. The collection should start on an empty bladder. The exact time the test starts isn't important but it's commonly started in the morning.

A client with acute liver failure exhibits confusion, a declining level of consciousness, and slowed respirations. The nurse finds him very difficult to arouse. The diagnostic information which best explains the client's behavior is: 1. elevated liver enzymes and low serum protein level. 2. subnormal serum glucose and elevated serum ammonia levels. 3. subnormal clotting factors and platelet count. 4. elevated blood urea nitrogen and creatinine levels and hyperglycemia.

2. subnormal serum glucose and elevated serum ammonia levels. In acute liver failure, serum ammonia levels increase because the liver can't adequately detoxify the ammonia produced in the GI tract. In addition, serum glucose levels decline because the liver isn't capable of releasing stored glucose. Both elevated serum ammonia and subnormal serum glucose levels depress the level of a client's consciousness. The other diagnostic values aren't as directly related to the client's level of consciousness.

A client with peptic ulcer disease is prescribed aluminum-magnesium complex (Riopan). When teaching about this antacid preparation, the nurse should instruct the client to take it with: 1. fruit juice. 2. water. 3. a food rich in vitamin C. 4. a food rich in vitamin D.

2. water. Water helps transport an antacid to the stomach. The client shouldn't take an antacid with fruit juice or a food rich in vitamin C or D because the antacid may impair absorption of important nutrients in the juice or food.

A client with an indwelling urinary catheter is suspected of having a urinary tract infection. The nurse should collect a urine specimen for culture and sensitivity by: 1. disconnecting the tubing from the urinary catheter and letting the urine flow into a sterile container. 2. wiping the self-sealing aspiration port with antiseptic solution and aspirating urine with a sterile needle. 3. draining urine from the drainage bag into a sterile container. 4. clamping the tubing for 60 minutes and inserting a sterile needle into the tubing above the clamp to aspirate urine.

2. wiping the self-sealing aspiration port with antiseptic solution and aspirating urine with a sterile needle. Most catheters have a self-sealing port for obtaining a urine specimen. Antiseptic solution is used to reduce the risk of introducing microorganisms into the catheter. Tubing shouldn't be disconnected from the urinary catheter. Any break in the closed urine drainage system may allow the entry of microorganisms. Urine in urine drainage bags may not be fresh and may contain bacteria, giving false test results. When there is no urine in the tubing, the catheter may be clamped for no more than 30 minutes to allow urine to collect.

What is the typical dosage and concentration of IV potassium?

20 mEq in 1000 mL

The nurse is preparing to teach a client with a peptic ulcer about the diet that should be followed after discharge. The nurse should explain that the diet will most likely consist of which of the following? 1. Bland foods. 2. High-protein foods. 3. Any foods that are tolerated. 4. Large amounts of milk.

3. Any foods that are tolerated. Diet therapy for ulcer disease is a controversial issue. There is no scientific evidence that diet therapy promotes healing. Most clients are instructed to follow a diet that they can tolerate. There is no need for the client to ingest only a bland or high-protein diet. Milk may be included in the diet, but it is not recommended in excessive amounts.

How many lobes does that right lung have?

3

The nurse has been assigned to provide care for four clients at the beginning of the day shift. In what order should the nurse assess these clients? 1. The client awaiting hiatal hernia repair at 11 am. 2. A client with suspected gastric cancer who is on nothing-by-mouth (NPO) status for tests. 3. A client with peptic ulcer disease experiencing sudden onset of acute stomach pain. 4. A client who is requesting pain medication 2 days after surgery to repair a fractured jaw.

3, 4, 2, 1 The client with peptic ulcer disease who is experiencing a sudden onset of acute stomach pain should be assessed first by the nurse. The sudden onset of stomach pain could be indicative of a perforated ulcer, which would require immediate medical attention. It is also important for the nurse to thoroughly assess the nature of the client's pain. The client with the fractured jaw is experiencing pain and should be assessed next. The nurse should then assess the client who is NPO for tests to ensure NPO status and comfort. Last, the nurse can assess the client before surgery.

The nurse has been assigned to provide care for four clients at the beginning of the day shift. In what order should the nurse assess these clients? 1. The client awaiting hiatal hernia repair at 11 am. 2. A client with suspected gastric cancer who is on nothing-by-mouth (NPO) status for tests. 3. A client with peptic ulcer disease experiencing sudden onset of acute stomach pain. 4. A client who is requesting pain medication 2 days after surgery to repair a fractured jaw.

3, 4, 2, 1 The client with peptic ulcer disease who is experiencing a sudden onset of acute stomach pain should be assessed first by the nurse. The sudden onset of stomach pain could be indicative of a perforated ulcer, which would require immediate medical attention. It is also important for the nurse to thoroughly assess the nature of the client's pain. The client with the fractured jaw is experiencing pain and should be assessed next. The nurse should then assess the client who is NPO for tests to ensure NPO status and comfort. Last, the nurse can assess the client before surgery.

When obtaining a nursing history on a client with a suspected gastric ulcer, which signs and symptoms should the nurse expect to assess? Select all that apply. 1. Epigastric pain at night. 2. Relief of epigastric pain after eating. 3. Vomiting. 4. Weight loss. 5. Melena.

3, 4, 5. Vomiting and weight loss are common with gastric ulcers. The client may also have blood in the stools (melena) from gastric bleeding. Clients with a gastric ulcer are most likely to complain of a burning epigastric pain that occurs about 1 hour after eating. Eating frequently aggravates the pain. Clients with duodenal ulcers are more likely to complain about pain that occurs during the night and is frequently relieved by eating.

A client with a peptic ulcer is about to begin a therapeutic regimen that includes a bland diet, antacids, and famotidine (Pepcid). Before the client is discharged, the nurse should provide which instruction? 1. "Eat three balanced meals every day." 2. "Stop taking the drugs when your symptoms subside." 3. "Avoid aspirin and products that contain aspirin." 4. "Increase your intake of fluids containing caffeine."

3. "Avoid aspirin and products that contain aspirin." Aspirin is a gastric irritant and should be avoided by clients with peptic ulcer to prevent further erosion of the stomach lining. The client should eat small, frequent meals rather than three large ones. Antacids and ranitidine prevent acid accumulation in the stomach; they should be taken even after symptoms subside. Caffeine should be avoided because it increases acid production in the stomach.

A client with benign prostatic hypertrophy (BPH) asks a nurse what can be done to manage the symptoms. The correct response by the nurse would be: 1. "There is really nothing that can be done except surgical removal of the prostate." 2. "Increase the amount of the zinc in your diet." 3. "Herbal therapy such as saw palmetto may improve urinary flow." 4. "Vitamin E will reduce the size of the prostate."

3. "Herbal therapy such as saw palmetto may improve urinary flow." Dietary changes such as decreasing fats and increasing fruits may reduce symptoms. Reducing meat and dairy intake decreases hormone stimulation. Zinc has been shown to delay the development of BPH, but not reverse or manage the symptoms. Saw palmetto has been shown to improve urine flow bu blocking the ability of dihydrotestosterone (DHT) to stimulate prostate cell growth. Vitamin E has not been shown to reduce the size.

A client with a history of alcohol abuse was admitted with bleeding esophageal varices. After several days of treatment, the client is ready for discharge. The nurse enters the client's room to review discharge instructions with the client when he tells the nurse that he wants help quitting drinking. How should the nurse respond? 1. "Let me finish reviewing your discharge instructions, then we can discuss your concerns." 2. "I'll tell your family so they can make arrangements for you to enter an alcohol rehabilitation center." 3. "I'll notify your physician and call the social worker so she can discuss treatment options with you." 4. "I hope it's not too late; you've already done a lot of damage to your liver."

3. "I'll notify your physician and call the social worker so she can discuss treatment options with you." The nurse should notify the physician and call the social worker so the social worker can discuss treatment options with the client. The social worker may be able to arrange in-patient treatment for the patient immediately after discharge if the client wishes. Option 1 minimizes the client's concerns; option 2 breeches client confidentiality; and option 4 is judgmental.

The nurse is doing preoperative teaching with the client who is about to undergo creation of a Kock pouch. The nurse interprets that the client has the best understanding of the nature of the surgery if the client makes which statement? 1. "I will be able to pass stool by the rectum eventually." 2. "The drainage from this type of ostomy will be formed." 3. "I will need to drain the pouch regularly with a catheter." 4. "I will need to wear a drainage bag for the rest of my life."

3. "I will need to drain the pouch regularly with a catheter." A Kock pouch is a continent ileostomy. As the ileostomy begins to function , the client drains it every 3 to 4 hours and then decreases the draining to about three times a day, or as needed when full. The client does not need to wear a drainage bag but should wear an absorbent dressing to absorb mucous drainage from the stoma. Ileostomy drainage is liquid. The client would be able to pass stool only from the rectum if an ileal-anal pouch or anastomosis were created. This type of operation is a two-stage procedure.

The nurse has taught the client about an upcoming endoscopic retrograde cholangiopancreatography procedure. The nurse determines that the client needs further information if the client makes which statement? 1. "I know I must sign the consent form." 2. "I hope the throat spray keeps me from gagging." 3. "I'm glad I don't have to lie still for this procedure." 4. "I'm glad some IV medication will be given to relax me."

3. "I'm glad I don't have to lie still for this procedure." The client does have to lie still for endoscopic retrograde cholangiopancreatography (ERCP), which takes about 1 hour to perform. The client also has to sign a consent form. Intravenous sedation is given to relax the client, and an anesthetic spray is used to help keep the client from gagging as the endoscope is passed.

A client who has been treated for chronic renal failure (CRF) is ready for discharge. The nurse should reinforce which dietary instruction? 1. "Be sure to eat meat at every meal." 2. "Eat plenty of bananas." 3. "Increase your carbohydrate intake." 4. "Drink plenty of fluids, and use a salt substitute."

3. "Increase your carbohydrate intake." Extra carbohydrates are needed to prevent protein catabolism. In a client with CRF, unrestricted intake of sodium, protein, potassium, and fluid may lead to a dangerous accumulation of electrolytes and protein metabolic products, such as amino acids and ammonia. Therefore, the client must limit intake of sodium; meat, which is high in protein; bananas, which are high in potassium; and fluid, because the failing kidneys can't secrete adequate urine. Salt substitutes are high in potassium and should be avoided.

A client with type 1 diabetes mellitus has a highly elevated glycosylated hemoglobin (Hb) test result. In discussing the result with the client, the nurse would be most accurate in stating: 1. "The test needs to be repeated following a 12-hour fast." 2. "It looks like you aren't following the prescribed diabetic diet." 3. "It tells us about your sugar control for the last 3 months." 4. "Your insulin regimen needs to be altered significantly."

3. "It tells us about your sugar control for the last 3 months." The glycosylated Hb test provides an objective measure of glycemic control over a 3-month period. The test helps identify trends or practices that impair glycemic control, and it doesn't require a fasting period before blood is drawn. The nurse can't conclude that the result occurs from poor dietary management or inadequate insulin coverage.

The nurse is providing dietary instructions to a client with a history of pancreatitis. Which instruction is correct? 1. "Maintain a high-fat diet and drink at least 3 L of fluid a day." 2. "Maintain a high-sodium, high-calorie diet." 3. "Maintain a high-carbohydrate, low-fat diet." 4. "Maintain a high-fat, high-carbohydrate diet."

3. "Maintain a high-carbohydrate, low-fat diet." A client with a history of pancreatitis should avoid foods and beverages that stimulate the pancreas, such as fatty foods, caffeine, and gas-forming foods; should avoid eating large meals; and should eat plenty of carbohydrates, which are easily metabolized. Therefore, the only correct instruction is to maintain a high-carbohydrate, low-fat diet. An increased sodium or fluid intake isn't necessary because chronic pancreatitis isn't associated with hyponatremia or fluid loss.

A client with type 1 diabetes mellitus calls the nurse to report recurrent episodes of hypoglycemia with exercising. Which statement by the client indicates an inadequate understanding of the peak action of NPH insulin and exercise? 1. "The best time for me to exercise is after I eat." 2. "The best time for me to exercise is after breakfast." 3. "The best time for me to exercise is mid- to late afternoon." 4. "The best time for me to exercise is after my morning snack."

3. "The best time for me to exercise is mid- to late afternoon." Rationale: A hypoglycemic reaction may occur in response to increased exercise. Clients should avoid exercise during the peak time of insulin. NPH insulin peaks at 4 to 12 hours; therefore, afternoon exercise takes place during the peak of the medication. Options 1, 2, and 4 do not address peak action times.

Sources of Vitamin B9/Folate

beans, legumes, citrus fruit, whole grains, dark green leafy vegetables, poultry, pork, shellfish, liver

A client with acute diarrhea is prescribed paregoric, 5 ml by mouth up to four times daily, until acute diarrhea subsides. The client asks the nurse how soon the medication will start to work after the first dose is taken. How should the nurse respond? 1. "Within 5 minutes" 2. "Within 20 minutes" 3. "Within 1 hour" 4. "Within 2 to 4 hours"

3. "Within 1 hour" Paregoric starts to act within 1 hour after administration. Onset of action isn't as rapid as 5 or 20 minutes or as slow as 2 to 4 hours.

The nurse is administering medications to a client diagnosed with hepatitis B. When the nurse hands the client his medications, the client says, "I would rather not take that pill or any others. I know there is no cure for hepatitis B." The nurse recognizes that the client is expressing feelings of hopelessness about his diagnosis. Which response by the nurse respects the client's rights concerning medication administration? 1. "You seem frustrated; however, you still must take this medication, it will help you." 2. "Legally, I have to give you this medication." 3. "You have the right to refuse any medication. Would you like to discuss your feelings about this disease." 4. "I will document that you are noncompliant with your treatment regimen."

3. "You have the right to refuse any medication. Would you like to discuss your feelings about this disease." Option 3 is the correct response. The client has the right to refuse any medical treatment, regardless of the consequences. The client is displaying hopelessness over the diagnosis; therefore, the nurse should encourage the client to discuss these feelings using therapeutic conversation. Options 1, 2, and 4 are inappropriate responses that are judgmental and dismiss the client's feelings.

The nurse is caring for a client with type 1 diabetes mellitus who exhibits confusion, light-headedness, and aberrant behavior. The client is still conscious. The nurse should first administer: 1. I.M. or subcutaneous glucagon. 2. I.V. bolus of dextrose 50%. 3. 15 to 20 g of a fast-acting carbohydrate such as orange juice. 4. 10 U of fast-acting insulin.

3. 15 to 20 g of a fast-acting carbohydrate such as orange juice. This client is having a hypoglycemic episode. Because the client is conscious, the nurse should first administer a fast-acting carbohydrate, such as orange juice, hard candy, or honey. If the client has lost consciousness, the nurse should administer either I.M. or subcutaneous glucagon or an I.V. bolus of dextrose 50%. The nurse shouldn't administer insulin to a client who's hypoglycemic; this action will further compromise the client's condition.

A client who has been treated for diverticulitis is being discharged on oral propantheline bromide (Pro-Banthine). The nurse should instruct the client to take the drug at which times? 1. With meals and at bedtime 2. Immediately before meals and at bedtime 3. 30 minutes before meals and at bedtime 4. 1 hour after meals and at bedtime

3. 30 minutes before meals and at bedtime Propantheline bromide is used to reduce secretions and spasms of the GI tract in clients with diverticulitis, a condition characterized by bowel inflammation and colonic irritability and spasticity. The nurse should instruct the client to take the drug 30 minutes before meals and at bedtime to reduce GI motility, thus relieving spasticity. Taking it with a meal, immediately before a meal, or 1 hour after a meal would interfere with the drug's action and absorption, thereby reducing its effectiveness.

The nurse administered neutral protamine Hagedorn (NPH) insulin to a diabetic client at 7 a.m. At what time would the nurse expect the client to be most at risk for a hypoglycemic reaction? 1. 10 a.m. 2. Noon 3. 4 p.m. 4. 10 p.m.

3. 4 p.m. NPH is an intermediate-acting insulin that peaks 8 to 12 hours after administration. Because the nurse administered NPH insulin at 7 a.m., the client is at greatest risk for hypoglycemia from 3 to 7 p.m.

The registered nurse and nursing assistant are caring for a group of clients. Which client's care can safely be delegated to the nursing assistant? 1. A 35-year-old client who underwent surgery 12 hours ago and has a suprapubic catheter in place that is draining burgundy colored urine 2. A 63-year-old client with uncontrolled diabetes mellitus who underwent radical suprapubic prostatectomy 1 day ago and has an indwelling urinary catheter draining yellow urine with clots 3. A 45-year-old client diagnosed with renal calculi who must ambulate four times daily and drink plenty of fluids. 4. A 19-year-old client who requires neurological assessment every four hours after sustaining a spinal cord injury in a motor vehicle accident that left him with paraplegia

3. A 45-year-old client diagnosed with renal calculi who must ambulate four times daily and drink plenty of fluids. The care of the client in option 3 can safely be delegated to the nursing assistant. The client in option 1 had surgery 12 hours ago; therefore, the registered nurse should care for the client because the client requires close assessment. The client in option 2 also requires careful assessment by the registered nurse because the client's diabetes mellitus is uncontrolled. In addition, the registered nurse should care for the client in option 4 because the client requires neurological assessment, which isn't within the scope of practice for the nursing assistant.

After checking the client's chart for possible contraindications, the nurse is administering meperidine (Demerol), 50 mg I.M., to a client with pain after an appendectomy. Which type of drug therapy would contraindicate the use of meperidine? 1. An antibiotic 2. An antiemetic 3. A monoamine oxidase (MAO) inhibitor 4. A loop diuretic

3. A monoamine oxidase (MAO) inhibitor MAO inhibitors increase the effects of meperidine and can cause rigidity, hypotension, and excitation. The client shouldn't receive meperidine within 14 days after administration of an MAO inhibitor. Antibiotics, antiemetics, and loop diuretics don't cause significant drug interactions when administered concurrently with meperidine.

The nurse is preparing to teach a client with a peptic ulcer about the diet that should be followed after discharge. The nurse should explain that the diet will most likely consist of which of the following? 1. Bland foods. 2. High-protein foods. 3. Any foods that are tolerated. 4. Large amounts of milk.

3. Any foods that are tolerated. Diet therapy for ulcer disease is a controversial issue. There is no scientific evidence that diet therapy promotes healing. Most clients are instructed to follow a diet that they can tolerate. There is no need for the client to ingest only a bland or high-protein diet. Milk may be included in the diet, but it is not recommended in excessive amounts.

Laboratory studies indicate a client's blood glucose level is 185 mg/dl. Two hours have passed since the client ate breakfast. Which test would yield the most conclusive diagnostic information about the client's glucose utilization? 1. A fasting blood glucose test 2. A 6-hour glucose tolerance test 3. A test of serum glycosylated hemoglobin (Hb A1c) 4. A test for urine ketones

3. A test of serum glycosylated hemoglobin (Hb A1c) Hb A1c is the most reliable indicator of glucose utilization because it reflects blood glucose levels for the prior 3 months. While a fasting blood glucose test and 6-hour glucose tolerance test yield information about a client's utilization of glucose, the results are influenced by other factors such as whether the client recently ate breakfast. Presence of ketones in the urine also provides information about glucose utilization but is limited in its diagnostic significance.

Paralytic ileus is described by the nurse as: 1. Edema of the intestinal mucosa. 2. Acute dilation of the colon. 3. Absent, diminished, or uncoordinated autonomic stimulation of peristalsis. 4. High-pitched, tinkling bowel sounds over the area of obstruction.

3. Absent, diminished, or uncoordinated autonomic stimulation of peristalsis.

The nurse correctly identifies a urine sample with a pH of 4.3 as being which type of solution? 1. Neutral 2. Alkaline 3. Acidic 4. Basic

3. Acidic Normal urine pH is 4.5 to 8.0; a value of 4.3 reveals acidic urine pH. A pH above 7.0 is considered an alkaline or basic solution. A pH of 7.0 is considered neutral.

A nurse is caring for a client admitted to the emergency department with diabetic ketoacidosis (DKA). In the acute phase, the priority nursing action is to prepare to: 1. Correct the acidosis. 2. Administer 5% dextrose intravenously. 3. Administer regular insulin intravenously. 4. Apply a monitor for an electrocardiogram.

3. Administer regular insulin intravenously. Rationale: Lack (absolute or relative) of insulin is the primary cause of DKA. Treatment consists of insulin administration (regular insulin), intravenous fluid administration (normal saline initially), and potassium replacement, followed by correcting acidosis. Applying an electrocardiogram monitor is not a priority action.

A client with type 1 diabetes presents with a decreased level of consciousness and a fingerstick glucose level of 39 mg/dl. Her family reports that she has been skipping meals in an effort to lose weight. Which nursing intervention is most appropriate? 1. Inserting a feeding tube and providing tube feedings 2. Administering a 500-ml bolus of normal saline solution 3. Administering 1 ampule of 50% dextrose solution, per physician's order 4. Observing the client for 1 hour, then rechecking the fingerstick glucose level

3. Administering 1 ampule of 50% dextrose solution, per physician's order The nurse should administer 50% dextrose solution to restore the client's physiological integrity. Feeding through a feeding tube isn't appropriate for this client. A bolus of normal saline solution doesn't provide the client with the much needed glucose. Observing the client for 1 hour delays treatment. The client's blood glucose level could drop further during this time, placing her at risk for irreversible brain damage.

A client with a history of alcohol abuse comes to the emergency department and complains of abdominal pain. Laboratory studies help confirm a diagnosis of acute pancreatitis. The client's vital signs are stable, but the client's pain is worsening and radiating to his back. Which intervention takes priority for this client? 1. Placing the client in a semi-Fowler's position 2. Maintaining nothing-by-mouth status 3. Administering morphine I.V. as prescribed 4. Providing mouth care

3. Administering morphine I.V. as prescribed The nurse should address the client's pain issues first by administering morphine I.V. as prescribed. The other interventions don't take priority over addressing the client's pain issues.

A client has just had a hemorrhoidectomy. What nursing intervention is appropriate for this client? 1. Instruct the client to limit fluid intake to avoid urinary retention. 2. Instruct the client to eat low-fiber foods to decrease the bulk of the stool. 3. Apply and maintain ice packs over the dressing until the packing is removed. 4. Help the client to a Fowler's position to place pressure on the rectal area and decrease bleeding.

3. Apply and maintain ice packs over the dressing until the packing is removed. Nursing interventions after a hemorrhoidectomy are aimed at management of pain and avoidance of bleeding. An ice pack will increase comfort and decrease bleeding. Options 1, 2, and 4 are incorrect interventions.

The nurse is reviewing the record of a client with a diagnosis of cirrhosis and notes that there is documentation of the presence of asterixis. How should the nurse assess for its presence? 1. Dorsiflex the client's foot. 2. Measure the abdominal girth. 3. Ask the client to extend the arms. 4. Instruct the client to lean forward.

3. Ask the client to extend the arms. Asterixis is irregular flapping movements of the fingers and wrists when the hands and arms are outstretched, with the palms down , wrists bent up, and fingers spread. Asterixis is the most common and reliable sign that hepatic encephalopathy is developing. Options 1, 2, and 4 are incorrect.

The home care nurse is making a visit with a client who had a double barrel colostomy created after bowel surgery. While the nurse is changing the client's appliance there is a knock on the door. The nurse answers the door. The client's next-door neighbor wants to visit with the client. Which intervention by the nurse is most appropriate? 1. Allow the neighbor to enter. 2. Have the neighbor wait in the next room until the appliance is applied. 3. Ask the neighbor to come back in 20 minutes. 4. Suggest that the neighbor come in and learn how to apply the appliance in case the client needs help.

3. Ask the neighbor to come back in 20 minutes. The home care nurse should ask the neighbor to come back in 20 minutes when the visit is over. Client privacy is a priority even in the home care setting. Options 1 and 2 violate client privacy and confidentiality. Option 4 is inappropriate because the client didn't request help from the neighbor.

A client who underwent abdominal surgery who has a nasogastric (NG) tube in place begins to complain of abdominal pain that he describes as "feeling full and uncomfortable." Which assessment should the nurse perform first? 1. Measure abdominal girth. 2. Auscultate bowel sounds. 3. Assess patency of the NG tube. 4. Assess vital signs.

3. Assess patency of the NG tube. When an NG tube is no longer patent, stomach contents collect in the stomach giving the client a sensation of fullness. The nurse should begin by assessing patency of the NG tube. The nurse can measure abdominal girth, auscultate bowels, and assess vital signs, but she should check NG tube patency first to help relieve the client's discomfort.

After undergoing transurethral resection of the prostate to treat benign prostatic hyperplasia, a client returns to the room with continuous bladder irrigation. On the first day after surgery, the client reports bladder pain. What should the nurse do first? 1. Increase the I.V. flow rate. 2. Notify the physician immediately. 3. Assess the irrigation catheter for patency and drainage. 4. Administer morphine sulfate, 2 mg I.V., as prescribed.

3. Assess the irrigation catheter for patency and drainage. Although postoperative pain is expected, the nurse should make sure that other factors, such as an obstructed irrigation catheter, aren't the cause of the pain. After assessing catheter patency, the nurse should administer an analgesic, such as morphine sulfate, as prescribed. Increasing the I.V. flow rate may worsen the pain. Notifying the physician isn't necessary unless the pain is severe or unrelieved by the prescribed medication.

A client is to take one daily dose of ranitidine (Zantac) at home to treat her peptic ulcer. The client understands proper drug administration of ranitidine when she says that she will take the drug at which of the following times? 1. Before meals. 2. With meals. 3. At bedtime. 4. When pain occurs.

3. At bedtime. Ranitidine blocks secretion of hydrochloric acid. Clients who take only one daily dose of ranitidine are usually advised to take it at bedtime to inhibit nocturnal secretion of acid. Clients who take the drug twice a day are advised to take it in the morning and at bedtime. It is not necessary to take the drug before meals. The client should take the drug regularly, not just when pain occurs.

A client is to take one daily dose of ranitidine (Zantac) at home to treat her peptic ulcer. The client understands proper drug administration of ranitidine when she says that she will take the drug at which of the following times? 1. Before meals. 2. With meals. 3. At bedtime. 4. When pain occurs.

3. At bedtime. Ranitidine blocks secretion of hydrochloric acid. Clients who take only one daily dose of ranitidine are usually advised to take it at bedtime to inhibit nocturnal secretion of acid. Clients who take the drug twice a day are advised to take it in the morning and at bedtime. It is not necessary to take the drug before meals. The client should take the drug regularly, not just when pain occurs.

A client with acute pyelonephritis receives a prescription for co-trimoxazole (Septra) P.O. twice daily for 10 days. Which finding best demonstrates that the client has followed the prescribed regimen? 1. Urine output increases to 2,000 ml/day. 2. Flank and abdominal discomfort decrease. 3. Bacteria are absent on urine culture. 4. The red blood cell (RBC) count is normal.

3. Bacteria are absent on urine culture.Co-trimoxazole is a sulfonamide antibiotic used to treat urinary tract infections. Therefore, absence of bacteria on urine culture indicates that the drug has achieved its desired effect. Although flank pain may decrease as the infection resolves, this isn't a reliable indicator of the drug's effectiveness. Co-trimoxazole doesn't affect urine output or the RBC count.

A 71-year-old client is admitted with hyperosmolar hyperglycemic nonketotic syndrome (HHNS). Which laboratory finding would the nurse expect in this client? 1. Arterial pH 7.25 2. Plasma bicarbonate 12 mEq/L 3. Blood glucose level 1,300 mg/dl 4. Blood urea nitrogen 15 mg/dl

3. Blood glucose level 1,300 mg/dl Hyperosmolar hyperglycemic nonketotic syndrome (HHNS) occurs most frequently in older clients. It can occur in clients with either type 1 or type 2 diabetes mellitus but occurs most frequently in those with type 2. The blood glucose level rises to above 600 mg/dl in response to illness or infection. As the blood glucose level rises, the body attempts to rid itself of the excess glucose by producing urine. Initially the client produces large quantities of urine, if fluid intake isn't increased at this time the client becomes dehydrated causing blood urea nitrogen levels to rise. Arterial pH and plasma bicarbonate levels typically remain within normal limits.

A client with chronic renal failure (CRF) is admitted to the urology unit. Which diagnostic test results are consistent with CRF? 1. Increased pH with decreased hydrogen ions 2. Increased serum levels of potassium, magnesium, and calcium 3. Blood urea nitrogen (BUN) 100 mg/dl and serum creatinine 6.5 mg/dl 4. Uric acid analysis 3.5 mg/dl and phenolsulfonphthalein (PSP) excretion 75%

3. Blood urea nitrogen (BUN) 100 mg/dl and serum creatinine 6.5 mg/dl The normal BUN level ranges 8 to 23 mg/dl; the normal serum creatinine level ranges from 0.7 to 1.5 mg/dl. The test results in option 3 are abnormally elevated, reflecting CRF and the kidneys' decreased ability to remove nonprotein nitrogen waste from the blood. CRF causes decreased pH and increased hydrogen ions — not vice versa. CRF also increases serum levels of potassium, magnesium, and phosphorous, and decreases serum levels of calcium. A uric acid analysis of 3.5 mg/dl falls within the normal range of 2.7 to 7.7 mg/dl; PSP excretion of 75% also falls within the normal range of 60% to 75%.

The nursing staff has just been trained how to use and care for a new blood glucose monitor. Which nursing intervention demonstrates proper use of a blood glucose monitor? 1. Ungloving the hands when removing the test strip 2. Smearing the drop of blood onto the reagent pad 3. Calibrating the machine after installing a new battery 4. Starting the timer on the machine while gathering supplies

3. Calibrating the machine after installing a new battery To obtain accurate readings, the nurse should calibrate the machine whenever a new battery is installed. To adhere to standard precautions and prevent contact with blood, the nurse's hands should remain gloved throughout blood glucose testing. The nurse should drop the blood — not smear it — on the reagent pad because smearing can cause an inaccurate reading. To help ensure accurate results, the nurse shouldn't start the timer before the blood sample is collected.

A client has a colostomy following surgery for colon cancer. Which assessment finding would require immediate action by a nurse? 1. Stoma raised 2 cm from the abdominal wall. 2. Bleeding was noted from the stoma during care. 3. Complaint of pain with light touch. 4. Stoma was pinkish-red in color.

3. Complaint of pain with light touch. The stoma will be 1 to 2 cm above the skin. Some bleeding is normal. There are no nerves in the mucous membranes, there should be no pain when touching. Pain would possibly indicate a problem internally.

A client develops decreased renal function and requires a change in antibiotic dosage. On which factor would the physician base the dosage change? 1. GI absorption rate 2. Therapeutic index 3. Creatinine clearance 4. Liver function studies

3. Creatinine clearance The physician orders tests for creatinine clearance to gauge the kidney's glomerular filtration rate; this is important because most drugs are excreted at least partially by the kidneys. The GI absorption rate, therapeutic index, and liver function studies don't help determine dosage change in a client with decreased renal function.

A client with long-standing type 1 diabetes mellitus is admitted to the hospital with unstable angina pectoris. After the client's condition stabilizes, the nurse evaluates the diabetes management regimen. The nurse learns that the client sees the physician every 4 weeks, injects insulin after breakfast and dinner, and measures blood glucose before breakfast and at bedtime. Consequently, the nurse should formulate a nursing diagnosis of: 1. Impaired adjustment. 2. Defensive coping. 3. Deficient knowledge (treatment regimen). 4. Health-seeking behaviors (diabetes control).

3. Deficient knowledge (treatment regimen). The client should inject insulin before, not after, breakfast and dinner — 30 minutes before breakfast for the a.m. dose and 30 minutes before dinner for the p.m. dose. Therefore, the client has a knowledge deficit regarding when to administer insulin. By taking insulin, measuring blood glucose levels, and seeing the physician regularly, the client has demonstrated the ability and willingness to modify the lifestyle as needed to manage the disease. This eliminates the nursing diagnoses of Impaired adjustment and Defensive coping. Because the nurse, not the client, questioned the client's health practices related to diabetes management, the nursing diagnosis of Health-seeking behaviors isn't warranted.

What should be the first nursing action if an NG tube is not draining? 1. Irrigate the tube. 2. Reposition the client. 3. Determine tube placement. 4. Remove the tube and reinsert.

3. Determine tube placement. Placement must be confirmed before irrigating. The lack of drainage may mean the tube is no longer in the correct location. Placement must be determined before removal.

Which factor can cause hepatitis A? 1. Contact with infected blood 2. Blood transfusions with infected blood 3. Eating contaminated shellfish 4. Sexual contact with an infected person

3. Eating contaminated shellfish Hepatitis A can be caused by consuming contaminated water, milk, or food — especially shellfish from contaminated water. Hepatitis B is caused by blood and sexual contact with an infected person. Hepatitis C is usually caused by contact with infected blood, including receiving blood transfusions.

A client complains of not having had a bowel movement since being admitted 2 days previously for multiple fractures of both lower legs. The client is on bedrest and skeletal traction. Which intervention would be the most appropriate nursing action? 1. Administer an enema. 2. Put the client on the bedpan every 2 hours. 3. Ensure maximum fluid intake (3,000mL/day) 4. Perform range-of-motion exercises to all extremities.

3. Ensure maximum fluid intake (3,000mL/day) Enema may not be necessary and requires a doctor's order. The bedpan requires a great deal of exertion when the client is not expressing the urge to defecate. The best early intervention is to increase fluid intake b/ constipation is common when activity is decreased or usual routines have been interrupted. It would be impossible to exercise extremities that have unhealed fractures.

Which diagnostic test would be used first to evaluate a client with upper GI bleeding? 1. Endoscopy 2. Upper GI series 3. Hemoglobin (Hb) levels and hematocrit (HCT) 4. Arteriography

3. Hemoglobin (Hb) levels and hematocrit (HCT) Hemoglobin and hematocrit are typically performed first in clients with upper GI bleeding to evaluate the extent of blood loss. Endoscopy is then performed to directly visualize the upper GI tract and locate the source of bleeding. An upper GI series, or barium study, usually isn't the diagnostic method of choice, especially in a client with acute active bleeding who's vomiting and unstable. An upper GI series is also less accurate than endoscopy. Although an upper GI series might confirm the presence of a lesion, it wouldn't necessarily reveal whether the lesion is bleeding. Arteriography is an invasive study associated with life-threatening complications and wouldn't be used for an initial evaluation.

For a client with hyperglycemia, which assessment finding best supports a nursing diagnosis of Deficient fluid volume? 1. Cool, clammy skin 2. Distended neck veins 3. Increased urine osmolarity 4. Decreased serum sodium level

3. Increased urine osmolarity In hyperglycemia, urine osmolarity (the measurement of dissolved particles in the urine) increases as glucose particles move into the urine. The client experiences glucosuria and polyuria, losing body fluids and experiencing deficient fluid volume. Cool, clammy skin; distended neck veins; and a decreased serum sodium level are signs of fluid volume excess, the opposite imbalance.

A client is admitted with increased ascites related to cirrhosis. Which nursing diagnosis should receive top priority? 1. Fatigue 2. Excess fluid volume 3. Ineffective breathing pattern 4. Imbalanced nutrition: Less than body requirements

3. Ineffective breathing pattern In ascites, accumulation of large amounts of fluid causes extreme abdominal distention, which may put pressure on the diaphragm and interfere with respiration. If uncorrected, this may lead to atelectasis or pneumonia. Although fluid volume excess is present, the diagnosis Ineffective breathing pattern takes precedence because it can lead more quickly to life-threatening consequences. The nurse can deal with fatigue and altered nutrition after the client establishes and maintains an effective breathing pattern.

The nurse explains to a client that she will administer his first insulin dose in his abdomen. How does absorption at the abdominal site compare to absorption at other sites? 1. Insulin is absorbed more slowly at abdominal injection sites than at other sites. 2. Insulin is absorbed rapidly regardless of the injection site. 3. Insulin is absorbed more rapidly at abdominal injection sites than at other sites. 4. Insulin is absorbed unpredictably at all injection sites.

3. Insulin is absorbed more rapidly at abdominal injection sites than at other sites. Subcutaneous insulin is absorbed most rapidly at abdominal injection sites, more slowly at sites on the arms, and slowest at sites on the anterior thigh. Absorption after injection of the buttocks is less predictable.

A client is admitted to a hospital with a diagnosis of diabetic ketoacidosis (DKA). The initial blood glucose level was 950 mg/ dL. A continuous intravenous infusion of regular insulin is initiated, along with intravenous rehydration with normal saline. The serum glucose level is now 240 mg/ dL. The nurse would next prepare to administer which of the following? 1. Ampule of 50% dextrose 2. NPH insulin subcutaneously 3. Intravenous fluids containing 5% dextrose 4. Phenytoin (Dilantin) for the prevention of seizures

3. Intravenous fluids containing 5% dextrose Rationale: During management of DKA, when the blood glucose level falls to 250 to 300 mg/ dL, the infusion rate is reduced and a 5% dextrose in 0.45% saline is added to maintain a blood glucose level of about 250 mg/ dL, or until the client recovers from ketosis. NPH insulin is not used to treat DKA. Fifty percent dextrose is used to treat hypoglycemia. Phenytoin (Dilantin) is not a usual treatment measure for DKA.

A client is brought to the emergency department in an unresponsive state, and a diagnosis of hyperglycemic hyperosmolar nonketotic syndrome is made. The nurse would immediately prepare to initiate which of the following anticipated physician's prescriptions? 1. Endotracheal intubation 2. 100 units of NPH insulin 3. Intravenous infusion of normal saline 4. Intravenous infusion of sodium bicarbonate

3. Intravenous infusion of normal saline Rationale: The primary goal of treatment in hyperglycemic hyperosmolar nonketotic syndrome (HHNS) is to rehydrate the client to restore fluid volume and to correct electrolyte deficiency. Intravenous fluid replacement is similar to that administered in diabetic ketoacidosis (DKA) and begins with IV infusion of normal saline. Regular insulin, not NPH insulin, would be administered. The use of sodium bicarbonate to correct acidosis is avoided because it can precipitate a further drop in serum potassium levels. Intubation and mechanical ventilation are not required to treat HHNS.

The nurse is caring for a client following a Billroth II procedure. Which postoperative prescription should the nurse question and verify? 1. Leg exercises 2. Early ambulation 3. Irrigating the nasogastric tube 4. Coughing and deep-breathing exercises

3. Irrigating the nasogastric tube In a Billroth II procedure, the proximal remnant of the stomach is anastomosed to the proximal jejunum. Patency of the nasogastric tube is critical for preventing the retention of gastric secretions. The nurse should never irrigate or reposition the gastric tube after gastric surgery, unless specifically prescribed by the physician. In this situation, the nurse should clarify the prescription. Options 1, 2, and 4 are appropriate postoperative interventions.

Which statement by the nurse correctly describes a double-barrel colostomy? 1. It is the least common type of colostomy, and it discharges liquid or unformed stool. 2. A single loop of the transverse colon is exteriorized and supported by a glass rod. There are two openings, a proximal loop and distal loop. 3. It has two stomas. A proximal loop discharges feces, and a distal loop discharges mucus. 4. It is most often permanent and is done to treat disorders of the sigmoid colon.

3. It has two stomas. A proximal loop discharges feces, and a distal loop discharges mucus. Number 1 is an example of an ascending colostomy. Number 2 is a transverse loop colostomy. Number 4 is a descending colostomy.

A diabetic client develops sinusitis and otitis media accompanied by a fever of 100.8° F (38.2° C). What effect may this have on his need for insulin? 1. It will have no effect. 2. It will decrease the need for insulin. 3. It will increase the need for insulin. 4. It will cause wide fluctuations in the need for insulin.

3. It will increase the need for insulin. Insulin requirements are increased by growth, pregnancy, increased food intake, stress, surgery, infection, illness, increased insulin antibodies, and some medications. Insulin requirements are decreased by hypothyroidism, decreased food intake, exercise, and some medications.

The nurse is caring for a client diagnosed with acute renal failure. The nurse notes on the intake and output record that the total urine output for the previous 24 hours was 95 ml. Urine output that's less than 100 ml in 24 hours is known as: 1. oliguria. 2. polyuria. 3. anuria. 4. hematuria.

3. anuria. Urine output less than 100 ml in 24 hours is called anuria. Urine output of less than 400 ml but more than 100 ml in 24 hours is called oliguria. Polyuria is excessive urination. Hematuria is the presence of blood in the urine.

After undergoing retropubic prostatectomy, a client returns to his room. The client is on nothing-by-mouth status and has an I.V. infusing in his right forearm at a rate of 100 ml/hour. The client also has an indwelling urinary catheter that is draining light pink urine. While assessing the client, the nurse notes that his urine output is red and has dropped to 15 ml and 10 ml for the last two consecutive hours. How can the nurse best explain this drop in urine output? 1. It's a normal finding caused by blood loss during surgery. 2. It's a normal finding associated with the client's nothing-by-mouth status. 3. It's an abnormal finding that requires further assessment. 4. It's an abnormal finding that will correct itself when the client ambulates.

3. It's an abnormal finding that requires further assessment. The drop in urine output to less than 30 ml/hour is abnormal and requires further assessment. The reduction in urine output may be caused by an obstruction in the urinary catheter tubing or deficient fluid volume from blood loss. The client's nothing-by-mouth status isn't the cause of the low urine output because the client is receiving I.V. fluid to compensate for the lack of oral intake. Ambulation promotes urination; however, the client should produce at least 30 ml of urine/hour.

For a client in the oliguric phase of acute renal failure (ARF), which nursing intervention is most important? 1. Encouraging coughing and deep breathing 2. Promoting carbohydrate intake 3. Limiting fluid intake 4. Providing pain-relief measures

3. Limiting fluid intake During the oliguric phase of ARF, urine output decreases markedly, possibly leading to fluid overload. Limiting oral and I.V. fluid intake can prevent fluid overload and its complications, such as heart failure and pulmonary edema. Encouraging coughing and deep breathing is important for clients with various respiratory disorders. Promoting carbohydrate intake may be helpful in ARF but doesn't take precedence over fluid limitation. Controlling pain isn't important because ARF rarely causes pain.

The nurse should know that the drainage is normal 4 days after a sigmoid colostomy when the material is: 1. Green liquid. 2. Solid formed. 3. Loose, bloody. 4. Semiformed.

3. Loose, bloody. Normal bowel function and soft-formed stool usually do not occur until around the seventh day following surgery.

A client with pancreatitis has been receiving total parenteral nutrition (TPN) for the past week. Which nursing intervention helps determine if TPN is providing adequate nutrition? 1. Accelerating the infusion if it falls behind schedule 2. Ensuring that the TPN tubing has an in-line filter 3. Monitoring the client's weight every day 4. Recording fluid intake and output

3. Monitoring the client's weight every day By weighing the client every day, the nurse helps the team evaluate the client's response to TPN. Maintenance of the current weight is one indicator of adequate nutrition; weight loss may indicate inadequate nutrition, whereas weight gain may indicate adequate nutrition or fluid retention. The nurse shouldn't accelerate a TPN infusion that has fallen behind because this can cause wide fluctuations in the blood glucose level. Use of an in-line filter on TPN tubing traps bacteria and particles but has no effect on nutrition. The nurse records intake and output to evaluate fluid replacement — not the nutritional adequacy of TPN.

The nurse is caring for a client with active upper GI bleeding. What is the appropriate diet for this client during the first 24 hours after admission? 1. Regular diet 2. Skim milk 3. Nothing by mouth 4. Clear liquids

3. Nothing by mouth Shock and bleeding must be controlled before oral intake, so the client should receive nothing by mouth. A regular diet is incorrect. When the bleeding is controlled, the diet is gradually increased, starting with ice chips and then clear liquids. Skim milk shouldn't be given because it increases gastric acid production, which could prolong bleeding. A liquid diet is the first diet offered after bleeding and shock are controlled.

The nurse is planning a group teaching session on the topic of urinary tract infection (UTI) prevention. Which point would the nurse want to include? 1. Limit fluid intake to reduce the need to urinate. 2. Take medication prescribed for a UTI until the symptoms subside. 3. Notify the physician if urinary urgency, burning, frequency, or difficulty occurs. 4. Wear only nylon underwear to reduce the chance of irritation.

3. Notify the physician if urinary urgency, burning, frequency, or difficulty occurs. Urgency, burning, frequency, and difficulty urinating are all common symptoms of a UTI. The client should notify his physician so that a microscopic urinalysis can be done and appropriate treatment can be initiated. The client should be instructed to drink 2 to 3 L of fluid per day to dilute the urine and reduce irritation of the bladder mucosa. The full amount of antibiotics prescribed for UTIs must be taken despite the fact that the symptoms may have subsided. This will help to prevent recurrences of UTI. Women are told to avoid scented toilet tissue and bubble baths and to wear cotton underwear, not nylon, to reduce the chance of irritation.

The nurse obtains a fingerstick glucose level of 45 mg/dl on the client newly diagnosed with diabetes mellitus. The client is alert and oriented, and the client's skin is warm and dry. How should the nurse intervene? 1. Give the client 4 oz. of milk and a graham cracker containing peanut butter. 2. Obtain a serum glucose level 3. Obtain a repeat fingerstick glucose level. 4. Notify the physician.

3. Obtain a repeat fingerstick glucose level.The nurse should recheck the fingerstick glucose level to verify the original result because the client isn't exhibiting signs of hypoglycemia. The nurse should give the client milk and a graham cracker containing peanut butter or a glass of orange juice after confirming the low glucose level. It isn't necessary to notify the physician or to obtain a serum glucose level at this time.

The nurse is doing an admission assessment on a client with a history of duodenal ulcer. To determine whether the problem is currently active, the nurse should assess the client for which symptom( s) of duodenal ulcer? 1. Weight loss 2. Nausea and vomiting 3. Pain relieved by food intake 4. Pain radiating down the right arm

3. Pain relieved by food intake A frequent symptom of duodenal ulcer is pain that is relieved by food intake. These clients generally describe the pain as a burning, heavy, sharp, or "hungry" pain that often localizes in the midepigastric area . The client with duodenal ulcer usually does not experience weight loss or nausea and vomiting. These symptoms are more typical in the client with a gastric ulcer.

The nurse is irrigating a colostomy when the client complains of cramping. What is the most appropriate initial action for the nurse? 1. Increase the flow of solution. 2. Ask the client to turn to his other side. 3. Pinch the tubing the interrupt the flow of solution. 4. Remove the tube from the colostomy.

3. Pinch the tubing the interrupt the flow of solution.

Sources of Vitamin A

Carotene source - green leafy vegetables & intensely colored fruits and vegetables - precursor to vitamin A, converted by body Retinal source - eggs, meat, dairy - converted from carotene by animal who eats it - we eat animal

Which steps should the nurse follow to insert a straight urinary catheter? 1. Create a sterile field, drape the client, clean the meatus, and insert the catheter only 6". 2. Put on gloves, prepare equipment, create a sterile field, expose the urinary meatus, and insert the catheter 6". 3. Prepare the client and equipment, create a sterile field, put on gloves, clean the urinary meatus, and insert the catheter until urine flows. 4. Prepare the client and equipment, create a sterile field, test the catheter balloon, clean the meatus, and insert the catheter until urine flows.

3. Prepare the client and equipment, create a sterile field, put on gloves, clean the urinary meatus, and insert the catheter until urine flows. Option 3 describes all the vital steps for inserting a straight catheter. Option 1 is incorrect because the nurse must prepare the client and equipment before creating a sterile field. Option 2 is incorrect because the nurse put on gloves before creating a sterile field and performing the other tasks. Option 4 describes the procedure for inserting a retention catheter, rather than a straight catheter.

A client is scheduled for a renal arteriogram. When the nurse checks the chart for allergies to shellfish or iodine, the nurse finds no allergies recorded. The client is unable to provide the information. During the procedure, the nurse should be alert for which finding that may indicate an allergic reaction to the dye used during the arteriogram. 1. Increased alertness 2. Hypoventilation 3. Pruritus 4. Unusually smooth skin

3. Pruritus The nurse should be alert for urticaria and pruritus, which may indicate a mild anaphylactic reaction to the arteriogram dye. Decreased (not increased) alertness may occur as well as dyspnea (not hypoventilation). Unusually smooth skin isn't a sign of anaphylaxis.

The nurse is caring for a client with cirrhosis. Which assessment findings indicate that the client has deficient vitamin K absorption caused by this hepatic disease? 1. Dyspnea and fatigue 2. Ascites and orthopnea 3. Purpura and petechiae 4. Gynecomastia and testicular atrophy

3. Purpura and petechiae A hepatic disorder, such as cirrhosis, may disrupt the liver's normal use of vitamin K to produce prothrombin (a clotting factor). Consequently, the nurse should monitor the client for signs of bleeding, including purpura and petechiae. Dyspnea and fatigue suggest anemia. Ascites and orthopnea are unrelated to vitamin K absorption. Gynecomastia and testicular atrophy result from decreased estrogen metabolism by the diseased liver.

Which laboratory value supports a diagnosis of pyelonephritis? 1. Myoglobinuria 2. Ketonuria 3. Pyuria 4. Low white blood cell (WBC) count

3. Pyuria Pyelonephritis is diagnosed by the presence of leukocytosis, hematuria, pyuria, and bacteriuria. The client exhibits fever, chills, and flank pain. Because there is often a septic picture, the WBC count is more likely to be high rather than low, as indicated in option 4. Ketonuria indicates a diabetic state.

Which symptom is a warning sign of colon cancer? 1. Hoarseness 2. Indigestion 3. Rectal bleeding 4. Sore that doesn't heal

3. Rectal bleeding Rectal bleeding of dark to bright red blood is a warning sign of colon cancer. Hoarseness, indigestion, and a sore that doesn't heal are potential warning signs of other types of cancer.

The purpose of giving neomycin before ileostomy surgery is to: 1. Decrease the incidence of postoperative atelectasis due to decreased depth of respirations. 2. Increase the effectiveness of the body's immunologic response following surgical trauma. 3. Reduce the incidence of wound infections by decreasing the number of intestinal organisms. 4. Prevent postoperative bladder atony due to catheterization.

3. Reduce the incidence of wound infections by decreasing the number of intestinal organisms. Neomycin is not effective in reducing postop atelectasis. The ability of the body to ward off infection is a result of the decrease in intestinal organisms. Neomycin is administered postoperatively because it is a poorly absorbed antibiotic and therefore is effective in reducing the number of intestinal organisms that may cause infection of the suture line. Bladder atony is generally due to decreased parasympathetic outflow and bladder tone secondary to anesthesia.

Which of the following is an appropriate nursing diagnosis for a client with renal calculi? 1. Ineffective renal tissue perfusion 2. Functional urinary incontinence 3. Risk for infection 4. Decreased cardiac output

3. Risk for infection Infection can occur with renal calculi from urine stasis caused by obstruction. Options 1 and 4 aren't appropriate for this client, and retention of urine, rather than incontinence, usually occurs.

A client with renal failure is undergoing continuous ambulatory peritoneal dialysis. Which nursing diagnosis is most appropriate for this client? 1. Impaired urinary elimination 2. Toileting self-care deficit 3. Risk for infection 4. Activity intolerance

3. Risk for infection The peritoneal dialysis catheter and regular exchanges of the dialysis bag provide a direct portal for bacteria to enter the body. Therefore, the client is at risk for infection. If the client experiences repeated peritoneal infections, continuous ambulatory peritoneal dialysis may no longer be effective in clearing waste products. The other options may be pertinent but are secondary to the risk for infection.

Which disorder is characterized by a sudden drop in blood glucose, followed by rebound hyperglycemia caused by the gradual and excessive administration of insulin? 1. Diabetes insipidus 2. Diabetic ketoacidosis 3. Somogyi phenomenon 4. Hyperosmolar hyperglycemic nonketotic syndrome (HHNS)

3. Somogyi phenomenon The Somogyi phenomenon is characterized by a sudden fall in blood glucose, followed by rebound hyperglycemia caused by gradual, excessive administration of insulin. Diabetes insipidus isn't associated with insulin administration. Diabetic ketoacidosis and HHNS aren't characterized by this phenomenon.

A 37-year-old man with ulcerative colitis has a new ileostomy. To promote a positive self-image for the client, the nurse will: 1. Teach the client to inspect the stoma daily. 2. Invite the client's wife and two sons to observe while the client changes the ostomy appliance. 3. Support the client in changing the ostomy appliance, but realize he may not like it. 4. Acknowledge the presence of odor during ostomy appliance changes by holding her breath.

3. Support the client in changing the ostomy appliance, but realize he may not like it. Teaching the client to inspect the stoma daily does not directly address the client's self-image. Providing privacy is appropriate unless the client gives permission for the family to observe and they will be participating in the care of the stoma. Answer 3 is correct because this behavior conveys to the client that the nurse understands and accepts some of his feelings about the new ostomy. If the nurse expresses distaste, verbally or nonverbally, this reinforces the distastefulness of the ostomy to the client.

A nurse performs a physical assessment on a client with type 2 diabetes mellitus. Findings include a fasting blood glucose of 120 mg/ dL , temperature of 101 ° F, pulse of 88 beats/ min, respirations of 22 breaths/ min, and blood pressure of 100/ 72 mm Hg. Which finding would be of most concern to the nurse? 1. Pulse 2. Respiration 3. Temperature 4. Blood pressure

3. Temperature Rationale: An elevated temperature may indicate infection. Infection is a leading cause of hyperglycemic hyperosmolar nonketotic syndrome or diabetic ketoacidosis. The other findings noted in the question are within normal limits.

The physician calls the nurse for an update on his client who underwent abdominal surgery five hours ago. The physician asks the nurse for the total amount of drainage collected in the Hemovac since surgery. The nurse reports that according to documentation, no drainage has been recorded. When the nurse finishes on the telephone, she goes to assess the client. Which assessment finding explains the absence of drainage? 1. The client has been lying on his side for two hours with the drain positioned upward. 2. The client has a nasogastric (NG) tube in place that drained 400 ml. 3. The Hemovac drain isn't compressed; instead it's fully expanded. 4. There is a moderate amount of dry drainage on the outside of the dressing.

3. The Hemovac drain isn't compressed; instead it's fully expanded. The Hemovac must be compressed to establish suction. If the Hemovac is allowed to fully expand suction is no longer present causing the drain to malfunction. The client who requires major abdominal surgery typically produces abdominal drainage despite the client's position. An NG tube drains stomach contents, not incisional contents. Therefore, the NG tube drainage of 400 ml is normal in this client and is not related to the absence of Hemovac drainage. Dry drainage on the dressing indicates leakage from the incision; it isn't related to the Hemovac drainage.

A client has been taking aluminum hydroxide (Amphojel) 30 mL six times per day at home to treat his peptic ulcer. He tells the nurse that he has been unable to have a bowel movement for 3 days. Based on this information, the nurse would determine that which of the following is the most likely cause of the client's constipation? 1. The client has not been including enough fiber in his diet. 2. The client needs to increase his daily exercise. 3. The client is experiencing an adverse effect of the aluminum hydroxide. 4. The client has developed a gastrointestinal obstruction.

3. The client is experiencing an adverse effect of the aluminum hydroxide. It is most likely that the client is experiencing an adverse effect of the antacid. Antacids with aluminum salt products, such as aluminum hydroxide, form insoluble salts in the body. These precipitate and accumulate in the intestines, causing constipation. Increasing dietary fiber intake or daily exercise may be a beneficial lifestyle change for the client but is not likely to relieve the constipation caused by the aluminum hydroxide. Constipation, in isolation from other symptoms, is not a sign of a bowel obstruction.

A client has been taking aluminum hydroxide (Amphojel) 30 mL six times per day at home to treat his peptic ulcer. He tells the nurse that he has been unable to have a bowel movement for 3 days. Based on this information, the nurse would determine that which of the following is the most likely cause of the client's constipation? 1. The client has not been including enough fiber in his diet. 2. The client needs to increase his daily exercise. 3. The client is experiencing an adverse effect of the aluminum hydroxide. 4. The client has developed a gastrointestinal obstruction.

3. The client is experiencing an adverse effect of the aluminum hydroxide. It is most likely that the client is experiencing an adverse effect of the antacid. Antacids with aluminum salt products, such as aluminum hydroxide, form insoluble salts in the body. These precipitate and accumulate in the intestines, causing constipation. Increasing dietary fiber intake or daily exercise may be a beneficial lifestyle change for the client but is not likely to relieve the constipation caused by the aluminum hydroxide. Constipation, in isolation from other symptoms, is not a sign of a bowel obstruction.

A client with suspected renal insufficiency is scheduled for a comprehensive diagnostic workup. After the nurse explains the diagnostic tests, the client asks which part of the kidney "does the work." Which answer is correct? 1. The glomerulus 2. Bowman's capsule 3. The nephron 4. The tubular system

3. The nephron The nephron is the functioning unit of the kidney. The glomerulus, Bowman's capsule, and tubular system are components of the nephron.

Which statement best describes the therapeutic action of loop diuretics? 1. They block reabsorption of potassium on the collecting tubule. 2. They promote sodium secretion into the distal tubule. 3. They block sodium reabsorption in the ascending loop and dilate renal vessels. 4. They promote potassium secretion into the distal tubule and constrict renal vessels.

3. They block sodium reabsorption in the ascending loop and dilate renal vessels. Loop diuretics block sodium reabsorption in the ascending loop of Henle, which promotes water diuresis. They also dilate renal vessels. Loop diuretics block potassium reabsorption, but this isn't a therapeutic effect. Thiazide diuretics promote sodium secretion into the distal tubule.

For a diabetic client with a foot ulcer, the physician orders bed rest, a wet-to-dry dressing change every shift, and blood glucose monitoring before meals and bedtime. Why are wet-to-dry dressings used for this client? 1. They contain exudate and provide a moist wound environment. 2. They protect the wound from mechanical trauma and promote healing. 3. They debride the wound and promote healing by secondary intention. 4. They prevent the entrance of microorganisms and minimize wound discomfort.

3. They debride the wound and promote healing by secondary intention. For this client, wet-to-dry dressings are most appropriate because they clean the foot ulcer by debriding exudate and necrotic tissue, thus promoting healing by secondary intention. Moist, transparent dressings contain exudate and provide a moist wound environment. Hydrocolloid dressings prevent the entrance of microorganisms and minimize wound discomfort. Dry, sterile dressings protect the wound from mechanical trauma and promote healing.

A client has a diagnosis of type 2 diabetes mellitus. The physician prescribes tolazamide (Tolinase), 100 mg P.O. daily. How does the onset of action of tolazamide compare with that of the other sulfonylureas? 1. All sulfonylureas, including tolazamide, have a slow onset of action. 2. All sulfonylureas, including tolazamide, have an immediate onset of action. 3. Tolazamide has a slower onset of action than the other sulfonylureas. 4. Tolazamide has a faster onset of action than the other sulfonylureas.

3. Tolazamide has a slower onset of action than the other sulfonylureas. Tolazamide has a slower onset of action than other sulfonylureas.

The client arrives at the emergency department with complaints of low abdominal pain and hematuria. The client is afebrile. The nurse next assesses the client to determine a history of: 1. Pyelonephritis 2. Glomerulonephritis 3. Trauma to the bladder or abdomen 4. Renal cancer in the client's family

3. Trauma to the bladder or abdomen Bladder trauma or injury should be considered or suspected in the client with low abdominal pain and hematuria. Glomerulonephritis and pyelonephritis would be accompanied by fever and are thus not applicable to the client described in this question. Renal cancer would not cause pain that is felt in the low abdomen; rather pain would be in the flank area.

A client comes to the emergency department complaining of acute GI distress. When obtaining the client's history, the nurse inquires about the family history. Which disorder has a familial basis? 1. Hepatitis 2. Iron deficiency anemia 3. Ulcerative colitis 4. Chronic peritonitis

3. Ulcerative colitis Ulcerative colitis is more common in people who have family members with the disease. (The same is true of some types of GI cancers, ulcers, and Crohn's disease.) Hepatitis, iron deficiency anemia, and chronic peritonitis are acquired disorders that don't run in families.

Which intervention is essential when performing dressing changes on a client with a diabetic foot ulcer? 1. Applying a heating pad 2. Debriding the wound three times per day 3. Using sterile technique during the dressing change 4. Cleaning the wound with a povidone-iodine solution

3. Using sterile technique during the dressing change The nurse should perform the dressing changes using sterile technique to prevent infection. Heating application should be avoided in a client with diabetes because of the risk for injury. Cleaning the wound with povidone-iodine solution and debriding the wound with each dressing change prevents the development of granulation tissue, which is essential in the wound healing process.

When obtaining a nursing history on a client with a suspected gastric ulcer, which signs and symptoms should the nurse expect to assess? Select all that apply. 1. Epigastric pain at night. 2. Relief of epigastric pain after eating. 3. Vomiting. 4. Weight loss. 5. Melena.

3. Vomiting. 4. Weight loss. 5. Melena. Vomiting and weight loss are common with gastric ulcers. The client may also have blood in the stools (melena) from gastric bleeding. Clients with a gastric ulcer are most likely to complain of a burning epigastric pain that occurs about 1 hour after eating. Eating frequently aggravates the pain. Clients with duodenal ulcers are more likely to complain about pain that occurs during the night and is frequently relieved by eating.

A client is admitted to the health care facility with a diagnosis of a bleeding gastric ulcer. The nurse expects this client's stools to be: 1. coffee-ground-like. 2. clay-colored. 3. black and tarry. 4. bright red.

3. black and tarry. Black, tarry stools are a sign of bleeding high in the GI tract, as from a gastric ulcer, and result from the action of digestive enzymes on the blood. Vomitus associated with upper GI tract bleeding commonly is described as coffee-ground-like. Clay-colored stools are associated with biliary obstruction. Bright red stools indicate lower GI tract bleeding.

The nurse is developing a teaching plan for a client with diabetes mellitus. A client with diabetes mellitus should: 1. use commercial preparations to remove corns. 2. cut the toenails by rounding edges. 3. wash and inspect the feet daily. 4. walk barefoot at least once each day.

3. wash and inspect the feet daily. A client with diabetes mellitus should wash and inspect his feet daily and should wear nonconstrictive shoes. Corns should be treated by a podiatrist — not with commercial preparations. Nails should be filed straight across. Clients with diabetes mellitus should never walk barefoot.

A client in the short-procedure unit is recovering from renal angiography in which a femoral puncture site was used. When providing postprocedure care, the nurse should: 1. keep the client's knee on the affected side bent for 6 hours. 2. apply pressure to the puncture site for 30 minutes. 3. check the client's pedal pulses frequently. 4. remove the dressing on the puncture site after vital signs stabilize.

3. check the client's pedal pulses frequently. After renal angiography involving a femoral puncture site, the nurse should check the client's pedal pulses frequently to detect reduced circulation to the feet caused by vascular injury. The nurse also should monitor vital signs for evidence of internal hemorrhage and should observe the puncture site frequently for fresh bleeding. The client should be kept on bed rest for several hours so the puncture site can seal completely. Keeping the client's knee bent is unnecessary. By the time the client returns to the short-procedure unit, manual pressure over the puncture site is no longer needed because a pressure dressing is in place. The nurse shouldn't remove this dressing for several hours — and only if instructed to do so.

The physician orders a stool culture to help diagnose a client with prolonged diarrhea. The nurse who obtains the stool specimen should: 1. take the specimen to the laboratory immediately. 2. apply a solution to the stool specimen. 3. collect the specimen in a sterile container. 4. store the specimen on ice.

3. collect the specimen in a sterile container. The nurse should collect the stool specimen using sterile technique and a sterile stool container. The stool may be collected for 3 consecutive days; no follow-up care is needed. Although a stool culture should be taken to the laboratory as soon as possible, it need not be delivered immediately (unlike stool being examined for ova and parasites). Applying a solution to a stool specimen would contaminate it; this procedure is done when testing stool for occult blood, not organisms. The nurse shouldn't store a stool culture on ice because the abrupt temperature change could kill the organisms.

A client with chronic renal failure (CRF) has developed faulty red blood cell (RBC) production. The nurse should monitor this client for: 1. nausea and vomiting. 2. dyspnea and cyanosis. 3. fatigue and weakness. 4. thrush and circumoral pallor.

3. fatigue and weakness. RBCs carry oxygen throughout the body. Decreased RBC production diminishes cellular oxygen, leading to fatigue and weakness. Nausea and vomiting may occur in CRF but don't result from faulty RBC production. Dyspnea and cyanosis are associated with fluid excess, not CRF. Thrush, which signals fungal infection, and circumoral pallor, which reflects decreased oxygenation, aren't signs of CRF.

The physician diagnoses type 1 diabetes mellitus in a client who has classic manifestations of the disease and a random blood glucose level of 350 mg/dl. In addition to dietary modifications, the physician prescribes insulin. Initially, most clients receive the least antigenic form of insulin. Therefore, the nurse expects the physician to prescribe: 1. beef insulin. 2. fish insulin. 3. human insulin. 4. pork insulin.

3. human insulin. Human insulin is the least antigenic form of insulin because its composition is identical to that of endogenous insulin. Animal insulins, such as beef, fish, and pork insulins, differ in composition from endogenous insulin and are therefore more antigenic.

The nurse is caring for a client in acute renal failure. The nurse should expect hypertonic glucose, insulin infusions, and sodium bicarbonate to be used to treat: 1. hypernatremia. 2. hypokalemia. 3. hyperkalemia. 4. hypercalcemia.

3. hyperkalemia. Hyperkalemia is a common complication of acute renal failure. It's life-threatening if immediate action isn't taken to reverse it. The administration of glucose and regular insulin, with sodium bicarbonate if necessary, can temporarily prevent cardiac arrest by moving potassium into the cells and temporarily reducing serum potassium levels. Hypernatremia, hypokalemia, and hypercalcemia don't usually occur with acute renal failure and aren't treated with glucose, insulin, or sodium bicarbonate.

A client with cholecystitis is receiving propantheline bromide. The client is given this medication because it: 1. reduces gastric solution production and hypermobility. 2. slows emptying of the stomach and reduces chyme in the duodenum. 3. inhibits contraction of the bile duct and gallbladder. 4. decreases bile secretions.

3. inhibits contraction of the bile duct and gallbladder. Propantheline bromide is classified as a GI anticholinergic; the medication inhibits muscarinic actions of acetylcholine at postganglionic parasympathetic neuroeffector sites. For gallbladder disease, propantheline has an antispasmodic effect on the bile duct and gallbladder. Although the medication reduces the production of gastric solutions as well as hypermobility, these aren't the main reasons for the medication. The drug doesn't slow emptying of the stomach or reduce chyme in the duodenum.

When planning care for a client with a small-bowel obstruction, the nurse should consider the primary goal to be: 1. reporting pain relief. 2. maintaining body weight. 3. maintaining fluid balance. 4. reestablishing a normal bowel pattern.

3. maintaining fluid balance. Because a client with a small-bowel obstruction can't tolerate oral intake, fluid volume deficit may occur and can be life-threatening. Therefore, maintaining fluid balance is the primary goal. The other options are secondary; pain relief and maintaining body weight don't reflect life-threatening conditions, and the client's normal bowel pattern can be reestablished after fluid volume is stabilized.

In a client with enteritis and frequent diarrhea, the nurse should anticipate an acid-base imbalance of: 1. respiratory acidosis. 2. respiratory alkalosis. 3. metabolic acidosis. 4. metabolic alkalosis.

3. metabolic acidosis. Diarrhea causes a bicarbonate deficit. With loss of the relative alkalinity of the lower GI tract, the relative acidity of the upper GI tract predominates leading to metabolic acidosis. Loss of acid, which occurs with severe vomiting, may lead to metabolic alkalosis. Diarrhea doesn't lead to respiratory acid-base imbalances, such as respiratory acidosis and respiratory alkalosis.

The nurse is providing postprocedure care for a client who underwent percutaneous lithotripsy. In this procedure, an ultrasonic probe inserted through a nephrostomy tube into the renal pelvis generates ultra-high-frequency sound waves to shatter renal calculi. The nurse should instruct the client to: 1. limit oral fluid intake for 1 to 2 weeks. 2. report the presence of fine, sandlike particles through the nephrostomy tube. 3. notify the physician about cloudy or foul-smelling urine. 4. report bright pink urine within 24 hours after the procedure.

3. notify the physician about cloudy or foul-smelling urine. The client should report the presence of foul-smelling or cloudy urine. Unless contraindicated, the client should be instructed to drink large quantities of fluid each day to flush the kidneys. Sandlike debris is normal because of residual stone products. Hematuria is common after lithotripsy.

Capillary glucose monitoring is being performed every 4 hours for a client diagnosed with diabetic ketoacidosis. Insulin is administered using a scale of regular insulin according to glucose results. At 2 p.m., the client has a capillary glucose level of 250 mg/dl for which he receives 8 U of regular insulin. The nurse should expect the dose's: 1. onset to be at 2 p.m. and its peak to be at 3 p.m. 2. onset to be at 2:15 p.m. and its peak to be at 3 p.m. 3. onset to be at 2:30 p.m. and its peak to be at 4 p.m. 4. onset to be at 4 p.m. and its peak to be at 6 p.m.

3. onset to be at 2:30 p.m. and its peak to be at 4 p.m. Regular insulin, which is a short-acting insulin, has an onset of 15 to 30 minutes and a peak of 2 to 4 hours. Because the nurse gave the insulin at 2 p.m., the expected onset would be from 2:15 to 2:30 p.m. and the peak from 4 to 6 p.m.

A 68-year-old male is being admitted to the hospital with abdominal pain, anemia, and bloody stools. He complains of feeling weak and dizzy. He has rectal pressure and needs to urinate and move his bowels. The nurse should help him: 1. to the bathroom. 2. to the bedside commode. 3. onto the bedpan. 4. to a standing position so he can urinate.

3. onto the bedpan. A client who's dizzy and anemic is at risk for injury because of his weakened state. Assisting him with the bedpan would best meet his needs at this time without risking his safety. The client may fall if walking to the bathroom, left alone to urinate, or trying to stand up.

Nursing assessment of a client with peritonitis (acute or chronic inflammation of the peritoneum) reveals hypotension, tachycardia, and signs and symptoms of dehydration. The nurse also expects to find: 1. tenderness and pain in the right upper abdominal quadrant. 2. jaundice and vomiting. 3. severe abdominal pain with direct palpation or rebound tenderness. 4. rectal bleeding and a change in bowel habits.

3. severe abdominal pain with direct palpation or rebound tenderness. Peritonitis decreases intestinal motility and causes intestinal distention. A classic sign of peritonitis is a sudden, diffuse, severe abdominal pain that intensifies in the area of the underlying causative disorder (such as appendicitis, diverticulitis, ulcerative colitis, or a strangulated obstruction). The client also has direct or rebound tenderness. Tenderness and pain in the right upper abdominal quadrant suggest cholecystitis. Jaundice and vomiting are signs of cirrhosis of the liver. Rectal bleeding or a change in bowel habits may indicate colorectal cancer.

A client with type 1 diabetes mellitus has been on a regimen of multiple daily injection therapy. He's being converted to continuous subcutaneous insulin therapy. While teaching the client about continuous subcutaneous insulin therapy, the nurse would be accurate in telling him the regimen includes the use of: 1. intermediate- and long-acting insulins. 2. short- and long-acting insulins. 3. short-acting insulin only. 4. short- and intermediate-acting insulins.

3. short-acting insulin only. Continuous subcutaneous insulin regimen uses a basal rate and boluses of short-acting insulin. Multiple daily injection therapy uses a combination of short-acting and intermediate- or long-acting insulins.

A female client reports to the nurse that she experiences a loss of urine when she jogs. The nurse's assessment reveals no nocturia, burning, discomfort when voiding, or urine leakage before reaching the bathroom. The nurse explains to the client that this type of problem is called: 1. functional incontinence. 2. reflex incontinence. 3. stress incontinence. 4. total incontinence.

3. stress incontinence. Stress incontinence is a small loss of urine with activities that increase intra-abdominal pressure, such as running, laughing, sneezing, jumping, coughing, and bending. These symptoms occur only in the daytime. Functional incontinence is the inability of a usually continent client to reach the toilet in time to avoid unintentional loss of urine. Reflex incontinence is an involuntary loss of urine at predictable intervals when a specific bladder volume is reached. Total incontinence occurs when a client experiences a continuous and unpredictable loss of urine.

A client with viral hepatitis A is being treated in an acute care facility. Because the client requires enteric precautions, the nurse should: 1. place the client in a private room. 2. wear a mask when handling the client's bedpan. 3. wash the hands after touching the client. 4. wear a gown when providing personal care for the client.

3. wash the hands after touching the client. To maintain enteric precautions, the nurse must wash the hands after touching the client or potentially contaminated articles and before caring for another client. A private room is warranted only if the client has poor hygiene — for instance, if the client is unlikely to wash the hands after touching infective material or is likely to share contaminated articles with other clients. For enteric precautions, the nurse need not wear a mask and must wear a gown only if soiling from fecal matter is likely.

The daily dosage for prophylactic isoniazid therapy is

300mg

Nursing considerations for Alpha Beta Blockers

Close monitoring of BP, don't administer to patients with COPD.

1 Gram Carbohydrates =

4 calories

1 Gram Protein =

4 calories

When teaching a client about insulin administration, the nurse should include which instruction? 1. "Take insulin after the first meal of the day." 2. "Inject insulin at a 45-degree angle into the deltoid muscle." 3. "Shake the insulin vial vigorously before withdrawing the medication." 4. "Draw up clear insulin first when mixing two types of insulin in one syringe."

4. "Draw up clear insulin first when mixing two types of insulin in one syringe." When mixing two types of insulin, the client should draw clear (regular) insulin into the syringe first. The daily insulin dose typically is administered before the first meal of the day and is injected into fatty tissue at a 90-degree angle. If cloudy (NPH or Humulin N) insulin must be administered, the client should roll the vial between the palms gently before withdrawing the medication.

During a client-teaching session, which instruction should the nurse give to a client receiving kaolin and pectin (Kaopectate) for treatment of diarrhea? 1. "Take the medication after every other loose bowel movement, up to five doses per day." 2. "Avoid taking this medication for more than 4 days." 3. "Consult the physician if you have more than six bowel movements in 1 day." 4. "Drink 8 to 13 8-oz glasses (2 to 3 L) of fluid daily."

4. "Drink 8 to 13 8-oz glasses (2 to 3 L) of fluid daily." The nurse should tell the client to drink 8 to 13 8-oz glasses of fluid daily to replace fluids lost through diarrhea. Kaolin and pectin mixtures should be taken after each loose bowel movement for up to eight doses daily. The client should avoid self-medication for longer than 48 hours. The client should consult a physician if diarrhea persists longer than 48 hours despite treatment.

A client is taking glyburide (DiaBeta), 1.25 mg P.O. daily, to treat type 2 diabetes mellitus. Which statement indicates the need for further client teaching about management of this disease? 1. "I always carry hard candy to eat in case my blood sugar level drops." 2. "I avoid exposure to the sun as much as possible." 3. "I always wear my medical identification bracelet." 4. "I often skip lunch because I don't feel hungry."

4. "I often skip lunch because I don't feel hungry." A client who is receiving an oral antidiabetic agent should eat meals on a regular schedule because skipping a meal increases the risk of hypoglycemia. Carrying hard candy, avoiding exposure to the sun, and always wearing a medical identification bracelet indicate effective teaching.

A nurse provides instructions to a client newly diagnosed with type 1 diabetes mellitus. The nurse recognizes accurate understanding of measures to prevent diabetic ketoacidosis when the client states: 1. "I will stop taking my insulin if I'm too sick to eat." 2. "I will decrease my insulin dose during times of illness." 3. "I will adjust my insulin dose according to the level of glucose in my urine." 4. "I will notify my physician if my blood glucose level is higher than 250 mg/ dL."

4. "I will notify my physician if my blood glucose level is higher than 250 mg/ dL." Rationale: During illness, the client should monitor blood glucose levels and should notify the physician if the level is higher than 250 mg/ dL. Insulin should never be stopped. In fact, insulin may need to be increased during times of illness. Doses should not be adjusted without the physician's advice and are usually adjusted based on blood glucose levels, not urinary glucose readings.

A client is taking an antacid for treatment of a peptic ulcer. Which of the following statements best indicates that the client understands how to correctly take the antacid? 1. "I should take my antacid before I take my other medications." 2. "I need to decrease my intake of fluids so that I don't dilute the effects of my antacid." 3. "My antacid will be most effective if I take it whenever I experience stomach pains." 4. "It is best for me to take my antacid 1 to 3 hours after meals."

4. "It is best for me to take my antacid 1 to 3 hours after meals." Antacids are most effective if taken 1 to 3 hours after meals and at bedtime. When an antacid is taken on an empty stomach, the duration of the drug's action is greatly decreased. Taking antacids 1 to 3 hours after a meal lengthens the duration of action, thus increasing the therapeutic action of the drug. Antacids should be administered about 2 hours after other medications to decrease the chance of drug interactions. It is not necessary to decrease fluid intake when taking antacids. If antacids are taken more frequently than recommended, the likelihood of developing adverse effects increases. Therefore, the client should not take antacids as often as desired to control pain.

A client is taking an antacid for treatment of a peptic ulcer. Which of the following statements best indicates that the client understands how to correctly take the antacid? 1. "I should take my antacid before I take my other medications." 2. "I need to decrease my intake of fluids so that I don't dilute the effects of my antacid." 3. "My antacid will be most effective if I take it whenever I experience stomach pains." 4. "It is best for me to take my antacid 1 to 3 hours after meals."

4. "It is best for me to take my antacid 1 to 3 hours after meals." Antacids are most effective if taken 1 to 3 hours after meals and at bedtime. When an antacid is taken on an empty stomach, the duration of the drug's action is greatly decreased. Taking antacids 1 to 3 hours after a meal lengthens the duration of action, thus increasing the therapeutic action of the drug. Antacids should be administered about 2 hours after other medications to decrease the chance of drug interactions. It is not necessary to decrease fluid intake when taking antacids. If antacids are taken more frequently than recommended, the likelihood of developing adverse effects increases. Therefore, the client should not take antacids as often as desired to control pain.

A nurse is interviewing a client with type 2 diabetes mellitus. Which statement by the client indicates an understanding of the treatment for this disorder? 1. "I take oral insulin instead of shots." 2. "By taking these medications, I am able to eat more." 3. "When I become ill, I need to increase the number of pills I take." 4. "The medications I'm taking help release the insulin I already make."

4. "The medications I'm taking help release the insulin I already make." Rationale: Clients with type 2 diabetes mellitus have decreased or impaired insulin secretion. Oral hypoglycemic agents are given to these clients to facilitate glucose uptake. Insulin injections may be given during times of stress-induced hyperglycemia . Oral insulin is not available because of the breakdown of the insulin by digestion. Options 1, 2, and 3 are incorrect.

The spouse of a client who had a transurethral resection of the prostate (TURP) 24 hours ago is upset because the irrigation seems to be increasing the client's pain. Which is the best response by a nurse? 1. "I will stop the bladder irrigation and the pain should subside." 2. "I will add a local anesthetic to the irrigant to treat the pain." 3. "Perhaps you should go home and get some rest." 4. "The rate of irrigant should be slowed when the drainage is pale pink."

4. "The rate of irrigant should be slowed when the drainage is pale pink." The nurse cannot stop the irrigation without an order. The nurse does not have the authority to add anesthetic. The purpose of the irrigant is to prevent clots. A pale pink would indicate that bleeding is diminishing.

A client who has just been diagnosed with hepatitis A asks, "How could I have gotten this disease?" What is the nurse's best response? 1. "You could have gotten it by using I.V. drugs." 2. "You must have received an infected blood transfusion." 3. "You probably got it by engaging in unprotected sex." 4. "You may have eaten contaminated restaurant food."

4. "You may have eaten contaminated restaurant food." Hepatitis A virus typically is transmitted by the oral-fecal route — commonly by consuming food contaminated by infected food handlers. The virus isn't transmitted by the I.V. route, blood transfusions, or unprotected sex. Hepatitis B can be transmitted by I.V. drug use or blood transfusion. Hepatitis C can be transmitted by unprotected sex.

The nurse teaches a diabetic client that diet plays a crucial role in managing diabetes mellitus. When evaluating dietary intake, the nurse knows the client is eating the right foods if total daily caloric intake consists of: 1. 30% to 35% carbohydrate, 40% fat, and 25% to 30% protein. 2. 40% to 45% carbohydrate, 40% fat, and 15% to 20% protein. 3. 50% to 55% carbohydrate, 35% fat, and 10% to 15% protein. 4. 55% to 60% carbohydrate, 30% fat, and 10% to 15% protein.

4. 55% to 60% carbohydrate, 30% fat, and 10% to 15% protein. A client with diabetes mellitus should get 55% to 60% of total daily calories from carbohydrates, no more than 30% from fats, and the remainder (10% to 15%) from proteins. A diet in which carbohydrates account for less than 55% of calories has a higher fat content than recommended for a healthy diet. Because diabetes mellitus is a risk factor for cardiovascular disease, excessive fat intake further increases the client's risk for cardiovascular disease.

On a medical surgical floor, the nurse is caring for a cluster of clients who were diagnosed with diabetes mellitus. Which client should the nurse assess first? 1. An 80-year-old client with a blood glucose level of 350 mg/dl 2. A 20-year-old client with a blood glucose level of 70 mg/dl 3. A 60-year-old client experiencing nausea and vomiting 4. A 55-year-old complaining of chest pressure

4. A 55-year-old complaining of chest pressure The nurse should assess the client with chest pressure first because he might be experiencing a myocardial infarction. The blood glucose levels in options 2 and 3 are abnormal, but not life threatening; therefore, those clients don't require immediate attention. After assessing the client with chest pressure, the nurse should assess the client experiencing nausea and vomiting.

The nurse is assessing the stoma of a client following a ureterostomy. Which of the following should the nurse expect to note? 1. A dry stoma 2. A pale stoma 3. A dark-colored stoma 4. A red and moist stoma

4. A red and moist stoma Following ureterostomy, the stoma should be red and moist. A pale stoma may indicate an inadequate amount of vascular supply. A dry stoma may indicate a body fluid deficit. Any sign of darkness or duskiness in the stoma may indicate a loss of vascular supply and must be reported immediately or necrosis can occur.

The nurse is monitoring a client with a diagnosis of peptic ulcer. Which assessment finding would most likely indicate perforation of the ulcer? 1. Bradycardia 2. Numbness in the legs 3. Nausea and vomiting 4. A rigid, board-like abdomen

4. A rigid, board-like abdomen Perforation of an ulcer is a surgical emergency and is characterized by sudden, sharp, intolerable severe pain beginning in the midepigastric area and spreading over the abdomen, which becomes rigid and board -like. Nausea and vomiting may occur. Tachycardia may occur as hypovolemic shock develops. Numbness in the legs is not an associated finding.

The nurse caring for a client with small-bowel obstruction would plan to implement which nursing intervention first? 1. Administering pain medication 2. Obtaining a blood sample for laboratory studies 3. Preparing to insert a nasogastric (NG) tube 4. Administering I.V. fluids

4. Administering I.V. fluids I.V. infusions containing normal saline solution and potassium should be given first to maintain fluid and electrolyte balance. For the client's comfort and to assist in bowel decompression, the nurse should prepare to insert an NG tube next. A blood sample is then obtained for laboratory studies to aid in the diagnosis of bowel obstruction and guide treatment. Blood studies usually include a complete blood count, serum electrolyte levels, and blood urea nitrogen level. Pain medication often is withheld until obstruction is diagnosed because analgesics can decrease intestinal motility.

The nurse is teaching the client about risk factors for diabetes mellitus. Which risk factor for diabetes mellitus is nonmodifiable? 1. Poor control of blood glucose levels 2. Inappropriate foot care 3. Current or recent foot trauma 4. Advanced age

4. Advanced age Nonmodifiable risk factors are ones that the client doesn't have the ability to change. Therefore, advanced age is the correct answer. The other choices are ones over which the client can exert some control.

The client has undergone esophagogastroduodenoscopy. The nurse places highest priority on which item as part of the client's care plan? 1. Monitoring the temperature 2. Monitoring complaints of heartburn 3. Giving warm gargles for a sore throat 4. Assessing for the return of the gag reflex

4. Assessing for the return of the gag reflex The nurse places highest priority on assessing for return of the gag reflex. This assessment addresses the client's airway. The nurse also monitors the client's vital signs and for a sudden increase in temperature, which could indicate perforation of the gastrointestinal tract. This complication would be accompanied by other signs as well, such as pain. Monitoring for sore throat and heartburn are also important; however, the client's airway is the priority.

A client is frustrated and embarrassed by urinary incontinence. Which measure should the nurse include in a bladder retraining program? 1. Establishing a predetermined fluid intake pattern for the client 2. Encouraging the client to increase the time between voidings 3. Restricting fluid intake to reduce the need to void 4. Assessing present elimination patterns

4. Assessing present elimination patterns The guidelines for initiating bladder retraining include assessing the client's intake patterns, voiding patterns, and reasons for each accidental voiding. Lowering the client's fluid intake won't reduce or prevent incontinence. The client should actually be encouraged to drink 1.5 to 2 L of water per day. A voiding schedule should be established after assessment.

The nurse expects to note an elevated serum glucose level in a client with hyperosmolar hyperglycemic nonketotic syndrome (HHNS). Which other laboratory finding should the nurse anticipate? 1. Elevated serum acetone level 2. Serum ketone bodies 3. Serum alkalosis 4. Below-normal serum potassium level

4. Below-normal serum potassium level A client with HHNS has an overall body deficit of potassium resulting from diuresis, which occurs secondary to the hyperosmolar, hyperglycemic state caused by the relative insulin deficiency. An elevated serum acetone level and serum ketone bodies are characteristic of diabetic ketoacidosis. Metabolic acidosis, not serum alkalosis, may occur in HHNS.

The client is admitted to the hospital with a diagnosis of benign prostatic hyperplasia, and a transurethral resection of the prostate is performed. Four hours after surgery, the nurse takes the client's vital signs and empties the urinary drainage bag. Which of the following assessment findings would indicate the need to notify the physician? 1. Red bloody urine 2. Pain related to bladder spasms 3. Urinary output of 200 mL higher than intake 4. Blood pressure, 100/ 50 mm Hg; pulse, 130 beats/ min

4. Blood pressure, 100/ 50 mm Hg; pulse, 130 beats/ min Frank bleeding (arterial or venous) may occur during the first day after surgery. Some hematuria is usual for several days after surgery. A urinary output of 200 mL more than intake is adequate. Bladder spasms are expected to occur following surgery. A rapid pulse with a low blood pressure is a potential sign of excessive blood loss. The physician should be notified.

The physician prescribes a single dose of trimethoprim/sulfamethoxazole (Bactrim) by mouth for a client diagnosed with an uncomplicated urinary tract infection (UTI). The pharmacy sends three unit-dose tablets. The nurse verifies the physician's order. What should the nurse do next? 1. Administer the three tablets as the single dose. 2. Call the physician to verify the order. 3. Give one tablet, three times per day. 4. Call the hospital pharmacist and question the medication supplied.

4. Call the hospital pharmacist and question the medication supplied. The nurse should call the hospital pharmacy and question the medication supplied. The hospital pharmacist should be able to tell the nurse whether three tablets are necessary for the single dose or whether a dispensing error occurred. It isn't clear whether the three tablets are the single dose because they were packaged as a unit-dose. The physician order was clearly written, so clarifying the order with the physician isn't necessary. Administering the tablets without clarification might cause a medication error.

A client requires hemodialysis. Which type of drug should be withheld before this procedure? 1. Phosphate binders 2. Insulin 3. Antibiotics 4. Cardiac glycosides

4. Cardiac glycosides Cardiac glycosides such as digoxin should be withheld before hemodialysis. Hypokalemia is one of the electrolyte shifts that occur during dialysis, and a hypokalemic client is at risk for arrhythmias secondary to digitalis toxicity. Phosphate binders and insulin can be administered because they aren't removed from the blood by dialysis. Some antibiotics are removed by dialysis and should be administered after the procedure to ensure their therapeutic effects. The nurse should check a formulary to determine whether a particular antibiotic should be administered before or after dialysis.

A client with diabetic ketoacidosis (DKA) was admitted to the intensive care unit 4 hours ago and has these laboratory results: blood glucose level 450 mg/dl, serum potassium level 2.5 mEq/L, serum sodium level 140 mEq/L, and urine specific gravity 1.025. The client has two I.V. lines in place with normal saline solution infusing through both. Over the past 4 hours, his total urine output has been 50 ml. Which physician order should the nurse question? 1. Infuse 500 ml of normal saline solution over 1 hour. 2. Hold insulin infusion for 30 minutes. 3. Add 40 mEq KCL to an infusion of half normal saline solution and infuse at a rate of 10 mEq/hour. 4. Change the second I.V. solution to dextrose 5% in water.

4. Change the second I.V. solution to dextrose 5% in water. The nurse should question the physician's order to change the second I.V. solution to dextrose 5% in water. The client should receive normal saline solution through the second I.V. site until the client's blood glucose level reaches 250 mg/dl. The client should receive a fluid bolus of 500 ml of normal saline solution. The client's urine output is low and his specific gravity is high which reveals dehydration. The nurse should expect to hold the insulin infusion for 30 minutes until the potassium replacement has been initiated. Insulin administration causes potassium to enter the cells, which further lowers the serum potassium level. Further lowering the serum potassium level, places the client at risk for life-threatening cardiac arrhythmias.

The client is scheduled for urinary diversion surgery to treat bladder cancer. Before surgery, the health care team consisting of a nurse, dietician, social worker, enterostomal therapist, surgeon, client educator, and mental health worker meet with the client. After the meeting, the client states, "My life won't ever be the same. What am I going to do?" Which health team member should the nurse consult to help with the client's concerns? 1. Social worker 2. Surgeon 3. Dietician 4. Client educator

4. Client educator The nurse should consult the client educator to help the client with his fears and concerns. Providing the client with information can greatly allay the client's fears. The social worker can provide the client with services he may need after discharge. The dietician can help with dietary concerns but can't provide help with direct concerns about the surgery.

During clindamycin (Cleocin) therapy, the nurse monitors a client for pseudomembranous colitis. This serious adverse reaction to clindamycin results from superinfection with which organism? 1. Staphylococcus aureus 2. Bacteroides fragilis 3. Escherichia coli 4. Clostridium difficile

4. Clostridium difficile Pseudomembranous colitis may result from a superinfection with C. difficile during clindamycin therapy. Clindamycin-induced pseudomembranous colitis isn't caused by S. aureus, B. fragilis, or E. coli.

One year ago, a client was diagnosed with cirrhosis of the liver caused by alcohol abuse. Since then, he has been noncompliant with the prescribed protein-restricted diet. After a friend finds him semiconscious at home, the client is admitted to the hospital. When initial laboratory test results show an elevated ammonia level, he's diagnosed with hepatic encephalopathy. The physician prescribes lactulose (Cephulac), 200 g diluted in 700 ml of tap water, given as a retention enema every 4 hours. For which other condition is lactulose prescribed? 1. Hyperkalemia 2. Lactic acidosis 3. Hypoglycemia 4. Constipation

4. Constipation Lactulose also may be used to treat constipation because it produces osmotic diarrhea. It isn't therapeutic in the treatment of hyperkalemia, lactic acidosis, or hypoglycemia.

Factors that increase BUN include

GI bleeding, dehydration, increased protein intake, fever

A 28-year-old client is admitted with inflammatory bowel syndrome (Crohn's disease). Which therapies should the nurse expect to be part of the care plan? 1. Lactulose therapy 2. High-fiber diet 3. High-protein milkshakes 4. Corticosteroid therapy 5. Antidiarrheal medications

4. Corticosteroid therapy 5. Antidiarrheal medications Corticosteroids, such as prednisone, reduce the signs and symptoms of diarrhea, pain, and bleeding by decreasing inflammation. Antidiarrheals, such as diphenoxylate (Lomotil), combat diarrhea by decreasing peristalsis. Lactulose is used to treat chronic constipation and would aggravate the symptoms. A high-fiber diet and milk and milk products are contraindicated in clients with Crohn's disease because they may promote diarrhea.

A client is diagnosed with diabetes mellitus. Which assessment finding best supports a nursing diagnosis of Ineffective coping related to diabetes mellitus? 1. Recent weight gain of 20 lb 2. Failure to monitor blood glucose levels 3. Skipping insulin doses during illness 4. Crying whenever diabetes is mentioned

4. Crying whenever diabetes is mentioned A client who cries whenever diabetes is mentioned is demonstrating ineffective coping. A recent weight gain and failure to monitor blood glucose levels would support a nursing diagnosis of Noncompliance: Failure to adhere to therapeutic regimen. Skipping insulin doses during illness would support a nursing diagnosis of Deficient knowledge related to treatment of diabetes mellitus.

The nurse is taking the history of a client who has had benign prostatic hyperplasia in the past. To determine whether the client currently is experiencing difficulty, the nurse asks the client about the presence of which early symptom? 1. Nocturia 2. Urinary retention 3. Urge incontinence 4. Decreased force in the stream of urine

4. Decreased force in the stream of urine Decreased force in the stream of urine is an early sign of benign prostatic hyperplasia. The stream later becomes weak and dribbling. The client then may develop hematuria, frequency, urgency, urge incontinence, and nocturia. If untreated, complete obstruction and urinary retention can occur.

Which finding best indicates that the nursing assistant has an understanding of blood glucose meter use? 1. Verbalizing an understanding of blood glucose meter use 2. Documenting a normal blood glucose level 3. Providing documentation of previous certification 4. Demonstrating correct technique

4. Demonstrating correct technique The best way to validate blood glucose meter use is to allow the nursing assistant to demonstrate correct technique. Verbalizing understanding doesn't demonstrate that the nursing assistant knows proper technique. Options 2 and 3 don't demonstrate blood glucose meter use.

Diagnostic Tests for Laryngeal Tumor

Larygoscopy/bx, MRI/CT

The nurse is assessing a client 24 hours following a cholecystectomy. The nurse notes that the T-tube has drained 750 mL of green-brown drainage since the surgery. Which nursing intervention is appropriate? 1. Clamp the T-tube. 2. Irrigate the T-tube. 3. Notify the physician. 4. Document the findings.

4. Document the findings. Following cholecystectomy, drainage from the T-tube is initially bloody and then turns to a greenish-brown color. The drainage is measured as output. The amount of expected drainage will range from 500 to 1000 mL/ day. The nurse would document the output.

The male client has a tentative diagnosis of urethritis. The nurse assesses the client for which of the following manifestations of the disorder? 1. Hematuria and pyuria 2. Dysuria and proteinuria 3. Hematuria and urgency 4. Dysuria and penile discharge

4. Dysuria and penile discharge Urethritis in the male client often results from chlamydial infection and is characterized by dysuria, which is accompanied by a clear to mucopurulent discharge. Because this disorder often coexists with gonorrhea, diagnostic tests are done for both and include culture and rapid assays.

Respiratory acidosis lab values

pH < 7.35, CO2 > 45 mmHg

A client with a diagnosis of diabetic ketoacidosis (DKA) is being treated in an emergency department. Which finding would a nurse expect to note as confirming this diagnosis? 1. Comatose state 2. Decreased urine output 3. Increased respirations and an increase in pH 4. Elevated blood glucose level and low plasma bicarbonate level

4. Elevated blood glucose level and low plasma bicarbonate level Rationale: In DKA, the arterial pH is lower than 7.35, plasma bicarbonate is lower than 15 mEq/ L, the blood glucose level is higher than 250 mg/ dL, and ketones are present in the blood and urine. The client would be experiencing polyuria, and Kussmaul's respirations would be present . A comatose state may occur if DKA is not treated, but coma would not confirm the diagnosis.

A client with newly diagnosed type 2 diabetes mellitus is admitted to the metabolic unit. The primary goal for this admission is education. Which of the following goals should the nurse incorporate into her teaching plan? 1. Maintenance of blood glucose levels between 180 and 200 mg/dl 2. Smoking reduction but not complete cessation 3. An eye examination every 2 years until age 50 4. Exercise and a weight reduction diet

4. Exercise and a weight reduction diet Type 2 diabetes is often obesity-related; therefore, weight reduction may enhance the normalization of the blood glucose level. Weight reduction should be achieved by a healthy diet and exercise to increase carbohydrate metabolism. Blood glucose levels should be maintained within normal limits to prevent the development of diabetic complications. Clients with type 1 or 2 diabetes shouldn't smoke because of the increased risk of cardiovascular disease. A funduscopic examination should be done yearly to identify early signs of diabetic retinopathy.

An elderly client continues to have fecal incontinence with 6 to 7 small brown liquid stools each day. The client eats a soft diet and does not receive any stool softeners or laxatives. The client's primary form of activity is sitting in the wheelchair for 2 hours twice a day. What is the correct explanation for the frequent diarrhea stools? 1. Inadequate roughage in the diet. 2. Inactivity from sedentary lifestyle. 3. Gastrointestinal virus. 4. Fecal impaction.

4. Fecal impaction. Inadequate roughage would not cause diarrhea. Fecal impaction is likely causing diarrhea. Fiber can be added to a soft diet to prevent constipation. Inactivity contributes to the potential for constipation and fecal impaction, but the impaction is causing the liquid stools. Pressure on the colonic mucosa causes seepage of liquid stool around the area of impaction.

The client has just had surgery to create an ileostomy. The nurse assesses the client in the immediate postoperative period for which most frequent complication of this type of surgery? 1. Folate deficiency 2. Malabsorption of fat 3. Intestinal obstruction 4. Fluid and electrolyte imbalance

4. Fluid and electrolyte imbalance A frequent complication that occurs following ileostomy is fluid and electrolyte imbalance. The client requires constant monitoring of intake and output to prevent this from occurring. Losses require replacement by intravenous infusion until the client can tolerate a diet orally. Intestinal obstruction is a less frequent complication. Fat malabsorption and folate deficiency are complications that could occur later in the postoperative period.

An external insulin pump is prescribed for a client with diabetes mellitus and the client asks the nurse about the functioning of the pump. The nurse bases the response on the information that the pump: 1. Is timed to release programmed doses of regular or NPH insulin into the bloodstream at specific intervals 2. Continuously infuses small amounts of NPH insulin into the bloodstream while regularly monitoring blood glucose levels 3. Is surgically attached to the pancreas and infuses regular insulin into the pancreas, which in turn releases the insulin into the bloodstream 4. Gives a small continuous dose of regular insulin subcutaneously, and the client can self-administer a bolus with an additional dose from the pump before each meal

4. Gives a small continuous dose of regular insulin subcutaneously, and the client can self-administer a bolus with an additional dose from the pump before each meal Rationale: An insulin pump provides a small continuous dose of regular insulin subcutaneously throughout the day and night, and the client can self-administer a bolus with an additional dose from the pump before each meal as needed. Regular insulin is used in an insulin pump. An external pump is not attached surgically to the pancreas.

An adult is admitted with probable large bowel obstruction. The nurse auscultates the abdomen. Which finding, if present, is consistent with an intestinal obstruction? 1. Hyperactive bowel sounds in all 4 quadrants. 2. Hypoactive bowel sounds in all 4 quadrants. 3. Coffee grounds emesis and tarry stools. 4. High pitched bowel sounds in the right quadrants; hypoactive bowel sounds in the left quadrants.

4. High pitched bowel sounds in the right quadrants; hypoactive bowel sounds in the left quadrants.

The nurse is caring for a client with acute renal failure. The nurse should expect that hypertonic glucose, insulin infusions, and sodium bicarbonate will be used to treat what complication of acute renal failure? 1. Hypokalemia 2. Hyperphosphatemia 3. Hypophosphatemia 4. Hyperkalemia

4. Hyperkalemia Hyperkalemia is a common complication of acute renal failure. The administration of glucose and regular insulin infusions, with sodium bicarbonate if necessary, can temporarily prevent cardiac arrest by moving potassium into the cells and temporarily reducing potassium levels. This treatment isn't used to treat hyperphosphatemia or hypophosphatemia.

A client with a peptic ulcer has been instructed to avoid intense physical activity and stress. Which strategy should the client incorporate into the home care plan? 1. Conduct physical activity in the morning so that he can rest in the afternoon. 2. Have the family agree to perform the necessary yard work at home. 3. Give up jogging and substitute a less demanding hobby. 4. Incorporate periods of physical and mental rest in his daily schedule.

4. Incorporate periods of physical and mental rest in his daily schedule. It would be most effective for the client to develop a health maintenance plan that incorporates regular periods of physical and mental rest in the daily schedule. Strategies should be identified to deal with the types of physical and mental stressors that the client needs to cope with in the home and work environments. Scheduling physical activity to occur only in the morning would not be restful or practical. There is no need for the client to avoid yard work or jogging if these activities are not stressful.

A client with a peptic ulcer has been instructed to avoid intense physical activity and stress. Which strategy should the client incorporate into the home care plan? 1. Conduct physical activity in the morning so that he can rest in the afternoon. 2. Have the family agree to perform the necessary yard work at home. 3. Give up jogging and substitute a less demanding hobby. 4. Incorporate periods of physical and mental rest in his daily schedule.

4. Incorporate periods of physical and mental rest in his daily schedule. It would be most effective for the client to develop a health maintenance plan that incorporates regular periods of physical and mental rest in the daily schedule. Strategies should be identified to deal with the types of physical and mental stressors that the client needs to cope with in the home and work environments. Scheduling physical activity to occur only in the morning would not be restful or practical. There is no need for the client to avoid yard work or jogging if these activities are not stressful.

A client is scheduled to undergo an exploratory laparoscopy. The registered nurse asks the licensed practical nurse (LPN) to prepare the client for surgery. The registered nurse must confirm that the LPN has specialized training before delegating which task? 1. Weighing the client 2. Teaching the client coughing and deep breathing exercises 3. Teaching the client how to collect a urine specimen 4. Initiating intravenous therapy, as ordered

4. Initiating intravenous therapy, as ordered The registered nurse must confirm that the LPN has specialized I.V. training before asking her to begin I.V. therapy for this client. Initiating I.V. therapy is beyond the usual scope of practice for an LPN. Options 1, 2, and 3 are within the scope of LPN practice and don't require additional training.

When caring for a client with hepatitis B, the nurse should monitor closely for the development of which finding associated with a decrease in hepatic function? 1. Jaundice 2. Pruritus of the arms and legs 3. Fatigue during ambulation 4. Irritability and drowsiness

4. Irritability and drowsiness Although all the options are associated with hepatitis B, the onset of irritability and drowsiness suggests a decrease in hepatic function. To detect signs and symptoms of disease progression, the nurse should observe for disorientation, behavioral changes, and a decreasing level of consciousness and should monitor the results of liver function tests, including the blood ammonia level. If hepatic function is decreased, the nurse should take safety precautions.

A client with peptic ulcer disease is taking ranitidine (Zantac). What is the expected outcome of this drug? 1. Heal the ulcer. 2. Protect the ulcer surface from acids. 3. Reduce acid concentration. 4. Limit gastric acid secretion.

4. Limit gastric acid secretion. Histamine-2 (H2) receptor antagonists, such as ranitidine, reduce gastric acid secretion. Antisecretory, or proton-pump inhibitors, such as omeprazole (Prilosec), help ulcers heal quickly in 4 to 8 weeks. Cytoprotective drugs, such as sucralfate (Carafate), protect the ulcer surface against acid, bile, and pepsin. Antacids reduce acid concentration and help reduce symptoms.

A client with peptic ulcer disease is taking ranitidine (Zantac). What is the expected outcome of this drug? 1. Heal the ulcer. 2. Protect the ulcer surface from acids. 3. Reduce acid concentration. 4. Limit gastric acid secretion.

4. Limit gastric acid secretion. Histamine-2 (H2) receptor antagonists, such as ranitidine, reduce gastric acid secretion. Antisecretory, or proton-pump inhibitors, such as omeprazole (Prilosec), help ulcers heal quickly in 4 to 8 weeks. Cytoprotective drugs, such as sucralfate (Carafate), protect the ulcer surface against acid, bile, and pepsin. Antacids reduce acid concentration and help reduce symptoms.

What should the nurse include in health teaching for a client who has an ileostomy? 1. Eat high-fiber foods. 2. Take laxatives for any blockage. 3. Eat raw fruits and vegetables. 4. Maintain adequate fluid intake.

4. Maintain adequate fluid intake. High-fiber foods should be avoided. Laxatives are not needed; drainage is liquid. Increased peristalsis may contribute to FVD and discomfort. Certain raw fruits and vegetables have high cellulose content and should be avoided because they could cause blockage. Due to lack of reabsorption of fluid from the large intestine, the client is at risk for FVD.

A client with a history of chronic cystitis comes to the outpatient clinic with signs and symptoms of this disorder. To prevent cystitis from recurring, the nurse recommends maintaining an acid-ash diet to acidify the urine, thereby decreasing the rate of bacterial multiplication. On an acid-ash diet, the client must restrict which beverage? 1. Cranberry juice 2. Coffee 3. Prune juice 4. Milk

4. Milk A client on an acid-ash diet must avoid milk and milk products because these make the urine more alkaline, encouraging bacterial growth. Other foods to avoid on this diet include all vegetables except corn and lentils; all fruits except cranberries, plums, and prunes; and any food containing large amounts of potassium, sodium, calcium, or magnesium. Cranberry and prune juice are encouraged because they acidify the urine. Coffee and tea are considered neutral because they don't alter the urine pH.

A client with decreased urine output refractory to fluid challenges is evaluated for renal failure. Which condition may cause the intrinsic (intrarenal) form of acute renal failure? 1. Poor perfusion to the kidneys 2. Damage to cells in the adrenal cortex 3. Obstruction of the urinary collecting system 4. Nephrotoxic injury secondary to use of contrast media

4. Nephrotoxic injury secondary to use of contrast media Intrinsic renal failure results from damage to the kidney, such as from nephrotoxic injury caused by contrast media, antibiotics, corticosteroids, or bacterial toxins. Poor perfusion to the kidneys may result in prerenal failure. Damage to the epithelial cells of the renal tubules results from nephrotoxic injury, not damage to the adrenal cortex. Obstruction of the urinary collecting system may cause postrenal failure.

Which of the following signs and symptoms would be seen in a client experiencing hypoglycemia? 1. Polyuria, headache, and fatigue 2. Polyphagia and flushed, dry skin 3. Polydipsia, pallor, and irritability 4. Nervousness, diaphoresis, and confusion

4. Nervousness, diaphoresis, and confusion Signs and symptoms associated with hypoglycemia include nervousness, diaphoresis, weakness, light-headedness, confusion, paresthesia, irritability, headache, hunger, tachycardia, and changes in speech, hearing, or vision. If untreated, signs and symptoms may progress to unconsciousness, seizures, coma, and death. Polydipsia, polyuria, and polyphagia are symptoms associated with hyperglycemia.

Which of the following is an adverse reaction to glipizide (Glucotrol)? 1. Headache 2. Constipation 3. Hypotension 4. Photosensitivity

4. Photosensitivity Glipizide may cause adverse skin reactions, such as rash, pruritus, and photosensitivity. It doesn't cause headache, constipation, or hypotension.

A 78-year-old client with pancreatic cancer has the following blood chemistry profile: Glucose, fasting: 204 mg/dL; BUN: 12 mg/dL; Creatinine: 0.9 mg/dL; Sodium: 136 mEq/L; Potassium: 2.2 mEq/L; Chloride: 99 mEq/L; CO2: 33 mEq/L. Which result should the nurse identify as critical and report immediately? 1. CO2 2. Sodium 3. Chloride 4. Potassium

4. Potassium A normal potassium level is 3.8 to 5.5mEq/L. Severe hypokalemia can cause cardiac and respiratory arrest, possibly leading to death. Hypokalemia also depresses the release of insulin and results in glucose intolerance. The glucose level is above normal (normal is 75 to 110 mg/dL) and the chloride level is a bit low (normal is 100 to 110 mEq/L). Although these levels should be reported, neither is life-threatening. The BUN (normal is 8 to 26 mg/dL) and creatinine (normal is 0.8 to 1.4 mg/dL) are within normal range.

Which of the following terms is used to describe the amount of stretch on the myocardium at the end of diastole? a. Afterload b. Cardiac index c. Cardiac output d. Preload

4. Preload is the amount of stretch of the cardiac muscle fibers at the end of diastole. The volume of blood in the ventricle at the end of diastole determines the preload. Afterload is the force against which the ventricle must expel blood. Cardiac index is the individualized measurement of cardiac output, based on the client's body surface area. Cardiac output is the amount of blood the heart is expelling per minute.

A client with chronic renal failure has a serum potassium level of 6.8 mEq/L. What should the nurse assess first? 1. Blood pressure 2. Respirations 3. Temperature 4. Pulse

4. Pulse An elevated serum potassium level may lead to a life-threatening cardiac arrhythmia, which the nurse can detect immediately by palpating the pulse. The client's blood pressure may change, but only as a result of the arrhythmia. Therefore, the nurse should assess blood pressure later. The nurse also can delay assessing respirations and temperature because these aren't affected by the serum potassium level.

The nurse is assessing a client who is experiencing an acute episode of cholecystitis. Where should the nurse anticipate the location of the pain? 1. Right lower quadrant, radiating to the back 2. Right lower quadrant, radiating to the umbilicus 3. Right upper quadrant, radiating to the left scapula and shoulder 4. Right upper quadrant, radiating to the right scapula and shoulder

4. Right upper quadrant, radiating to the right scapula and shoulder During an acute episode of cholecystitis, the client may complain of severe right upper quadrant pain that radiates to the right scapula and shoulder . This is determined by the pattern of dermatomes in the body. The other options are incorrect.

When teaching a client about insulin therapy, the nurse should instruct the client to avoid which over-the-counter preparation that can interact with insulin? 1. Antacids 2. Acetaminophen preparations 3. Vitamins with iron 4. Salicylate preparations

4. Salicylate preparations Salicylates may interact with insulin to cause hypoglycemia. Antacids, acetaminophen preparations, and vitamins with iron don't interact with insulin.

The nurse is teaching a group of middle-aged men about peptic ulcers. When discussing risk factors for peptic ulcers, the nurse should mention: 1. a sedentary lifestyle and smoking. 2. a history of hemorrhoids and smoking. 3. alcohol abuse and a history of acute renal failure. 4. alcohol abuse and smoking.

4. alcohol abuse and smoking. Risk factors for peptic (gastric and duodenal) ulcers include alcohol abuse, smoking, and stress. A sedentary lifestyle and a history of hemorrhoids aren't risk factors for peptic ulcers. Chronic renal failure, not acute renal failure, is associated with duodenal ulcers.

A client with a tentative diagnosis of hyperosmolar hyperglycemic nonketotic syndrome (HHNS) has a history of type 2 diabetes that is being controlled with an oral diabetic agent, tolazamide (Tolinase). Which of the following is the most important laboratory test for confirming this disorder? 1. Serum potassium level 2. Serum sodium level 3. Arterial blood gas (ABG) values 4. Serum osmolarity

4. Serum osmolarity Serum osmolarity is the most important test for confirming HHNS; it's also used to guide treatment strategies and determine evaluation criteria. A client with HHNS typically has a serum osmolarity of more than 350 mOsm/L. Serum potassium, serum sodium, and ABG values are also measured, but they aren't as important as serum osmolarity for confirming a diagnosis of HHNS. A client with HHNS typically has hypernatremia and osmotic diuresis. ABG values reveal acidosis, and the potassium level is variable.

A client with nausea, vomiting, and abdominal cramps and distention is admitted to the health care facility. Which test result is most significant? 1. Blood urea nitrogen (BUN) level of 29 mg/dl 2. Serum sodium level of 132 mEq/L 3. Urine specific gravity of 1.025 4. Serum potassium level of 3 mEq/L

4. Serum potassium level of 3 mEq/L A serum potassium level of 3 mEq/L is below normal, indicating hypokalemia. Because hypokalemia may cause cardiac arrhythmias and asystole, it's the most significant finding. In a client with a potential fluid volume imbalance, such as from vomiting, the other options are expected and none is as life-threatening as hypokalemia. A BUN level of 29 mg/dl indicates slight dehydration, probably caused by vomiting. A serum sodium level of 132 mEq/L is slightly below normal but not life-threatening. A urine specific gravity of 1.025 is normal.

A client is admitted with nausea, vomiting, and diarrhea. His blood pressure on admission is 74/30 mm Hg. The client is oliguric and his blood urea nitrogen (BUN) and creatinine levels are elevated. The physician will most likely write an order for which treatment? 1. Force oral fluids. 2. Administer furosemide (Lasix) 20 mg I.V. 3. Start hemodialysis after a temporary access is obtained. 4. Start I.V. fluids with a normal saline solution bolus followed by a maintenance dose.

4. Start I.V. fluids with a normal saline solution bolus followed by a maintenance dose. The client is in prerenal failure caused by hypovolemia. I.V. fluids should be given with a bolus of normal saline solution followed by maintenance I.V. therapy. This treatment should rehydrate the client, causing his blood pressure to rise, his urine output to increase, and the BUN and creatinine levels to normalize. The client wouldn't be able to tolerate oral fluids because of the nausea, vomiting, and diarrhea. The client isn't fluid-overloaded so his urine output won't increase with furosemide, which would actually worsen the client's condition. The client doesn't require dialysis because the oliguria and elevated BUN and creatinine levels are caused by dehydration.

A client with heart failure admitted to an acute care facility and is found to have a cystocele. When planning care for this client, the nurse is most likely to formulate which nursing diagnosis? 1. Total urinary incontinence 2. Functional urinary incontinence 3. Reflex urinary incontinence 4. Stress urinary incontinence

4. Stress urinary incontinence Stress urinary incontinence is a urinary problem associated with cystocele — herniation of the bladder into the birth canal. Other problems associated with this disorder include urinary frequency, urinary urgency, urinary tract infection (UTI), and difficulty emptying the bladder. Total incontinence, functional incontinence, and reflex incontinence usually result from neurovascular dysfunction, not cystocele.

Which outcome indicates effective client teaching to prevent constipation? 1. The client verbalizes consumption of low-fiber foods. 2. The client maintains a sedentary lifestyle. 3. The client limits water intake to three glasses per day. 4. The client reports engaging in a regular exercise regimen.

4. The client reports engaging in a regular exercise regimen. A regular exercise regimen promotes peristalsis and contributes to regular bowel elimination patterns. A low-fiber diet, a sedentary lifestyle, and limited water intake would predispose the client to constipation.

A client has a newly created colostomy. After participating in counseling with the nurse and receiving support from the spouse, the client decides to change the colostomy pouch unaided. Which behavior suggests that the client is beginning to accept the change in body image? 1. The client closes the eyes when the abdomen is exposed. 2. The client avoids talking about the recent surgery. 3. The client asks the spouse to leave the room. 4. The client touches the altered body part.

4. The client touches the altered body part. By touching the altered body part, the client recognizes the body change and establishes that the change is real. Closing the eyes, not looking at the abdomen when the colostomy is exposed, or avoiding talking about the surgery reflects denial, instead of acceptance of the change. Asking the spouse to leave the room signifies that the client is ashamed of the change and not coping with it.

A client is admitted to the hospital after vomiting bright red blood and is diagnosed with a bleeding duodenal ulcer. The client develops a sudden, sharp pain in the midepigastric region along with a rigid, boardlike abdomen. These clinical manifestations most likely indicate which of the following? 1. An intestinal obstruction has developed. 2. Additional ulcers have developed. 3. The esophagus has become inflamed. 4. The ulcer has perforated.

4. The ulcer has perforated. The body reacts to perforation of an ulcer by immobilizing the area as much as possible. This results in boardlike abdominal rigidity, usually with extreme pain. Perforation is a medical emergency requiring immediate surgical intervention because peritonitis develops quickly after perforation. An intestinal obstruction would not cause midepigastric pain. The development of additional ulcers or esophageal inflammation would not cause a rigid, boardlike abdomen.

A client is scheduled to undergo a left hemicolectomy for colorectal cancer. The physician prescribes phenobarbital (Luminal), 100 mg I.M. 60 minutes before surgery, for sedation. Which statement accurately describes administration of phenobarbital? 1. The onset of action for I.M. administration is 30 to 60 minutes. 2. This drug can be mixed and given with other medications. 3. This drug should be used within 24 hours after opening. 4. This drug should be injected into a large muscle mass.

4. This drug should be injected into a large muscle mass. Phenobarbital should be injected into a large muscle mass. The onset of action by the I.M. route is 10 to 30 minutes. Barbiturates are involved in many drug interactions, so the drug shouldn't be mixed or given with other medications. The drug solution should be used within 30 minutes after opening to minimize deterioration.

A client is undergoing an extensive diagnostic workup for a suspected GI problem. The nurse discovers that the client has a family history of ulcer disease. Which blood type also is a risk factor for duodenal ulcers? 1. Type A 2. Type B 3. Type AB 4. Type O

4. Type O Duodenal ulcers are more common in people with type O blood, suggesting a genetic basis. Types A, B, and AB blood aren't associated with an increased incidence of duodenal ulcers.

A client admitted with a gunshot wound to the abdomen is transferred to the intensive care unit after an exploratory laparotomy. I.V. fluid is being infused at 150 ml/hr. Which assessment finding suggests that the client is experiencing acute renal failure (ARF)? 1. Blood urea nitrogen (BUN) level of 22 mg/dl 2. Serum creatinine level of 1.2 mg/dl 3. Temperature of 100.2° F (37.8° C) 4. Urine output of 250 ml/24 hours

4. Urine output of 250 ml/24 hours ARF, characterized by abrupt loss of kidney function, commonly causes oliguria, which is demonstrated by a urine output of 250 ml/24 hours. A serum creatinine level of 1.2 mg/dl isn't diagnostic of ARF. A BUN level of 22 mg/dl or a temperature of 100.2° F (37.8° C) wouldn't result from this disorder.

The client underwent a transurethral resection of the prostate gland 24 hours ago and is on continuous bladder irrigation. Which nursing intervention is appropriate? 1. Tell the client to try to urinate around the catheter to remove blood clots. 2. Restrict fluids to prevent the client's bladder from becoming distended. 3. Prepare to remove the catheter. 4. Use aseptic technique when irrigating the catheter.

4. Use aseptic technique when irrigating the catheter. If the catheter is blocked by blood clots, it may be irrigated according to physician's orders or facility protocol. The nurse should use sterile technique to reduce the risk of infection. Urinating around the catheter can cause painful bladder spasms. Encourage the client to drink fluids to dilute the urine and maintain urine output. The catheter remains in place for 2 to 4 days after surgery and is only removed with a physician's order.

A client with liver and renal failure has severe ascites. On initial shift rounds, his primary nurse finds his indwelling urinary catheter collection bag too full to store more urine. The nurse empties more than 2,000 ml from the collection bag. One hour later, she finds the collection bag full again. The nurse notifies the physician, who suspects that a bladder rupture is allowing the drainage of peritoneal fluid. The physician orders a urinalysis to be obtained immediately. If the physician's suspicion is correct, the urine will abnormally contain: 1. creatinine. 2. urobilinogen. 3. chloride. 4. albumin.

4. albumin. Albumin is an abnormal finding in a routine urine specimen. Ascites present in liver failure contain albumin; therefore, if the bladder ruptured, ascites containing albumin would drain from the indwelling urinary catheter because the catheter is no longer contained in the bladder. Creatinine, urobilinogen, and chloride are normally found in urine.

A client with liver and renal failure has severe ascites. On initial shift rounds, his primary nurse finds his indwelling urinary catheter collection bag too full to store more urine. The nurse empties more than 2,000 ml from the collection bag. One hour later, she finds the collection bag full again. The nurse notifies the physician, who suspects that a bladder rupture is allowing the drainage of peritoneal fluid. The physician orders a urinalysis to be obtained immediately. If the physician's suspicion is correct, the urine will abnormally contain: 1. creatinine. 2. urobilinogen. 3. chloride. 4. albumin.

4. albumin. Albumin is an abnormal finding in a routine urine specimen. Ascites present in liver failure contain albumin; therefore, if the bladder ruptured, ascites containing albumin would drain from the indwelling urinary catheter because the catheter is no longer contained in the bladder. Creatinine, urobilinogen, and chloride are normally found in urine.

Extracorporeal shock wave lithotripsy (ESWL)

A noninvasive mechanical procedure for breaking up stones located in the kidney or upper ureter so that they can pass spontaneously or be removed by other methods No incision is made and no drains are placed; a stent may be placed to facilitate passing stone fragments. Fluoroscopy is used to visualize the stone and ultrasonic waves are delivered to the area of the stone to disintegrate it. The stones are passed in the urine within a few days.

A client with a bleeding peptic ulcer is admitted to an acute care facility. As part of therapy, the physician prescribes cimetidine (Tagamet) I.V. The nurse must avoid administering this drug too rapidly because doing so may cause: 1. tetany. 2. bronchospasms. 3. hallucinations. 4. bradycardia.

4. bradycardia. When given by rapid I.V. injection, cimetidine may cause profound bradycardia and other cardiotoxic effects. Tetany and bronchospasms aren't associated with cimetidine. Although the drug may cause hallucinations, this adverse reaction doesn't result simply from rapid administration.

A client is in the late stage of cirrhosis. When planning the client's diet, the nurse should focus on providing increased amounts of: 1. fat. 2. fiber. 3. protein. 4. carbohydrate.

4. carbohydrate. Normally, the liver performs many metabolic functions that provide energy for the body. In cirrhosis, the liver's metabolic function is compromised, increasing the client's need for dietary carbohydrates and other energy sources to provide for cellular metabolism. The nurse should limit the client's fat intake to prevent satiation and should restrict protein intake because a cirrhotic liver can't metabolize protein efficiently. Increasing fiber intake isn't significant for a client with cirrhosis.

The nurse is providing dietary instructions to a client with hypoglycemia. To control hypoglycemic episodes, the nurse should recommend: 1. increasing saturated fat intake and fasting in the afternoon. 2. increasing intake of vitamins B and D and taking iron supplements. 3. eating a candy bar if light-headedness occurs. 4. consuming a low-carbohydrate, high-protein diet and avoiding fasting.

4. consuming a low-carbohydrate, high-protein diet and avoiding fasting. To control hypoglycemic episodes, the nurse should instruct the client to consume a low-carbohydrate, high-protein diet, avoid fasting, and avoid simple sugars. Increasing saturated fat intake and increasing vitamin supplementation wouldn't help control hypoglycemia.

A client with hepatitis C develops liver failure and GI hemorrhage. The blood products that would most likely bring about hemostasis in the client are: 1. whole blood and albumin. 2. platelets and packed red blood cells. 3. fresh frozen plasma and whole blood. 4. cryoprecipitate and fresh frozen plasma.

4. cryoprecipitate and fresh frozen plasma. The liver is vital in the synthesis of clotting factors, so when it's diseased or dysfunctional, as in hepatitis C, bleeding occurs. Treatment consists of administering blood products that aid clotting. These include fresh frozen plasma containing fibrinogen and cryoprecipitate, which have most of the clotting factors. Although administering whole blood, albumin, and packed cells will contribute to hemostasis, those products aren't specifically used to treat hemostasis. Platelets are helpful, but the best answer is cryoprecipitate and fresh frozen plasma.

Which medication should the nurse expect to administer to a client with constipation? 1. lorazepam (Ativan) 2. loperamide (Imodium) 3. flurbiprofen (Ansaid) 4. docusate sodium (Colace)

4. docusate sodium (Colace) Docusate sodium, a laxative, is used to treat constipation. It softens the stool by stimulating the secretion of intestinal fluid into the stool. Lorazepam, an antianxiety agent, has no laxative effect. Administering loperamide, an antidiarrheal agent, could cause the constipation to worsen. Flurbiprofen is a nonsteroidal anti-inflammatory agent with no laxative effect.

A client tells the nurse that she has been working hard for the last 3 months to control her type 2 diabetes mellitus with diet and exercise. To determine the effectiveness of the client's efforts, the nurse should check: 1. urine glucose level. 2. fasting blood glucose level. 3. serum fructosamine level. 4. glycosylated hemoglobin level.

4. glycosylated hemoglobin level. Because some of the glucose in the bloodstream attaches to some of the hemoglobin and stays attached during the 120-day life span of red blood cells, glycosylated hemoglobin levels provide information about blood glucose levels during the previous 3 months. Fasting blood glucose and urine glucose levels only give information about glucose levels at the point in time when they were obtained. Serum fructosamine levels provide information about blood glucose control over the past 2 to 3 weeks.

The physician prescribes lactulose (Cephulac), 30 ml three times daily, when a client with cirrhosis develops an increased serum ammonia level. To evaluate the effectiveness of lactulose, the nurse should monitor: 1. urine output. 2. abdominal girth. 3. stool frequency. 4. level of consciousness (LOC).

4. level of consciousness (LOC). In cirrhosis, the liver fails to convert ammonia to urea. Ammonia then builds up in the blood and is carried to the brain, causing cerebral dysfunction. When this occurs, lactulose is administered to promote ammonia excretion in the stool and thus improve cerebral function. Because LOC is an accurate indicator of cerebral function, the nurse can evaluate the effectiveness of lactulose by monitoring the client's LOC. Monitoring urine output, abdominal girth, and stool frequency helps evaluate the progress of cirrhosis, not the effectiveness of lactulose.

Alterations in hepatic blood flow resulting from a drug interaction also can affect: 1. onset and duration. 2. distribution and excretion. 3. absorption and metabolism. 4. metabolism and excretion.

4. metabolism and excretion. Alterations in hepatic blood flow resulting from a drug interaction can affect metabolism and excretion. Changes in hepatic blood flow don't affect absorption, so a drug's onset of action is also unchanged. Further, alterations in hepatic blood flow don't affect distribution.

Following an earthquake, a client who was rescued from a collapsed building is seen in the emergency department. He has blunt trauma to the thorax and abdomen. The nursing observation that most suggests the client is bleeding is: 1. a prolonged partial thromboplastin time (PTT). 2. a recent history of warfarin (Coumadin) usage. 3. diminished breath sounds. 4. orthostatic hypotension.

4. orthostatic hypotension. Bleeding is a volume-loss problem, which causes a drop in blood pressure. As the bleeding persists and the body's ability to compensate declines, orthostatic hypotension becomes evident. A prolonged PTT and a history of warfarin usage are causes of bleeding but aren't evidence of bleeding. As bleeding persists and the client's level of consciousness declines, breathing will become more shallow and breath sounds will diminish; however, this is a late and unreliable manifestation of bleeding.

A 25-year-old female client seeks care for a possible infection. Her symptoms include burning on urination and frequent, urgent voiding of small amounts of urine. She's placed on trimethoprim-sulfamethoxazole (Bactrim) to treat possible infection. Another medication is prescribed to decrease the pain and frequency. Which is the most likely medication prescribed for the pain? 1. nitrofurantoin (Macrodantin) 2. ibuprofen (Motrin) 3. acetaminophen with codeine 4. phenazopyridine (Pyridium)

4. phenazopyridine (Pyridium) Phenazopyridine may be prescribed in conjunction with an antibiotic for painful bladder infections to promote comfort. Because of its local anesthetic action on the urinary mucosa, phenazopyridine specifically relieves bladder pain. Nitrofurantoin is a urinary antiseptic with no analgesic properties. Although ibuprofen and acetaminophen with codeine are analgesics, they don't exert a direct effect on the urinary mucosa.

The nurse is assigned to care for a postoperative client who has diabetes mellitus. During the assessment interview, the client reports that he's impotent and says he's concerned about its effect on his marriage. In planning this client's care, the most appropriate intervention would be to: 1. encourage the client to ask questions about personal sexuality. 2. provide time for privacy. 3. provide support for the spouse or significant other. 4. suggest referral to a sex counselor or other appropriate professional.

4. suggest referral to a sex counselor or other appropriate professional. The nurse should refer this client to a sex counselor or other professional. Making appropriate referrals is a valid part of planning the client's care. The nurse doesn't normally provide sex counseling.

Assessment for PAD

5 Ps: pain, pallor, pulselessness, parasthesia, poikilothermia - skin appearance (pale/shiny)/temperature (cool), ulcers, may have depression/anxiety re: amputation

Signs of Bowel Perforation and Peritonitis

Guarding of the abdomen Increased fever and chills Pallor Progressive abdominal distention and abdominal pain Restlessness Tachycardia and tachypnea

PaO2 range for O2 deprivation

50 - 60 mmHg

Normal ejection fraction

50 - 70%

Prophylactic isoniazid therapy must continue for

6 to 12 months

Approximately what percentage of an adult is fluid?

60% - varies with age, body size, and gender

Hypocalcemia lab value

< 8.5 mEq/L

How many times in an hour should a patient have sustained, voluntary inflation with the incentive spirometer?

8 - 10

Normal PaO2

80 - 100 mmHg

Normal blood glucose level

80 - 20 mg/dl

1 Gram Fat =

9 calories

Normal O2 Saturation

95 - 100%

Hypomagnesemia lab value

< 1.5 mEq/L

Hypophosphatemia lab value

< 1.7 mEq/L

Hyponatremia lab value

< 135 mEq/L

Hyperphosphatemia lab value

< 2.5 mEq/L

Hypokalemia lab value

< 3.5 mEq/L

Metabolic acidosis lab values

pH < 7.35, bicarb < 22 mEq/L

t-tube

A T-tube is placed after surgical exploration of the common bile duct. The tube preserves the patency of the duct and ensures drainage of bile until edema resolves and bile is effectively draining into the duodenum. A gravity drainage bag is attached to the T-tube to collect the drainage.

Crohn's disease

A chronic progressive disease of the liver characterized by diffuse degeneration and destruction of hepatocytes. Repeated destruction of hepatic cells causes the formation of scar tissue.

Kock pouch

A continent internal ileal reservoir created from a segment of the ileum and ascending colon. The ureters are implanted into the side of the reservoir, and a special nipple valve is constructed to attach the reservoir to the skin. Postoperatively, the client will have a Foley catheter in place to drain urine continuously until the pouch has healed. The catheter is irrigated gently with normal saline to prevent obstruction from mucus or clots. Following removal of the catheter, the client is instructed in how to self-catheterize and to drain the reservoir at 4- to 6-hour intervals

Polycystic kidney disease

A cystic formation and hypertrophy of the kidneys, which leads to cystic rupture, infection, formation of scar tissue, and damaged nephrons There is no specific treatment to arrest the progress of the destructive cysts. The ultimate result of this disease is renal failure.

Lower gastrointestinal tract study (barium enema)

A fluoroscopic and radiographic examination of the large intestine is performed after rectal instillation of barium sulfate. The study may be done with or without air. Preprocedure: low-fiber diet is given for 1 to 2 days before the test - A clear liquid diet and laxative are given the evening before the test. - NPO after midnight the day of the test - Cleansing enemas may be prescribed on the morning of the test. Postprocedure: Instruct the client to increase oral fluid intake to help pass the barium. - Administer a mild laxative as prescribed to facilitate emptying of the barium. - Monitor stools for the passage of barium. - Notify the physician if a bowel movement does not occur within 2 days.

Albumin

A main plasma protein of blood - Maintains oncotic pressure and transports bilirubin, fatty acids, medications, hormones, and other substances that are insoluble in water Increased in conditions such as dehydration, diarrhea, and metastatic carcinoma; decreased in conditions such as acute infection, ascites, and alcoholism Presence of detectable albumin, or protein, in the urine is indicative of abnormal renal function

Liver biopsy

A needle is inserted through the abdominal wall to the liver to obtain a tissue sample for biopsy and microscopic examination. Preprocedure: Assess results of coagulation tests (prothrombin time, partial thromboplastin time, platelet count). - Administer a sedative as prescribed. - c. Note that the client is placed in the supine or left lateral position during the procedure to expose the right side of the upper abdomen. Postprocedure - Assess vital signs. - Assess biopsy site for bleeding. - Monitor for peritonitis - Maintain bedrest for several hours. - Place the client on the right side with a pillow under the costal margin to decrease the risk of hemorrhage, and instruct the client to avoid coughing and straining. - Instruct the client to avoid heavy lifting and strenuous exercise for 1 week.

Continuous Bladder Irrigation

A three-way (lumen) irrigation is used to decrease bleeding and to keep the bladder free from clots—one lumen is for inflating the balloon (30 mL); one lumen is for instillation (inflow); one lumen is for outflow.

You are preparing to give an enteral feeding through a nasogastric tube. Place the steps in the correct order. a. Assess for bowel sounds. b. Auscultate tube placement and check pH. c. Flush the tube with water. d. Reflush the tube with water. e. Administer the feeding. f. Check for residual volume.

A, B, F, C, E & D - Assessment is the first step. Checking for tube placement prevents accidentally instilling feeding/medication into the lungs. The amount of residual volume determines whether the amount of the scheduled feeding is appropriate or whether the physician should be notified. Flushing the tube before and after feeding helps maintain tube patency.

In educating a client with gastroesophageal reflux disease (GERD), you will teach the client that the drug therapy is a "step-up" approach that depends on the response to the medication. For the drugs listed, what is the anticipated order that the physician will try in the treatment plan? a. Magnesium trisilicate (Gaviscon) and Famotidine (Pepci AC) b. Ranitidine (Zantac) 150 mg c. Pantoprazole (Protonix)

A, B & C - First, antacids and over-the-counter histamine - 2 blockers are used. In step two, prescription histamine - 2 blockers are prescribed. Finally, proton-pump inhibitors are used.

Signs and Symptoms of GERD

Heartburn, Epigastric pain, Dyspepsia, Regurgitation, Pain and difficulty with swallowing, Hypersalivation

Signs and Symptoms of Hiatal Hernia

Heartburn, Regurgitation or vomiting, Dysphagia, Feeling of fullness

Respiratory alkalosis lab values

pH > 7.45, CO2 < 35 mmHg

What to assess on patient on O2

ABGs, O2 sat, vital signs, lung sounds, hx, ekg changes, restlessness/confusion

Hiatal Hernia

AKA esophageal or diaphragmatic hernia. A portion of the stomach herniates through the diaphragm and into the thorax. Herniation results from weakening of the muscles of the diaphragm and is aggravated by factors that increase abdominal pressure such as pregnancy, ascites, obesity, tumors, and heavy lifting. Complications include ulceration, hemorrhage, regurgitation and aspiration of stomach contents, strangulation, and incarceration of the stomach in the chest with possible necrosis, peritonitis, and mediastinitis.

Metabolic alkalosis lab values

pH > 7.45, bicarb > 26 MeQ/L

S&S pancreatitis

Abdominal pain, including a sudden onset at a midepigastric or left upper quadrant location with radiation to the back Pain aggravated by a fatty meal, alcohol, or lying in a recumbent position - flexion of spine may relieve Abdominal tenderness and guarding Nausea and vomiting Weight loss Absent or decreased bowel sounds Elevated white blood cell count, and glucose, bilirubin, alkaline phosphatase, and urinary amylase levels Elevated serum lipase and amylase levels

Partial compensation ABB disorder

Abnormal pH, both CO2 and bicarb either high or low

Basic/Primary ABB disorder

Abnormal pH, either CO2 or bicarb normal and other one abnormal

Tachypnea

Abnormally rapid respirations

Cirrhosis and Ascites

Accumulation of fluid in the peritoneal cavity that results from venous congestion of the hepatic capillaries. Capillary congestion leads to plasma leaking directly from the liver surface and portal vein. Due to increased BP, increased osmotic pressure, decreased colloidal oncotic pressure, and hyperaldosteronism.

captopril, enalapril

Ace Inhibitors

Action of Central Acting Inhibitors

Act on the CNS in the brain, inhibit vasoconstriction

Signs and Symptoms of Gastritis

Acute: Abdominal discomfort, Anorexia, nausea, and vomiting, Headache, Hiccupping Chronic: Anorexia, nausea, and vomiting, Belching, Heartburn after eating, Sour taste in the mouth, Vitamin B12 deficiency

Risk factors for hypertension

Age, sex, race, fam hx, obesity, smoking, Na intake, lipid levels, alcohol, sedentary lifestyle, diabetes, stress

Interventions for Bronchitis

Airway clearance, O2, antibiotics, hydration, bronchodilators, chest physiotherapy, breathing exercise

How does the endocrine system control blood pressure?

Aldosterone causes the retention of Na and water.

Hyperaldosteronism and cirrhosis

Aldosterone is not being metabolized by the liver and causes increased reabsorption of sodium and water retention, K+ loss - contributes to ascites

Coreg (cavedilol), labetolol

Alpha Beta blockers

Aluminum Based Antacids

Amphojel, Maalox, Tums side effect: constipation contain significant amounts of sodium

Pyelonephritis

An inflammation of the renal pelvis and the parenchyma commonly caused by bacterial invasion. Acute pyelonephritis often occurs after bacterial contamination of the urethra or following an invasive procedure of the urinary tract. Chronic pyelonephritis most commonly occurs following chronic urinary flow obstruction with reflux. Escherichia coli is the most common causative bacterial organism.

Chron's disease

An inflammatory disease that can occur anywhere in the gastrointestinal tract but most often affects the terminal ileum and leads to thickening and scarring, a narrowed lumen, fistulas, ulcerations, and abscesses. Characterized by remissions and exacerbations.

Renal angiography

An injection of a radiopaque dye through a catheter inserted into the femoral artery to examine the renal blood vessels and renal arterial supply Assess and mark the peripheral pulses. Maintain bed rest and apply a sandbag or other device that will provide pressure to prevent bleeding, if prescribed, at the insertion site for 4 to 8 hours. Instruct the client to maintain a supine position with the leg straight (the head of the bed should not be elevated greater than 20 degrees for 8 hours, (or as prescribed). Assess the temperature, color, movement, and sensation (CMS) of the toes of the involved extremity with each vital sign check. Inspect the catheter insertion site for bleeding or swelling with each vital sign check.

Kock ileostomy (continent ileostomy)

An intraabdominal pouch constructed from the terminal ileum. The pouch is connected to the stoma with a nipple-like valve constructed from a portion of the ileum. The stoma is flush with the skin.

Ureterolithotomy/pyelolithotomy and nephrolithotomy

An open surgical procedure performed if lithotripsy is not effective for removal of a stone in the ureter, renal pelvis, or kidney.

KUB (kidneys, ureters, and bladder) radiography

An x-ray of the urinary system and adjacent structures to detect urinary calculi.

Intravenous pyelography

An x-ray procedure in which an intravenous injection of a radiopaque dye is used to visualize and identify abnormalities in the renal system. Obtain an informed consent. Assess the client for allergies to iodine, seafood, and radiopaque dyes. Withhold food and fluids after midnight on the night before the test. Monitor vital signs. Monitor urinary output.

Pericarditis - treatment

Analgesics, antibiotics if bacterial, steroids, high fowler's leaning forward, monitor for signs of cardiac tamponade

Signs and symptoms of hypertensive crisis

Angina and dyspnes related to increased in SVR Headache, nausea, vomiting, seizure, confusion, coma, blurred vision, transient blindness related to cerebral edema and spasm of cerebral vessels

High Residue Diet

Increased fiber

Milk of Magnesia

Antacid - magnesium based side effect: diarrhea 5cc PO TID PC assess for hx of renal disease b/c can cause renal insufficiency and calcinosis

Nutritional Assessment

Anthropometrics (age, weight, height, sex, skinfold and arm circumfrence) Biochemical tests Clinical observation Dietary and personal Hx

Interventions for Bronchiectasis

Antibiotics, hydration, rest, cdb, surgical

Pharmacological therapy for urinary incontinence

Anticholinergic agents inhibit bladder contraction and are considered first-line medications for urge incontinence. Several tricyclic antidepressant medications can also decrease bladder contractions as well as increase bladder neck resistance. Pseudoephedrine sulfate (Sudafed), which acts on alpha-adrenergic receptors, causing urinary retention, may be used to treat stress incontinence; it needs to be used with caution in men with prostatic hyperplasia. Hormone therapy (eg, estrogen) taken orally, transdermally, or topically was once the treatment of choice for urinary incontinence in postmenopausal women because it restores the mucosal, vascular, and muscular integrity of the urethra.

Bentyl

Antispasmodic

Bypass procedures for PAD

Aortobifemoral, femoropopliteal (fem-pop), femorotibial

Veins vs Arteries

Arteries - carry oxygenated blood - thick walled/muscular - high pressure system Veins - carry deoxygenated blood - thin walls/can expand - have valves - low pressure system

Risk factors for MI

Aterosclerosis, CAD, elevated cholesterol, smoking, hypertension, obesity, physical inactivity, impaired glucose tolerance, stress

Treatment for sinus bradycardia

Atropine, transcutaneous or transvenous pacer

Preventing Dumping Syndrome

Avoid sugar, salt, and milk. Eat a high-protein, high-fat, low-carbohydrate diet. Eat small meals and avoid consuming fluids with meals. Lie down after meals. Take antispasmodic medications as prescribed to delay gastric emptying.

Water Soluble Vitamins

B & C - cannot be stored in the body - megadoses of C and B6 can lead to toxicity

Population most at risk for Hepatitis A

Commonly seen in young children, individuals in institutionalized settings, and health care personnel.

Diagnostic Tests for Bronchiectasis

Bronchoscopy, bronchogram, PFTs

Signs and symptoms of duodenal ulcers

Burning pain midepigastric area 1.5 - 3 hours after a meal and during the night melena pain is relieved by ingestion of food

Oxygen Hemoglobin Dissociation Curve

Compares PaO2 with SaO2 If PaO2 decreases from 100 -> 80, hgb remain saturated and hypoxia does not occur

Obesity

BMI of 30 or more high rate of hypertension, gout, cardiovascular disease, degenerative joint disease, Type 2 Diabetes, stroke, breast cancer, and sleep apnea (Pickwickian syndrome)

Vitamin B1/Thiamine Deficiency

Beriberi - anorexia, weight loss, muscle weakness/wasting, peripheral neuropathy - can progress to right sided heart failure

metoprolol, atenolol, nadolol

Beta blockers

Action of Beta Blockers

Blocks response to SNS stimulation thereby preventing vasoconstriction and tachycardia. Results in decreased heart rate and BP.

Cirrhosis and coagulation defects

Decreased synthesis of bile fats in the liver prevents the absorption of fat-soluble vitamins. Without vitamin K and clotting factors II, VII, IX, and X, the client is prone to bleeding.

Biguanides

Decreases glucose production by the liver and makes muscle more sensitive to insulin Less likely to cause hypoglycemic reaction Help improve lipid levels Do not take if kidney disease Side effects: nausea, diarrhea, metallic taste in mouth ie. Glucophage (metformin)

You are preparing to administer TPN through a central line. Place the steps for administration in the correct order. a. Use aseptic technique when handling the injection cap. b. Thread the IN tubing through an infusion pump. c. Check the solution for cloudiness or turbidity. d. Connect the tubing to the central line. e. Select the correct tubing and filter. f. Set infusion pump at prescribed rate.

C, E, B, A, D & F - The solution should not be cloudy or turbid. Prepare the equipment by priming the tubing and threading the pump. To prevent infection, use aseptic technique when inserting the connector into the injection cap and connecting the tubing to the central line. Set the pump at the prescribed rate.

Etiology of hypertensive crisis

poorly controlled hypertensives, hypertensives who have stopped taking their medications, substance abuse, ecclampsia

Holter monitor

portable 24hr EKG - patient is given a diary to record symptoms

% Carbs, Fats, Protein (energy req)

Carbs 50-60% Fat 30% Protein 10-20%

Action of Vasodilators

Causes systemic vasodilation, decrease preload and afterload/SVR, increase blood flow to the heart

Nursing considerations for Vasodilators

Check BOm can cause headaches, flushing, and palpatations

Nursing considerations for Calcium channel blockers

Check BP and pulse. Do not give to patients with heart block. Avoid grapefruit.

Nursing considerations for Beta blockers

Check BP and pulse. Don't administer to patients with COPD. Don't stop abruptly.

How is hypertension diagnosed?

Check BP in both arms three times and take highest number.

Major Nutrients in Protein Group

Complete and incomplete proteins, Vitamin B complex, Zinc, Iron, Phosphorus

Clinical presentation of MI

Chest pain not relieved by nitroglycerine Pallor Diaphoresis Nausea and vomiting Dyspnea Impending doom Altered LOC

Diabetic retinopathy

Chronic and progressive impairment of the retinal circulation that eventually causes hemorrhage. Permanent vision changes and blindness can occur. The client has difficulty with carrying out the daily tasks of blood glucose testing and insulin injections.

Biliary Cirrhosis

Cirrhosis develops from chronic biliary obstruction, bile stasis, and inflammation, resulting in severe obstructive jaundice.

Laënnec's Cirrhosis

Cirrhosis is alcohol-induced, nutritional, or portal. Cellular necrosis causes eventual widespread scar tissue, with fibrotic infiltration of the liver.

Cardiac Cirrhosis

Cirrhosis is associated with severe, right-sided congestive heart failure and results in an enlarged, edematous, congested liver. The liver becomes anoxic, resulting in liver cell necrosis and fibrosis.

Postnecrotic Cirrhosis

Cirrhosis occurs after massive liver necrosis. Cirrhosis results as a complication of hepatitis or exposure to hepatotoxins. Scar tissue causes destruction of liver lobules and entire lobes.

Shift of O2 Hgb Dissociation Curve

Curve is affected by CO2, H+ conc., temperature, and 2,3-DPG Increase in factors shifts curve to right, there is a decrease in O2's affinity for hgb Decrease in factors shifts curve to left, O2's affinity for hgb increases and will travel through circulation without being used

Esophageal Varices

Dilated and tortuous veins in the submucosa of the esophagus - Caused by portal hypertension, often associated with liver cirrhosis; are at high risk for rupture if portal circulation pressure rises

To prevent tracheal dilation in a patient with a tracheostomy tube...

a minimal-leak technique should be used and the pressure should be kept at less than 25 cm H2O

Predisposing factors for varicose veins

prolonged standing, familial, obesity, pregnancy

Ventilation Perfusion Scan

Detects P.E.

Urea breath test

Detects the presence of Helicobacter pylori, the bacteria that cause peptic ulcer disease. The client consumes a capsule of carbon-labeled urea and provides a breath sample 10 to 20 minutes later. H. pylori can also be detected by assessing serum antibody levels.

Dyspnea

Difficulty breathing, labored breathing, SOB

Hypokalemia increases the risk of toxicity for which drug?

Digitalis

Causes of PVCs

Digoxin toxicity, hypoxia, hypokalemia, fever, acidosis, exercise

Drugs used to teat valve disease

Digoxin, coumadin, amioderone, antibiotics

Treatment for A-Fib and A-Flutter

Digoxin, coumadin/pradaxa, calcium channel blockers/beta blockers, antiarrhythmics/amioderone

Bronchoscopy

Direct examination of larynx, trachea, and bronchi using and endoscope - invasive (consent) - make sure gag reflex returns - stridor and sore throat NL

Clubbing

Distal phalynx of fingers is rounded and bulbous - nail plate is more convex - caused by chronic hypoxia

Pharmacologic Management of Hypertenstion

Diuretics, beta blockers, calcium channel blockers, vasodilators, alpha andrenergic blockers, ace inhibitors

Causes of Hypokalemia

Diuretics, diarrhea/vomiting, gastric suction, hyperaldosteroneism, bulimia

How is PAD diagnosed?

Doppler studies, aniogram, ankle brachial index (compare pressures), pedal pulses

Thoracentesis

Drainage of excess pleural fluid - relieves pain - diagnostic - risk for pneumothorax

Signs and Symptoms of Respiratory Disorders

Dyspnea Tachypnea Increases sputum production/cough Hemoptysis Wheezing/stridor/hypoxia chest pain clubbing

Diagnostic tests for CAD/Angina

EKG (shows ischemia), cardiac catheterization (most definitive), blood lipid level, cardiac enzymes, stress test, holter monitor, echocardiogram, TEE, MUGA (ejection fraction)

Surgical management for Pancreatitis

ERCP percutaneous drainage of pseudocyst/abscess Whipple procedure (cancer)

Non-surgical procedures for cholecystitis

ERCP - remove stones with stent placement ESWL - shock waves used to disintegrate stones

What aggravates the symptoms of PAD?

Elevation of extremity - pain occurs distal to the occlusion - gravity aids in blood flow.

What is the drug of choice for treating Legionnaires' disease?

Erythromycin (Erythrocin)

Upper gastrointestinal tract study (barium swallow)

Examination of the upper gastrointestinal tract under fluoroscopy after the client drinks barium sulfate Preprocedure: NPO after midnight the day of the test Postprocedure: A laxative may be prescribed, Instruct the client to increase oral fluid intake to help pass the barium, Monitor stools (may use laxative) for the passage of barium (stools will appear chalky white) because barium can cause a bowel obstruction

Polycthemia

Excess # of circulating RBCs - risk for clots - may develop in patient's with chronic hypoxia/hypoxemia

Exercise and diabetes

Exercise lowers the blood glucose level, encourages weight loss, reduces cardiovascular risks, improves circulation and muscle tone, decreases total cholesterol and triglyceride levels, and decreases insulin resistance and glucose intolerance. Instruct the client in dietary adjustments when exercising; dietary adjustments are individualized. If the client requires extra food during exercise to prevent hypoglycemia, it need not be deducted from the regular meal plan. If the blood glucose level is higher than 250 mg/ dL and urinary ketones (type 1 diabetes mellitus) are present, the client is instructed not to exercise until the blood glucose level is closer to normal and urinary ketones are absent.

Hemoptysis

Expectoration of blood from the respiratory tract

Hemoglobin and Hematocrit Normals

Female: 12-14/40-48 Male: 13-16/42-48

HHNK/HHNS

Extreme hyperglycemia occurs without ketosis or acidosis. The syndrome occurs most often in individuals with type 2 diabetes mellitus. The major difference between HHNS and DKA is that ketosis and acidosis do not occur with HHNS; enough insulin is present with HHNS to prevent breakdown of fats for energy, thus preventing ketosis.

Colonoscopy

Fiberoptic endoscopy study in which the lining of the large intestine is visually examined; biopsies and polypectomies can be performed. - performed with the client lying on the left side with the knees drawn up to the chest; position may be changed during the test to facilitate passing of the scope. Preprocedure: Adequate cleansing of the colon is necessary, as prescribed by the physician. - A clear liquid diet is started at noon on the day before the test. - Consult with the physician regarding medications that must be withheld before the test. - Client is NPO after midnight on the day of the test. A mild sedative is administered intravenously. - Glucagon may be administered to relax smooth muscle. Postprocedure: Provide bedrest until alert. - Monitor for signs of bowel perforation and peritonitis - Instruct the client to report any bleeding to the physician.

Upper gastrointestinal fiberoscopy AKA esophagogastroduodenoscopy

Following sedation, an endoscope is passed down the esophagus to view the gastric wall, sphincters, and duodenum; tissue specimens can be obtained. Preprocedure: The client must be NPO for 6 to 12 hours before the test. - A local anesthetic (spray or gargle) is administered along with medication that provides conscious sedation and relieves anxiety - Atropine sulfate may be administered to reduce secretions, and glucagon may be administered to relax smooth muscle. - Client is positioned on the left side to facilitate saliva drainage and to provide easy access of the endoscope. Airway patency is monitored during the test and pulse oximetry is used to monitor oxygen saturation; emergency equipment should be readily available. Postprocedure: Client must be NPO until the gag reflex returns (1 to 2 hours). - Monitor for signs of perforation (pain, bleeding, unusual difficulty swallowing, elevated temperature). - Maintain bedrest for the sedated client until alert. - Lozenges, saline gargles, or oral analgesics can relieve a minor sore throat (not given to the client until the gag reflex returns).

Respiration

Gas exchange between the atmospheric air and the blood and between the blood and cells of body

Gastric analysis

Gastric analysis requires the passage of a nasogastric tube into the stomach to aspirate gastric contents for the analysis of acidity (pH), appearance, and volume; the entire gastric contents are aspirated, and then specimens are collected every 15 minutes for 1 hour. Histamine or pentagastrin may be administered subcutaneously to stimulate gastric secretions; these medications may produce a flushed feeling. Esophageal reflux of gastric acid may be diagnosed by ambulatory pH monitoring; a probe is placed just above the lower esophageal sphincter and connected to an external recording device. It provides a computer analysis and graphic display of results. Preprocedure: Fasting for 8 to 12 hours is required before the test. - Use of tobacco and chewing gum are avoided for 6 hours before the test. - Medications that stimulate gastric secretions are withheld for 24 to 48 hours. Postprocedure: Client may resume normal activities. - Refrigerate gastric samples if not tested within 4 hours.

Diabetic neuropathy

General deterioration of the nervous system throughout the body b. Complications include the development of nonhealing ulcers of the feet, gastric paresis, and erectile dysfunction.

GRAS

Generally Recognized As Safe - FDA list for food additives

Phosphorus plays a role in

Generation of bone tissue, metabolism of glucose and lipids, ABB - has inverse relationship with calcium levels

Signs and symptoms of gastric ulcers

Gnawing sharp pain in or left of the midepigastric region 30 - 60 minutes after a meal food ingestion accentuates pain hematemesis

Stridor

Harsh high pitched sound heard on inspiration, usually without need of stethoscope, secondary to upper airway obstruction

Assessment/signs/symptoms for esophageal varices

Hematemesis; Melena; Tarry stools; Ascites; Jaundice; Hepatomegaly; splenomegaly; Dilated abdominal veins; Signs of shock

Questions to ask a patient with diarrhea

History of travel? Diet? Medications? Previous surgery? Family history? Personal contacts?

Signs and Symptoms of Laryngeal Tumor

Hoarseness, dysphagia, dyspnea, pain

Pathophysiology of Bronchitis

Hyperactivity and enlargement of mucous secreting glands causing inflammation, narrow airways, and decreased cilliary efficiency

Hallmark of excess parathyroid hormone levels

Hypercalcemia

Which electrolyte imbalance causes tall T waves on EKG?

Hyperkalemia

Causes of repiratory alkalosis

Hyperventilation, stimulation of CNS resp centers, mechanical overventilation

Diuretics can cause which electrolyte imbalance?

Hypokalemia

Muscle weakness, diarrhea, and hypo reflex of DTRs indicates which electrolyte imbalance?

Hypokalemia

Dysphasia is common with which electrolyte imbalance?

Hypomagnesemia

Nursing management for fluid volume deficit

I&O, VS, oral care, oral and parenteral fluids, monitor sx - skin turgor, urinary output, mental status

Nursing management for fluid volume excess

I&O, daily weights, assess lung sounds, edema, monitor response to meds (diuretics), promote adherence to fluid restriction, patient ed on fluid restriction, avoid sodium, promote rest, semi fowler's position for orthopnea, skin care, positioning/turning

Interventions for Hyperkalemia

IV 10-20% glucose with insulin, kayexalate, monitor EKG, dialysis

PVCs are treated with

IV Lidocaine

Interventions for Hypomagnesemia

IV and oral magnesium

Endocarditis - treatment

IV antibiotics (maybe at home, PIC), rest, ROM, valve repair, anticoagulants

Interventions for Hypernatremia

IV solutions without Na (hypotonic), decrease Na intake

Sulfonylureas

Increase insulin production by pancreas by stimulating beta cells - also increases receptor sensitivity 1st and 2nd generation - 2nd gen have fewer side effects and more expensive Used in patients > 40 w/ diabetes < 5 years ie. Diabinese, Glucotrol, Amaryl

Thiazolidinediones

Increase uptake of sugar by muscle and fat cells - also reduce glucose production in the liver ie. Actos

Respiratory assessment findings for FVE

Increased RR, shallow resp, dyspnea, crackles

Causes of Bronchiectasis

Infection, TB, fungi, cystic fibrosis, lung abcess

Pneumonia

Infection/inflammation of lung parenchyma

Bronchitis - definition and etiliology

Inflammation and infection of bronchioles and trachea, can be chronic or acute - etiology can be bacterial, viral, or due to pulmonary irritants

diverticulitis

Inflammation of one or more diverticula from penetration of fecal matter through the thin-walled diverticula, resulting in local abscess formation. A perforated diverticulum can progress to intra-abdominal perforation with generalized peritonitis.

Appendicitis

Inflammation of the appendix. When the appendix becomes inflamed or infected, rupture may occur within a matter of hours, leading to peritonitis and sepsis.

Gastritis

Inflammation of the stomach or gastric mucosa Acute gastritis is caused by the ingestion of food contaminated with disease-causing microorganisms or food that is irritating or too highly seasoned, the overuse of aspirin or other nonsteroidal anti-inflammatory drugs (NSAIDs), excessive alcohol intake, bile reflux, or radiation therapy. Chronic gastritis is caused by benign or malignant ulcers or by the bacteria H. pylori, and also may be caused by autoimmune diseases, dietary factors, medications, alcohol, smoking, or reflux.

Action of Ace inhibitors

Inhibits conversionj of angiotensis I to angiotensin II, thereby inhibiting vasoconstriction

Renal biopsy

Insertion of a needle into the kidney to obtain a sample of tissue for examination; usually done percutaneously Preprocedure interventions: Assess vital signs. Assess baseline coagulation studies; notify the physician if abnormal results are noted. Obtain an informed consent. NPO Postprocedure interventions: Monitor vital signs, especially for hypotension and tachycardia, which could indicate bleeding. Provide pressure to the biopsy site for 30 minutes. Monitor the hemoglobin and hematocrit levels for decreases, which could indicate bleeding. Place the client in the supine position and on bed rest for 8 hours as prescribed. Check the biopsy site and under the client for bleeding. Encourage fluid intake of 1500 to 2000 mL as prescribed. Observe the urine for gross and microscopic bleeding. Instruct the client to avoid heavy lifting and strenuous activity for 2 weeks. Instruct the client to notify the physician if either a temperature greater than 100 ° F or hematuria occurs after the first 24 hours postprocedure.

Gluten Free Diet

Intolerance to barley, rye, oat, and wheat

3 Grades of Myocardial Insult

Ischemia - not enough O2 Injury - enzymes released Infarction - permanent irreversible damage

Later Manifestations of Cirrhosis

Jaundice Skin lesions Clotting defects Gynecomastia Loss of axillary and pubic hair Impotence Decreased libido Sodium and water retention Peripheral neuropathy

Trace Minerals

Less than 100mg needed/day - copper, fluoride, iodine, iron, and zinc

Causes for cystectomy with urinary diversion

cancer of bladder neurogenic bladder congenital anomalies trauma chronic infection

Hepatocellular Jaundice

Liver's inability to take up bilirubin and excrete it - happens with hepatitis, cirrhosis, and hepatic carcinoma

Complications of insulin therapy

Local allergic reactions Insulin lipodystrophy Lipohypertrophy Insulin resistance

Functional incontinence

Lower urinary tract function is intact but other factors, such as severe cognitive impairment (eg, Alzheimer's dementia), make it difficult for the patient to identify the need to void or physical impairments make it difficult or impossible for the patient to reach the toilet in time for voiding.

Osmotic (Saline) Cathartics

MOM, Fleets enema, Fleet Phosphate Soda, Epsom Salts

How do the kidney's maintain ABB?

Maintain bicarbonate levels, excrete H+, conserve bicarbonate

How do the lungs maintain ABB?

Maintain carbonic acid levels

Interventions for patient on O2

Maintain patent airway Respiratory assessment O2 delivery system assessment O2 humidification Mouth/nose care Psychosocial support Patient/family ed

Lung Compliance

Measure of the force required to expand or inflate the lungs, ability of lungs to expand and recoil

Nursing considerations for Loop diuretics

Monitor BP, K, I&O, dehydration

irreducible hernia

cannot be replaced into cavity

Nursing interventions for gastric ulcers

Monitor vital signs and for signs of bleeding. Administer small, frequent bland feedings during the active phase. Administer H2-receptor antagonists or proton pump inhibitors as prescribed to decrease the secretion of gastric acid. Administer antacids as prescribed to neutralize gastric secretions. Administer anticholinergics as prescribed to reduce gastric motility. Administer mucosal barrier protectants as prescribed 1 hour before each meal. Administer prostaglandins as prescribed for their protective and antisecretory actions.

Ventilation

Movement of air in and out of airways

First degree heart block

PR interval > .20 seconds - conduction is delayed at AV node - related to digoxin or vagal stimulation

Interventions for appendicitis

NPO Administer fluids intravenously to prevent dehydration. Monitor for changes in level of pain. Monitor for signs of ruptured appendix and peritonitis Position the client in a right side-lying or low to semi-Fowler's position to promote comfort. Monitor bowel sounds. Apply ice packs to the abdomen for 20 to 30 minutes every hour as prescribed. (Avoid heat - heat can cause rupture) Administer antibiotics as prescribed. Pain meds after dx is made. Schedule appendectomy.

Diet for acute pancreatitis

NPO TPN eventually progress to small frequent feedings high carb and protein, low fat no stimulants ie. coffee, alcohol supplement w/ fat soluble vitamins

Types of O2 Delivery Systems

Nasal canula Mask Venturi mask Partial rebreather mask Ventilator

Medications used to treat hepatic encephalopathy

Neomycin sulfate Lactulose Levdopa

Chemo receptors

Nerve endings located in the aortic arch and carotid bodies that are stimulated by hypoxemia and that subsequently transmit impulses to the CNS.

Advantages of Lap Band Surgery

No cutting of the stomach No stapling of the stomach Calibrated pouch and stoma size Can be adjusted to patient's needs after surgery with no operation to adjust the stoma Laparoscopic removal possible Fully reversible Short hospital stay (does not exceed 48 hours)

Serum calcium level is controlled by

PTH and calcitonin

The most significant and direct indicator of the effectiveness of oxygen therapy is...

PaO2

mode of oxygen delivery can deliver levels of the fraction of inspired oxygen (FIO2) as high as 100%?

Nonrebreather mask

Somogyi effect

Normal or elevated blood glucose levels are present at bedtime; hypoglycemia occurs at about 2 to 3 AM, which causes an increase in the production of counterregulatory hormones. By about 7 AM, in response to the counterregulatory hormones, the blood glucose rebounds significantly to the hyperglycemic range. Treatment includes decreasing the evening (predinner or bedtime) dose of intermediate-acting insulin or increasing the bedtime snack.

Increased fat intake leads to...

Obesity Diarrhea Increased cholesterol Increased gall stones

Cirrhosis and jaundice

Occurs because the liver is unable to metabolize bilirubin and because the edema, fibrosis, and scarring of the hepatic bile ducts interfere with normal bile and bilirubin secretion

Antidiarrheal

Opioids, Lomotil, Immodium

Oral Anesthetics

Orajel

Interventions for Hypophosphatemia

Oral phosphorous w/ vitamin d

Interventions of Hypocalcemia

Oral/IV calcium

Interventions for Hypokalemia

Oral/IV supplements, monitor EKG and ABGs

SaO2

Oxygen saturation Percentage of hemoglobin saturated with O2

Common therapies used to treat pulmonary hypertension...

Oxygen, diuretics, and vasodilators

PaO2

Partial pressure of alveolar oxygen. Indirect measure of O2 content in arterial blood. NL = 80 - 100

Esophageal speech

Patient compresses air in esophagus and expels it, conscious belching that is transformed into speech, a voice prosthesis is inserted into stoma. Most like real speech.

Nursing considerations for hypertensive crisis

Patients have an arterial line for direct BP measurment and BP is checked every 2-3 minutes. Monitor for hypotension. Neuro checks assessing for stroke, assess for MI, visual changes, renal function with hourly urine output.

Vitamin B3/Niacin and Tryptophan deficiency

Pellagra

Absorbant Antidiarrheal Drugs

Pepto Bismol, Kaopectate

Nephrectomy

Performed for extensive kidney damage, renal infection, severe obstruction from stones or tumors, and prevention of stone recurrence.

Cholecystography

Performed to detect gallstones and assess the ability of the gallbladder to fill, concentrate its contents, contract, and empty Preprocedure: Assess for allergies to iodine or seafood. - Contrast agents may be administered 10 to 12 hours (evening before) before the test. - Client is NPO after the contrast agent is administered. - Instruct the client that if a rash, itching , hives, or difficulty in breathing occurs after taking the contrast agent to report to the emergency department. Postprocedure: Inform the client that dysuria is common because the contrast agent is excreted in the urine. - A normal diet may be resumed (a fatty meal may enhance excretion of the contrast agent).

Action of Alpha andrenergic blockers

Peripheral vasodilators act directly on the blood vessel

Which electrolyte is essential for RBC production?

Phosphate

Complications of Cirrhosis

Portal hypertension Ascites Bleeding esophageal varices Coagulation defects Jaundice Portal systemic encephalopathy Hepatorenal syndrome

Interventions to Improve Respiration

Positioning/Posture Environmental control Activity/rest Decrease anxiety Hydration Infection prevention Nutrition O2/Ventilation

Signs & Symptoms CAD

Possibly normal findings during asymptomatic periods, chest pain, palpitations, dyspnea, syncope, cough, hemoptysis, excessive fatigue

Action of Thiazide diuretics

Prevents sodium and water reabsorption, promotes K excretion

Fortification

Process of adding nutritive substances not naturally occurring in the given food to increase its nutritional value

Diabetic nephropathy

Progressive decrease in kidney function

Hepatorenal syndrome

Progressive renal failure associated with hepatic failure characterized by a sudden decrease in urinary output, elevated blood urea nitrogen and creatinine levels, decreased urine sodium excretion, and increased urine osmolarity

Pharmacological treatment for BPH

Proscar - long term - blocks testosterone formation causing regression of hyoerplastic tissue and decreases the size of the prostate Minipress, Hytrin, Flomax - relax smooth muscle and increase urinary flow

Hyperventilating and hyperoxygenating the client before and during (or after) suctioning helps prevent...

cardiac arrhythmias

Vitamin C (function)

promotes healthy immune system, helps wounds heal, maintains connective tissue, aids in absorption of iron

Vitamin B2/Riboflavin (function)

promotes normal growth

Vitamin B3/Niacin (function)

promotes normal growth

Glomerulonephritis

Term that includes a variety of disorders, most of which are caused by an immunological reaction. Results in proliferative and inflammatory changes within the glomerular structure. Destruction, inflammation, and sclerosis of the glomeruli of both kidneys occur. The inflammation of the glomeruli results from an antigen-antibody reaction produced from an infection or autoimmune process elsewhere in the body. Loss of kidney function occurs.

Early Manifestations of Cirrhosis

RUQ pain Anorexia Dyspepsia Flatulence Change in bowel patterns Fever Weight loss Fatigue Enlarged liver/spleen

Treatment for Laryngeal Cancer

Radiation, chemotherapy, laryngectomy (full/partial)

Hypertensive Crisis

Rapid rise in BP. Systolic > 240, Diastolic > 120 Can cause organ damage: heart, brain, kidneys BP must be reduced gradually b/c rapid decreased can result in stroke

Cheyne-Stokes

Regular cycle where the rate a depth of breathing increase, then decrease until apnea (usually 20 sec) - duration of apnea may progressively lengthen - associated with heart failure and damage to resp center

Action of Calcium channel blockers

Relax smooth muscle in arterial walls, dilate coronary arteries

Transurethral resection of the prostate (TURP)

Removal of benign prostatic tissue surrounding the urethra with use of a resectoscope introduced through the urethra; there is little risk of impotence and it is most commonly used for BPH.

Interventions for DKA

Restore circulating blood volume and protect against cerebral, coronary, and renal hypoperfusion. Treat dehydration with rapid IV infusions of 0.9% or 0.45% normal saline (NS) as prescribed; dextrose is added to IV fluids (D 5NS, or 5% dextrose in 0.45% saline) when the blood glucose level reaches 250 to 300 mg/ dL. Treat hyperglycemia with regular insulin administered intravenously as prescribed. Correct electrolyte imbalances (potassium level may be elevated as a result of dehydration and acidosis). Monitor potassium level closely because when the client receives treatment for the dehydration and acidosis, the serum potassium level will decrease and potassium replacement may be required.

Myocardial Infarction

Results from sustained cardiac ischemia causing cellular death (after 20 min) and necrosis of myocardium - scar tissue replaces necrotic tissue after 6 weeks

Vitamin C Deficiency

Scurvy

Pneumocystitis Carinni Pneumonia

Secondary to AIDS

Byetta

Signals the pancreas to make the right amount of insulin after eating - stops the liver from making too much glucose - slows how quickly food leaves the stomach - may reduce appetite and aid in weight loss

Lubricants

Soften feces ie. mineral oil (PO or enema)

Baroreceptors

Specialized nerve endings located in the walls of the aortic arch and carotid sinuses. They are affected by changes in blood pressure. Increases in BP stimulate baroreceptors and heart rate and BP decrease. Decrease in BP leads to less stimulation of baroreceptors, vasoconstriction occurs and heart rate increases.

Meglitinides

Stimulate the beta cells to release more insulin

Treatment for Burger's disease

Stop smoking, keep extremities warm, administer calcium channel blockers (blocks spasms)

Surgical Interventions for Gastric Ulcers

Total gastrectomy Vagotomy Gastric resection Billroth I Billroth II Pyloroplasty

Risk factors for Pneumonia

Streptococcus, smokers, elderly, immuocompromised, immobility, alcoholics, malnourished

Thrombolytic therapy for MI

Streptokinase - TPA - may be prescribed within first 6 hours

Elasticity/Recoil

Stretchability

Billroth I

Surgical intervention for gastric ulcers Partial gastrectomy, with the remaining segment anastomosed to the duodenum; also called gastroduodenostomy

Gastric resection

Surgical intervention for gastric ulcers Removal of the lower half of the stomach and usually includes a vagotomy; also called antrectomy

Total gastrectomy

Surgical intervention for gastric ulcers Removal of the stomach with attachment of the esophagus to the jejunum or duodenum; also called esophagojejunostomy or esophagoduodenostomy

Vagotomy

Surgical intervention for gastric ulcers Surgical division of the vagus nerve to eliminate the vagal impulses that stimulate hydrochloric acid secretion in the stomach

Mechanics of Respiration

Takes place as a results of pressure gradients. Intrapulmonary pleural pressure < atmosheric Changes in cavity size and pressure allow for inspiration and expiration

Apnea

Temporary cessation of breathing

Stool specimens

Testing of stool specimens includes inspecting the specimen for consistency and color and testing for occult blood. Tests for fecal urobilinogen, fat, nitrogen, parasites, pathogens, food substances, and other substances may be performed; these tests require that the specimen be sent to the laboratory. Random specimens are sent promptly to the laboratory. Quantitative 24- to 72-hour collections must be kept refrigerated until they are taken to the laboratory. Some specimens require that a certain diet be followed or that certain medications be withheld; check agency guidelines regarding specific procedures.

What is happening when the heart is in diastole?

The atria are filling with blood and the heart is relaxed. The tricuspid and mitral valves are closed.

GERD

The backflow of gastric and duodenal contents into the esophagus. The reflux is caused by an incompetent lower esophageal sphincter, pyloric stenosis, or motility disorder. Leads to Barrett's esophagus, aspiration pneumonia, dental erosion.

Surgical interventions for ulcerative colitis

Total proctocolectomy with permanent ileostomy Kock ileostomy (continent ileostomy) Ileoanal reservoir

Insulin resistance

The client receiving insulin develops immune antibodies that bind the insulin, thereby decreasing the insulin available for use in the body. Treatment consists of administering a purer insulin preparation

Posticteric Stage of Hepatitis

The convalescent stage of hepatitis, in which the jaundice decreases and the color of the urine and stool return to normal Assess for increased energy levels, subsiding of pain, minimal to absent gastrointestinal symptoms, and serum bilirubin and enzyme levels return to normal.

Diagnostic tests for DVT

Venous ultrasound, venogram, lung scan, PT/PTT

Causes of Hypernatremia

Water loss, hypertonic tube feeding, diabetes insipidus, hyperventilation, diarrhea, renal failure, heat stroke

Creatinine clearance test

The creatinine clearance test evaluates how well the kidneys remove creatinine from the blood. The test includes obtaining a blood sample and timed urine specimens. Blood is drawn when the urine specimen collection is complete. The urine specimen for the creatinine clearance is usually collected for 24 hours, but shorter periods such as 8 or 12 hours could be prescribed.

Preicteric Stage of Hepatitis

The first stage of hepatitis preceding the appearance of jaundice; includes flu-like symptoms Assess for flu-like symptoms— malaise, fatigue Anorexia, nausea, vomiting, diarrhea Pain— headache, muscle aches, polyarthritis Serum bilirubin and enzyme levels are elevated.

TIPS

The nonsurgical procedure uses the normal vascular anatomy of the liver to create a shunt with the use of a metallic stent. The shunt is between the portal and systemic venous system in the liver and is aimed at relieving portal hypertension.

Arterial pressure

The pressure of the blood against the arterial walls.

Endoscopy sclerotherapy

The procedure involves the injection of a sclerosing agent into and around bleeding varices. Complications include chest pain, pleural effusion, aspiration pneumonia, esophageal stricture, and perforation of the esophagus.

Icteric Stage of Hepatitis

The second stage of hepatitis; includes the appearance of jaundice and associated symptoms such as elevated bilirubin levels, dark or tea-colored urine, and clay -colored stools Assess for jaundice, pruritus, dark or tea-colored urine, clay colored stools, and a decrease in preicteric-phase symptoms

Metabolism

The sum of all the physiological and chemical reactions that take place in the body

Tidal Volume

The volume of air inhaled and exhaled with each breath. May not vary even with severe disease. NL = 500mL

Residual Volume

The volume of air remaining in the lungs after a maximum exhalation. May be increased with obstructive disease. NL = 1200mL

Purpose of O2 Therapy

To prevent hypoxia and provide tissue oxygenation

Risk factors for Laryngeal Tumors

Tobacco, alcohol, toxins

PTCA/Angioplasty

Treatment for CAD - compresses plaque against walls and dilates vessel

Treatment for HHNS

Treatment is similar to that for DKA - includes fluid replacement, correction of electrolyte imbalances, and insulin administration. Fluid replacement in the older client must be done very carefully because of the potential for heart failure. Insulin plays a less critical role in the treatment of HHNS than it does for the treatment of DKA because ketosis and acidosis do not occur; rehydration alone may decrease glucose levels.

Complications of BPH

UTIs Hydronephrosis Pyelonephritis

ulcerative colitis

Ulcerative and inflammatory disease of the bowel that results in poor absorption of nutrients. Acute ulcerative colitis results in vascular congestion, hemorrhage, edema, and ulceration of the bowel mucosa. Chronic ulcerative colitis causes muscular hypertrophy, fat deposits, and fibrous tissue with bowel thickening, shortening, and narrowing.

Diagnostics for GERD

Upper GI series, Barium swallow, endoscopy, Gastric pH monitoring, Gastric emptying study

ileal conduit

Ureters are implanted into a segment of the ileum, with the formation of an abdominal stoma. The urine flows into the conduit and is propelled continuously out through the stoma by peristalsis. The client is required to wear an appliance over the stoma to collect the urine. Complications include obstruction, pyelonephritis, leakage at the anastomosis site, stenosis, hydronephrosis, calculi, skin irritation and ulceration, and stomal defects.

GI Ultrasound

Used in detecting cystic structures - performed before Barium studies - NPO for gallbladder

Nursing considerations for ACE II - Angiotension receptor blockers (ARBS)

Useful with renal patients, check BP, pulse, headache, dizzy, cough

Echocardiogram

Uses ultrasound to look at valves and left ventricle

Major Nutrients in Vegetable and Fruit Groups

Vitamins A, B, and C, Incomplete proteins, Calcium, Magnesium, Iron, Fiber

Pericarditis - diagnostic tests

WBC/sed rate, blood culture, EKG, edho, BP, temp

Main Functions of Upper Respiratory System

Warm, filter, and humidify air.

What is happening when the heart is in systole?

When the pressure in the ventricles increases the valves are forced open, the heart contracts and blood is ejected into the aorta.

Antiviral Agents GI

Zovirax

Duodenal ulcer

a break in the mucosa of the duodenum

Inflammatory bowel disease

a chronic recurrent inflammation of the intestinal tract that has exacerbations and remissions. Ulcerative colitis Chron's disease

Inspiratory and expiratory stridor is asscoiated with...

a foreign body obstructing the trachea or mainstem bronchi

Asterixis

a coarse tremor characterized by rapid, nonrhythmic extensions and flexions in the wrist and fingers

Arthrectomy

a device with a tiny rotating blade is used to shave away the plaque from the inside of the artery

Type 1 diabetes mellitus

a nearly absolute deficiency of insulin due to detruction of beta cells; if insulin is not given, fats are metabolized for energy, resulting in ketonemia (acidosis) - onset of hyperglycemic symptoms are more rapid and acute

BiPAP

a type of continuous positive airway pressure in which both inspiratory and expiratory pressures are set above atmospheric pressure - assists clients with chronic obstructive pulmonary disease who retain PaCO2

Regurgitant valve

a valve that cannot close completely - sometimes called insufficient

Stenotic valve

a valve that cannot open fully

A pt recovering from surgery has an indwelling urinary catheter. The nurse would contact the pt's primary healthcare provider with which of the following 24-hour urine output volumes? a) 600 mL b) 750 mL c) 1000 mL d) 1200 mL

a) 600 mL

For a client with an endotracheal (ET) tube, which nursing action is most essential? a) Auscultating the lungs for bilateral breath sounds b) Turning the client from side to side every 2 hours c) Monitoring serial blood gas values every 4 hours d) Providing frequent oral hygiene

a) Auscultating the lungs for bilateral breath sounds

A nurse is performing a respiratory assessment on a client with pneumonia. She asks the client to say "ninety-nine" several times. Through her stethoscope, she hears the words clearly over his left lower lobe. What term should the nurse use to document this finding? a) Bronchophony b) Tactile fremitus c) Crepitation d) Egophony

a) Bronchophony

A nurse is caring for a client who has a tracheostomy and temperature of 103° F (39.4° C). Which intervention will most likely lower the client's arterial blood oxygen saturation? a) Endotracheal suctioning b) Encouragement of coughing c) Use of cooling blanket d) Incentive spirometry

a) Endotracheal suctioning

A client comes to the emergency department complaining of sudden onset of diarrhea, anorexia, malaise, cough, headache, and recurrent chills. Based on the client's history and physical findings, the physician suspects Legionnaires' disease. While awaiting diagnostic test results, the client is admitted to the facility and started on antibiotic therapy. What is the drug of choice for treating Legionnaires' disease? a) Erythromycin (Erythrocin) b) Rifampin (Rifadin) c) Amantadine (Symmetrel) d) Amphotericin B (Fungizone)

a) Erythromycin (Erythrocin)

A pt with a history of stomach ulcers is diagnosed with hypophosphatemia. Which of the following interventions should the nurse include in this pt's plan of care? a) Request a dietitian consult for selecting foods high in phosphorous. b) Provide aluminum hydroxide antacids as prescribed. c) Instruct pt to avoid poultry, peanuts, & seeds. d) Instruct to avoid the intake of sodium phosphate.

a) Request a dietitian consult for selecting foods high in phosphorous.

The home health nurse sees a client with end-stage chronic obstructive pulmonary disease. An outcome identified for this client is preventing infection. Which finding indicates that this outcome has been met? a) Decreased oxygen requirements b) Increased sputum production c) Decreased activity tolerance d) Normothermia

a) Decreased oxygen requirements

A pt who is taking digoxin (Lanoxin) is admitted with possible hypokalemia. Which of the following does the nurse realize might occur with this pt? a) Digoxin toxicity may occur. b) A higher dose of digoxin (Lanoxin) may be needed. c) A diuretic may be needed. d) Fluid volume deficit may occur.

a) Digoxin toxicity may occur.

What does a positive Chvostek's sign indicate? a) Hypocalcemia b) Hyponatremia c) Hypokalemia d) Hypermagnesemia

a) Hypocalcemia

A home health nurse is visiting a home care client with advanced lung cancer. Upon assessing the client, the nurse discovers wheezing, bradycardia, and a respiratory rate of 10 breaths/min. These signs are associated with which condition? a) Hypoxia b) Delirium c) Hyperventilation d) Semiconsciousness

a) Hypoxia

The pt has a serum phosphate level of 4.7 mg/dL. Which interdisciplinary treatments would the nurse expect for this pt? Select all that apply. a) IV normal saline b) calcium containing antacids c) IV potassium phosphate d) encouraging milk intake e) increasing vitamin D intake

a) IV normal saline b) calcium containing antacids

The nurse is caring for a client with chest trauma. Which nursing diagnosis takes highest priority? a) Impaired gas exchange b) Anxiety c) Decreased cardiac output d) Ineffective cardiopulmonary tissue perfusion

a) Impaired gas exchange

After receiving an oral dose of codeine for an intractable cough, the client asks the nurse, "How long will it take for this drug to work?" How should the nurse respond? a) In 30 minutes b) In 1 hour c) In 2.5 hours d) In 4 hours

a) In 30 minutes

A client with pneumococcal pneumonia is admitted to an acute care facility. The client in the next room is being treated for mycoplasmal pneumonia. Despite the different causes of the various types of pneumonia, all of them share which feature? a) Inflamed lung tissue b) Sudden onset c) Responsiveness to penicillin d) Elevated white blood cell (WBC) count

a) Inflamed lung tissue

A client suffers adult respiratory distress syndrome as a consequence of shock. The client's condition deteriorates rapidly, and endotracheal (ET) intubation and mechanical ventilation are initiated. When the high-pressure alarm on the mechanical ventilator sounds, the nurse starts to check for the cause. Which condition triggers the high-pressure alarm? a) Kinking of the ventilator tubing b) A disconnected ventilator tube c) An ET cuff leak d) A change in the oxygen concentration without resetting the oxygen level alarm

a) Kinking of the ventilator tubing

A client with suspected inhalation anthrax is admitted to the emergency department. Which action by the nurse takes the highest priority? a) Monitor vital signs and oxygen saturation every 15 to 30 minutes. b) Suction the client as needed to obtain a sputum specimen for culture and sensitivity. c) Assess intake and output and maintain adequate hydration. d) Reassure the client that intubation and mechanical ventilation will be temporary.

a) Monitor vital signs and oxygen saturation every 15 to 30 minutes.

A client with chronic obstructive pulmonary disease presents with respiratory acidosis and hypoxemia. He tells the nurse that he doesn't want to be placed on a ventilator. What action should the nurse take? a) Notify the physician immediately so he can determine client competency. b) Have the client sign a do-not-resuscitate (DNR) form. c) Determine whether the client's family was consulted about his decision. d) Consult the palliative care group to direct care for the client.

a) Notify the physician immediately so he can determine client competency.

A 33-year-old woman with primary pulmonary hypertension is being evaluated for a heart-lung transplant. The nurse asks her what treatments she is currently receiving for her disease. She is likely to mention which treatments? a) Oxygen b) Aminoglycosides c) Diuretics d) Vasodilators e) Antihistamines f) Sulfonamides

a) Oxygen c) Diuretics d) Vasodilators

A client has hypoxemia of pulmonary origin. What portion of arterial blood gas results is most useful in distinguishing between acute respiratory distress syndrome and acute respiratory failure? a) Partial pressure of arterial oxygen (PaO2) b) Partial pressure of arterial carbon dioxide (PaCO2) c) pH d) Bicarbonate (HCO3-)

a) Partial pressure of arterial oxygen (PaO2)

A client is admitted to the emergency department with an acute asthma attack. The physician prescribes ephedrine sulfate, 25 mg subcutaneously (S.C.). How soon should the ephedrine take effect? a) Rapidly b) In 3 minutes c) In 1 hour d) In 2 hours

a) Rapidly

The nurse administers albuterol (Proventil), as prescribed, to a client with emphysema. Which finding indicates that the drug is producing a therapeutic effect? a) Respiratory rate of 22 breaths/minute b) Dilated and reactive pupils c) Urine output of 40 ml/hour d) Heart rate of 100 beats/minute

a) Respiratory rate of 22 breaths/minute

The nurse is planning care for a pt with fluid volume overload & hyponatremia. Which of the following should be included in this pt's plan of care? a) Restrict fluids. b) Administer intravenous fluids. c) Provide Kayexalate. d) Administer intravenous normal saline with furosemide.

a) Restrict fluids.

A pregnant pt is admitted with excessive thirst, increased urination, & has a medical diagnosis of diabetes insipidus. The nurse chooses which of the following nursing diagnoses as most appropriate? a) Risk for Imbalanced Fluid Volume b) Excess Fluid Volume c) Imbalanced Nutrition d) Ineffective Tissue Perfusion

a) Risk for Imbalanced Fluid Volume

A client with respiratory acidosis is admitted to the intensive care unit for close observation. The nurse should stay alert for which complication associated with respiratory acidosis? a) Shock b) Stroke c) Seizures d) Hyperglycemia

a) Shock

A client has been hospitalized for treatment of acute bacterial pneumonia. Which outcome indicates an improvement in the client's condition? a) The client has a partial pressure of arterial oxygen (PaO2) value of 90 mm Hg or higher. b) The client has a partial pressure of arterial carbon dioxide (PaCO2) value of 65 mm Hg or higher. c) The client exhibits restlessness and confusion. d) The client exhibits bronchial breath sounds over the affected area.

a) The client has a partial pressure of arterial oxygen (PaO2) value of 90 mm Hg or higher.

When analyzing an arterial blood gas report of a pt with COPD & respiratory acidosis, the nurse anticipates that compensation will develop through which of the following mechanisms? a) The kidneys retain bicarbonate. b) The kidneys excrete bicarbonate. c) The lungs will retain carbon dioxide. d) The lungs will excrete carbon dioxide.

a) The kidneys retain bicarbonate.

The nurse is caring for a pt diagnosed with renal failure. Which of the following does the nurse recognize as compensation for the acid-base disturbance found in pts with renal failure? a) The pt breathes rapidly to eliminate carbon dioxide. b) The pt will retain bicarbonate in excess of normal. c) The pH will decrease from the present value. d) The pt's oxygen saturation level will improve.

a) The pt breathes rapidly to eliminate carbon dioxide.

A client has a serum calcium level of 7.2 mg/dl. During the physical examination, the nurse expects to assess: a) Trousseau's sign. b) Homans' sign. c) Hegar's sign. d) Goodell's sign.

a) Trousseau's sign.

An elderly pt is at home after being diagnosed with fluid volume overload. Which of the following should the home care nurse instruct this pt to do? a) Wear support hose. b) Keep legs in a dependent position. c) Avoid wearing shoes while in the home. d) Try to sleep without extra pillows.

a) Wear support hose.

A pt is diagnosed with hyperphosphatemia. The nurse realizes that this pt might also have an imbalance of which of the following electrolytes? a) calcium b) sodium c) potassium d) chloride

a) calcium

A pt is diagnosed with hypokalemia. After reviewing the pt's current medications, which of the following might have contributed to the pt's health problem? a) corticosteroid b) thiazide diuretic c) narcotic d) muscle relaxer

a) corticosteroid

An elderly pt who is being medicated for pain had an episode of incontinence. The nurse realizes that this pt is at risk for developing a) dehydration. b) over-hydration. c) fecal incontinence. d) a stroke.

a) dehydration.

A pt is receiving intravenous fluids postoperatively following cardiac surgery. Nursing assessments should focus on which postoperative complication? a) fluid volume excess b) fluid volume deficit c) seizure activity d) liver failure

a) fluid volume excess

A pt prescribed spironolactone is demonstrating ECG changes & complaining of muscle weakness. The nurse realizes this pt is exhibiting signs of which of the following? a) hyperkalemia b) hypokalemia c) hypercalcemia d) hypocalcemia

a) hyperkalemia

An elderly pt with peripheral neuropathy has been taking magnesium supplements. The nurse realizes that which of the following symptoms can indicate hypomagnesaemia? a) hypotension, warmth, & sweating b) nausea & vomiting c) hyperreflexia d) excessive urination

a) hypotension, warmth, & sweating

A client with chronic obstructive pulmonary disease (COPD) is admitted to the medical-surgical unit. To help this client maintain a patent airway and achieve maximal gas exchange, the nurse should: a) instruct the client to drink 2 L of fluid daily. b) maintain the client on bed rest. c) administer anxiolytics, as prescribed, to control anxiety. d) administer pain medication as prescribed.

a) instruct the client to drink 2 L of fluid daily.

The nurse is planning care for a pt with severe burns. Which of the following is this pt at risk for developing? a) intracellular fluid deficit b) intracellular fluid overload c) extracellular fluid deficit d) interstitial fluid deficit

a) intracellular fluid deficit

When caring for a pt diagnosed with hypocalcemia, which of the following should the nurse additionally assess in the pt? a) other electrolyte disturbances b) hypertension c) visual disturbances d) drug toxicity

a) other electrolyte disturbances

A client who weighs 175 lb (79.4 kg) is receiving aminophylline (Aminophyllin) (400 mg in 500 ml) at 50 ml/hour. The theophylline level is reported as 6 mcg/ml. The nurse calls the physician who instructs the nurse to change the dosage to 0.45 mg/kg/hour. The nurse should: a) question the order because the dosage is too low. b) question the order because the dosage is too high. c) set the pump at 45 ml/hour. d) stop the infusion and have the laboratory repeat the theophylline measurement.

a) question the order because the dosage is too low.

A slightly obese client with a history of allergy-induced asthma, hypertension, and mitral valve prolapse is admitted to an acute care facility for elective surgery. The nurse obtains a complete history and performs a thorough physical examination, paying special attention to the cardiovascular and respiratory systems. When percussing the client's chest wall, the nurse expects to elicit: a) resonant sounds. b) hyperresonant sounds. c) dull sounds. d) flat sounds.

a) resonant sounds.

An elderly postoperative pt is demonstrating lethargy, confusion, & a resp rate of 8 per minute. The nurse sees that the last dose of pain medication administered via a pt controlled anesthesia (PCA) pump was within 30 minutes. Which of the following acid-base disorders might this pt be experiencing? a) respiratory acidosis b) metabolic acidosis c) respiratory alkalosis d) metabolic alkalosis

a) respiratory acidosis

What is the nurse's primary concern regarding fluid & electrolytes when caring for an elderly pt who is intermittently confused? a) risk of dehydration b) risk of kidney damage c) risk of stroke d) risk of bleeding

a) risk of dehydration

A pt is diagnosed with severe hyponatremia. The nurse realizes this pt will mostly likely need which of the following precautions implemented? a) seizure b) infection c) neutropenic d) high-risk fall

a) seizure

A pt, experiencing multisystem fluid volume deficit, has the symptoms of tachycardia, pale, cool skin, & decreased urine output. The nurse realizes these findings are most likely a direct result of which of the following? a) the body's natural compensatory mechanisms b) pharmacological effects of a diuretic c) effects of rapidly infused intravenous fluids d) cardiac failure

a) the body's natural compensatory mechanisms

The nurse is reviewing a pt's blood pH level. Which of the systems in the body regulate blood pH? Select all that apply. a) renal b) cardiac c) buffers d) respiratory

a, c, d

Which signs cause the nurse to suspect cardiac tamponade after a client has cardiac surgery? Check all that apply. a. Tachycardia b. Hypertension c. Increased CVP d. Increased urine output e. Jugular vein distention

a, c, e. Blood in the pericardial sac compresses the heart so the ventricles cannot fill; this leads to a rapid thready pulse. Tamponade causes hypotension and a narrowed pulse pressure. As the tamponade increases, pressure on the heart interferes with the ejection of blood from the left ventricle, resulting in an increased pressure in the right side of the heart and the systemic circulation. As the heart because more inefficient, there is a decrease in kidney perfusion and therefore urine output. The increased venous pressure caused JVD.

When interpreting an ECG, the nurse would keep in mind which of the following about the P wave? Select all that apply. a. Reflects electrical impulse beginning at the SA node. b. Indicated electrical impulse beginning at the AV node. c. Reflects atrial muscle depolarization. d. Identifies ventricular muscle depolarization. e. Has duration of normally 0.11 seconds or less.

a, c, e. In a client who has had an ECG, the P wave represents the activation of the electrical impulse in the SA node, which is then transmitted to the AV node. In addition, the P wave represents atrial muscle depolarization, not ventricular depolarization. The normal duration of the P wave is 0.11 seconds or less in duration and 2.5 mm or more in height.

Which pts are at risk for the development of hypercalcemia? Select all that apply. a) the pt with a malignancy b) the pt taking lithium c) the pt who uses sunscreen to excess d) the pt with hyperparathyroidism e) the pt who overuses antacids

a,b,d,e

The pt who has a serum magnesium level of 1.4 mg/dL is being treated with dietary modification. Which foods should the nurse suggest for this pt? Select all that apply. a) bananas b) seafood c) white rice d) lean red meat e) chocolate

a,b,e

The staff nurse is caring for a client who is a potential heart donor. The client's family is concerned that the recipient will have access to personal donor information. Which response by the nurse demonstrates knowledge of the organ donation process? a. "I will have the transplant coordinator speak with you to answer your questions." b. "There is never contact between the donor's family and the recipient." c. "The recipient is allowed to ask questions about the donor and have them answered." d. "It is important that the recipient know where to send Thank-You cards."

a. "I will have the transplant coordinator speak with you to answer your questions." The transplant coordinator, a specially trained person with knowledge of the donation, procurement, and transplantation process, typically speaks to family members and answers their questions. Contact is permitted after the procedure with consent from the donor's family and the recipient. Typically, the transplant organization coordinates the communication. Confidentiality of the potential donor is always maintained unless the recipient and donor families both sign confidentiality waivers.

Before discharge, which instruction should the nurse give to a client receiving flecainide (Tambocor) to reduce the risk of heart failure? a. "Limit your fluid intake." b. "Take a diuretic before going to bed." c. "Limit your potassium intake." d. "Have your serum electrolyte levels measured weekly."

a. "Limit your fluid intake." The nurse should tell the client receiving flecainide to limit fluid intake. The client shouldn't take a diuretic unless prescribed by the physician; if prescribed, the diuretic should be taken early in the day to prevent nocturia. Sodium (not potassium) should be limited because excessive sodium intake causes water retention. The client's electrolyte levels don't need to be measured weekly

Which statements by a client prior to a cardiac catheterization would require a nurse to take further action? Select all that apply. a. "My kidney doctor has been suggesting I have this done for a year." b. "I haven't had anything to eat since dinner last night." c. "The last time I had a catheterization I had respiratory problems." d. "I am glad I can go home right after the procedure." e. "During the procedure I will feel warm all over."

a. "My kidney doctor has been suggesting I have this done for a year." c. "The last time I had a catheterization I had respiratory problems." d. "I am glad I can go home right after the procedure."

A female client who has just been diagnosed with hepatitis A asks, "How could I have gotten this disease?" What is the nurse's best response? a. "You may have eaten contaminated restaurant food." b. "You could have gotten it by using I.V. drugs." c. "You must have received an infected blood transfusion." d. "You probably got it by engaging in unprotected sex."

a. "You may have eaten contaminated restaurant food." Hepatitis A virus typically is transmitted by the oral-fecal route — commonly by consuming food contaminated by infected food handlers. The virus isn't transmitted by the I.V. route, blood transfusions, or unprotected sex. Hepatitis B can be transmitted by I.V. drug use or blood transfusion. Hepatitis C can be transmitted by unprotected sex.

Nurse Ryan is assessing for correct placement of a nosogartric tube. The nurse aspirates the stomach contents and check the contents for pH. The nurse verifies correct tube placement if which pH value is noted? a. 3.5 b. 7.0 c. 7.35 d. 7.5

a. 3.5 If the nasogastric tube is in the stomach, the pH of the contents will be acidic. Gastric aspirates have acidic pH values and should be 3.5 or lower. Option B indicates a slightly acidic pH. Option C indicates a neutral pH. Option D indicates an alkaline pH.

The normal serum level for potassium is a. 3.5 - 5.5 mEq/L b. 96 - 106 mEq/L c. 8.5 - 10.5 mEq/L d. 135 - 145 mEq/L

a. 3.5 - 5.5 mEq/L

The nurse knows that serum amylase levels return to normal within which timeframe? a. 48 hours b. 36 hours c. 12 hours d. 24 hours

a. 48 hours Serum amylase usually returns to normal within 48 - 72 hours.

A nurse is assigned to care for a group of clients. On review of the clients' medical records, the nurse determines that which client is at risk for a fluid volume deficit? a. A client with a colostomy b. A client with congestive heart failure c. A client on long-term corticosteroid therapy d. A client receiving frequent wound irrigations

a. A client with a colostomy

A nurse instructs a client about taking an angiotestin-converting enzyme (ACE). The client should be warned to continue therapy even when which common side effect occurs? a. A dry, persistent cough. b. Exacerbation of heart failure. c. Sedation. d. Urinary incontinence.

a. A dry, persistent cough.

Which of the following instructions should be included in the discharge teaching for a patient discharged with a transdermal nitroglycerin patch? a. "Apply the patch to a nonhairy, nonfatty area of the upper torso or arms." b. "Apply the patch to the same site each day to maintain consistent drug absorption." c. "If you get a headache, remove the patch for 4 hours and then reapply." d. "If you get chest pain, apply a second patch right next to the first patch."

a. A nitroglycerin patch should be applied to a nonhairy, nonfatty area for the best and most consistent absorption rates. Sites should be rotated to prevent skin irritation, and the drug should be continued if headache occurs because tolerance will develop. Sublingual nitroglycerin should be used to treat chest pain.

Which of the following neuroregulators increase gastric acid secretion? a. Acetylcholine b. Norepinephrine c. Secretin d. Gastrin

a. Acetylcholine Acetylcholine causes increased gastric acid. Norepinephrine inhibits secretions of the GI tract. Gastrin increases secretion of gastric juice, which is rich in HCl. Secretin in the stomach inhibits gastric secretion somewhat.

Which of the following terms describes the force against which the ventricle must expel blood? a. Afterload b. Cardiac output c. Overload d. Preload

a. Afterload refers to the resistance normally maintained by the aortic and pulmonic valves, the condition and tone of the aorta, and the resistance offered by the systemic and pulmonary arterioles. Cardiac output is the amount of blood expelled from the heart per minute. Overload refers to an abundance of circulating volume. Preload is the volume of blood in the ventricle at the end of diastole.

A charge nurse on a medical-surgical floor observes a certified nursing assistant (CNA) taking care of a client with impaire vascular circulation. What behavior by the aide requires immediate action by the RN? a. Aide places client's feet in warm water. b. Aide elevates client's legs. c. Aide places sheepskin between overlapped toes. d. Aide applies lanolin to client's legs.

a. Aide places client's feet in warm water.

A nurse reviews a client's laboratory report and notes that the client's serum phosphorus level is 2mg/dl. Which condition most likely caused this serum phosphorus level? a. Alcoholism b. Renal insufficiency c. Hypoparathyroidism d. Tumor lysis syndrome

a. Alcoholism

A 60-year-old male client comes into the emergency department with complaints of crushing chest pain that radiates to his shoulder and left arm. The admitting diagnosis is acute myocardial infarction. Immediate admission orders include oxygen by NC at 4L/minute, blood work, chest x-ray, an ECG, and 2mg of morphine given intravenously. The nurse should first: a. Administer the morphine. b. Obtain a 12-lead ECG. c. Obtain the lab work. d. Order the chest x-ray.

a. Although obtaining the ECG, chest x-ray, and blood work are all important, the nurse's priority action would be to relieve the crushing chest pain.

Interventions for peritonitis

antibiotics NG tube IV fluids analgesics surgery

The nurse is caring for a client who's experiencing sinus bradycardia with a pulse rate of 40 beats/minute. His blood pressure is 80/50 mm Hg and he complains of dizziness. Which medication would be used to treat his bradycardia? a. Atropine b. Dobutamine (Dobutrex) c. Amiodarone (Cordarone) d. Lidocaine (Xylocaine)

a. Atropine I.V. push atropine is used to treat symptomatic bradycardia. Dobutamine is used to treat heart failure and low cardiac output. Amiodarone is used to treat ventricular fibrillation and unstable ventricular tachycardia. Lidocaine is used to treat ventricular ectopy, ventricular tachycardia, and ventricular fibrillation.

Patient Education for Peptic Ulcer Disease

a. Avoid consuming alcohol and substances that contain caffeine or chocolate. b. Avoid smoking. c. Avoid aspirin or NSAIDs. d. Obtain adequate rest and reduce stress.

The nurse teaches the client with angina about the common expected side effects of nitroglycerin, including: a. Headache b. High blood pressure c. Shortness of breath d. Stomach cramps

a. Because of the widespread vasodilating effects, nitroglycerin often produces such side effects as headache, hypotension, and dizziness. The client should lie or **** down to avoid fainting. Nitro does not cause shortness of breath or stomach cramps.

A client is having frequent premature ventricular contractions. A nurse would place priority on assessment of which of the following items? a. Blood pressure and peripheral perfusion b. Sensation of palpitations c. Causative factors such as caffeine d. Precipitating factors such as infection

a. Blood pressure and peripheral perfusion Premature ventricular contractions can cause hemodynamic compromise. The shortened ventricular filling time with the ectopic beats leads to decreased stroke volume and, if frequent enough, to decreased cardiac output. The client may be asymptomatic or may feel palpations. PVCs can be caused by cardiac disorders or by any number of physiological stressors, such as infection, illness, surgery, or trauma, and by the intake of caffeine, alcohol, or nicotine.

A client is admitted to an emergency department with chest pain that is being ruled out for myocardial infarction. Vital signs are as follows at 11am, P: 92 beats/min, RR: 24 breaths/min, BP: 140/88 mm Hg; 11:15am P: 96 beats/min, RR: 26 breaths/min, BP: 128/82 mm Hg, 11:30am P: 104 beats/min, RR: 28 breaths/min, BP: 104/68 mm Hg, 11:45am P: 118 beats/min, RR: 32, breaths/min, BP: 88/58 mm Hg. The nurse should alert the physician because these changes are most consistent with which of the following complications? a. Cardiogenic shock b. Cardiac tamponade c. Pulmonary embolism d. Dissecting thoracic aortic anyeurysm

a. Cardiogenic shock Cardiogenic shock occurs with severe damage (more than 40%) to the left ventricle. Classic signs include hypertension, a rapid pulse that becomes weaker, decreased urine output, and cool, clammy skin. Respiratory rate increases as the body develops metabolic acidosis from shock. Cardiac tamponade is accompanied by distant, muffled heart sounds and prominent neck vessels. Pulmonary embolism presents suddenly with severe dyspnea accompanying the chest pain. Dissecting aortic aneyurysms usually are accompanied by back pain.

Patients with metal valve replacements require what type of therapy?

anticoagulant - lifetime

Which of the following nursing interventions should the nurse perform for a female client receiving enteral feedings through a gastrostomy tube? a. Change the tube feeding solutions and tubing at least every 24 hours. b. Maintain the head of the bed at a 15-degree elevation continuously. c. Check the gastrostomy tube for position every 2 days. d. Maintain the client on bed rest during the feedings.

a. Change the tube feeding solutions and tubing at least every 24 hours. Tube feeding solutions and tubing should be changed every 24 hours, or more frequently if the feeding requires it. Doing so prevents contamination and bacterial growth. The head of the bed should be elevated 30 to 45 degrees continuously to prevent aspiration. Checking for gastrostomy tube placement is performed before initiating the feedings and every 4 hours during continuous feedings. Clients may ambulate during feedings.

Which of the following electrolytes is a major anion in body fluid? a. Chloride b. Potassium c. Sodium d. Calcium

a. Chloride

A client enters the ER complaining of chest pressure and severe epigastric distress. His VS are 158/90, 94, 24, and 99*F. The doctor orders cardiac enzymes. If the client were diagnosed with an MI, the nurse would expect which cardiac enzyme to rise within the next 3 to 8 hours? a. Creatine kinase (CK or CPK) b. Lactic dehydrogenase (LDH) c. LDH-1 d. LDH-2

a. Creatine kinase (CK, formally known as CPK) rises in 3-8 hours if an MI is present. When the myocardium is damaged, CPK leaks out of the cell membranes and into the blood stream. Lactic dehydrogenase rises in 24-48 hours, and LDH-1 and LDH-2 rises in 8-24 hours.

A nurse should instruct a client with edema from venous stasis to avoid: Select all the apply. a. Crossing the legs while sitting. b. Keeping the legs in a dependent position. c. Elevating the legs whenever possible. d. Taking frequent hot baths. e. Wearing support hose when sleeping. f. Walking every day.

a. Crossing the legs while sitting. b. Keeping the legs in a dependent position. d. Taking frequent hot baths.

Good dental care is an important measure in reducing the risk of endocarditis. A teaching plan to promote good dental care in a client with mitral stenosis should include demonstration of the proper use of: a. A manual toothbrush b. An electric toothbrush c. An irrigation device d. Dental floss

a. Daily dental care and frequent checkups by a dentist who is informed about the client's condition are required to maintain good oral health. Use of an electric toothbrush, an irrigation device, or dental floss may cause gums to bleed and allow bacteria to enter mucous membranes and the blood stream, increasing the risk of endocarditis.

A client is recovering from coronary artery bypass graft (CABG) surgery. Which nursing diagnosis takes highest priority at this time? a. Decreased cardiac output related to depressed myocardial function, fluid volume deficit, or impaired electrical conduction b. Anxiety related to an actual threat to health status, invasive procedures, and pain c. Disabled family coping related to knowledge deficit and a temporary change in family dynamics d. Hypothermia related to exposure to cold temperatures and a long cardiopulmonary bypass time

a. Decreased cardiac output related to depressed myocardial function, fluid volume deficit, or impaired electrical conduction For a client recovering from CABG surgery, Decreased cardiac output is the most important nursing diagnosis because myocardial function may be depressed from anesthetics or a long cardiopulmonary bypass time, leading to decreased cardiac output. Other possible causes of decreased cardiac output in this client include fluid volume deficit and impaired electrical conduction. The other options may be relevant but take lower priority at this time because maintaining cardiac output is essential to sustaining the client's life.

Dr. Smith has determined that the client with hepatitis has contracted the infection form contaminated food. The nurse understands that this client is most likely experiencing what type of hepatitis? a. Hepatitis A b. Hepatitis B c. Hepatitis C d. Hepatitis D

a. Hepatitis A Hepatitis A is transmitted by the fecal-oral route via contaminated food or infected food handlers. Hepatitis B, C, and D are transmitted most commonly via infected blood or body fluids.

A client is recovering from coronary artery bypass graft (CABG) surgery. The nurse knows that for several weeks after this procedure, the client is at risk for certain conditions. During discharge preparation, the nurse should advise the client and family to expect which common symptom that typically resolves spontaneously? a. Depression b. Ankle edema c. Memory lapses d. Dizziness

a. Depression For the first few weeks after CABG surgery, clients commonly experience depression, fatigue, incisional chest discomfort, dyspnea, and anorexia. Depression typically resolves on its own and doesn't require medical intervention; however, family members should be aware that symptoms don't always resolve on their own. They should also be instructed about worsening symptoms of depression and when to seek care. Ankle edema seldom follows CABG surgery and may indicate right-sided heart failure; because this condition is a sign of cardiac dysfunction, the client should report ankle edema at once. Memory lapses reflect neurologic rather than cardiac dysfunction. Dizziness may result from decreased cardiac output, an abnormal condition after CABG surgery that warrants immediate physician notification.

A client is scheduled for outpatient bariatric surgery. Deep vein thrombosis (DVT) is best prevented by: a. Early ambulation b. Postoperative low-dose heparin. c. Alternating compression leg wraps. d. Range-of-motion exercises to wrist.

a. Early ambulation

A nurse notes 2+ bilateral edema in the lower extremities of a client with myocardial infarction who was admitted 2 days ago. The nurse would plan to do which of the following next? a. Review the intake and output records for the last 2 days. b. Change the time of diuretic administration from morning to evening. c. Request a sodium restriction of 1 g/day from the physician. d. Order daily weights starting the following morning.

a. Edema, the accumulation of excess fluid in the interstitial spaces, can be measured by intake greater than output and by a sudden increase in weight. Diuretics should be given in the morning whenever possible to avoid nocturia. Strict sodium restrictions are reserved for clients with severe symptoms.

In caring for a client with acute viral hepatitis, which task should be delegated to the nursing assistant? a. Empty the bedpan while wearing gloves. b. Suggest diversional activities. c. Monitor dietary preferences. d. Reports signs and symptoms of jaundice.

a. Empty the bedpan while wearing gloves. The nursing assistant should use infection control precautions for the protection of self, employees, and other clients. Planning and monitoring are RN responsibilities. While the nursing assistants can report valuable information, they should not be responsible for signs and symptoms that can be subtle or hard to detect, such as skin changes.

Which diagnostic test would be used first to evaluate a client with upper GI bleeding? a. Endoscopy b. Upper GI series c. Hemoglobin (Hb) levels and hematocrit (HCT) d. Arteriography

a. Endoscopy Endoscopy permits direct evaluation of the upper GI tract and can detect 90% of bleeding lesions. An upper GI series, or barium study, usually isn't the diagnostic method of choice, especially in a client with acute active bleeding who's vomiting and unstable. An upper GI series is also less accurate than endoscopy. Although an upper GI series might confirm the presence of a lesion, it wouldn't necessarily reveal whether the lesion is bleeding. Hb levels and HCT, which indicate loss of blood volume, aren't always reliable indicators of GI bleeding because a decrease in these values may not be seen for several hours. Arteriography is an invasive study associated with life-threatening complications and wouldn't be used for an initial evaluation.

A client with cirrhosis is at risk for developing complications. Which condition is the most serious and potentially life-threatening? a. Esophageal varices b. Ascites c. Peripheral edema d. Asterixis (liver flap)

a. Esophageal varices When a client has esophageal varices, the vessels become very fragile and massive hemorrhage can occur. The mortality rate is 30% - 50% after an episode of bleeding. Ascites and edema occur when liver production of albumin fails. Asterixis is a sign of hepatic encephalopathy.

A female client being seen in a physician's office has just been scheduled for a barium swallow the next day. The nurse writes down which instruction for the client to follow before the test? a. Fast for 8 hours before the test b. Eat a regular supper and breakfast c. Continue to take all oral medications as scheduled d. Monitor own bowel movement pattern for constipation

a. Fast for 8 hours before the test A barium swallow is an x-ray study that uses a substance called barium for contrast to highlight abnormalities in the gastrointestinal tract. The client should fast for 8 to 12 hours before the test, depending on physician instructions. Most oral medications also are withheld before the test. After the procedure, the nurse must monitor for constipation, which can occur as a result of the presence of barium in the gastrointestinal tract.

Assessment findings for acute pyelonephritis

a. Fever and chills b. Nausea c. Flank pain on the affected side d. Costovertebral angle tenderness e. Headache f. Dysuria g. Frequency and urgency h. Cloudy, bloody, or foul-smelling urine i. Increased white blood cells in the urine

Which signs and symptoms are present with a diagnosis of pericarditis? a. Fever, chest discomfort, and elevated erythrocyte sedimentation rate (ESR) b. Low urine output secondary to left ventricular dysfunction c. Lethargy, anorexia, and heart failure d. Pitting edema, chest discomfort, and nonspecific ST-segment elevation

a. Fever, chest discomfort, and elevated erythrocyte sedimentation rate (ESR) The classic signs and symptoms of pericarditis include fever, positional chest discomfort, nonspecific ST-segment elevation, elevated ESR, and pericardial friction rub. All other symptoms may result from acute renal failure.

A client with mild hypertension asks a nurse for suggestions to control blood pressure. What should a nurse recommend? a. Follow a regular exercise program. b. Attend a stress-reduction support group. c. Avoid use of tobacco and limit alcohol intake. d. Increase intake of fruit and vegetables.

a. Follow a regular exercise program.

Assessment finding for chronic pyelonephritis

a. Frequently diagnosed incidentally when a client is being evaluated for hypertension b. Poor urine-concentrating ability c. Pyuria d. Azotemia e. Proteinuria

PostOp ileostomy

a. Healthy stoma is red; a color change to dark blue or black should be reported to the physician. b. Postoperative drainage will be dark green and progress to yellow as the client begins to eat. c. Stool is liquid. d. Risk for dehydration and electrolyte imbalance exists. e. Do not administer medications such as suppositories through an ileostomy.

Which type of jaundice is the result of increased destruction of red blood cells? a. Hemolytic b. Obstructive c. Nonobstructive d. Hepatocellular

a. Hemolytic Hemolytic jaundice is the result of an increased destruction of the red blood cells. Hepatocellular jaundice is caused by the inability of the damaged liver cells to clear normal amounts of bilirubin from the blood. Obstructive jaundice resulting from extrahepatic obstruction may be caused by occlusion of the bile duct from a gall stone, inflammatory process, a tumor, or pressure from an enlarged organ. Nonobstructive jaundice occurs with hepatitis.

In presenting a workshop on parameters of cardiac function, which conditions should the nurse list as those most likely to lead to a decrease in preload? a. Hemorrhage, sepsis, and anaphylaxis b. Myocardial infarction, fluid overload, and diuresis c. Fluid overload, sepsis, and vasodilation d. Third spacing, heart failure, and diuresis

a. Hemorrhage, sepsis, and anaphylaxis Preload is the volume in the left ventricle at the end of diastole. It's also referred to as end-diastolic volume. Preload is reduced by any condition that reduces circulating volume, such as hemorrhage, sepsis, and anaphylaxis. Hemorrhage reduces circulating volume by loss of volume from the intravascular space. Sepsis and anaphylaxis reduce circulating volume by increased capillary permeability. Diuresis, vasodilation, and third spacing also reduce preload. Preload would increase with fluid overload and heart failure.

Amioderone

antidysrhytmic

A client, who is admitted in hypertesive crisis, has a blood pressure of 246/128 mm Hg. Which findings in the client's history should a nurse identify as being related to complications of hypertension? Select all that apply. a. History of brain attack (stroke). b. Elevated blood urea nitrogen. c. Bilateral glaucoma. d. Left ventricular hypertrophy. e. Intermittent claudication.

a. History of brain attack (stroke). d. Left ventricular hypertrophy. e. Intermittent claudication.

Nurse Oliver checks for residual before administering a bolus tube feeding to a client with a nasogastric tube and obtains a residual amount of 150 mL. What is appropriate action for the nurse to take? a. Hold the feeding b. Reinstill the amount and continue with administering the feeding c. Elevate the client's head at least 45 degrees and administer the feeding d. Discard the residual amount and proceed with administering the feeding

a. Hold the feeding Unless specifically indicated, residual amounts more than 100 mL require holding the feeding. Therefore options B, C, and D are incorrect. Additionally, the feeding is not discarded unless its contents are abnormal in color or characteristics.

A client has developed atrial fibrillation, which a ventricular rate of 150 beats per minute. A nurse assesses the client for: a. Hypotension and dizziness b. Nausea and vomiting c. Hypertension and headache d. Flat neck veins

a. Hypotension and dizziness The client with uncontrolled atrial fibrillation with a ventricular rate more than 150 beats a minute is at risk for low cardiac output because of loss of atrial kick. The nurse assesses the client for palpitations, chest pain or discomfort, hypotension, pulse deficit, fatigue, weakness, dizziness, syncope, shortness of breath, and distended neck veins.

PostOp colostomy

a. If a pouch system is not in place, apply a petroleum jelly gauze over the stoma to keep it moist, covered with a dry sterile dressing; place a pouch system on the stoma as soon as possible. b. Monitor the pouch system for proper fit and signs of leakage; empty the pouch when one-third full. c. Monitor the stoma for size, unusual bleeding, color changes, or necrotic tissue. d. Note that the normal stoma color is red or pink, indicating high vascularity. e. Note that a pale pink stoma indicates low hemoglobin and hematocrit levels. f. Assess the functioning of the colostomy. g. Expect that stool will be liquid postoperatively but will become more solid, depending on the area of the colostomy. h. Expect liquid stool from an ascending colon colostomy, loose to semiformed stool from a transverse colon colostomy, or close to normal stool from a descending colon colostomy. i. Fecal matter should not be allowed to remain on the skin. j. Administer analgesics and antibiotics as prescribed. k. Irrigate perineal wound if present and if prescribed, and monitor for signs of infection; provide comfort measures for perineal itching and pain. l. Instruct the client to avoid foods that cause excessive gas formation and odor. m. Instruct the client in stoma care and irrigations as prescribed. n. Instruct the client on how to resume normal activities, including work, travel, and sexual intercourse, as prescribed; provide psychosocial support.

Which sign or symptom suggest that a client's abdominal aortic aneurysm is extending? a. Increased abdominal and back pain b. Decreased pulse rate and blood pressure c. Retrosternal back pain radiating to the left arm d. Elevated blood pressure and rapid respirations

a. Increased abdominal and back pain Pain in the abdomen and back signify that the aneurysm is pressing downward on the lumbar nerve root and is causing more pain. The pulse rate would increase with aneurysm extension. Chest pain radiating down the arm would indicate myocardial infarction. Blood pressure would decrease with aneurysm extension, and the respiratory rate may not be affected.

Which of the following may occur with respiratory acidosis? a. Increased intracranial pressure b. Mental alertness c. Decreased pulse d. Decreased blood pressure

a. Increased intracranial pressure

While a female client is being prepared for discharge, the nasogastric (NG) feeding tube becomes clogged. To remedy this problem and teach the client's family how to deal with it at home, what should the nurse do? a. Irrigate the tube with cola. b. Advance the tube into the intestine. c. Apply intermittent suction to the tube. d. Withdraw the obstruction with a 30-ml syringe.

a. Irrigate the tube with cola. The nurse should irrigate the tube with cola because its effervescence and acidity are suited to the purpose, it's inexpensive, and it's readily available in most homes. Advancing the NG tube is inappropriate because the tube is designed to stay in the stomach and isn't long enough to reach the intestines. Applying intermittent suction or using a syringe for aspiration is unlikely to dislodge the material clogging the tube but may create excess pressure. Intermittent suction may even collapse the tube.

The blood pressure of a 28-year-old client who is overweight and does not have diabetes is 136/78 mm Hg. Client education should include: a. Losing 20 pounds would bring the blood pressure down by as much as 20 points and reduce the risk for stroke. b. The need to recheck blood pressure weekly so medications can be started as soon as it goes over 140/90 mm Hg. c. The need for medications in clients who are overweight with a systolic blood pressure over 130/80 mm Hg. d. Acknowledging that the client has maintained a blood pressure within normal limits.

a. Losing 20 pounds would bring the blood pressure down by as much as 20 points and reduce the risk for stroke.

Interventions for pancreatitis

a. Maintain NPO status and maintain hydration with IV fluids as prescribed. b. Administer parenteral nutrition for severe nutritional depletion. c. Administer supplemental preparations and vitamins and minerals to increase caloric intake if prescribed. d. Maintain nasogastric tube to decrease gastric distention and suppress pancreatic secretion. e. Administer meperidine hydrochloride (Demerol) as prescribed for pain because it causes less incidence of smooth muscle spasm of the pancreatic ducts and sphincter of Oddi than some other medications. f. Administer antacids as prescribed to neutralize gastric secretions. g. Administer H2-receptor antagonists or proton pump inhibitors as prescribed to decrease hydrochloric acid production and prevent activation of pancreatic enzymes. h. Administer anticholinergics as prescribed to decrease vagal stimulation, decrease gastrointestinal motility, and inhibit pancreatic enzyme secretion. i. Instruct the client in the importance of avoiding alcohol. j. Instruct the client in the importance of follow-up visits with the physician. k. Instruct the client to notify the physician if acute abdominal pain, jaundice, clay-colored stools, or dark-colored urine develops.

A nurse caring for a client with an ileostomy understands that the client is most at risk for developing which acid-base disorder? a. Metabolic acidosis b. Metabolic alkalosis c. Respiratory acidosis d. Respiratory alkalosis

a. Metabolic acidosis

A nurse notes that a client's arterial blood gas results reveal a pH of 7.50 and a PCO2 of 30 mm Hg. The nurse monitors that client for which clinical manifestations associated with these ABG results? Select all the apply. a. Nausea b. Confusion c. Bradypnea d. Tachycardia e. Hyperkalemia 6. Lightheadedness

a. Nausea b. Confusion d. Tachycardia 6. Lightheadedness

A client with hypertension visits the health clinic for a routine checkup. The nurse measures the client's blood pressure at 184/92 mm Hg and notes a 5-lb weight gain over the past month. Which nursing diagnosis reflects the most serious problem in managing a client with hypertension? a. Noncompliance (nonadherence to therapeutic regimen) b. Deficient knowledge (disease process) c. Excess fluid volume d. Imbalanced nutrition: More than body requirements

a. Noncompliance (nonadherence to therapeutic regimen)

A nurse is assessing an electrocardiogram rhythm strip. The P waves and QRS complexes are regular. The PR interval is 0.16 second, and QRS complexes measure 0.06 second. The overall heart rate is 64 beats per minute. The nurse assesses the cardiac rhythm as: a. Normal sinus rhythm b. Sinus bradycardia c. Sick sinus syndrome d. First-degree heart block.

a. Normal sinus rhythm measurements are normal, measuring 0.12 to 0.20 seconds and < .12 seconds, respectively.

A male client undergoes total gastrectomy. Several hours after surgery, the nurse notes that the client's nasogastric (NG) tube has stopped draining. How should the nurse respond? a. Notify the physician b. Reposition the tube c. Irrigate the tube d. Increase the suction level

a. Notify the physician An NG tube that fails to drain during the postoperative period should be reported to the physician immediately. It may be clogged, which could increase pressure on the suture site because fluid isn't draining adequately. Repositioning or irrigating an NG tube in a client who has undergone gastric surgery can disrupt the anastomosis. Increasing the level of suction may cause trauma to GI mucosa or the suture line.

Which of the following is a correct route of administration for potassium? a. Oral b. IV push c. Subcutaneous d. Intramuscular

a. Oral

Signs and symptoms of diabetes mellitus

a. Polyuria, polydipsia, polyphagia (more common in type 1 diabetes mellitus) b. Hyperglycemia (126 or > fasting) c. Weight loss (common in type 1 diabetes mellitus, rare in type 2 diabetes mellitus) d. Blurred vision e. Slow wound healing f. Vaginal infections g. Weakness and paresthesias h. Signs of inadequate circulation to the feet i. Signs of accelerated atherosclerosis (renal, cerebral, cardiac, peripheral)

Nurse Juvy is caring for a client with cirrhosis of the liver. To minimize the effects of the disorder, the nurse teaches the client about foods that are high in thiamine. The nurse determines that the client has the best understanding of the dietary measures to follow if the client states an intension to increase the intake of: a. Pork b. Milk c. Chicken d. Broccoli

a. Pork The client with cirrhosis needs to consume foods high in thiamine. Thiamine is present in a variety of foods of plant and animal origin. Pork products are especially rich in this vitamin. Other good food sources include nuts, whole grain cereals, and legumes. Milk contains vitamins A, D, and B2. Poultry contains niacin. Broccoli contains vitamins C, E, and K and folic acid

Because a client has mitral stenosis and is a prospective valve recipient, the nurse preoperatively assesses the client's past compliance with medical regimens. Lack of compliance with which of the following regimens would pose the greatest health hazard to this client? a. Medication therapy b. Diet modification c. Activity restrictions d. Dental care

a. Preoperatively, anticoagulants may be prescribed for the client with advanced valvular heart disease to prevent emboli. Post-op, all clients with mechanical valves and some with bioprostheses are maintained indefinitely on anticoagulation therapy. Adhering strictly to a dosage schedule and observing specific precautions are necessary to prevent hemorrhage or thromboembolism. Some clients are maintained on lifelong antibiotic prophylaxis to prevent recurrence from rheumatic fever. Episodic prophylaxis is required to prevent infective endocarditis after dental procedures or upper respiratory, GI, or GU surgery.

When teaching a client about propranolol hydrochloride, the nurse should base the information on the knowledge that propranolol hydrochloride: a. Blocks beta-adrenergic stimulation and thus causes decreased heart rate, myocardial contractility, and conduction. b. Increases norepinephrine secretion and thus decreases blood pressure and heart rate. c. Is a potent arterial and venous vasodilator that reduces peripheral vascular resistance and lowers blood pressure. d. Is an angiotensin-converting enzyme inhibitor that reduces blood pressure by blocking the conversion of angiotensin I to angiotensin II.

a. Propranolol hydrochloride is a beta-adrenergic blocking agent. Actions of propranolol hydrochloride include reducing heart rate, decreasing myocardial contractility, and slowing conduction.

A nurse should evaluate the effects of coumadin, used in the treatment of deep vein thrombosis, by looking at the results of which laboratory tests? a. Prothrmbn time (PT). b. Lee-White clotting time. c. Partial thromboplastin time (PTT). d. Fibrinogen clotting time (FCT).

a. Prothrmbn time (PT).

Following a treadmill test and cardiac catheterization, the client is found to have coronary artery disease, which is inoperative. He is referred to the cardiac rehabilitation unit. During his first visit to the unit he says that he doesn't understand why he needs to be there because there is nothing that can be done to make him better. The best nursing response is: a. "Cardiac rehabilitation is not a cure but can help restore you to many of your former activities." b. "Here we teach you to gradually change your lifestyle to accommodate your heart disease." c. "You are probably right but we can gradually increase your activities so that you can live a more active life." d. "Do you feel that you will have to make some changes in your life now?"

a. Such a response does not have false hope to the client but is positive and realistic. The answer tells the client what cardiac rehabilitation is and does not dwell upon his negativity about it.

A client is wearing a continuous cardiac monitor, which begins to sound its alarm. A nurse sees no electrocardiogram complexes on the screen. The first action of the nurse is to: a. Check the client status and lead placement. b. Press the recorder button on the electrocardiogram console. c. Call the physician. d. Call a code blue.

a. Sudden loss of electrocardiogram complexes indicates ventricular asystole or possible electrode displacement. Accurate assessment of the client and equipment is necessary to determine the cause and identify the appropriate intervention.

The nurse is monitoring a female client for the early signs and symptoms of dumping syndrome. Which of the following indicate this occurrence? a. Sweating and pallor b. Bradycardia and indigestion c. Double vision and chest pain d. Abdominal cramping and pain

a. Sweating and pallor Early manifestations of dumping syndrome occur 5 to 30 minutes after eating. Symptoms include vertigo, tachycardia, syncope, sweating, pallor, palpitations, and the desire to lie down.

After open-heart surgery a client develops a temperature of 102*F. The nurse notifies the physician, because elevated temperatures: a. Increase the cardiac output b. May indicate cerebral edema c. May be a forerunner of hemorrhage d. Are related to diaphoresis and possible chilling

a. Temperatures of 102*F or greater lead to an increased metabolism and cardiac workload.

Diet and diabetes

a. The total number of calories is individualized based on the client's current or desired weight and the presence of other existing health problems. b. Day-to-day consistency in timing and amount of food intake helps control the blood glucose level. c. As prescribed by the physician, the client may be advised to follow the food exchange recommendations of the American Diabetic Association d. Carbohydrate counting may be a simpler approach for some clients; it focuses on the total grams of carbohydrates eaten per meal. The client may be more compliant with carbohydrate counting, resulting in better glycemic control; it is usually necessary for clients undergoing intense insulin therapy. e. Incorporate the diet into individual client needs, lifestyle, and cultural and socioeconomic patterns.

Interventions for Chron's disease

antimicrobials corticosteroids immunosuppressive agents flagyl B12 injections high calorie, high protein, nitrogen, low fat, low residue diet - no dairy If for surgery then NPO and TPN surgery only done if not responsive to other therapy - high incidence of recurrence - not a cure

What criteria should the nurse use to determine normal sinus rhythm for a client on a cardiac monitor? Check all that apply. a. The RR intervals are relatively consistent b. One P wave precedes each QRS complex c. Four to eight complexes occur in a 6 second strip d. The ST segment is higher than the PR interval e. The QRS complex ranges from 0.12 to 0.20 second.

a. The RR intervals are relatively consistent b. One P wave precedes each QRS complex The consistency of the RR interval indicates regular rhythm. A normal P wave before each complex indicates the impulse originated in the SA node. The number of complexes in a 6 second strip is multiplied by 10 to approximate the heart rate; normal sinus rhythm is 60 to 100. Elevation of the ST segment is a sign of cardiac ischemia and is unrelated to the rhythm. The QRS duration should be less than 0.12 second; the PR interval should be 0.12 to 0.20 second.

The nurse is evaluating a client who had a myocardial infarction (MI) 7 days ago. Which outcome indicates that the client is responding favorably to therapy? a. The client demonstrates the ability to tolerate increasing activity without chest pain. b. The client exhibits a heart rate above 100 beats/minute. c. The client verbalizes the intention of making all necessary lifestyle changes except for stopping smoking. d. The client states that sublingual nitroglycerin usually relieves chest pain.

a. The client demonstrates the ability to tolerate increasing activity without chest pain. The ability to tolerate increasing activity without chest pain indicates a favorable response to therapy in a client who is recovering from an MI or who has a history of coronary artery disease. The client should have a normal electrocardiogram with no arrhythmias and a regular heart rate of 60 to 100 beats/minute. Smoking is a cardiovascular risk factor that the client must be willing to eliminate. A client who responds favorably to therapy shouldn't have chest pain.

Which of the following symptoms should the nurse teach the client with unstable angina to report immediately to her physician? a. A change in the pattern of her pain b. Pain during sex c. Pain during an argument with her husband d. Pain during or after an activity such as lawnmowing

a. The client should report a change in the pattern of chest pain. It may indicate increasing severity of CAD.

After receiving shift report, the registered nurse in the cardiac step-down unit, must prioritize her client care assignment. She has an ancillary staff member available to help her care for her clients. Which of these clients should the registered nurse assess first? a. The client with heart failure who is having some difficulty breathing. b. The anxious client who was diagnosed with an acute myocardial infarction (MI) two days ago and who was transferred from the coronary care unit today. c. The demanding client who underwent coronary bypass surgery three days ago. d. The client admitted during the previous shift with new-onset controlled atrial fibrillation who has her call light on.

a. The client with heart failure who is having some difficulty breathing. The registered nurse should care for the client with heart failure who is experiencing difficulty breathing. Breathing takes precedence over the other client needs. The ancillary staff member can answer the call light of the client admitted with controlled atrial fibrillation. She can also attend to the demanding client who underwent coronary bypass surgery three days ago. Although anxiety can be detrimental to a client with an MI, anxiety doesn't take precedence over another client's breathing difficulty.

Which assessment finding should indicate to a nurse that client has progression of intermittent claudication? a. The distance a client can walk before leg pain starts. b. Presence of pedal edema in the legs after sitting 20 minutes. c. Changes in strength of peripheral pulses in the affected leg. d. Changes in skin temperature and color of the feet.

a. The distance a client can walk before leg pain starts.

A nurse is reading a physician's progress notes in the client's record and read that the physician has documented "insensible fluid loss of approximately 800mL daily." The nurse interprets that this type of fluid loss can occur through: a. The skin b. Urinary output c. Wound drainage d. The gastrointestinal tract

a. The skin

Which condition leads to chronic respiratory acidosis in older adults? a. Thoracic skeletal change b. Erratic meal patterns c. Decreased renal function d. Overuse of sodium bicarbonate

a. Thoracic skeletal change

A nurse is assessing a client with a suspected diagnosis of hypocalcemia. Which of the following clinical manifestations would the nurse expect to note in the client? a. Twitching b. Hypoactive bowel sounds c. Negative Trousseau's sign d. Hypoactive deep tendon reflexes

a. Twitching

A nurse reviews a client's electrolyte laboratory report and notes that the potassium level is 3.2 mEq/L. Which of the following would the nurse note on the electrocardiogram as a result of the laboratory value? a. U waves b. absent P waves c. elevated T waves d. elevated ST segment

a. U waves

When the patient diagnosed with angina pectoris complains that he is experiencing chest pain more frequently even at rest, the period of pain is longer, and it take less stress for the pain to occur, the nurse recognizes that the patient is describing which type of angina? a. Unstable b. Intractable c. Refractory d. Variant

a. Unstable

A client with rapid rate atrial fibrillation asks a nurse why the physician is going to perform carotid massage. The nurse responds that this procedure may stimulate the: a. Vagus nerve to slow the heart rate b. Vagus nerve to increase the heart rate; overdriving the rhythm. c. Diaphragmic nerve to slow the heart rate d. Diaphragmic nerve to overdrive the rhythm

a. Vagus nerve to slow the heart rate Carotid sinus massage is one of the maneuvers used for vagal stimulation to decrease a rapid heart rate and possibly terminate a tachydysrhythmia. The others include inducing the gag reflex and asking the client to strain or bear down. Medication therapy often is needed as an adjunct to keep the rate down or maintain the normal rhythm.

Which of the following signs and symptoms would most likely be found in a client with mitral regurgitation? a. Exertional dyspnea b. Confusion c. Elevated creatine phosphokinase concentration d. Chest pain

a. Weight gain, due to fluid retention and worsening heart failure, causes exertional dyspnea in clients with mitral regurgitation. The rise in left atrial pressure that accompanies mitral valve disease is transmitted backward into pulmonary veins, capillaries, and arterioles and eventually to the right ventricle. Signs and symptoms of pulmonary and systemic venous congestion follow.

Which food would be restricted in a low sodium diet? a. Whole milk. b. Ginger ale. c. Orange juice. d. Black coffee.

a. Whole milk.

The nurse is caring for a cardiac client who requires various cardiac medications. When the nurse helps the client out of bed for breakfast, the client becomes dizzy and asks to lie down. The nurse helps the client lie down, puts up the side rails, and obtains the client's blood pressure, which is 84/50 mm Hg. It's time for the nurse to administer the client's medications: nitroglycerin, metoprolol (Lopressor), and furosemide (Lasix). Which action is best taken by the nurse? a. Withhold the medications and notify the physician. b. Administer the medications immediately. c. Encourage the client to sit up and eat breakfast. d. Administer the nitroglycerin and metoprolol and withhold the furosemide.

a. Withhold the medications and notify the physician. The nurse should withhold the three medications and notify the physician. Each of these medications has the potential to lower the client's blood pressure. Administering them together when the client is already hypotensive may severely lower the client's blood pressure. The client may continue to experience dizziness when sitting up so breakfast should be held until his blood pressure stabilizes.

A client with chest pain doesn't respond to nitroglycerin. On admission to the emergency department, the health care team obtains an electrocardiogram and begins infusing I.V. morphine. The physician also considers administering alteplase (Activase). This thrombolytic agent must be administered how soon after onset of myocardial infarction (MI) symptoms? a. Within 12 hours b. Within 24 hours c. Within 24 to 48 hours d. Within 5 to 7 days

a. Within 12 hours For the best chance of salvaging the client's myocardium, a thrombolytic agent must be administered within 12 hours after onset of chest pain or other signs or symptoms of MI. Within the first 24 hours after an MI, sudden death is most likely to occur. I.V. heparin therapy begins after administration of a thrombolytic agent and usually continues for 5 to 7 days.

An obese white male client, age 49, is diagnosed with hypercholesterolemia. The physician prescribes a low-fat, low-cholesterol, low-calorie diet to reduce blood lipid levels and promote weight loss. This diet is crucial to the client's well-being because his race, sex, and age increase his risk for coronary artery disease (CAD). To determine if the client has other major risk factors for CAD, the nurse should assess for: a. a history of diabetes mellitus. b. elevated high-density lipoprotein (HDL) levels. c. a history of ischemic heart disease. d. alcoholism.

a. a history of diabetes mellitus. Diabetes mellitus, smoking, and hypertension are other major risk factors for CAD. Elevated HDL levels aren't a risk factor for CAD; in fact, increased HDL levels seem to protect against CAD. Ischemic heart disease is another term for CAD, not a risk factor. Alcoholism hasn't been identified as a major risk factor for CAD.

A client with a myocardial infarction (MI) develops pulmonary crackles and dyspnea. A chest X-ray shows evidence of pulmonary edema. The specific type of MI the client had is most probably: a. anterior. b. posterior. c. lateral. d. inferior.

a. anterior. An anterior MI causes left ventricular dysfunction and can lead to manifestations of heart failure, which include pulmonary crackles and dyspnea. The other types of MI aren't usually associated with heart failure.

A client is in hemorrhagic shock. To determine the effectiveness of fluid replacement therapy, the nurse should monitor the client's: a. blood pressure. b. hemoglobin level. c. temperature. d. heart rate.

a. blood pressure. With adequate fluid replacement, fluid volume in the intravascular space expands, raising the client's blood pressure. The hemoglobin level reflects red blood cell concentration, not overall fluid status. Temperature and heart rate aren't directly related to fluid status.

When the appropriate complex follows the pacing spike, it is said to be a. captured. b. nonsynchronous. c. inhibited. d. triggered.

a. captured.

After a myocardial infarction, a client develops a complication requiring a continuous infusion of lidocaine. To monitor the effectiveness of this infusion, the nurse should focus primarily on: a. electrocardiogram (ECG). b. urine output. c. creatine kinase (CK) and troponin levels. d. blood pressure and heart rate.

a. electrocardiogram (ECG). Lidocaine is an antiarrhythmic given to treat cardiac irritability and ventricular arrhythmias. The best indicator of its effectiveness is the reduction or disappearance of ventricular arrhythmias as seen on the ECG. Urine output is an indicator of pump effectiveness. CK and troponin levels monitor myocardial damage. Blood pressure and heart rate are too nonspecific to be indicators of lidocaine's effectiveness.

Which of the following is characteristic of the intrarenal category of acute renal failure? a. increased BUN b. high specific gravity c. decreased urine sodium d. decreased creatinine

a. increased BUN The intrarenal category of acute renal failure encompasses an increased BUN, increased creatinine, a low specific gravity of urine, and an increased urine sodium.

A nurse reviews the blood gas results of a client with atelactasis. The nurse analyzes the results and determines that the client is experiencing respiratory acidosis. Which of the following validates that nurse's findings? a. pH 7.25, PCO2 50 mm Hg b. 7.35, PCO2 40 mm Hg c. pH 7.50, PCO2 52 mm Hg d. pH 7.52, PCO2 28 mm Hg

a. pH 7.25, PCO2 50 mm Hg

A client reports recent onset of chest pain that occurs sporadically with exertion. The client also has fatigue and mild ankle swelling, which is most pronounced at the end of the day. The nurse suspects a cardiovascular disorder. When exploring the chief complaint, the nurse should find out if the client has any other common cardiovascular symptoms, such as: a. shortness of breath. b. insomnia. c. irritability. d. lower substernal abdominal pain.

a. shortness of breath. Common signs and symptoms of cardiovascular dysfunction include shortness of breath, chest pain, dyspnea, palpitations, fainting, fatigue, and peripheral edema. Insomnia seldom indicates a cardiovascular problem. Although irritability may occur if cardiovascular dysfunction leads to cerebral oxygen deprivation, this symptom more commonly reflects a respiratory or neurologic dysfunction. Lower substernal abdominal pain occurs with some GI disorders.

A client with chronic heart failure is receiving digoxin (Lanoxin), 0.25 mg by mouth (P.O.) daily, and furosemide (Lasix), 20 mg P.O. twice daily. The nurse instructs the client to notify the physician if nausea, vomiting, diarrhea, or abdominal cramps occur because these signs and symptoms may signal digitalis toxicity. Digitalis toxicity also may cause: a. visual disturbances. b. taste and smell alterations. c. dry mouth and urine retention. d. nocturia and sleep disturbances.

a. visual disturbances. Digitalis toxicity may cause visual disturbances (such as flickering light flashes, colored or halo vision, photophobia, blurring, diplopia, and scotomata), central nervous system abnormalities (such as headache, fatigue, lethargy, depression, irritability and, if profound, seizures, delusions, hallucinations, and memory loss), and cardiovascular abnormalities (abnormal heart rate and arrhythmias). Taste and smell alterations aren't associated with digitalis toxicity. Dry mouth and urine retention typically occur with anticholinergic agents, not inotropic agents such as digoxin. Nocturia and sleep disturbances are adverse effects of furosemide — especially if the client takes the second daily dose in the evening, which may cause diuresis at night.

A male client with cholelithiasis has a gallstone lodged in the common bile duct. When assessing this client, the nurse expects to note: a. yellow sclerae. b. light amber urine. c. circumoral pallor. d. black, tarry stools.

a. yellow sclerae. Yellow sclerae may be the first sign of jaundice, which occurs when the common bile duct is obstructed. Urine normally is light amber. Circumoral pallor and black, tarry stools don't occur in common bile duct obstruction; they are signs of hypoxia and GI bleeding, respectively.

pace maker - sensing

ability to recognize spontaneous atrial or ventricular activity

Hernia

abnormal protrusion of an organ through the structures normally containing it

Achalasia

absence of peristalsis of the lower esophagus and tightened LES

Bulk Forming Laxatives

absorb water into feces and increase bulk to produce stools ie. senakot, citrucel, metamucil, fibercon

Vitamin D (function)

absorption of calcium and phosphorus

empyema

accumulation of prurulent material in the pleural space

Sodium bicarbonate is used to treat...

acidosis

Inspiration

active, diaphragm contracts/flattens, thoracic cage expands, chest volume increases, intrapulmonary pressure decreases, air moves in to lungs

Causes of Obstruction

air way edema and mucous --> bronchoconstriction

Causes of mechanical intestinal obstruction

adhesions following surgery (50%) hernia (15%) cancer (15%) volvulus diverticular disease

Causes of Hyponatremia

adrenal insufficiency, water intoxication, SIADH, vomiting, diarrhea, sweating, diuretics

Factors Affecting BMR

age, sex, climate, state of health, sleep, activity

Causes of Pancreatitis

alcohol abuse biliary tract disease infection drugs (steroids, thiazide, diuretics, sulfonamides, NSAIDs) trauma metabolic disorders - hyperlipidemia, hyperparathyroidism, renal failure Kaposi sarcoma

Causes of Hypophosphatemia

alcohol withdrawl, resp alkalosis

Causes of Hypomagnesemia

alcoholism, gastric suction, malabsorption

Full Liquid Diet

all liquids and foods that become liquid at body temperature - used as a progression between clear liquids and soft diet

Neuromuscular assessment finding for FVE

altered LOC, headache, visual disturbances, muscle weakness, paresthsia

Intervention for hyperphosphatemia

aluminum hydroxide,

Antidysrhytmics and MI

amioderone, lidocaine

Respiratory excursion

an estimation of thoracic expansion - assessment for range and symmetry of excursion - decreased excursion may be caused by chronic fibrotic disease - asymmetric excursion may be due to splinting or unilateral bronchial obstruction

Bronchophony

an increased intensity and clarity of voice sounds heard over a bronchus surrounded by consolidated lung tissue

Peritonitis

an inflammatory process in the peritoneum that results in extracellular fluid shifts

The common feature of all types of pneumonia is...

an inflammatory pulmonary response

A high-pressure alarm on a continuous mechanical ventilator indicates...

an obstruction in the flow of oxygen from the machine to the client - the nurse should suction the client's artificial airway to remove respiratory secretions that could be causing the obstruction

Arterial vs Venous ulcers

arterial - pale, not a lot of blood, cool, usually on upper feet and toes venous - not as painful, red tissue, bloods, warm

Interventions for Angina

assess pain (1-10), administer O2, vital signs, administer nitrates, obtain 12 lead EKG

RDA

average daily dietary intake level that is sufficient to meet the nutrient requirement of nearly all healthy individuals in a particular life stage and gender group based on age, sex, pregnant/lactating, height/weight avg., lives in USA

Interventions for hypermagnesemia

avoid laxatives and antacids/meds containing magnesium, diuretics

An adult client with cystic fibrosis is admitted to an acute care facility with an acute respiratory infection. Prescribed respiratory treatment includes chest physiotherapy. When should the nurse perform this procedure? a) Immediately before a meal b) At least 2 hours after a meal c) When bronchospasms occur d) When secretions have mobilized

b) At least 2 hours after a meal

Before weaning a client from a ventilator, which assessment parameter is most important for the nurse to review? a) Fluid intake for the past 24 hours b) Baseline arterial blood gas (ABG) levels c) Prior outcomes of weaning d) Electrocardiogram (ECG) results

b) Baseline arterial blood gas (ABG) levels

Cor Pulmonale

cardiac problem with respiratory etiology characterized by right sided hypertrophy and ventricular impairment - leads to heart failure - secondary to pulmonary hypertension

A client with myasthenia gravis is receiving continuous mechanical ventilation. When the high-pressure alarm on the ventilator sounds, what should the nurse do? a) Check for an apical pulse. b) Suction the client's artificial airway. c) Increase the oxygen percentage. d) Ventilate the client with a handheld mechanical ventilator.

b) Suction the client's artificial airway.

A client with end-stage chronic obstructive pulmonary disease requires bi-level positive airway pressure (BiPAP). While caring for the client, the nurse determines that bilateral wrist restraints are required to prevent compromised care. Which client care outcome is associated with restraint use in the client who requires BiPAP? a) The client will remain infection-free. b) The client will maintain adequate oxygenation. c) The client will maintain adequate urine output. d) The client will remain pain-free.

b) The client will maintain adequate oxygenation.

A client undergoes a total laryngectomy and tracheostomy formation. On discharge, the nurse should give which instruction to the client and family? a) "Clean the tracheostomy tube with alcohol and water." b) "Family members should continue to talk to the client." c) "Oral intake of fluids should be limited for 1 week only." d) "Limit the amount of protein in the diet."

b) "Family members should continue to talk to the client."

A home care nurse visits a client with chronic obstructive pulmonary disease who requires oxygen. Which statement by the client indicates the need for additional teaching about home oxygen use? a) "I lubricate my lips and nose with K-Y jelly." b) "I make sure my oxygen mask is on tightly so it won't fall off while I nap." c) "I have a 'no smoking' sign posted at my front entry-way to remind guests not to smoke." d) "I clean my mask with water after every meal."

b) "I make sure my oxygen mask is on tightly so it won't fall off while I nap."

For a client with an acute pulmonary embolism, the physician prescribes heparin (Liquaemin), 25,000 U in 500 ml of dextrose 5% in water (D5W) at 1,100 U/hour. The nurse should administer how many milliliters per hour? a) 8 b) 22 c) 30 d) 50

b) 22

What is the normal pH range for arterial blood? a) 7 to 7.49 b) 7.35 to 7.45 c) 7.50 to 7.60 d) 7.55 to 7.65

b) 7.35 to 7.45

On auscultation, which finding suggests a right pneumothorax? a) Bilateral inspiratory and expiratory crackles b) Absence of breath sounds in the right thorax c) Inspiratory wheezes in the right thorax d) Bilateral pleural friction rub

b) Absence of breath sounds in the right thorax

A client with a pulmonary embolus has the following arterial blood gas (ABG) values: pH, 7.49; partial pressure of arterial oxygen (PaO2), 60 mm Hg; partial pressure of arterial carbon dioxide (PaCO2), 30 mm Hg; bicarbonate (HCO3-) 25 mEq/L. What should the nurse do first? a) Instruct the client to breathe into a paper bag. b) Administer oxygen by nasal cannula as prescribed. c) Auscultate breath sounds bilaterally every 4 hours. d) Encourage the client to deep-breathe and cough every 2 hours.

b) Administer oxygen by nasal cannula as prescribed.

The physician prescribes triamcinolone (Azmacort) and salmeterol (Serevent) for a client with a history of asthma. What action should the nurse take when administering these drugs? a) Administer the triamcinolone and then administer the salmeterol. b) Administer the salmeterol and then administer the triamcinolone. c) Allow the client to choose the order in which the drugs are administered. d) Monitor the client's theophylline level before administering the medications.

b) Administer the salmeterol and then administer the triamcinolone.

A client has a sucking stab wound to the chest. Which action should the nurse take first? a) Draw blood for a hematocrit and hemoglobin level. b) Apply a dressing over the wound and tape it on three sides. c) Prepare a chest tube insertion tray. d) Prepare to start an I.V. line.

b) Apply a dressing over the wound and tape it on three sides.

A client, confused and short of breath, is brought to the emergency department by a family member. The medical history reveals chronic bronchitis and hypertension. To learn more about the client's current respiratory problem, the physician orders a chest X-ray and arterial blood gas (ABG) analysis. When reviewing the ABG report, the nurse sees many abbreviations. What does a lowercase "a" in an ABG value represent? a) Acid-base balance b) Arterial blood c) Arterial oxygen saturation d) Alveoli

b) Arterial blood

Systemic effects of hypertension

cardiac, cerebral, peripheral vascular, renal, retinal

A client with acute bronchitis is admitted to the health care facility and is receiving supplemental oxygen via nasal cannula. When monitoring this client, the nurse suddenly hears a high-pitched whistling sound. What is the most likely cause of this sound? a) The water level in the humidifier reservoir is too low. b) The oxygen tubing is pinched. c) The client has a nasal obstruction. d) The oxygen concentration is above 44%.

b) The oxygen tubing is pinched.

A client with a pneumothorax receives a chest tube attached to a Pleur-evac. The nurse notices that the fluid of the second chamber of the Pleur-evac isn't bubbling. Which nursing assumption would be most invalid? a) The tubing from the client to the chamber is blocked. b) There is a leak somewhere in the tubing system. c) The client's affected lung has reexpanded. d) The tubing needs to be cleared of fluid.

b) There is a leak somewhere in the tubing system.

After receiving the wrong medication, the client's breathing stops. The nurse initiates the code protocol, and the client is emergently intubated. As soon as the client's condition stabilizes, the nurse completes an incident report. What should the nurse do next? a) Place the incident report on the client's chart. b) Document the incident in the nurses' notes. c) Document in the nurses' notes that an incident report was completed. d) Make a copy of the incident report for the client.

b) Document the incident in the nurses' notes.

A client with chronic obstructive pulmonary disease (COPD) is being evaluated for a lung transplant. The nurse performs the initial physical assessment. Which signs and symptoms should the nurse expect to find? a) Decreased respiratory rate b) Dyspnea on exertion c) Barrel chest d) Shortened expiratory phase e) Clubbed fingers and toes f) Fever

b) Dyspnea on exertion c) Barrel chest e) Clubbed fingers and toes

The nurse is planning care for a client after a tracheostomy. One of the client's goals is to overcome verbal communication impairment. Which of the following interventions should the nurse include in the care plan? a) Make an effort to read the client's lips to foster communication. b) Encourage the client's communication attempts by allowing him time to select or write words. c) Answer questions for the client to reduce his frustration. d) Avoid using a tracheostomy plug because it blocks the airway.

b) Encourage the client's communication attempts by allowing him time to select or write words.

Which condition would the nurse expect to find in a client diagnosed with hyperparathyroidism? a) Hypocalcemia b) Hypercalcemia c) Hyperphosphatemia d) Hypophosphaturia

b) Hypercalcemia

A nurse is completing her annual cardiopulmonary resuscitation training. The class instructor tells her that a client has fallen off a ladder and is lying on his back; he is unconscious and isn't breathing. What maneuver should the nurse use to open his airway? a) Head tilt-chin lift b) Jaw-thrust c) Heimlich d) Seldinger

b) Jaw-thrust

The physician orders an isotonic I.V. solution for a client. Which solution should the nurse plan to administer? a) 5% dextrose and normal saline solution b) Lactated Ringer's solution c) Half-normal saline solution d) 10% dextrose in water

b) Lactated Ringer's solution

A client in the emergency department reports that he has been vomiting excessively for the past 2 days. His arterial blood gas analysis shows a pH of 7.50, PaCO2 of 43 mm Hg, PaO2 of 75 mm Hg, and HCO3− of 42 mEq/L. Based on these findings, the nurse documents that the patient is experiencing which type of acid-base imbalance? a) Respiratory alkalosis b) Metabolic alkalosis c) Respiratory acidosis d) Metabolic acidosis

b) Metabolic alkalosis

A client abruptly sits up in bed, reports having difficulty breathing and has an arterial oxygen saturation of 88%. Which mode of oxygen delivery would most likely reverse the manifestations? a) Simple mask b) Nonrebreather mask c) Face tent d) Nasal cannula

b) Nonrebreather mask

An elderly client with influenza is admitted to an acute care facility. The nurse monitors the client closely for complications. What is the most common complication of influenza? a) Septicemia b) Pneumonia c) Meningitis d) Pulmonary edema

b) Pneumonia

A nurse detects bilateral crackles when auscultating a client's lungs. Which statement about crackles is true? a) They're usually heard on expiration and may clear with a cough. b) They're usually heard on inspiration and sometimes clear with a cough. c) They're hissing or musical and are usually heard on inspiration and expiration; if severe, they may be heard without a stethoscope. d) They're creaking and grating and are usually heard over the problem area on both inspiration and expiration.

b) They're usually heard on inspiration and sometimes clear with a cough.

Prednisone (Deltasone) is prescribed to control inflammation in a client with interstitial lung disease. During client teaching, the nurse stresses the importance of taking prednisone exactly as prescribed and cautions against discontinuing the drug abruptly. A client who discontinues prednisone abruptly may experience: a) hyperglycemia and glycosuria. b) acute adrenocortical insufficiency. c) GI bleeding. d) restlessness and seizures.

b) acute adrenocortical insufficiency.

A 47-year-old male client with unresolved hemothorax is febrile, with chills and sweating. He has a nonproductive cough and chest pain. His chest tube drainage is turbid. A possible explanation for these findings is: a) lobar pneumonia. b) empyema. c) Pneumocystis carinii pneumonia. d) infected chest tube wound site.

b) empyema.

A pt with fluid retention related to renal problems is admitted to the hospital. The nurse realizes that this pt could possibly have which of the following electrolyte imbalances? a) hypokalemia b) hypernatremia c) carbon dioxide d) magnesium

b) hypernatremia

An elderly pt comes into the clinic with the complaint of watery diarrhea for several days with abdominal & muscle cramping. The nurse realizes that this pt is demonstrating which of the following? a) hypernatremia b) hyponatremia c) fluid volume excess d) hyperkalemia

b) hyponatremia

A pt is admitted with burns over 50% of his body. The nurse realizes that this pt is at risk for which of the following electrolyte imbalances? a) hypercalcemia b) hypophosphatemia c) hypernatremia d) hypermagnesemia

b) hypophosphatemia

The nurse observes a pt's respirations & notes that the rate is 30 per minute & the respirations are very deep. The metabolic disorder this pt might be demonstrating is which of the following? a) hypernatremia b) increasing carbon dioxide in the blood c) hypertension d) pain

b) increasing carbon dioxide in the blood

The nurse is caring for a client who recently underwent a tracheostomy. The first priority when caring for a client with a tracheostomy is: a) helping him communicate. b) keeping his airway patent. c) encouraging him to perform activities of daily living. d) preventing him from developing an infection.

b) keeping his airway patent.

A pt is prescribed 40 mEq potassium as a replacement. The nurse realizes that this replacement should be administered a) directly into the venous access line. b) mixed in the prescribed intravenous fluid. c) via a rectal suppository. d) via intramuscular injection.

b) mixed in the prescribed intravenous fluid.

The nurse is caring for a client experiencing an acute asthma attack. The client stops wheezing and breath sounds aren't audible. This change occurred because: a) the attack is over. b) the airways are so swollen that no air can get through. c) the swelling has decreased. d) crackles have replaced wheezes.

b) the airways are so swollen that no air can get through.

A pt is prescribed 20 mEq of potassium chloride. The nurse realizes that the reason the pt is receiving this replacement is a) to sustain respiratory function. b) to help regulate acid-base balance. c) to keep a vein open. d) to encourage urine output.

b) to help regulate acid-base balance.

A client recovering from a pulmonary embolism is receiving warfarin (Coumadin). To counteract a warfarin overdose, the nurse would administer: a) heparin. b) vitamin K1 (phytonadione). c) vitamin C. d) protamine sulfate.

b) vitamin K1 (phytonadione).

The pt, newly diagnosed with diabetes mellitus, is admitted to the emergency department with nausea, vomiting, & abdominal pain. ABG results reveal a pH of 7.2 & a bicarbonate level of 20 mEq/L. Which other assessment findings would the nurse anticipate in this pt? Select all that apply. a) tachycardia b) weakness c) dysrhythmias d) Kussmaul's respirations 5. cold, clammy skin

b) weakness c) dysrhythmias d) Kussmaul's respirations

The pt is receiving intravenous potassium (KCL). Which nursing actions are required? Select all that apply. a) Administer the dose IV push over 3 minutes. b) Monitor the injection site for redness. c) Add the ordered dose to the IV hanging. d) Use an infusion controller for the IV. e) Monitor fluid intake & output.

b,d,e

A client is receiving conscious sedation while undergoing bronchoscopy. Which assessment finding should receive the nurse's immediate attention? a) Absent cough and gag reflexes b) Blood-tinged secretions c) Oxygen saturation of 90% d) Respiratory rate of 13 breaths/min

c) Oxygen saturation of 90%

To prevent gastroesophageal reflux in a male client with hiatal hernia, the nurse should provide which discharge instruction? a. "Lie down after meals to promote digestion." b. "Avoid coffee and alcoholic beverages." c. "Take antacids with meals." d. "Limit fluid intake with meals."

b. "Avoid coffee and alcoholic beverages." To prevent reflux of stomach acid into the esophagus, the nurse should advise the client to avoid foods and beverages that increase stomach acid, such as coffee and alcohol. The nurse also should teach the client to avoid lying down after meals, which can aggravate reflux, and to take antacids after eating. The client need not limit fluid intake with meals as long as the fluids aren't gastric irritants.

Before discharge, which instruction should the nurse give to a client receiving digoxin (Lanoxin)? a. "Take an extra dose of digoxin if you miss one dose." b. "Call the physician if your heart rate is above 90 beats/minute." c. "Call the physician if your pulse drops below 80 beats/minute." d. "Take digoxin with meals."

b. "Call the physician if your heart rate is above 90 beats/minute." The nurse should instruct the client to notify the physician if his heart rate is greater than 90 beats/minute because cardiac arrhythmias may occur with digitalis toxicity. To prevent toxicity, the client should be instructed never to take an extra dose of digoxin if a dose is missed. The nurse should show the client how to take her pulse and to call the physician if her pulse rate drops below 60 beats/minute — not 80 beats/minute, which is a normal pulse rate and doesn't warrant action. Digoxin shouldn't be administered with meals because this slows the absorption rate.

A client, who is overweight and had diabetes, has a blood pressure of 148/92 mm Hg. The client's urinalysis reveal microalbuminuria. The client does not want to take drugs despite a family history of brain attack (cerebrovascular accident [CVA]). The correct response by a nurse is: a. "Find herbal products to reduce blood pressure and weight." b. "Use a drug while lifestyle changes are instituted." c. Evaluate barriers to weight loss and make a plan for exercising and dieting." d. "Recheck blood pressure weekly."

b. "Use a drug while lifestyle changes are instituted."

The home care nurse is visiting a left-handed client who has an automated implantable cardioverter-defibrillator implanted in his left chest. The client tells the nurse how excited he is because he's planning to go rifle hunting with his grandson. How should the nurse respond? a. "Be sure to enjoy your time with your grandson." b. "You cannot shoot a rifle left-handed because the rifle's recoil will traumatize the AICD site." c. "Being that close to a rifle might make your AICD fire." d. "You will need to take an extra dose of your antiarrhythmic before you shoot."

b. "You cannot shoot a rifle left-handed because the rifle's recoil will traumatize the AICD site." The recoil from the rifle can damage the AICD so the client should be warned against shooting a rifle with his left hand. Close proximity to a rifle won't cause the AICD to fire inadvertently. The client shouldn't take an extra dose of his antiarrhythmic.

Which of the following is considered an isotonic solution? a. Dextran in normal saline b. 0.9% normal saline c. 0.45% normal saline d. 3% NaCl

b. 0.9% normal saline

During surgery, a client develops sinus bradycardia. The physician orders atropine sulfate. Which dose and route is the nurse most likely to administer? a. 0.6 mg I.M. b. 1 mg I.V. c. 2 mg I.M. d. 2 mg I.V.

b. 1 mg I.V. To reverse arrhythmias, bradycardia, or sinus arrest, the usual adult dosage of atropine is 0.4 to 1 mg I.V. every 2 hours as needed. The drug isn't administered I.M. for the treatment of bradycardia.

Following a percutaneous transluminal coronary angioplasty (PTCA), a client is monitored in the postprocedure unit. The client's heparin infusion was stopped 2 hours earlier. There is no evidence of bleeding or hematoma at the insertion site, and the pressure device is removed. The nurse should plan to remove the femoral sheath when the partial thromboplastin time (PTT) is: a. 25 seconds or less. b. 50 seconds or less. c. 75 seconds or less. d. 100 seconds or less.

b. 50 seconds or less. Heparin causes an elevation of the PTT and, thereby, increases the risk for bleeding. With a large cannulation such as a sheath used for angioplasty, the PTT should be 40 seconds or less before the sheath is removed. Removing the sheath prematurely can cause bleeding at the insertion site. The other PTT results are incorrect for determining when to remove the sheath.

Treatment for sinus tachycardia

carotid massage ice

The nurse on the telemetry unit is faced with various situations. Which situation takes priority? a. A client's cardiac monitor suddenly reveals sinus tachycardia with isolated premature ventricular contractions. b. A client's cardiac rhythm suddenly changes from normal sinus rhythm to uncontrolled atrial fibrillation. c. A client is requesting help to go to the bathroom. d. The cardiologist is asking the nurse to make rounds with him to his clients.

b. A client's cardiac rhythm suddenly changes from normal sinus rhythm to uncontrolled atrial fibrillation. The client whose cardiac rhythm suddenly changes from normal sinus rhythm to uncontrolled atrial fibrillation takes priority. This cardiac rhythm change may cause clots to shower from the atria placing the client at risk for a stroke. The client whose cardiac monitor reveals sinus tachycardia with isolated premature ventricular contractions isn't experiencing a life-threatening situation; therefore, he doesn't take priority. The nurse can ask her ancillary staff member to assist the client to the bathroom. Making rounds with the physician can wait until the nurse addresses the needs of the client in atrial fibrillation.

A client has driven himself to the ER. He is 50 years old, has a history of hypertension, and informs the nurse that his father died of a heart attack at 60 years of age. The client is presently complaining of indigestion. The nurse connects him to an ECG monitor and begins administering oxygen at 2 L/minute per NC. The nurse's next action would be to: a. Call for the doctor. b. Start an intravenous line. c. Obtain a portable chest radiograph. d. Draw blood for laboratory studies.

b. Advanced cardiac life support recommends that at least one or two intravenous lines be inserted in one or both of the antecubital spaces. Calling the physician, obtaining a portable chest radiograph, and drawing blood are important but secondary to starting the intravenous line.

A 35-year-old male was knifed in the street fight, admitted through the ER, and is now in the ICU. An assessment of his condition reveals the following symptoms: respirations shallow and rapid, CVP 15 cm H2O, BP 90 mm Hg systolic, skin cold and pale, urinary output 60-100 mL/hr for the last 2 hours. Analyzing these symptoms, the nurse will base a nursing diagnosis on the conclusion that the client has which of the following conditions? a. Hypovolemic shock b. Cardiac tamponade c. Wound dehiscence d. Atelectasis

b. All of the client's symptoms are found in both cardiac tamponade and hypovolemic shock except the increase in urinary output.

When do coronary arteries primarily receive blood flow? a. During inspiration b. During diastolic c. During expiration d. During systole

b. Although the coronary arteries may receive a minute portion of blood during systole, most of the blood flow to coronary arteries is supplied during diastole. Breathing patterns are irrelevant to blood flow.

A client comes to the physician's office for a complete physical examination required for employment. The physician assesses the client's arms and legs for evidence of peripheral vascular disease. What is the most commonly used overall indicator of arm and leg circulation? a. Exercise testing b. Ankle-brachial index c. Limb blood pressure d. Allen's test

b. Ankle-brachial index

Atherosclerosis impedes coronary blood flow by which of the following mechanisms? a. Plaques obstruct the vein b. Plaques obstruct the artery c. Blood clots form outside the vessel wall d. Hardened vessels dilate to allow blood to flow through

b. Arteries, not veins, supply the coronary arteries with oxygen and other nutrients. Atherosclerosis is a direct result of plaque formation in the artery. Hardened vessels can't dilate properly and, therefore, constrict blood flow.

The nurse is preparing a client for cardiac catheterization. The nurse knows that she must provide which nursing intervention when the client returns to his room after the procedure? a. Withhold analgesics for at least 6 hours after the procedure. b. Assess the puncture site frequently for hematoma formation or bleeding. c. Inform the client that he may experience numbness or pain in his leg. d. Restrict fluids for 6 hours after the procedure.

b. Assess the puncture site frequently for hematoma formation or bleeding. Because the diameter of the catheter used for cardiac catheterization is large, the puncture site must be checked frequently for hematoma formation and bleeding. The nurse should administer analgesics as prescribed and needed. If the femoral artery was accessed during the procedure, the client should be instructed to report any leg pain or numbness, which may indicate arterial insufficiency. Fluids should be encouraged to eliminate dye from the client's system.

A nurse is watching the cardiac monitor, and a client's rhythm suddenly changes. There are no P waves; instead there are wavy lines. The QRS complexes measure 0.08 second, but they are irregular, with a rate of 120 beats a minute. The nurse interprets this rhythm as: a. Sinus tachycardia b. Atrial fibrillation c. Ventricular tachycardia d. Ventricular fibrillation

b. Atrial fibrillation Atrial fibrillation is characterized by a loss of P waves; an undulating, wavy baseline; QRS duration that is often within normal limits; and an irregular ventricular rate, which can range from 60 to 100 beats per minute (when controlled with medications) to 100 to 160 beats per minute (when uncontrolled).

What laboratory finding is the primary diagnostic indicator for pancreatitis? a. Elevated blood urea nitrogen (BUN) b. Elevated serum lipase c. Elevated aspartate aminotransferase (AST) d. Increased lactate dehydrogenase (LD)

b. Elevated serum lipase Elevation of serum lipase is the most reliable indicator of pancreatitis because this enzyme is produced solely by the pancreas. A client's BUN is typically elevated in relation to renal dysfunction; the AST, in relation to liver dysfunction; and LD, in relation to damaged cardiac muscle.

The physician refers the client with unstable angina for a cardiac catherization. The nurse explains to the client that this procedure is being used in this specific case to: a. Open and dilate the blocked coronary arteries b. Assess the extent of arterial blockage c. Bypass obstructed vessels d. Assess the functional adequacy of the valves and heart muscle.

b. Cardiac catherization is done in clients with angina primarily to assess the extent and severity of the coronary artery blockage, A decision about medical management, angioplasty, or coronary artery bypass surgery will be based on the catherization results.

A client's ABGs reveal: pH 7.38, PO2 65, PCO2 55, HCO3 32. The nurse's interpretation of the blood gases is that he or she has: a. Uncompensated respiratory acidosis b. Compensated respiratory acidosis c. Uncompensated metabolic alkalosis d. Compensated metabolic alkalosis

b. Compensated respiratory acidosis

When ventricular fibrillation occurs in a CCU, the first person reaching the client should: a. Administer oxygen b. Defibrillate the client c. Initiate CPR d. Administer sodium bicarbonate intravenously

b. Defibrillate the client Ventricular fibrillation is a death-producing dysrhythmia and, once identified, must be terminated immediately by precordial shock (defibrillation). This is usually a standing physician's order in a CCU.

Which of the following is a modifiable risk factor for coronary artery disease ( CAD)? a. Race b. Diabetes mellitus c. Gender d. Increasing age

b. Diabetes mellitus

After extensive cardiac bypass surgery, a client returns to the intensive care unit on dobutamine (Dobutrex), 5 mcg/kg/minute I.V. Which classification best describes dobutamine? a. Indirect-acting dual-active agent b. Direct-acting beta-active agent c. Indirect-acting beta-active agent d. Direct-acting alpha-active agent

b. Direct-acting beta-active agent Dobutamine acts directly on beta receptors. Thus, the drug can be described as a direct-acting beta-active agent. Adrenergic agents are classified according to their method of action and the type of receptor they act on. Direct-acting agents act directly on the sympathetically innervated organ or tissue, whereas indirect-acting agents trigger the release of a neurotransmitter, usually norepinephrine. Dual-acting agents combine direct and indirect actions. Adrenergic agents act on alpha, beta, and dopamine receptors.

A client is prescribed lisinopril (Zestril) for treatment of hypertension. He asks the nurse about possible adverse effects. The nurse should teach him about which common adverse effects of angiotensin-converting enzyme (ACE) inhibitors? a. Constipation b. Dizziness c. Headache d. Hyperglycemia e. Hypotension f. Impotence

b. Dizziness c. Headache e. Hypotension

A client is suspected of having hepatitis. Which diagnostic test result will assist in confirming this diagnosis? a. Elevated hemoglobin level b. Elevated serum bilirubin level c. Elevated blood urea nitrogen level d. Decreased erythrocycle sedimentation rate

b. Elevated serum bilirubin level Laboratory indicators of hepatitis include elevated liver enzyme levels, elevated serum bilirubin levels, elevated erythrocyte sedimentation rates, and leukopenia. An elevated blood urea nitrogen level may indicate renal dysfunction. A hemoglobin level is unrelated to this diagnosis.

Which dietary modification is utilized for a patient diagnosed with acute pancreatitis? a. High-fat diet b. Elimination of coffee c. High-protein diet d. Low-carbohydrate diet

b. Elimination of coffee A high-carbohydrate, low-fat, and low-protein diet should be implemented.

A client has been admitted to the hospital with a diagnosis of suspected bacterial endocarditis. The complication the nurse will constantly observe for is: a. Presence of heart murmur b. Systemic emboli c. Fever d. Congestive heart failure

b. Emboli are the major problem; those arising in the right heart chambers will terminate in the lungs and left chamber emboli may travel anywhere in the arteries. Heart murmurs, fever, and night sweats may be present, but do not indicate a problem with emboli. CHF may be a result, but this is not as dangerous an outcome as emboli.

Identify the appropriate actions in the prevention of thrombophlebitis in a client on bedrest. Select all the apply. a. Elevate the knee gatch of the bed. b. Encourage exercises that dorsiflex and plantar flex the ankle. c. Apply sequential compression devices bilaterally. d. Prevent dehydration. e. Periodically elevate the feet and lower legs above level of heart. f. Apply warm, moist packs bilaterally to lower legs.

b. Encourage exercises that dorsiflex and plantar flex the ankle. c. Apply sequential compression devices bilaterally. d. Prevent dehydration. e. Periodically elevate the feet and lower legs above level of heart.

During CPR for ventricular fibrillation, which drug would an RN most like prepare first? a. Atropine sulfate b. Epinephrine c. Furosemide (Lasix) d. Lidocaine

b. Epinephrine - first line drug used during CPR to stimulate cardiac activity Atropine sulfate is used for bradycardia and first-degree AV blocks. Furosemide is a diuretic. Lidocaine is used for ventricular arrhythmias, specifically PVCs.

A client receives a pacemaker to treat a recurring arrhythmia. When monitoring the cardiac rhythm strip, the nurse observes extra pacemaker spikes that aren't followed by a beat. Which condition should the nurse suspect? a. Failure to pace b. Failure to capture c. Failure to sense d. Asystole

b. Failure to capture Extra pacemaker spikes that aren't followed by a beat may indicate failure to capture, in which the pacemaker fires but the heart doesn't conduct the beat. In failure to pace, the pacemaker doesn't fire when it should, causing hypotension and other signs of low cardiac output, accompanied by bradycardia or a heart rate slower than the pacemaker's preset rate. In failure to sense, the pacemaker can't sense the client's intrinsic heartbeat; on the rhythm strip, spikes may fall on T waves, or they may fall regularly but at points where they shouldn't appear. Asystole is characterized by an absent heart rate or rhythm as reflected by a flat line on the rhythm strip.

An example of an energy nutrient is: a. Water b. Fat c. Vitamins d. Minerals

b. Fat

Which of the following would be the least important assessment in a patient diagnosed with ascites? a. Measurement of abdominal girth b. Foul-smelling breath c. Weight d. Palpation of the abdomen for a fluid shift

b. Foul-smelling breath Foul-smelling breath would not be considered an important assessment for this patient. Measurement of abdominal girth, weight, and palpation of the abdomen for a fluid shift are all important assessment parameters for the patient diagnosed with ascites.

The nurse is caring for a client experiencing dyspnea, dependent edema, hepatomegaly, crackles, and jugular vein distention. What condition should the nurse suspect? a. Pulmonary embolism b. Heart failure c. Cardiac tamponade d. Tension pneumothorax

b. Heart failure A client with heart failure has decreased cardiac output caused by the heart's decreased pumping ability. A buildup of fluid occurs, causing dyspnea, dependent edema, hepatomegaly, crackles, and jugular vein distention. A client with pulmonary embolism experiences acute shortness of breath, pleuritic chest pain, hemoptysis, and fever. A client with cardiac tamponade experiences muffled heart sounds, hypotension, and elevated central venous pressure. A client with tension pneumothorax has a deviated trachea and absent breath sounds on the affected side as well as dyspnea and jugular vein distention.

When a client is started on oral or I.V. diltiazem (Cardizem), the nurse should monitor for which potential complication? a. Flushing b. Heart failure c. Renal failure d. Hypertension

b. Heart failure The chief adverse effects of diltiazem are hypotension, atrioventricular blocks, heart failure, and elevated liver enzyme levels. Other reactions that have been reported include flushing, nocturia, and polyuria, but not renal failure. Although flushing may occur, it's an adverse reaction, not a potential complication. Heart failure is a life-threatening reaction.

You are taking an initial history for a client seeking surgical treatment for obesity. Which of the following should be called to the attention of the surgeon before proceeding with additional history or physical assessment? a. Obesity for approximately 5 years b. History of counseling for body dysmorphic disorder c. Failure to reduce weight with other forms of therapy d. Body weight 100% above the ideal for age, gender and height

b. History of counseling for body dysmorphic disorder Body dysmorphic disorder is a preoccupation with an imagined physical defect. Corrective surgery can exacerbate this disorder when the client continues to feel dissatisfied with the results. The other criteria are indicators of candidacy for this treatment.

A nurse is caring for a client with unstable ventricular tachycardia. The nurse instructs the client to do which of the following, if prescribed, during an episode of ventricular tachycardia? a. Breathe deeply, regularly, and easily. b. Inhale deeply and cough forcefully every 1 to 3 seconds. c. Lie down flat in bed d. Remove any metal jewelry

b. Inhale deeply and cough forcefully every 1 to 3 seconds. Cough cardiopulmonary resuscitation (CPR) sometimes is used in the client with unstable ventricular tachycardia. The nurse tells the client to use cough CPR, if prescribed, by inhaling deeply and coughing forcefully every 1 to 3 seconds. Cough CPR may terminate the dysrhythmia or sustain the cerebral and coronary circulation for a short time until other measures can be implemented.

A client who suffered blunt chest trauma in a car accident complains of chest pain, which is exacerbated by deep inspiration. On auscultation, the nurse detects a pericardial friction rub — a classic sign of acute pericarditis. The physician confirms acute pericarditis and begins appropriate medical intervention. To relieve chest pain associated with pericarditis, the nurse should encourage the client to assume which position? a. Semi-Fowler's b. Leaning forward while sitting c. Supine d. Prone

b. Leaning forward while sitting When the client leans forward, the heart pulls away from the diaphragmatic pleurae of the lungs, helping relieve chest pain caused by pericarditis. The semi-Fowler, supine, and prone positions don't cause this pulling-away action and therefore don't relieve chest pain associated with pericarditis.

A client is in the early stage of heart failure. During this time, which compensatory mechanism occurs? a. Decreased renal blood flow causes the renin-angiotensin-aldosterone system to lower the secretion of aldosterone and antidiuretic hormone. b. Low blood pressure triggers the baroreceptors to increase sympathetic nervous system stimulation. c. Decreased renal blood flow causes the renin-angiotensin-aldosterone system to raise aldosterone secretion. d. Low blood pressure triggers the baroreceptors to decrease sympathetic nervous system stimulation.

b. Low blood pressure triggers the baroreceptors to increase sympathetic nervous system stimulation. In the early stage of heart failure, low blood pressure triggers baroreceptors in the carotid sinus and aortic arch to increase sympathetic nervous system stimulation, causing a faster heart rate, vasoconstriction, and increased myocardial oxygen consumption. Decreased renal blood flow causes the renin-angiotensin-aldosterone system to increase, not reduce, secretion of aldosterone and antidiuretic hormone, in turn causing sodium and water retention and arterial vasoconstriction.

A nurse is caring for a client with a nasogastric tube that is attached to low suction. The nurse monitors the client, knowing that the client is at risk for which acid-base disorder. a. Metabolic acidosis b. Metabolic alkalosis c. Respiratory acidosis d. Respiratory alkalosis

b. Metabolic alkalosis

While caring for a client who has sustained an MI, the nurse notes eight PVCs in one minute on the cardiac monitor. The client is receiving an IV infusion of D5W and oxygen at 2 L/minute. The nurse's first course of action should be to: a. Increase the IV infusion rate b. Notify the physician promptly c. Increase the oxygen concentration d. Administer a prescribed analgesic

b. Notify the physician promptly PVCs are often a precursor of life-threatening dysrhythmias, including ventricular tachycardia and ventricular fibrillation. An occasional PVC is not considered dangerous, but if PVCs occur at a rate greater than 5 or 6 per minute in the post MI client, the physician should be notified immediately. More than 6 PVCs per minute is considered serious and usually calls for decreasing ventricular irritability by administering medications such as lidocaine. Increasing the IV infusion rate would not decrease the number of PVCs. Increasing the oxygen concentration should not be the nurse's first course of action; rather, the nurse should notify the physician promptly. Administering a prescribed analgesic would not decrease ventricular irritability.

A client with myocardial infarction has been transferred from a coronary care unit to a general medical unit with cardiac monitoring via telemetry. A nurse plans to allow for which of the following client activities? a. Strict bed rest for 24 hours after transfer. b. Bathroom privileges and self-care activities. c. Unsupervised hallway ambulation with distances under 200 feet. d. Ad lib activities because the client is monitored.

b. On transfer from the CCU, the client is allowed self-care activities and bathroom privileges. Supervised ambulation for brief distances are encouraged, with distances gradually increased (50, 100, 200 feet).

As a nurse is assisting a client out of bed, which objective sign should indicate ti the RN that the client is experiencing hypotension? a. Pupil response is sluggish to light. b. Orientation level changes. c. Pulse rate increases 20 beats per minute. d. Increased muscle rigidity in legs.

b. Orientation level changes.

Which of the following is the major cause of morbidity and mortality in patients with chronic pancreatitis? a. Tetany b. Pancreatic necrosis c. MODS d. Shock

b. Pancreatic necrosis Pancreatic necrosis is a major cause of morbidity and mortality in patient with acute pancreatitis. Shock and multiple organ failure may occur with acute pancreatitis. Tetany is not a major cause or morbidity and mortality in patients with chronic pancreatitis.

Which condition is most likely to have a nursing diagnosis of fluid volume deficit? a. Appendicitis b. Pancreatitis c. Cholecystitis d. Gastric ulcer

b. Pancreatitis Hypovolemic shock from fluid shifts is a major factor in acute pancreatitis. The other conditions are less likely to exhibit fluid volume deficit.

Which of the following terms describes the passage of a hollow instrument into a cavity for the withdrawal of fluid? a. Dialysis b. Paracentesis c. Ascites d. Asterixis

b. Paracentesis Paracentesis may be used to withdraw ascitic fluid if the fluid accumulation is causing cardiorespiratory compromise. Asterixis refers to involuntary flapping movements of the the hands associated with metabolic liver dysfunction. Ascites refers to accumulation of serous fluid within the peritoneal cavity. Dialysis refers to a form of filtration to separate crystalloid from colloid substances.

A nurse is caring for a client who is on a mechanical ventilator. Blood gas results indicate a pH of 7.50 and a PCO2 of 30 mm Hg. The nurse has determined that the client is experiencing respiratory alkalosis. Which laboratory value would most likely be noted in this condition? a. Sodium level of 145 mEq/L b. Potassium level of 3 mEq/L c. Magnesium level of 2 mg/dL d. Phosphorus level of 4 mg/dL

b. Potassium level of 3 mEq/L

Which of the following ECG waveforms characterizes conduction of an electrical impulse through the left ventricle? a. P wave b. QRS complex c. PR interval d. QT interval

b. QRS complex

The nurse provides instructions to a client with a low potassium level about the foods that are high in potassium and tells that client to consume which foods? Select all the apply. a. Peas b. Raisins c. Potatoes d. Cauliflower e. Cantaloupe f. Strawberries

b. Raisins c. Potatoes e. Cantaloupe f. Strawberries

In preparing a client for a colonoscopy procedure, which task is most suitable to delegate to the nursing assistant? a. Explain the need for clear liquids 1 - 3 days prior to procedure. b. Reinforce NPO status 8 hours prior to procedure. c. Administer laxatives 1 - 3 days prior to procedure. d. Administer an enema the night before the procedure.

b. Reinforce NPO status 8 hours prior to procedure. The nursing assistant can reinforce dietary and fluid restrictions after the RN has explained the information to the client. It is also possible that the nursing assistant can administer the enema; however, special training is required and policies may vary between institutions. Medication administration should be performed by licensed personnel.

A nurse reviews the arterial blood gas results of a client and notes the following: pH 7.45, PCO2 of 30 mm Hg, and HCO3 of 22 mEq/L. The nurse analyzes these results as indicating which condition? a. Metabolic acidosis, compensated b. Respiratory alkalosis, compensated c. Metabolic alkalosis, uncompensated d. Respiratory acidosis, uncompensated

b. Respiratory alkalosis, compensated

The nurse is obtaining a history from a new client in the cardiovascular clinic. When investigating for childhood diseases and disorders associated with structural heart disease, the nurse should consider which finding significant? a. Croup b. Rheumatic fever c. Severe staphylococcal infection d. Medullary sponge kidney

b. Rheumatic fever Childhood diseases and disorders associated with structural heart disease include rheumatic fever and severe streptococcal (not staphylococcal) infections. Croup — a severe upper airway inflammation and obstruction that typically strikes children ages 3 months to 3 years — may cause latent complications, such as ear infection and pneumonia. However, it doesn't affect heart structures. Likewise, medullary sponge kidney, characterized by dilation of the renal pyramids and formation of cavities, clefts, and cysts in the renal medulla, eventually may lead to hypertension but doesn't damage heart structures.

A client with end-stage heart failure is preparing for discharge. The client and his caregiver meet with the home care nurse and voice their concern that setting up a hospital bed in the bedroom will leave him feeling isolated. Which suggestion by the home care nurse best addresses this concern? a. Place a chair in the bedroom so guests can visit with the client. b. Set up the hospital bed in the family room so the client can be part of household activities. c. Set up the hospital bed in the bedroom so the client can rest in a quiet environment. d. Set up the hospital bed in the bedroom so the client can be assessed in a quiet environment.

b. Set up the hospital bed in the family room so the client can be part of household activities. The client should be kept actively involved in the household to prevent feelings of isolation. This can be accomplished by setting up the hospital bed in the family room. Placing a chair in the bedroom allows the client periods of isolation when visitors aren't present. It's important for the client to have periods of rest; however, that can be accomplished without keeping the client isolated in a bedroom. The needs of the client should be considered before the needs of the nurse who assesses the client during an occasional visit.

Which of the following categories or oral antidiabetic agents exert their primary action by directly stimulating the pancreas to secrete insulin? a. Alpha glucosidase inhibitors b. Sulfonylureas c. Biguanides d. Thiazolidinediones

b. Sulfonylureas A functioning pancreas is necessary for sulfonylureas to be effective. Thiazolidinediones enhance insulin action at the receptor site without increasing insulin secretion from the beta cells of the pancreas. Biguanides facilitate insulin's action on peripheral receptor sites. Alpha glucosidase inhibitors delay the absorption of glucose in the intestinal system, resulting in a lower postprandial blood glucose level.

The nurse just received shift report for a group of clients on the telemetry unit. Which client should the nurse assess first. a. The client with a history of atrial fibrillation b. The client admitted with first-degree atrioventricular (AV) block whose cardiac monitor now reveals type II second-degree AV block c. The client with a history of heart failure who has bibasilar crackles and pitting edema in both feet d. The client with a demand pacemaker whose monitor shows normal sinus rhythm at a rate of 90 beats/minute

b. The client admitted with first-degree atrioventricular (AV) block whose cardiac monitor now reveals type II second-degree AV block The client whose cardiac rhythm now shows type II second-degree AV block should be assessed first. The client's rhythm has deteriorated from first-degree heart block to type II second-degree AV block and may continue to deteriorate into a lethal form of AV block (known as complete heart block). The client with a history of atrial fibrillation whose monitor reveals atrial fibrillation doesn't need to be assessed first. Because his rhythm is chronic, he has most likely been given an anticoagulant and isn't at immediate risk from this rhythm. The client with a history of heart failure may have chronic bibasilar crackles and pitting edema of both feet. Therefore, assessing this client first isn't necessary. The client's demand pacemaker fires only when the client's intrinsic heart rate falls below the pacemaker's set rate. In option 4, the pacemaker isn't firing because it most likely has been set at a slower rate than the client's intrinsic heart rate of 90 beats/minute.

A female client with dysphagia is being prepared for discharge. Which outcome indicates that the client is ready for discharge? a. The client doesn't exhibit rectal tenesmus. b. The client is free from esophagitis and achalasia. c. The client reports diminished duodenal inflammation. d. The client has normal gastric structures.

b. The client is free from esophagitis and achalasia. Dysphagia may be the reason why a client with esophagitis or achalasia seeks treatment. Dysphagia isn't associated with rectal tenesmus, duodenal inflammation, or abnormal gastric structures.

A nurse caring for a group of clients reviews the electrolyte laboratory results and notes a sodium level of 130 mEq/L on one client's laboratory report. The nurse understands that which client is at highest risk for the development of a sodium value at this level? a. The client with Cushing's syndrome b. The client who is taking diuretics c. The client with hyperaldoseroneism d. The client who is taking corticosteroids

b. The client who is taking diuretics

The nurse is assigned with an ancillary staff member to care for a group of cardiac clients. Which client should the nurse address first? a. The client admitted with unstable angina who wants to be discharged. b. The client who suffered an acute myocardial infarction (MI) who is complaining of constipation. c. The client who had a pacemaker inserted yesterday and who is complaining of incisional pain. d. The client who has his call light on.

b. The client who suffered an acute myocardial infarction (MI) who is complaining of constipation. The client who suffered an acute MI who is complaining of constipation should be addressed first. If the client strains at stool after an MI, the vagal response may be stimulated causing bradycardia thereby provoking arrhythmias. The nurse should delegate answering the call light to the ancillary personnel. She can also delegate some of the discharge preparation, such as packing the client's belongings. After addressing the MI client with constipation, the nurse should promptly address the pain relief needs of the client who had a pacemaker inserted the previous day.

A nurse is assigned to care for a group of clients. On review of the clients medical records, the nurse determines that which client is at risk for fluid volume excess? a. The client taking diuretcs b. The client with renal failure c. The client with an ileostomy d. The client who requires gastrointestinal suctioning

b. The client with renal failure

Before using a defibrillator to terminate ventricular fibrillation, the nurse should check the synchronizer switch. Why is this so important? a. The delivered shock must be synchronized with the client's QRS complex. b. The defibrillator won't deliver a shock if the synchronizer switch is turned on. c. The defibrillator won't deliver a shock if the synchronizer switch is turned off. d. The shock must be synchronized with the client's T wave.

b. The defibrillator won't deliver a shock if the synchronizer switch is turned on. The defibrillator won't deliver a shock to the client in ventricular fibrillation if the synchronizer switch is turned on because the defibrillator needs to recognize a QRS complex when the switch is turned on. The synchronizer switch should be turned on when attempting to terminate arrhythmias that contain QRS complexes, such as rapid atrial fibrillation that is resistant to pharmacologic measures.

When administered a thrombolytic drug to the client experiencing an MI, the nurse explains to him that the purpose of this drug is to: a. Help keep him well hydrated. b. Dissolve clots he may have. c. Prevent kidney failure. d. Treat potential cardiac arrhythmias.

b. Thrombolytic drugs are administered within the first 6 hours after onset of a MI to lyse clots and reduce the extent of myocardial damage.

A nurse notices frequent artifact on the ECG monitor for a client whose leads are connected by cable to a console at the bedside. The nurse examines the client to determine the cause. Which of the following items is unlikely to be responsible for the artifact? a. Frequent movement of the client b. Tightly secured cable connections c. Leads applied over hairy areas d. Leads applied to the limbs

b. Tightly secured cable connections Motion artifact, or "noise," can be caused by frequent client movement, electrode placement on limbs, and insufficient adhesion to the skin, such as placing electrodes over hairy areas of the skin. Electrode placement over bony prominences also should be avoided. Signal interference can also occur with electrode removal and cable disconnection.

The client's ABGs on room air are: pH 7.33, PaO2 77, HCO3 23. The nurse would instruct the client to: a. Try to breathe more slowly b. Used the bedside inspirometer hourly when awake. c. Where nasal cannula oxygen at 6 L/min. d. Increase fluid intake to flush the kidneys.

b. Used the bedside inspirometer hourly when awake.

A nurse notes that a client with sinus rhythm has a premature ventricular contraction that falls on the T wave of the preceding beat. The client's rhythm suddenly changes to one with no P waves or definable QRS complexes. Instead there are coarse wavy lines of varying amplitude. The nurse assesses this rhythm to be: a. Ventricular tachycardia b. Ventricular fibrillation c. Atrial fibrillation d. Asystole

b. Ventricular fibrillation Ventricular fibrillation is characterized by irregular, chaotic undulations of varying amplitudes. Ventricular fibrillation has no measurable rate and no visible P waves or QRS complexes and results from electrical chaos in the ventricles.

A nurse is watching the cardiac monitor and notices that the rhythm suddenly changes. There are no P waves, the QRS complexes are wide, and the ventricular rate is regular but over 100. The nurse determines that the client is experiencing: a. Premature ventricular contractions b. Ventricular tachycardia c. Ventricular fibrillation d. Sinus tachycardia

b. Ventricular tachycardia Ventricular tachycardia is characterized by the absence of P waves, wide QRS complexes (usually greater than 0.14 second), and a rate between 100 and 250 impulses per minute. The rhythm is usually regular.

The nurse is caring for a male client with a diagnosis of chronic gastritis. The nurse monitors the client knowing that this client is at risk for which vitamin deficiency? a. Vitamin A b. Vitamin B12 c. Vitamin C d. Vitamin E

b. Vitamin B12 Chronic gastritis causes deterioration and atrophy of the lining of the stomach, leading to the loss of the function of the parietal cells. The source of the intrinsic factor is lost, which results in the inability to absorb vitamin B12. This leads to the development of pernicious anemia. The client is not at risk for vitamin A, C, or E deficiency.

A client who has been receiving heparin therapy also is started on warfarin. The client asks a nurse why both medications are being administered. In formulating a response, the nurse incorporates the understanding that warfarin: a. Stimulates the breakdown of specific clotting factors by the liver, and it takes 2-3 days for this to exert an anticoagulant effect. b. Inhibits synthesis of specific clotting factors in the liver, and it takes 3-4 days for this medication to exert an anticoagulant effect. c. Stimulates production of the body's own thrombolytic substances, but it takes 2-4 days for this to begin. d. Has the same mechanism of action as Heparin, and the crossover time is needed for the serum level of warfarin to be therapeutic.

b. Warfarin works in the liver and inhibits synthesis of four vitamin K-dependent clotting factors (X, IX, VII, and II), but it takes 3 to 4 days before the therapeutic effect of warfarin is exhibited.

A client with left-sided heart failure complains of increasing shortness of breath and is agitated and coughing up pink-tinged, foamy sputum. The nurse should recognize these as signs and symptoms of: a. right-sided heart failure. b. acute pulmonary edema. c. pneumonia. d. cardiogenic shock.

b. acute pulmonary edema. Because of decreased contractility and increased fluid volume and pressure in clients with heart failure, fluid may be driven from the pulmonary capillary beds into the alveoli, causing pulmonary edema. In right-sided heart failure, the client would exhibit hepatomegaly, jugular vein distention, and peripheral edema. In pneumonia, the client would have a temperature spike and sputum that varies in color. Cardiogenic shock would show signs of hypotension and tachycardia.

A male client has just been diagnosed with hepatitis A. On assessment, the nurse expects to note: a. severe abdominal pain radiating to the shoulder. b. anorexia, nausea, and vomiting. c. eructation and constipation. d. abdominal ascites.

b. anorexia, nausea, and vomiting. Hallmark signs and symptoms of hepatitis A include anorexia, nausea, vomiting, fatigue, and weakness. Abdominal pain may occur but doesn't radiate to the shoulder. Eructation and constipation are common in gallbladder disease, not hepatitis A. Abdominal ascites is a sign of advanced hepatic disease, not an early sign of hepatitis A.

A client with substernal chest pain that radiates to the jaw is admitted to the coronary care unit. The client subsequently develops hypotension and suffers cardiac arrest. Which calcium preparation is injected into the ventricle during cardiac arrest? a. calcium carbonate (BioCal) b. calcium chloride c. calcium glubionate (Neo-Calglucon) d. calcium lactate

b. calcium chloride Calcium chloride is the only calcium preparation that should be injected into the ventricle during cardiac arrest, if appropriate.

A client with chest pain receives nitroglycerin on the way to the acute care facility. Based on an electrocardiogram obtained on admission, the physician suspects a myocardial infarction (MI) and prescribes I.V. morphine to relieve continuing pain. A primary goal of nursing care for this client is to recognize life-threatening complications of an MI. The major cause of death after an MI is: a. cardiogenic shock. b. cardiac arrhythmia. c. heart failure. d. pulmonary embolism.

b. cardiac arrhythmia. Cardiac arrhythmias cause roughly 40% to 50% of deaths after MI. Heart failure, in contrast, accounts for 33% and cardiogenic shock for 9% of post-MI deaths. Pulmonary embolism, another potential complication of an MI, is less common.

When evaluating a male client for complications of acute pancreatitis, the nurse would observe for: a. increased intracranial pressure. b. decreased urine output. c. bradycardia. d. hypertension.

b. decreased urine output. Acute pancreatitis can cause decreased urine output, which results from the renal failure that sometimes accompanies this condition. Intracranial pressure neither increases nor decreases in a client with pancreatitis. Tachycardia, not bradycardia, usually is associated with pulmonary or hypovolemic complications of pancreatitis. Hypotension can be caused by a hypovolemic complication, but hypertension usually isn't related to acute pancreatitis.

What is a major indicator of fluid volume deficit? a. full and bounding pulse b. drop in postural blood pressure c. elevated temperature d. pitting edema of lower extremities

b. drop in postural blood pressure

Women's bodies normally contain more _______ and less _______ than men's bodies. a. carbohydrate, protein b. fat, protein c. protein, ash d. water, fat

b. fat, protein

A client is recovering from surgical repair of a dissecting aortic aneurysm. The nurse should evaluate the client for signs of bleeding or recurring dissection. These signs include: a. hematuria and decreased urine output. b. hypotension and tachycardia. c. increased urine output and bradycardia. d. hypotension and bradycardia.

b. hypotension and tachycardia. When caring for a client recovering from surgical repair of a dissecting aortic aneurysm, the nurse must monitor for hypotension with reflex tachycardia, decreased urine output, and unequal or absent peripheral pulses — all potential signs of bleeding or recurring dissection. Hematuria, increased urine output, and bradycardia aren't signs of bleeding from aneurysm repair or recurring dissection.

A client is experiencing an acute myocardial infarction (MI) and I.V. morphine is prescribed. Morphine is given because it: a. eliminates pain, reduces cardiac workload, and increases myocardial contractility. b. lowers resistance, reduces cardiac workload, and decreases myocardial oxygen demand. c. raises the blood pressure, lowers myocardial oxygen demand, and eliminates pain. d. increases venous return, lowers resistance, and reduces cardiac workload.

b. lowers resistance, reduces cardiac workload, and decreases myocardial oxygen demand. When given to treat acute MI, morphine sulfate eliminates pain, reduces venous return to the heart, reduces vascular resistance, reduces myocardial workload, and reduces the oxygen demand of the heart. Morphine sulfate doesn't increase myocardial contractility, raise blood pressure, or increase venous return.

BMR

basal metabolic rate - maintenance of muscle tone and body temperature - glandular and cellular activity - circulation - respiration - men > women

A male client with pancreatitis complains of pain. The nurse expects the physician to prescribe meperidine (Demerol) instead of morphine to relieve pain because: a. meperidine provides a better, more prolonged analgesic effect. b. morphine may cause spasms of Oddi's sphincter. c. meperidine is less addictive than morphine. d. morphine may cause hepatic dysfunction.

b. morphine may cause spasms of Oddi's sphincter. For a client with pancreatitis, the physician will probably avoid prescribing morphine because this drug may trigger spasms of the sphincter of Oddi (a sphincter at the end of the pancreatic duct), causing irritation of the pancreas. Meperidine has a somewhat shorter duration of action than morphine. The two drugs are equally addictive. Morphine isn't associated with hepatic dysfunction.

When evaluating arterial blood gases (ABGs), which value is consistent with metabolic alkalosis? a. PaCO2 36 b. pH 7.48 c. HCO3 21 mEq/L d. O2 sat 95%

b. pH 7.48

A nurse us preparing to care for a client with a potassium deficit. The nurse reviews the client's record and determines that the client was at risk for developing potassium deficit because the client: a. sustained tissue damage b. requires nasogastric suction c. has a history or Addison's disease d. is taking a potassium sparing diuretic

b. requires nasogastric suction

Vasoactive drugs, which cause the arteries and veins to dilate thereby shunting much of the intravascular volume to the periphery and causing a reduction in preload and afterload, include agents such as a. furosemide (Lasix) b. sodium nitoprusside (Nipride) c. dopamine (Inotropin) d. norepinephrine (Levophed)

b. sodium nitoprusside (Nipride)

Considering a client's atrial fibrillation, the nurse must administer digoxin (Lanoxin) with caution because it: a. affects the sympathetic division of the autonomic nervous system, decreasing vagal tone. b. stimulates the parasympathetic division of the autonomic nervous system, increasing vagal tone. c. can induce hypertensive crisis by constricting arteries. d. can trigger proarrhythmia by increasing stroke volume.

b. stimulates the parasympathetic division of the autonomic nervous system, increasing vagal tone. The nurse must administer digoxin with caution in a client with atrial fibrillation because digoxin stimulates the parasympathetic division of the autonomic nervous system, increasing vagal tone. The vagal effect slows the heart rate, increases the refractory period, and slows conduction through the atrioventricular node and junctional tissue, thus increasing the potential for new arrhythmias to develop. Digoxin doesn't constrict arteries. Although it can trigger proarrhythmias, it does so by increasing vagal tone (not stroke volume).

A client with a history of myocardial infarction is admitted with shortness of breath, anxiety, and slight confusion. Assessment findings include a regular heart rate of 120 beats/minute, audible third and fourth heart sounds, blood pressure of 84/64 mm Hg, bibasilar crackles on lung auscultation, and a urine output of 5 ml over the past hour. The nurse anticipates preparing the client for transfer to the intensive care unit and pulmonary artery catheter insertion because: a. the client is experiencing heart failure. b. the client is going into cardiogenic shock. c. the client shows signs of aneurysm rupture. d. the client is in the early stage of right-sided heart failure.

b. the client is going into cardiogenic shock. This client's findings indicate cardiogenic shock, which occurs when the heart fails to pump properly, impeding blood supply and oxygen flow to vital organs. Cardiogenic shock also may cause cold, clammy skin and generalized weakness, fatigue, and muscle pain as lactic acid accumulates from poor blood flow, preventing waste removal. Left-sided and right-sided heart failure eventually cause venous congestion with jugular vein distention and edema as the heart fails to pump blood forward. A ruptured aneurysm causes severe hypotension and a quickly deteriorating clinical status from blood loss and circulatory collapse; this client has low but not severely decreased blood pressure. Also, in ruptured aneurysm, deterioration is more rapid and full cardiac arrest is common.

PreOp for arterial bypass procedures

bacteriostatic shower, antibiotics, foley, CVL, vital signs, baseline pulses, consent

Surgical treatments for valve disease

balloon valvuloplasty, aortic valve replacement, mitral valve replacement

Medications for sinus tachycardia

beta blockers - block beta receptors in heart and decrease heart rate - decrease force of contraction and decrease AV conduction

Complications of duodenal ulcers

bleeding, perforation, gastric outlet obstruction, and intractable disease

Diagnostic tests for diarrhea

blood (CBC, LFTs, iron/folate) stool culture (WBCs, blood, ova & parasites) endoscopy GI series w/ barium swallow

Causes of peritonitis

blood born infection cirrhosis of the liver perforation/rupture of bowel/appendix pancreatitis peritoneal dialysis abdominal surgery

ABG

blood taken from artery - measures O2, CO2, pH, and Bicarb - used as a guide for O2 delivery system

Troponin level

blood test used to diagnose MI - level rises within 3 hours and remains elevated 7 - 10 days

Creatine kinase level

blood test used to diagnose MI - level rises within 6 hours after onset of chest pain and peaks within 18 hours

CK-MB

blood test used to diagnose MI - peak elevation occurs 18 hours after onset of chest pain - level returns to normal 48 - 72 hours later

Calcium plays a role in

bone formation, coagulation, excitation of cardiac and skeletal muscle, conduction of neuromuscular impulses, regulation of endocrine/exocrine glands

Cardiac assessment findings for FVE

bounding pulse, elevated BP, distended neck and hand veins, elevated CVP, dysrhytmias

Catabolism

breakdown of chemical substances into simpler substances

A client with pneumonia develops respiratory failure and has a partial pressure of arterial oxygen of 55 mm Hg. He's placed on mechanical ventilation with a fraction of inspired oxygen (FIO2) of 0.9. The nursing goal should be to reduce the FIO2 to no greater than: a) 0.21. b) 0.35. c) 0.5 d) 0.7

c) 0.5

A client in the emergency department is diagnosed with a communicable disease. When complications of the disease are discovered, the client is admitted to the hospital and placed in respiratory isolation. Which infection warrants respiratory isolation? a) Chickenpox b) Impetigo c) Measles d) Cholera

c) Measles

Signs and symptoms of angina

chest pain/pressure, pain may radiate, pain relieved by nitroglycerin and rest, dyspnea, pallor/diaphoresis, indigestion, "impending doom"

A 35-year-old female pt comes into the clinic postoperative parathyroidectomy. Which of the following should the nurse instruct this pt? a) Drink one glass of red wine per day. b) Avoid the sun. c) Milk & milk-based products will ensure an adequate calcium intake. d) Red meat is the protein source of choice.

c) Milk & milk-based products will ensure an adequate calcium intake.

A client has undergone a left hemicolectomy for bowel cancer. Which activities prevent the occurrence of postoperative pneumonia in this client? a) Administering oxygen, coughing, breathing deeply, and maintaining bed rest b) Coughing, breathing deeply, maintaining bed rest, and using an incentive spirometer c) Coughing, breathing deeply, frequent repositioning, and using an incentive spirometer d) Administering pain medications, frequent repositioning, and limiting fluid intake

c) Coughing, breathing deeply, frequent repositioning, and using an incentive spirometer

A recent immigrant from Vietnam is diagnosed with pulmonary tuberculosis (TB). Which intervention is most important for the nurse to implement with this client? a) Client teaching about the cause of TB b) Reviewing the risk factors for TB c) Developing a list of people with whom the client has had contact d) Client teaching about the importance of TB testing

c) Developing a list of people with whom the client has had contact

The nurse is caring for a pt who is anxious & dizzy following a traumatic experience. The arterial blood gas findings include: pH 7.48, PaO2 110, PaCO2 25, & HCO3 24. The nurse would anticipate which initial intervention to correct this problem? a) Immediately administer oxygen via a mask & monitor oxygen saturation. b) Prepare to start an intravenous fluid bolus using isotonic fluids. c) Encourage the pt to breathe in & out slowly into a paper bag. d) Anticipate the administration of intravenous sodium bicarbonate.

c) Encourage the pt to breathe in & out slowly into a paper bag.

The physician orders a palliative care consult for a client with end-stage chronic obstructive pulmonary disease who wishes no further medical intervention. Which step should the nurse anticipate based on her knowledge of palliative care? a) Decreasing administration of pain medications b) Reducing oxygen requirements c) Increasing the need for antianxiety agents d) Decreasing the use of bronchodilators

c) Increasing the need for antianxiety agents

A client who sustained a pulmonary contusion in a motor vehicle accident develops a pulmonary embolism. Which nursing diagnosis takes priority with this client? a) Excess fluid volume related to excess sodium intake b) Acute pain related to tissue trauma c) Ineffective breathing pattern related to tissue trauma d) Activity intolerance related to insufficient energy to carry out activities of daily living

c) Ineffective breathing pattern related to tissue trauma

A client is prescribed rifampin (Rifadin), 600 mg P.O. daily. Which statement about rifampin is true? a) It's usually given alone. b) Its exact mechanism of action is unknown. c) It's tuberculocidal, destroying the offending bacteria. d) It acts primarily against resting bacteria.

c) It's tuberculocidal, destroying the offending bacteria.

Angina

chest pain resulting from myocardial ischemia caused by inadequate myocardial blood/O2 supply - demand exceeds supply - conversion from aerobic to anaerobic respiration causes lactic acid build up which causes pain

A client is receiving supplemental oxygen. When determining the effectiveness of oxygen therapy, which arterial blood gas value is most important? a) pH b) Bicarbonate (HCO3-) c) Partial pressure of arterial oxygen (PaO2) d) Partial pressure of arterial carbon dioxide (PaCO2)

c) Partial pressure of arterial oxygen (PaO2)

The nurse is developing a teaching plan for a client with asthma. Which teaching point has the highest priority? a) Avoid contact with fur-bearing animals. b) Change filters on heating and air conditioning units frequently. c) Take prescribed medications as scheduled. d) Avoid goose down pillows.

c) Take prescribed medications as scheduled.

A client with a history of type 1 diabetes is admitted to the hospital with community-acquired pneumonia. The client's blood glucose level in the emergency care unit was 576 mg/dl. The physician prescribes an I.V. containing normal saline solution, an insulin infusion, and I.V. levofloxacin (Levaquin). The nurse piggybacks the insulin infusion into the normal saline solution. She questions whether she can also piggyback the levofloxacin into the same I.V. line. Which health team member should she collaborate with to check the compatibility of these solutions? a) The physician who prescribed the medications b) The coworker with 20 years nursing experience c) The pharmacist covering the floor d) The infectious disease nurse

c) The pharmacist covering the floor

The nurse observes constant bubbling in the water-seal chamber of a closed chest drainage system. What should the nurse conclude? a) The system is functioning normally. b) The client has a pneumothorax. c) The system has an air leak. d) The chest tube is obstructed.

c) The system has an air leak.

A client in acute respiratory distress is brought to the emergency department. After endotracheal (ET) intubation and initiation of mechanical ventilation, the client is transferred to the intensive care unit. Before suctioning the ET tube, the nurse hyperventilates and hyperoxygenates the client. What is the rationale for these interventions? a) They help prevent subcutaneous emphysema. b) They help prevent pneumothorax. c) They help prevent cardiac arrhythmias. d) They help prevent pulmonary edema.

c) They help prevent cardiac arrhythmias.

A client with a suspected pulmonary disorder undergoes pulmonary function tests. To interpret test results accurately, the nurse must be familiar with the terminology used to describe pulmonary functions. Which term refers to the volume of air inhaled or exhaled during each respiratory cycle? a) Vital capacity b) Functional residual capacity c) Tidal volume d) Maximal voluntary ventilation

c) Tidal volume

The nurse is teaching a client with chronic bronchitis about breathing exercises. Which instruction should the nurse include in the teaching? a) Make inhalation longer than exhalation. b) Exhale through an open mouth. c) Use diaphragmatic breathing. d) Use chest breathing.

c) Use diaphragmatic breathing.

or a client with advanced chronic obstructive pulmonary disease (COPD), which nursing action best promotes adequate gas exchange? a) Encouraging the client to drink three glasses of fluid daily b) Keeping the client in semi-Fowler's position c) Using a high-flow Venturi mask to deliver oxygen as prescribed d) Administering a sedative as prescribed

c) Using a high-flow Venturi mask to deliver oxygen as prescribed

A client admitted with multiple traumatic injuries receives massive fluid resuscitation. Later, the physician suspects that the client has aspirated stomach contents. The nurse knows that this client is at highest risk for: a) chronic obstructive pulmonary disease (COPD). b) bronchial asthma. c) adult respiratory distress syndrome (ARDS). d) renal failure.

c) adult respiratory distress syndrome (ARDS).

A client with chronic obstructive pulmonary disease (COPD) is recovering from a myocardial infarction. Because the client is extremely weak and can't produce an effective cough, the nurse should monitor closely for: a) pleural effusion. b) pulmonary edema. c) atelectasis. d) oxygen toxicity.

c) atelectasis.

A pt is admitted with hypernatremia caused by being stranded on a boat in the Atlantic Ocean for five days without a fresh water source. Which of the following is this pt at risk for developing? a) pulmonary edema b) atrial dysrhythmias c) cerebral bleeding d) stress fractures

c) cerebral bleeding

A postoperative pt is diagnosed with fluid volume overload. Which of the following should the nurse assess in this pt? a) poor skin turgor b) decreased urine output c) distended neck veins d) concentrated hemoglobin & hematocrit levels

c) distended neck veins

After undergoing a left thoracotomy, a client has a chest tube in place. When caring for this client, the nurse must: a) report fluctuations in the water-seal chamber. b) clamp the chest tube once every shift. c) encourage coughing and deep breathing. d) milk the chest tube every 2 hours.

c) encourage coughing and deep breathing.

A client with chronic sinusitis comes to the outpatient department complaining of headache, malaise, and a nonproductive cough. When examining the client's paranasal sinuses, the nurse detects tenderness. To evaluate this finding further, the nurse should transilluminate the: a) frontal sinuses only. b) sphenoidal sinuses only. c) frontal and maxillary sinuses. d) sphenoidal and ethmoidal sinuses.

c) frontal and maxillary sinuses.

Inspiratory and expiratory stridor may be heard in a client who: a) is experiencing an exacerbation of goiter. b) is experiencing an acute asthmatic attack. c) has aspirated a piece of meat. d) has severe laryngotracheitis.

c) has aspirated a piece of meat.

A client's chest X-ray reveals bilateral white-outs, indicating adult respiratory distress syndrome (ARDS). This syndrome results from: a) cardiogenic pulmonary edema. b) respiratory alkalosis. c) increased pulmonary capillary permeability. d) renal failure.

c) increased pulmonary capillary permeability.

A client with advanced acquired immunodeficiency syndrome (AIDS) is diagnosed with active tuberculosis. Which of the following regimens would the nurse expect the physician to prescribe? a) isoniazid (Laniazid) and rifampin (Rifadin) b) ethambutol (Myambutol), pyrazinamide, and isoniazid c) isoniazid, rifampin, ethambutol, and pyrazinamide d) ethambutol, ciprofloxacin (Cipro), pyrazinamide, and streptomycin

c) isoniazid, rifampin, ethambutol, and pyrazinamide

A 21-year-old client with cystic fibrosis develops pneumonia. To decrease the viscosity of respiratory secretions, the physician prescribes acetylcysteine (Mucomyst). Before administering the first dose, the nurse checks the client's history for asthma. Acetylcysteine must be used cautiously in a client with asthma because: a) it's a respiratory depressant. b) it's a respiratory stimulant. c) it may induce bronchospasm. d) it inhibits the cough reflex.

c) it may induce bronchospasm.

The blood gases of a pt with an acid-base disorder show a blood pH outside of normal limits. The nurse realizes that this pt is a) fully compensated. b) demonstrating anaerobic metabolism. c) partially compensated. d) in need of intravenous fluids

c) partially compensated.

A client with severe acute respiratory syndrome (SARS) privately informs the nurse that he doesn't want to be placed on a ventilator if his condition worsens. The client's wife and children have repeatedly expressed their desire that everything be done for the client. The most appropriate action by the nurse would be to: a) inform the family of the client's wishes. b) assure the family that everything possible will be done. c) support the client's decision. d) assure the client that everything possible will be done.

c) support the client's decision.

The nurse is caring for a client who has a tracheostomy tube and is undergoing mechanical ventilation. The nurse can help prevent tracheal dilation, a complication of tracheostomy tube placement, by: a) suctioning the tracheostomy tube frequently. b) using a cuffed tracheostomy tube. c) using the minimal air leak technique with cuff pressure less than 25 cm H2O. d) keeping the tracheostomy tube plugged.

c) using the minimal air leak technique with cuff pressure less than 25 cm H2O.

Action of Alpha Beta Blockers

can be administer IV in hypertensive emergency

reducible hernia

can be manually replaced into cavity

A client with angina pectoris must learn how to reduce risk factors that exacerbate this condition. When developing the client's care plan, the nurse should include which expected outcome? a. "Client will verbalize an understanding of the need to call the physician if acute pain lasts more than 2 hours." b. "Client will verbalize the intention to avoid exercise." c. "Client will verbalize the intention to stop smoking." d. "Client will verbalize an understanding of the need to restrict dietary fat, fiber, and cholesterol."

c. "Client will verbalize the intention to stop smoking." The client with angina pectoris should stop smoking at once because smoking increases the blood carboxyhemoglobin level; this, in turn, reduces the heart's oxygen supply and may induce angina. The client must seek immediate medical attention if chest pain doesn't subside after three nitroglycerin doses taken 10 to 15 minutes apart; serious myocardial damage or even sudden death may occur if chest pain persists for 2 hours. To improve coronary circulation and promote weight management, the client should get regular daily exercise. The client should eat plenty of fiber, which may decrease serum cholesterol and triglyceride levels and minimize hypertension, in turn reducing the risk for atherosclerosis (which plays a role in angina).

Which of the following solutions is hypotonic? a. 5% NaCl b. 0.9% NaCl c. 0.45% NaCl d. Lactated Ringer's Solution

c. 0.45% NaCl

What is the average daily urinary output in an adult? a. 0.5L b. 1.0L c. 1.5L d. 2.5L

c. 1.5L

Nurse Berlinda is assigned to a 41-year-old client who has a diagnosis of chronic pancreatitis. The nurse reviews the laboratory result, anticipating a laboratory report that indicates a serum amylase level of: a. 45 units/L b. 100 units/L c. 300 units/L d. 500 units/L

c. 300 units/L The normal serum amylase level is 25 to 151 units/L. With chronic cases of pancreatitis, the rise in serum amylase levels usually does not exceed three times the normal value. In acute pancreatitis, the value may exceed five times the normal value. Options A and B are within normal limits. Option D is an extremely elevated level seen in acute pancreatitis.

When auscultating the apical pulse of a client who has atrial fibrillation, the nurse would expect to hear a rhythm that is characterized by: a. The presence of occasional coupled beats b. Long pauses in an otherwise regular rhythm c. A continuous and totally unpredictable irregularity d. Slow but strong and regular beats

c. A continuous and totally unpredictable irregularity In atrial fibrillation, multiple ectopic foci stimulate the atria to contract. The AV node is unable to transmit all of these impulses to the ventricles, resulting in a pattern of highly irregular ventricular contractions.

A nurse is viewing the cardiac monitor in a client's room and notes that the client has just gone into ventricular tachycardia. The client is awake and alert and has good skin color. The nurse would prepare to do which of the following? a. Immediately defibrillate b. Prepare for pacemaker insertion c. Administer amiodarone (Cordarone) intravenously d. Administer epinephrine (Adrenaline) intravenously

c. Administer amiodarone (Cordarone) intravenously First-line treatment of ventricular tachycardia in a client who is hemodynamically stable is the use of anti-dysrhythmics such as amiodarone (Cordarone), lidocaine (Xylocaine), and procainamide (Pronestyl). Cardioversion also may be needed to correct the rhythm (cardioversion is recommended for stable ventricular tachycardia). Defibrillation is used with pulseless ventricular tachycardia. Epinephrine would stimulate and already excitable ventricle and is contraindicated.

Which of the following is the most common cause of symptomatic hypomagnesemia? a. IV drug use b. Burns c. Alcoholism d. Sedentary lifestyle

c. Alcoholism

In planning the post-operative care for a morbidly obese client, how can the expertise of the LPN/LVN best be applied? a. Obtain an oversized blood pressure cuff and a large-size bed. b. Set up a reinforced trapeze bar. c. Assist in the planning of bathing, turning, and ambulation. d. Design alternatives for routine tasks such as daily weights.

c. Assist in the planning of bathing, turning, and ambulation. The LPN/LVN can assist in the planning of interventions, but the RN should take ultimate responsibility for planning or designing. Obtaining equipment should be delegated to the nursing assistant. Contact physical therapy to set up specialized equipment.

An 84-year-old male is returning from the operating room (OR) after inguinal hernia repair. The nurse notes that he has fluid volume excess from the operation and is at risk for left-sided heart failure. Which sign or symptom indicates left-sided heart failure? a. Jugular vein distention b. Right upper quadrant pain c. Bibasilar fine crackles d. Dependent edema

c. Bibasilar fine crackles Bibasilar fine crackles are a sign of alveolar fluid, a sequelae of left ventricular fluid, or pressure overload. Jugular vein distention, right upper quadrant pain (hepatomegaly), and dependent edema are caused by right-sided heart failure, usually a chronic condition.

A nurse should hold administration of a nitrate if: a. Pulse rate is under 60. b. Client is allergic to sulfa. c. Blood pressure is less than 90/50 mm Hg. d. Drug therapy has exceeded 2 weeks.

c. Blood pressure is less than 90/50 mm Hg.

During a cardiac catherization blood samples from the right atrium, right ventricle, and pulmonary artery are analyzed for their oxygen content. Normally: a. All contain less CO2 than does pulmonary vein blood b. All contain more oxygen than does pulmonary vein blood c. The samples of blood all contain about the same amount of oxygen d. Pulmonary artery blood contains more oxygen than the other samples

c. Blood samples from the right atrium, right ventricle, and pulmonary artery would all be about the same with regard to oxygen concentration. Such blood contains slightly less oxygen than does systemic arterial blood.

Which of the following are the insensible mechanisms of fluid loss? a. Bowel elimination b. Urination c. Breathing d. Nausea

c. Breathing

For a patient who has undergone peripheral arteriography, how should the nurse assess the adequacy of peripheral circulation? a. By checking for cardiac dysrhythmias b. By observing the client for bleeding c. By checking peripheral pulses d. By hemodynamic monitoring

c. By checking peripheral pulses

A client with no history of cardiovascular disease comes into the ambulatory clinic with flu-like symptoms. The client suddenly complains of chest pain. Which of the following questions would best help a nurse to discriminate pain caused by a non-cardiac problem? a. "Have you ever had this pain before?" b. "Can you describe the pain to me?" c. "Does the pain get worse when you breathe in?" d. "Can you rate the pain on a scale of 1-10, with 10 being the worst?"

c. Chest pain is assessed by using the standard pain assessment parameters. Options 1, 2, and 4 may or may not help discriminate the origin of pain. Pain of pleuropulmonary origin usually worsens on inspiration.

Mandy, an adolescent girl is admitted to an acute care facility with severe malnutrition. After a thorough examination, the physician diagnoses anorexia nervosa. When developing the plan of care for this client, the nurse is most likely to include which nursing diagnosis? a. Hopelessness b. Powerlessness c. Chronic low self esteem d. Deficient knowledge

c. Chronic low self esteem Young women with Chronic low self esteem — are at highest risk for anorexia nervosa because they perceive being thin as a way to improve their self-confidence. Hopelessness and Powerlessness are inappropriate nursing diagnoses because clients with anorexia nervosa seldom feel hopeless or powerless; instead, they use food to control their desire to be thin and hope that restricting food intake will achieve this goal. Anorexia nervosa doesn't result from a knowledge deficit, such as one regarding good nutrition.

Nurse Joy is preparing to administer medication through a nasogastric tube that is connected to suction. To administer the medication, the nurse would: a. Position the client supine to assist in medication absorption b. Aspirate the nasogastric tube after medication administration to maintain patency c. Clamp the nasogastric tube for 30 minutes following administration of the medication d. Change the suction setting to low intermittent suction for 30 minutes after medication administration

c. Clamp the nasogastric tube for 30 minutes following administration of the medication If a client has a nasogastric tube connected to suction, the nurse should wait up to 30 minutes before reconnecting the tube to the suction apparatus to allow adequate time for medication absorption. Aspirating the nasogastric tube will remove the medication just administered. Low intermittent suction also will remove the medication just administered. The client should not be placed in the supine position because of the risk for aspiration.

Which of the following is clinical manifestation of cholelithiasis? a. Upper left quadrant abdominal pain b. Epigastric distress prior to a meal c. Clay-colored stools d. Nonpalpable abdominal mass

c. Clay-colored stools The patient with gallstones has clay-colored stools, and excruciating upper right quadrant pain that radiates to the back or right shoulder. The patient develops a fever and may have a palpable abdominal mass.

Which of the following would be inconsistent as a cause of DKA? a. Decreased or missed dose of insulin b. Illness of infection c. Competency in injecting insulin d. Undiagnosed and untreated diabetes

c. Competency in injecting insulin

A client is admitted to the emergency department after complaining of acute chest pain radiating down his left arm. Which laboratory studies would be indicated? a. Hemoglobin and hematocrit b. Serum glucose c. Creatinine phosphokinase (CPK) d. Troponin T and troponin I e. Myoglobin f. Blood urea nitrogen (BUN)

c. Creatinine phosphokinase (CPK) d. Troponin T and troponin I e. Myoglobin Levels of CPK, troponin T, and troponin I elevate because of cellular damage. Myoglobin elevation is an early indicator of myocardial damage. Hemoglobin, hematocrit, serum glucose, and BUN levels don't provide information related to myocardial ischemia.

A male client who is recovering from surgery has been advanced from a clear liquid diet to a full liquid diet. The client is looking forward to the diet change because he has been "bored" with the clear liquid diet. The nurse would offer which full liquid item to the client? a. Tea b. Gelatin c. Custard d. Popsicle

c. Custard Full liquid food items include items such as plain ice cream, sherbet, breakfast drinks, milk, pudding and custard, soups that are strained, and strained vegetable juices. A clear liquid diet consists of foods that are relatively transparent. The food items in options A, B, and D are clear liquids.

You are caring for a client with a nasogastric (NG) tube. Which task can be delegated to the experienced nursing assistant? a. Remove the NG tube per physician order. b. Secure the tape if the client accidentally dislodges the tube. c. Disconnect the suction to allow ambulation to the toilet. d. Reconnect the suction after the client has ambulated.

c. Disconnect the suction to allow ambulation to the toilet. Disconnecting the tube from suction is an appropriate task to delegate. Suction should be reconnected by the nurse, so that correct pressure is checked. If the nursing assistant is permitted to reconnect the tube, the RN is still responsible for checking that the pressure setting is correct. During removal of the tube, there is a potential for aspiration, so the nurse should perform this task. If the tube is dislodged, the nurse should recheck placement before it is secured.

The nurse is awaiting the arrival of a client from the emergency department. The client has a left ventricular myocardial infarction and is being admitted. In caring for this client, the nurse should be alert for which signs and symptoms of left-sided heart failure? a. Jugular vein distention b. Hepatomegaly c. Dyspnea d. Crackles e. Tachycardia f. Right upper quadrant pain

c. Dyspnea d. Crackles e. Tachycardia Signs and symptoms of left-sided heart failure include dyspnea, orthopnea, and paroxysmal nocturnal dyspnea; fatigue; nonproductive cough and crackles; hemoptysis; point of maximal impulse displaced toward the left anterior axillary line; tachycardia and S3 and S4 heart sounds; and cool, pale skin. Jugular vein distention, hepatomegaly, and right upper quadrant pain are all signs of right-sided heart failure.

Which of the following factors can cause hepatitis A? a. Contact with infected blood b. Blood transfusions with infected blood c. Eating contaminated shellfish d. Sexual contact with an infected person

c. Eating contaminated shellfish Hepatitis A can be caused by consuming contaminated water, milk, or food — especially shellfish from contaminated water. Hepatitis B is caused by blood and sexual contact with an infected person. Hepatitis C is usually caused by contact with infected blood, including receiving blood transfusions.

A client comes to the emergency department complaining of chest pain. An electrocardiogram (ECG) reveals myocardial ischemia and an anterior-wall myocardial infarction (MI). Which ECG characteristic indicates myocardial ischemia? a. Prolonged PR interval b. Absent Q wave c. Elevated ST segment d. Widened QRS complex

c. Elevated ST segment Ischemic myocardial tissue changes cause elevation of the ST segment, a peaked or inverted T wave, and a pathological Q wave. A prolonged PR interval occurs with first-degree heart block, the least dangerous atrioventricular heart block; this disorder may arise in healthy people but sometimes results from drug toxicity, electrolyte or metabolic disturbances, rheumatic fever, or chronic degenerative disease of the conduction system. An absent Q wave is normal; an MI may cause a significant Q wave. A widened QRS complex indicates a conduction delay in the His-Purkinje system.

A client enters the ER complaining of severe chest pain. A myocardial infarction is suspected. A 12 lead ECG appears normal, but the doctor admits the client for further testing until cardiac enzyme studies are returned. All of the following will be included in the nursing care plan. Which activity has the highest priority? a. Monitoring vital signs b. Completing a physical assessment c. Maintaining cardiac monitoring d. Maintaining at least one IV access site

c. Even though initial tests seem to be within normal range, it takes at least 3 hours for the cardiac enzyme studies to register. In the meantime, the client needs to be watched for bradycardia, heart block, ventricular irritability, and other arrhythmias. Other activities can be accomplished around the MI monitoring.

One hour after administering IV furosemide (Lasix) to a client with heart failure, a short burst of ventricular tachycardia appears on the cardiac monitor. Which of the following electrolyte imbalances should the nurse suspect? a. Hypocalcemia b. Hypermagnesemia c. Hypokalemia d. Hypernatremia

c. Furosemide is a potassium-depleting diuretic than can cause hypokalemia. In turn, hypokalemia increases myocardial excitability, leading to ventricular tachycardia.

A client has just returned from having a femoral arteriogram. The first assessment by a nurse should be: a. Vital signs. b. Bowel sounds. c. Groin area. d. Respiratory status.

c. Groin area.

A nurse is caring for a client with acute congestive heart failure who is receiving high doses of a diuretic. On assessment, the nurse notes that the client has flat neck veins, generalized muscle weakness, and diminished deep tendon reflexes. The nurse suspects hyponatremia. What additional signs would the nurse expect to note in a client with hyponatremia? a. Extreme thirst b. Decreased urinary output c. Hyperactive bowel sounds d. Increased specific gravity of the urine

c. Hyperactive bowel sounds

A hospitalized client experiences digoxin- (Lanoxin-) induced premature ventricular contractions (PVCs). Which type of effect do such contractions represent? a. Toxic b. Secondary c. Iatrogenic d. Idiosyncratic

c. Iatrogenic Digoxin-induced PVCs are iatrogenic because the drug is mimicking a cardiac disorder. Because the client is experiencing an apparent pathological disorder, this effect isn't considered toxic, secondary, or idiosyncratic.

A 68-year-old woman is scheduled to undergo mitral valve replacement for severe mitral stenosis and mitral regurgitation. Although the diagnosis was made during childhood, she did not have any symptoms until 4 years ago. Recently, she noticed increased symptoms, despite daily doses of digoxin and furosemide. During the initial interview with the nice lady, the nurse would most likely learn that the client's childhood health history included: a. Chicken pox b. poliomyelitis c. Rheumatic fever d. meningitis

c. Most clients with mitral stenosis have a history of rheumatic fever or bacterial endocarditis.

17. The nurse is reviewing the medication record of a female client with acute gastritis. Which medication, if noted on the client's record, would the nurse question? a. Digoxin (Lanoxin) b. Furosemide (Lasix) c. Indomethacin (Indocin) d. Propranolol hydrochloride (Inderal)

c. Indomethacin (Indocin) Indomethacin (Indocin) is a nonsteroidal anti-inflammatory drug and can cause ulceration of the esophagus, stomach, or small intestine. Indomethacin is contraindicated in a client with gastrointestinal disorders. Furosemide (Lasix) is a loop diuretic. Digoxin is a cardiac medication. Propranolol (Inderal) is a β-adrenergic blocker. Furosemide, digoxin, and propranolol are not contraindicated in clients with gastric disorders.

Which of the following nursing interventions is required to prepare a patient with cardiac dysrhythmia for an elective electrical cardioversion? a. Monitor blood pressure every 4 hours b. Administer digitalis and diuretics 24 hours before cardioversion c. Instruct the patient to restrict food and oral intake d. Facilitate CPR until the patient is prepared for cardioversion

c. Instruct the patient to restrict food and oral intake

The 30 year old client with asthma is brought into the emergency department by a friend because of difficulty breathing throughout the day and is now confused. The client's ABGs are: pH 7.19, PaO2 46, PaCO2 88, HCO3 25. The nurse would anticipate that the client's immediate treatment will be directed at. a. increasing O2 via nasal cannula to maintain pulse oximetry saturation greater than 94% b. Starting IV with Solu-Medrol c. Intubation and mechanical ventilation d. Administration of STAT bronchodilator via handheld nubulizer

c. Intubation and mechanical ventilation

The nurse is caring for a female client following a Billroth II procedure. Which postoperative order should the nurse question and verify? a. Leg exercises b. Early ambulation c. Irrigating the nasogastric tube d. Coughing and deep-breathing exercises

c. Irrigating the nasogastric tube In a Billroth II procedure, the proximal remnant of the stomach is anastomosed to the proximal jejunum. Patency of the nasogastric tube is critical for preventing the retention of gastric secretions. The nurse should never irrigate or reposition the gastric tube after gastric surgery, unless specifically ordered by the physician. In this situation, the nurse should clarify the order. Options A, B, and D are appropriate postoperative interventions.

Which of the following are byproducts of fat breakdown, which accumulate in the blood and urine? a. Creatinine b. Hemoglobin c. Ketones d. Cholesterol

c. Ketones Ketones are byproducts of fat breakdown, and they accumulate in the blood and urine. Creatinine, hemoglobin, and cholesterol are not byproducts of fat breakdown.

The nurse is teaching a client who receives nitrates for the relief of chest pain. Which instruction should the nurse emphasize? a. Repeat the dose of sublingual nitroglycerin every 15 minutes for three doses. b. Store the drug in a cool, well-lit place. c. Lie down or sit in a chair for 5 to 10 minutes after taking the drug. d. Restrict alcohol intake to two drinks per day.

c. Lie down or sit in a chair for 5 to 10 minutes after taking the drug. Nitrates act primarily to relax coronary smooth muscle and produce vasodilation. They can cause hypotension, which makes the client dizzy and weak. Nitrates are taken at the first sign of chest pain and before activities that might induce chest pain. Sublingual nitroglycerin is taken every 5 minutes for three doses. If the pain persists, the client should seek medical assistance immediately. Nitrates must be stored in a dark place in a closed container. Sunlight causes the medication to lose its effectiveness. Alcohol is prohibited because nitrates may enhance the effects of the alcohol.

The nurse is providing discharge instructions to a male client following gastrectomy and instructs the client to take which measure to assist in preventing dumping syndrome? a. Ambulate following a meal b. Eat high carbohydrate foods c. Limit the fluid taken with meal d. Sit in a high-Fowler's position during meals

c. Limit the fluid taken with meal Dumping syndrome is a term that refers to a constellation of vasomotor symptoms that occurs after eating, especially following a Billroth II procedure. Early manifestations usually occur within 30 minutes of eating and include vertigo, tachycardia, syncope, sweating, pallor, palpitations, and the desire to lie down. The nurse should instruct the client to decrease the amount of fluid taken at meals and to avoid high-carbohydrate foods, including fluids such as fruit nectars; to assume a low-Fowler's position during meals; to lie down for 30 minutes after eating to delay gastric emptying; and to take antispasmodics as prescribed.

The nurse would obtain serum levels of which electrolytes in a client with frequent episodes of ventricular tachycardia? a. Calcium and magnesium b. Potassium and calcium c. Magnesium and potassium d. Potassium and sodium

c. Magnesium and potassium Hypomagnesemia as well as hypokalemia and hyperkalemia are common causes of ventricular tachycardia. Calcium imbalances cause changes in the QT interval and ST segment. Alterations in sodium level don't cause rhythm disturbances.

Which of the following nursing interventions must a nurse perform when administering prescribed vasopressors to a patient with a cardiac dysrhythmia? a. Keep the patient flat for one hour after administration b. Administer every five minutes during cardiac resuscitation c. Monitor vital signs and cardiac rhythm d. Document heart rate before and after administration

c. Monitor vital signs and cardiac rhythm The nurse should monitor the patient's vital signs and cardiac rhythm for effectiveness of the medication and for side effects and should always have emergency life support equipment available when caring for an acutely ill patient. The side effects of vasopressor drugs are hypertension, dysrhythmias, pallor, and oliguria.

The nurse is reviewing the physician's orders written for a male client admitted to the hospital with acute pancreatitis. Which physician order should the nurse question if noted on the client's chart? a. NPO status b. Nasogastric tube inserted c. Morphine sulfate for pain d. An anticholinergic medication

c. Morphine sulfate for pain Meperidine (Demerol) rather than morphine sulfate is the medication of choice to treat pain because morphine sulfate can cause spasms in the sphincter of Oddi. Options A, B, and D are appropriate interventions for the client with acute pancreatitis.

During the previous few months, a 56-year-old woman felt brief twinges of chest pain while working in her garden and has had frequent episodes of indigestion. She comes to the hospital after experiencing severe anterior chest pain while raking leaves. Her evaluation confirms a diagnosis of stable angina pectoris. After stabilization and treatment, the client is discharged from the hospital. At her follow-up appointment, she is discouraged because she is experiencing pain with increasing frequency. She states that she is visiting an invalid friend twice a week and now cannot walk up the second flight of steps to the friend's apartment without pain. Which of the following measures that the nurse could suggest would most likely help the client deal with this problem? a. Visit her friend earlier in the day. b. Rest for at least an hour before climbing the stairs. c. Take a nitroglycerin tablet before climbing the stairs. d. Lie down once she reaches the friend's apartment.

c. Nitroglycerin may be used prophylactically before stressful physical activities such as stair climbing to help the client remain pain free. Visiting her friend early in the day would have no impact on decreasing pain episodes. Resting before or after an activity is not as likely to help prevent an activity-related pain episode.

As an initial step in treating a client with angina, the physician prescribes nitroglycerin tablets, 0.3mg given sublingually. This drug's principle effects are produced by: a. Antispasmotic effect on the pericardium b. Causing an increased mycocardial oxygen demand c. Vasodilation of peripheral vasculature d. Improved conductivity in the myocardium

c. Nitroglycerin produces peripheral vasodilation, which reduces myocardial oxygen consumption and demand. Vasodilation in coronary arteries and collateral vessels may also increase blood flow to the ischemic areas of the heart. Nitroglycerin decreases myocardial oxygen demand. Nitroglycerin does not have an effect on pericardial spasticity or conductivity in the myocardium.

The nurse is caring for a female client with active upper GI bleeding. What is the appropriate diet for this client during the first 24 hours after admission? a. Regular diet b. Skim milk c. Nothing by mouth d. Clear liquids

c. Nothing by mouth Shock and bleeding must be controlled before oral intake, so the client should receive nothing by mouth. A regular diet is incorrect. When the bleeding is controlled, the diet is gradually increased, starting with ice chips and then clear liquids. Skim milk shouldn't be given because it increases gastric acid production, which could prolong bleeding. A liquid diet is the first diet offered after bleeding and shock are controlled.

Which of the following electrolytes is a major cation in body fluid? a. Phosphate b. Chloride c. Potassium d. Bicarbonate

c. Potassium

A nurse caring for a client with hypocalcemia would expect to note which of the following changes on the elctrocardiogram? a. Widened T wave b. Prominent U wave c. Prolonged QT interval d. Shortened ST segment

c. Prolonged QT interval

The physician orders blood coagulation tests to evaluate a client's blood-clotting ability. The nurse knows that such tests are important in assessing clients at risk for thrombi, such as those with a history of atrial fibrillation, infective endocarditis, prosthetic heart valves, or myocardial infarction. Which test is used to determine a client's response to oral anticoagulant drugs? a. Bleeding time b. Platelet count c. Prothrombin time (PT) d. Partial thromboplastin time (PTT)

c. Prothrombin time (PT) PT determines a client's response to oral anticoagulant therapy. This test measures the time required for a fibrin clot to form in a citrated plasma sample after calcium ions and tissue thromboplastin are added and compares this time with the fibrin clotting time in a control sample. Anticoagulant dosages should be adjusted, as needed, to maintain PT at 1.5 to 2.5 times the control value. PTT determines the effectiveness of heparin therapy and helps evaluate bleeding tendencies. Roughly 99% of bleeding disorders are diagnosed from PT and PTT values. Bleeding time indicates how long it takes for a small puncture wound to stop bleeding. The platelet count reveals the number of circulating platelets in venous or arterial blood.

A client with mitral stenosis is scheduled for mitral valve replacement. Which condition may arise as a complication of mitral stenosis? a. Left-sided heart failure b. Myocardial ischemia c. Pulmonary hypertension d. Left ventricular hypertrophy

c. Pulmonary hypertension Mitral stenosis, or severe narrowing of the mitral valve, impedes blood flow through the stenotic valve, increasing pressure in the left atrium and pulmonary circulation. This may lead to low cardiac output, pulmonary hypertension, edema, and right-sided (not left-sided) heart failure. Other potential complications of mitral stenosis include mural thrombi, pulmonary hemorrhage, and embolism to vital organs. Myocardial ischemia may occur in a client with coronary artery disease. Left ventricular hypertrophy is a potential complication of aortic stenosis.

The nurse is caring for a male client with cirrhosis. Which assessment findings indicate that the client has deficient vitamin K absorption caused by this hepatic disease? a. Dyspnea and fatigue b. Ascites and orthopnea c. Purpura and petechiae d. Gynecomastia and testicular atrophy

c. Purpura and petechiae A hepatic disorder, such as cirrhosis, may disrupt the liver's normal use of vitamin K to produce prothrombin (a clotting factor). Consequently, the nurse should monitor the client for signs of bleeding, including purpura and petechiae. Dyspnea and fatigue suggest anemia. Ascites and orthopnea are unrelated to vitamin K absorption. Gynecomastia and testicular atrophy result from decreased estrogen metabolism by the diseased liver.

A male client with a peptic ulcer is scheduled for a vagotomy and the client asks the nurse about the purpose of this procedure. Which response by the nurse best describes the purpose of a vagotomy? a. Halts stress reactions b. Heals the gastric mucosa c. Reduces the stimulus to acid secretions d. Decreases food absorption in the stomach

c. Reduces the stimulus to acid secretions A vagotomy, or cutting of the vagus nerve, is done to eliminate parasympathetic stimulation of gastric secretion. Options A, B, and D are incorrect descriptions of a vagotomy.

Which of the following blood tests is most indicative of cardiac damage? a. Lactate dehydrogenase b. Complete blood count (CBC) c. Troponin I d. Creatine kinase (CK)

c. Troponin I levels rise rapidly and are detectable within 1 hour of myocardial injury. Troponin levels aren't detectable in people without cardiac injury.

In caring for a client with GERD, which task would be appropriate to assign to the nursing assistant? a. Share successful strategies for weight reduction. b. Encourage the client to express concerns about lifestyle modification. c. Remind the client not to lie down for 2 - 3 hours after eating. d. Explain the rationale for small frequent meals.

c. Remind the client not to lie down for 2 - 3 hours after eating. Reminding the client to follow through on advice given by the nurse is an appropriate task for the nursing assistant. The RN should take responsibility for teaching rationale and discussing strategies for the treatment plan and assessing client concerns.

A nurse plans care for a client with COPD, understanding that the client is most likely to experience that type of acid-base imbalance? a. Metabolic acidosis b. Metabolic alkalosis c. Respiratory acidosis d. Respiratory alkalosis

c. Respiratory acidosis

A nurse should recognize the signs of deep vein thrombosis (DVT) if a client reports: Select all the apply. a. Leg feeling cool with no pain. b. Numbness of legs with diaphoresis. c. Sudden swelling of one leg with dependent edema. d. Dizziness and blurred vision. e. Pain behind the knee with dorsiflexion of foot.

c. Sudden swelling of one leg with dependent edema. e. Pain behind the knee with dorsiflexion of foot.

A nurse is preparing to remove a nasogartric tube from a female client. The nurse should instruct the client to do which of the following just before the nurse removes the tube? a. Exhale b. Inhale and exhale quickly c. Take and hold a deep breath d. Perform a Valsalva maneuver

c. Take and hold a deep breath When the nurse removes a nasogastric tube, the client is instructed to take and hold a deep breath. This will close the epiglottis. This allows for easy withdrawal through the esophagus into the nose. The nurse removes the tube with one smooth, continuous pull.

When administering a beta blocker, a nurse should first: a. Take the client's temperature and heart rate. b. Place the client in an upright position before swallowing the drug. c. Take the client's blood pressure and heart rate. d. Ask the client if there have been any adverse effects.

c. Take the client's blood pressure and heart rate.

A client is experiencing tachycardia. The nurse's understanding of the physiological basis for this symptom is explained by which of the following statements? a. The demand for oxygen is decreased because of pleural involvement b. The inflammatory process causes the body to demand more oxygen to meet its needs. c. The heart has to pump faster to meet the demand for oxygen when there is lowered arterial oxygen tension. d. Respirations are labored.

c. The arterial oxygen supply is lowered and the demand for oxygen is increased, which results in the heart's having to beat faster to meet the body's needs for oxygen.

Sublingual nitroglycerin tablets begin to work within 1 to 2 minutes. How should the nurse instruct the client to use the drug when chest pain occurs? a. Take one tablet every 2 to 5 minutes until the pain stops. b. Take one tablet and rest for 10 minutes. Call the physician if pain persists after 10 minutes. c. Take one tablet, then an additional tablet every 5 minutes for a total of 3 tablets. Call the physician if pain persists after three tablets. d. Take one tablet. If pain persists after 5 minutes, take two tablets. If pain still persists 5 minutes later, call the physician.

c. The correct protocol for nitroglycerin used involves immediate administration, with subsequent doses taken at 5-minute intervals as needed, for a total dose of 3 tablets. Sublingual nitroglycerin appears in the blood stream within 2 to 3 minutes and is metabolized within about 10 minutes.

Which of the following may be a potential cause of hypoglycemia in the patient diagnosed with diabetes mellitus. a. The patient has not been exercising. b. The patient has not been compliant with the prescribed treatment regimen. c. The patient has not consumed food and continues to take insulin or oral antidiabetic medications. d. The patient has consumed food and has not taken or received insulin.

c. The patient has not consumed food and continues to take insulin or oral antidiabetic medications. Hypoglycemia occurs when a patient with diabetes is not eating at all and continues to take insulin or oral antidiabetic medications. Hypoglycemia does not occur when the patient has not been compliant with the prescribed treatment regimen. If the patient has eaten and has not taken or received insulin, DKA is more likely to develop.

A woman with severe mitral stenosis and mitral regurgitation has a pulmonary artery catheter inserted. The physician orders pulmonary artery pressure monitoring, including pulmonary capillary wedge pressures. The purpose of this is to help assess the: a. Degree of coronary artery stenosis b. Peripheral arterial pressure c. Pressure from fluid within the left ventricle d. Oxygen and carbon dioxide concentration is the blood

c. The pulmonary artery pressures are used to assess the heart's ability to receive and pump blood. The pulmonary capillary wedge pressure reflects the left ventricle end-diastolic pressure and guides the physician in determining fluid management for the client. The degree of coronary artery stenosis is assessed during a cardiac catherization. The peripheral arterial pressure is assessed with an arterial line.

A client is at risk for pulmonary embolism and is on anticoagulant therapy with warfarin (Coumadin). The client's prothrombin time is 20 seconds, with a control of 11 seconds. The nurse assesses that this result is: a. The same as the client's own baseline level b. Lower than the needed therapeutic level c. Within the therapeutic range d. Higher than the therapeutic range

c. The therapeutic range for prothrombin time is 1.5 to 2 times the control for clients at risk for thrombus. Based on the client's control value, the therapeutic range for this individual would be 16.5 to 22 seconds. Therefore the result is within therapeutic range.

In order to prevent the development of tolerance, the nurse instructs the patient to: a. Apply the nitroglycerin patch every other day b. Switch to sublingual nitroglycerin when the patient's systolic blood pressure elevates to >140 mm Hg c. Apply the nitroglycerin patch for 14 hours each and remove for 10 hours at night d. Use the nitroglycerin patch for acute episodes of angina only

c. Tolerance can be prevented by maintaining an 8- to 12-hour nitrate-free period each day.

Which of the following is also termed preinfarction angina? a. Stable angina b. Variant angina c. Unstable angina d. Silent ischemia

c. Unstable angina

When assessing a client with left-sided heart failure, the nurse expects to note: a. ascites. b. jugular vein distention. c. air hunger. d. pitting edema of the legs.

c. air hunger. With left-sided heart failure, the client typically has air hunger and other signs of pulmonary congestion. Ascites, jugular vein distention, and pitting edema of the legs are signs of right-sided heart failure.

The nurse is caring for a client with congestive heart failure. On assessment, the nurse notes that the client is dyspneic and that crackles are audible on auscultation. The nurse suspects excess fluid volume. What additional signs would the nurse expect to note in this client is excess fluid volume is present? a. weight loss b. flat neck and hand veins c. an increase in blood pressure d. a decreased central venous pressure

c. an increase in blood pressure

Which complication of cardiac surgery occurs when there is fluid and clot accumulation in the pericardial sac, which compresses the heart, preventing blood from filling the ventricles? a. hypothermia b. hypertension c. cardiac tamponade d. fluid overload

c. cardiac tamponade

A client is admitted for treatment of Prinzmetal's angina. When developing the care plan, the nurse keeps in mind that this type of angina is triggered by: a. activities that increase myocardial oxygen demand. b. an unpredictable amount of activity. c. coronary artery spasm. d. the same type of activity that caused previous angina episodes.

c. coronary artery spasm. Prinzmetal's angina results from coronary artery spasm. Activities that increase myocardial oxygen demand may trigger angina of effort. An unpredictable amount of activity may precipitate unstable angina. Worsening angina is brought on by the same type or level of activity that caused previous angina episodes; however, anginal pain is increasingly severe.

Oncotic pressure refers to the a. excretion of substances such as glucose through increased urine output b. number of dissolved particles contained in a unit of fluid. c. osmotic pressure exerted by proteins. d. amount of pressure needed to stop the flow of water by osmosis.

c. osmotic pressure exerted by proteins.

The nurse is caring for a client with acute pulmonary edema. To immediately promote oxygenation and relieve dyspnea, the nurse should: a. administer oxygen. b. have the client take deep breaths and cough. c. place the client in high Fowler's position. d. perform chest physiotherapy.

c. place the client in high Fowler's position. The high Fowler's position will initially promote oxygenation in the client and relieve shortness of breath. Additional measures include administering oxygen to increase content in the blood. Deep breathing and coughing will improve oxygenation postoperatively but may not immediately relieve shortness of breath. Chest physiotherapy results in expectoration of secretions, which isn't the primary problem in pulmonary edema.

A nursing student needs to administer potassium chloride intravenously as prescribed to a client with hypokalemia. The nursing instructor determines that the student is unprepared for this procedure if the student states that which of the following is part of the plan for preparation and administration of the potassium? a. obtaining a controlled IV infusion pump b. monitoring urine output during administration c. preparing the medication for bolus administration d. diluting the medication in appropriate amount of normal saline

c. preparing the medication for bolus administration

A client with severe angina and ST-segment elevation on the electrocardiogram is being seen in the emergency department. In terms of diagnostic laboratory testing, it's most important for the nurse to advocate ordering a: a. creatine kinase level. b. hemoglobin (Hb) level. c. troponin level. d. liver panel.

c. troponin level. Troponin is a myocardial cell protein that is elevated in the serum when myocardial damage has occurred during a myocardial infarction (MI). It's the best serum indicator of MI and is more indicative of cardiac damage than creatine kinase. Hb values and liver panel components aren't as useful in the diagnosis of MI as a troponin level.

A female client with viral hepatitis A is being treated in an acute care facility. Because the client requires enteric precautions, the nurse should: a. place the client in a private room. b. wear a mask when handling the client's bedpan. c. wash the hands after touching the client. d. wear a gown when providing personal care for the client.

c. wash the hands after touching the client. To maintain enteric precautions, the nurse must wash the hands after touching the client or potentially contaminated articles and before caring for another client. A private room is warranted only if the client has poor hygiene — for instance, if the client is unlikely to wash the hands after touching infective material or is likely to share contaminated articles with other clients. For enteric precautions, the nurse need not wear a mask and must wear a gown only if soiling from fecal matter is likely.

Sources of Vitamin K

cabbage, cauliflower, spinach, green leafy vegetables, cereals - synthesized by normal intestinal bacteria

Interventions for Hypercalcemia

calcitonin, dialysis, fluids, fiber

cardiac catheterization/angiogram

catheter inserted via femoral artery and to cardiac artery - dye is injected - patient may experience palpatations, hot flashes, cough - NPO for 6 - 8 hours - should get baseline vital signs, lung & heart sounds - mark pedal pulses - risks are stroke, mi, bleeding, infection, and death

Angioplasty

catheter with balloon used to compress plaque against wall of artery - a stent may be inserted

Pellagra

caused by an inability to absorb niacin (B3) or tryptophan (aa) - scaly sores, mucosal changes, mental sx

Cheilosis

caused by riboflavin deficiency - lesions of lips and mouth

Keratomalacia

caused by vitamin a deficiency

Scurvy

caused by vitamin c deficiency - tender sore gums that bleed easily, small skin hemorrhages - late stages, breakdown of collagen causing delayed wound healing, teeth fall out

Sources of Vitamin B2/Riboflavin

cereal, nuts, milk, eggs, green leafy vegetables, lean meat

Surgical procedures for cholecystitis

cholecystectomy laparoscopic cholecystectomy t-tube

CAL

chronic airflow limitation, ie. asthma

Bronchiectasis

chronic dilation of large sized bronchioles - destruction of bronchial walls accompanied by infection

Causes of constipation

chronic disorders - IBS, diverticular drug induced - antacids, antidepressants, anticholingergics, barium sulfate, iron endocrine - hypothyroidism, diabetes scleroderma neurogenic - megacolon, MS, parkinson's

Carbohydrates

composed of carbon, hydrogen, & oxygen - obtained primarily from plant foods - primary animal carb is lactose - 1g = 4 cal - provide energy - protein sparing - antiketogenic - too much leads to storage as fat - high carb diets usually lack essential nutrients

Fats

composed of carbon, hydrogen, & oxygen - saturated=animal - unsaturated=plant - provides energy - part of cell structure - fatty tissue position and protect vital organs - insulates/prevents loss of body heat - satiety - lubricates bowel - assist in absorption of fate soluble vitamins

Protein

composed of carbon, hydrogen, oxygen, and nitrogen - contain complete and incomplete proteins - necessary for growth, maintenance, and repair of tissues

t-tube care

connected to drain blood tinged drainage changes to greenish/brown may drain 500mL in first 24hrs 200mL drainage after 2-3 days fowler's position when drainage subsides t tube can be clamped before and after meals

Kidney compensation for alkalosis

conserve H+, excrete bicarb

Kidney compensation for acidosis

conserve bicarb, excrete H+

Antiemetics

control vomiting ie. Vistaril, Atarax, Tigan, Compazine, Phenergan

Aldosterone controls ECFV by

controlling amount of sodium excreted by the kidneys

How does kidney function effect blood pressure?

controls Na excretion and retention and therefore controls water - if kidneys are not well perfused then renin is released and angiotensin 1 and 2 are stimulated which are powerful vasoconstrictors

Excess potassium loss through the kidneys is often caused by such meds as

corticosteroids, potassium-wasting diuretics, amphotericin B, & large doses of some antibiotics.

Clinical manifestations of pneumonia

cough, sob, fever, malaise, chills, pleuritic pain, increase tactile fremitus, rales/rhonchi

S2

created by closure of aortic and pulmonic valves

S1

created by the closure of the mitral and tricuspid valves - indicates beginning of ventricular systole

CPK/CK

creatinine kinase/creatinine phosphonase - released into blood when there is tissue death

The physician determines that a client has been exposed to someone with tuberculosis. The nurse expects the physician to order which of the following? a) Daily oral doses of isoniazid (Nydrazid) and rifampin (Rifadin) for 6 months to 2 years b) Isolation until 24 hours after antitubercular therapy begins c) Nothing, until signs of active disease arise d) Daily doses of isoniazid, 300 mg for 6 months to 1 year

d) Daily doses of isoniazid, 300 mg for 6 months to 1 year

The nurse is assessing a client who comes to the clinic for care. Which findings in this client suggest bacterial pneumonia? a) Nonproductive cough and normal temperature b) Sore throat and abdominal pain c) Hemoptysis and dysuria d) Dyspnea and wheezing

d) Dyspnea and wheezing

The nurse is caring for a client with pneumonia. As part of prescribed therapy, the client must use a bedside incentive spirometer to promote maximal deep breathing. The nurse checks to make sure the client is using the spirometer properly. During each waking hour, the client should perform a minimum of how many sustained, voluntary inflation maneuvers? a) One to two b) Three to four c) Five to seven d) Eight to ten

d) Eight to ten

An elderly pt with a history of sodium retention arrives to the clinic with the complaints of "heart skipping beats" & leg tremors. Which of the following should the nurse ask this pt regarding these symptoms? a) "Have you stopped taking your digoxin medication?" b) "When was the last time you had a bowel movement?" c) "Were you doing any unusual physical activity?" d) "Are you using a salt substitute?"

d) "Are you using a salt substitute?"

After diagnosing a client with pulmonary tuberculosis, the physician tells family members that they must receive isoniazid (INH [Laniazid]) as prophylaxis against tuberculosis. The client's teenage daughter asks the nurse how long the drug must be taken. What is the usual duration of prophylactic isoniazid therapy? a) 3 to 5 days b) 1 to 3 weeks c) 2 to 4 months d) 6 to 12 months

d) 6 to 12 months

Clinical manifestations of metabolic alkalosis

dizziness, tachycardia, nausea/vomiting, anorexia, muscle cramps, tetany, hypoventilation, seizures, tingling

When caring for a client with acute respiratory failure, the nurse should expect to focus on resolving which set of problems? a) Hypotension, hyperoxemia, and hypercapnia b) Hyperventilation, hypertension, and hypocapnia c) Hyperoxemia, hypocapnia, and hyperventilation d) Hypercapnia, hypoventilation, and hypoxemia

d) Hypercapnia, hypoventilation, and hypoxemia

On arrival at the intensive care unit, a critically ill client suffers respiratory arrest and is placed on mechanical ventilation. The physician orders pulse oximetry to monitor the client's arterial oxygen saturation (SaO2) noninvasively. Which vital sign abnormality may alter pulse oximetry values? a) Fever b) Tachypnea c) Tachycardia d) Hypotension

d) Hypotension

The nurse is admitting a pt who was diagnosed with acute renal failure. Which of the following electrolytes will be most affected with this disorder? a) calcium b) magnesium c) phosphorous d) potassium

d) potassium

A client hospitalized for treatment of a pulmonary embolism develops respiratory alkalosis. Which clinical findings commonly accompany respiratory alkalosis? a) Nausea or vomiting b) Abdominal pain or diarrhea c) Hallucinations or tinnitus d) Light-headedness or paresthesia

d) Light-headedness or paresthesia

A client has the following arterial blood gas (ABG) values: pH, 7.12; partial pressure of arterial carbon dioxide (PaCO2), 40 mm Hg; and bicarbonate (HCO3-), 15 mEq/L. These ABG values suggest which disorder? a) Respiratory alkalosis b) Respiratory acidosis c) Metabolic alkalosis d) Metabolic acidosis

d) Metabolic acidosis

A client undergoes a tracheostomy after many failed attempts at weaning him from a mechanical ventilator. Two days after tracheostomy, while the client is being weaned, the nurse detects a mild air leak in the tracheostomy tube cuff. What should the nurse do first? a) Call the physician. b) Remove the malfunctioning cuff. c) Add more air to the cuff. d) Suction the client, withdraw residual air from the cuff, and reinflate it.

d) Suction the client, withdraw residual air from the cuff, and reinflate it.

A nurse on the medical-surgical unit just received report on her client care assignment. Which client should she assess first? a) The client with anorexia, weight loss, and night sweats b) The client with crackles and fever who is complaining of pleuritic pain c) The client who had difficulty sleeping, daytime fatigue, and morning headache d) The client with petechiae over the chest who's complaining of anxiety and shortness of breath

d) The client with petechiae over the chest who's complaining of anxiety and shortness of breath

The nurse observes a new environmental services employee enter the room of a client with severe acute respiratory syndrome (SARS). Which action by the employee requires immediate intervention by the nurse? a) The employee wears a gown, gloves, N95 respirator, and eye protection when entering the room. b) The employee doesn't remove the stethoscope, blood pressure cuff, and thermometer that are kept in the room. c) The employee removes all personal protective equipment and washes her hands before leaving the client's room. d) The employee enters the room wearing a gown, gloves, and a mask.

d) The employee enters the room wearing a gown, gloves, and a mask.

A woman whose husband was recently diagnosed with active pulmonary tuberculosis (TB) is a tuberculin skin test converter. Management of her care would include: a) scheduling her for annual tuberculin skin testing. b) placing her in quarantine until sputum cultures are negative. c) gathering a list of persons with whom she has had recent contact. d) advising her to begin prophylactic therapy with isoniazid (INH).

d) advising her to begin prophylactic therapy with isoniazid (INH).

At 11 p.m., a client is admitted to the emergency department. He has a respiratory rate of 44 breaths/min. He's anxious, and wheezes are audible. The client is immediately given oxygen by face mask and methylprednisolone (Depo-medrol) I.V. At 11:30 p.m., the client's arterial blood oxygen saturation is 86%, and he's still wheezing. The nurse should plan to administer: a) alprazolam (Xanax). b) propranolol (Inderal). c) morphine. d) albuterol (Proventil).

d) albuterol (Proventil).

The nurse assessing a client for tracheal displacement should know that the trachea will deviate toward the: a) contralateral side in a simple pneumothorax. b) affected side in a hemothorax. c) affected side in a tension pneumothorax. d) contralateral side in a hemothorax.

d) contralateral side in a hemothorax.

Before administering ephedrine, the nurse assesses the client's history. Because of ephedrine's central nervous system (CNS) effects, it is not recommended for: a) clients with an acute asthma attack. b) clients with narcolepsy. c) clients under age 6. d) elderly clients.

d) elderly clients.

A client with chronic obstructive pulmonary disease tells the nurse that he feels short of breath. The client's respiratory rate is 36 breaths/min and the nurse auscultates diffuse wheezes. His arterial oxygen saturation is 84%. The nurse calls the assigned respiratory therapist to administer a prescribed nebulizer treatment. The therapist says, "I have several more nebulizer treatments to do on the unit where I am now. As soon as I'm done, I'll come assess the client." The nurse's most appropriate action is to: a) notify the primary physician immediately. b) stay with the client until the therapist arrives. c) administer the treatment by metered-dose inhaler. d) give the nebulizer treatment herself.

d) give the nebulizer treatment herself.

A 28-year-old male pt is admitted with diabetic ketoacidosis. The nurse realizes that this pt will have a need for which of the following electrolytes? a) sodium b) potassium c) calcium d) magnesium

d) magnesium

A pt's blood gases show a pH greater of 7.53 & bicarbonate level of 36 mEq/L. The nurse realizes that the acid-base disorder this pt is demonstrating is which of the following? a) respiratory acidosis b) metabolic acidosis c) respiratory alkalosis d) metabolic alkalosis

d) metabolic alkalosis

A pt is admitted for treatment of hypercalcemia. The nurse realizes that this pt's intravenous fluids will most likely be which of the following? a) dextrose 5% & water b) dextrose 5% & ? normal saline c) dextrose 5% & ? normal saline d) normal saline

d) normal saline

A client with Guillain-Barré syndrome develops respiratory acidosis as a result of reduced alveolar ventilation. Which combination of arterial blood gas (ABG) values confirms respiratory acidosis? a) pH, 7.5; PaCO2 30 mm Hg b) pH, 7.40; PaCO2 35 mm Hg c) pH, 7.35; PaCO2 40 mm Hg d) pH, 7.25; PaCO2 50 mm Hg

d) pH, 7.25; PaCO2 50 mm Hg

A client admitted with acute anxiety has the following arterial blood gas (ABG) values: pH, 7.55; partial pressure of arterial oxygen (PaO2), 90 mm Hg; partial pressure of arterial carbon dioxide (PaCO2), 27 mm Hg; and bicarbonate (HCO3-), 24 mEq/L. Based on these values, the nurse suspects: a) metabolic acidosis. b) metabolic alkalosis. c) respiratory acidosis. d) respiratory alkalosis.

d) respiratory alkalosis.

The nurse prepares to perform postural drainage. How should the nurse ascertain the best position to facilitate clearing the lungs? a) Inspection b) Chest X-ray c) Arterial blood gas (ABG) levels d)Auscultation

d)Auscultation

A paradoxical pulse occurs in a client who had a coronary artery bypass graft (CABG) surgery 2 days ago. Which of the following surgical complications should the nurse suspect? a. Left-sided heart failure b. Aortic regurgitation c. Complete heart block d. Pericardial tamponade

d. A paradoxical pulse (a palpable decrease in pulse amplitude on quiet inspiration) signals pericardial tamponade, a complication of CABG surgery. Left-sided heart failure can cause pulsus alternans (pulse amplitude alternation from beat to beat, with a regular rhythm). Aortic regurgitation may cause bisferious pulse (an increased arterial pulse with a double systolic peak). Complete heart block may cause a bounding pulse (a strong pulse with increased pulse pressure).

A client comes to the physician's office for a follow-up visit 4 weeks after suffering a myocardial infarction (MI). The nurse takes this opportunity to evaluate the client's knowledge of the prescribed cardiac rehabilitation program. Which evaluation statement suggests that the client needs more instruction? a. "Client performs relaxation exercises three times per day to reduce stress." b. "Client's 24-hour dietary recall reveals low intake of fat and cholesterol." c. "Client verbalizes an understanding of the need to seek emergency help if the heart rate increases markedly while at rest." d. "Client walks 4 miles in 1 hour every day."

d. "Client walks 4 miles in 1 hour every day." Four weeks after an MI, a client's walking program should aim for a goal of 2 miles in less than 1 hour. Walking 4 miles in 1 hour is excessive and may induce another MI by increasing the heart's oxygen demands. Therefore, this client requires appropriate exercise guidelines and precautions. The other options indicate understanding of the cardiac rehabilitation program. For example, the client should reduce stress, which speeds the heart rate and thus increases myocardial oxygen demands. Reducing dietary fat and cholesterol intake helps lower the risk of atherosclerosis. A sudden rise in the heart rate while at rest warrants emergency medical attention because it may signal a life-threatening arrhythmia and increase myocardial oxygen demands.

The monitor technician on the telemetry unit asks the charge nurse why every client whose monitor shows atrial fibrillation is receiving warfarin (Coumadin). Which response by the charge nurse is best? a. "It's just a coincidence; most clients with atrial fibrillation don't receive warfarin." b. "Warfarin controls heart rate in the client with atrial fibrillation." c. "Warfarin prevents atrial fibrillation from progressing to a lethal arrhythmia." d. "Warfarin prevents clot formation in the atria of clients with atrial fibrillation."

d. "Warfarin prevents clot formation in the atria of clients with atrial fibrillation." Blood pools in the atria of clients with atrial fibrillation. As the blood pools, clots form. These clots can be forced from the atria as the heart beats, placing the client at risk for stroke. Warfarin is prescribed in most clients with atrial fibrillation to prevent clot formation and decrease the risk of stroke, not to control heart rate. Digoxin is typically prescribed to control heart rate in atrial fibrillation. Atrial fibrillation doesn't typically progress to a lethal arrhythmia such as ventricular fibrillation.

Below which serum sodium level may convulsions or coma occur? a. 145 mEq/L b. 140 mEq/L c. 142 mEq/L d. 135 mEq/L

d. 135 mEq/L

How long after oral administration can the nurse expect to see digoxin's (Lanoxin) peak effect? a. 2 to 5 minutes b. 10 to 20 minutes c. 30 minutes to 2 hours d. 2 to 6 hours

d. 2 to 6 hours The peak effect of digoxin occurs 2 to 6 hours after an oral dose and 1 to 4 hours after an I.V. dose. Digoxin's onset of action ranges from 30 minutes to 2 hours after an oral dose and from 5 to 30 minutes after an I.V. dose.

What percentage of potassium that is excreted daily, leaves the body by way of the kidneys? a. 60 b. 40 c. 20 d. 80

d. 80

You would be most concerned about which client having an order for TPN (total parental nutrition) fat emulsion? a. A client with gastrointestinal obstruction b. A client with severe anorexia nervosa c. A client with chronic diarrhea and vomiting d. A client with a fractured femur

d. A client with a fractured femur A client with fractured femur is at risk for fat embolism, so fat emulsion should be used with caution. Vomiting may be a problem if the emulsion is infused too rapidly. TPN is commonly used for gastrointestinal obstruction, severe anorexia nervosa, and chronic diarrhea or vomiting.

Care of which of these clients is most appropriate to assign to the LPN/LVN, under the supervision of an RN? a. A client with oral cancer who is scheduled in the morning for glossectomy b. An obese client returned from surgery following a vertical banded gastroplasty c. A client with anorexia nervosa with muscle weakness and decreased urine output d. A client with intractable nausea and vomiting related to chemotherapy

d. A client with intractable nausea and vomiting related to chemotherapy Nausea and vomiting are common after chemotherapy. Administration of antiemetics and fluid monitoring can be done by an LPN/LVN. The RN should do the pre-operative teaching for the glossectomy client. Clients returning from surgery need extensive assessment. The client with anorexia is showing signs of hypokalemia and is at risk for cardiac dysrhythmias.

The nurse is monitoring a female client with a diagnosis of peptic ulcer. Which assessment findings would most likely indicate perforation of the ulcer? a. Bradycardia b. Numbness in the legs c. Nausea and vomiting d. A rigid, board-like abdomen

d. A rigid, board-like abdomen Perforation of an ulcer is a surgical emergency and is characterized by sudden, sharp, intolerable severe pain beginning in the midepigastric area and spreading over the abdomen, which becomes rigid and board-like. Nausea and vomiting may occur. Tachycardia may occur as hypovolemic shock develops. Numbness in the legs is not an associated finding.

The nurse is monitoring a female client with a diagnosis of peptic ulcer. Which assessment findings would most likely indicate perforation of the ulcer? a. Bradycardia b. Numbness in the legs c. Nausea and vomiting d. A rigid, board-like abdomen

d. A rigid, board-like abdomen Perforation of an ulcer is a surgical emergency and is characterized by sudden, sharp, intolerable severe pain beginning in the midepigastric area and spreading over the abdomen, which becomes rigid and board-like. Nausea and vomiting may occur. Tachycardia may occur as hypovolemic shock develops. Numbness in the legs is not an associated finding.

A client in the emergency department complains of squeezing substernal pain that radiates to the left shoulder and jaw. He also complains of nausea, diaphoresis, and shortness of breath. What should the nurse do? a. Complete the client's registration information, perform an electrocardiogram, gain I.V. access, and take vital signs. b. Alert the cardiac catheterization team, administer oxygen, attach a cardiac monitor, and notify the physician. c. Gain I.V. access, give sublingual nitroglycerin, and alert the cardiac catheterization team. d. Administer oxygen, attach a cardiac monitor, take vital signs, and administer sublingual nitroglycerin.

d. Administer oxygen, attach a cardiac monitor, take vital signs, and administer sublingual nitroglycerin. Cardiac chest pain is caused by myocardial ischemia. Administering supplemental oxygen increases the myocardial oxygen supply. Cardiac monitoring helps detect life-threatening arrhythmias. Ensure that the client isn't hypotensive before giving sublingual nitroglycerin for chest pain. Registration information may be delayed until the client is stabilized. Alerting the cardiac catheterization team before completing the initial assessment is premature.

A client with a 3-day history or nausea and vomiting presents to the emergency department. Arterial blood gases are drawn and the nurse reviews the results, expecting to note which of the following? a. A decreased pH and an increased CO2 b. An increased pH and a decreased CO2 c. A decreased pH and a decreased HCO3- d. An increased pH with an increase HCO3-

d. An increased pH with an increase HCO3-

A nurse is assessing the blood pressure of a client diagnosed with primary hypertension. The nurse ensures accurate measurement by avoiding which of the following? a. Seating the client with arm bared, supported, and at heart level. b. Measuring the blood pressure after the client has been seated quietly for 5 minutes. c. Using a cuff with a rubber bladder that encircles at least 80% of the limb. d. Taking a blood pressure within 15 minutes after nicotine or caffeine ingestion.

d. BP should be taken with the client seated with the arm bared, positioned with support and at heart level. The client should sit with the legs on the floor, feet uncrossed, and not speak during the recording. The client should not have smoked tobacco or taken in caffeine in the 30 minutes preceding the measurement. The client should rest quietly for 5 minutes before the reading is taken. The cuff bladder should encircle at least 80% of the limb being measured. Gauges other than a mercury sphygnomanometer should be calibrated every 6 months to ensure accuracy.

You are caring for a client with peptic ulcer disease. Which assessment finding is the most serious? a. Projectile vomiting b. Burning sensation 2 hours after eating c. Coffee-grounded emesis d. Board-like abdomen with shoulder pain

d. Board-like abdomen with shoulder pain A board-like abdomen with shoulder pain is a symptom of a perforation, which is most lethal complication of peptic ulcer disease. A burning sensation is a typical complaint, which can be controlled with medications. Projectile vomiting can signal an obstruction. Coffee-ground emesis is typical of slower bleeding and will require diagnostic testing.

Which of the following discharge instructions for self-care should the nurse provide to a patient who has undergone a percutaneous transluminal coronary angioplasty (PTCA) procedure? a. Refrain from sexual activity for one month. b. Cleanse the site with disinfectants and dress the wound appropriately. c. Normal activities of daily living can be resumed the first day post op. d. Cleanse the site with soap and water and remove any dressing.

d. Cleanse the site with soap and water and remove any dressing.

A nurse caring for a client who has been receiving IV diuretics suspects that the client is experiencing a fluid volume deficit. Which assessment finding would the nurse note in a client with this condition? a. Lung congestion b. Decreased hematocrit c. Increased blood pressure d. Decreased central venous pressure (CVP)

d. Decreased central venous pressure (CVP)

The nurse receives emergency laboratory results for a client with chest pain and immediately informs the physician. An increased myoglobin level suggests which of the following? a. Cancer b. Hypertension c. Liver disease d. Myocardial infarction

d. Detection of myoglobin is one diagnostic tool to determine whether myocardial damage has occurred. Myoglobin is generally detected about one hour after a heart attack is experienced and peaks within 4 to 6 hours after infarction (Remember, less than 90 mg/L is normal).

The nurse records a client's history and discovers several risk factors for coronary artery disease. Which cardiac risk factors are considered controllable? a. Diabetes, hypercholesterolemia, and heredity b. Diabetes, age, and gender c. Age, gender, and heredity d. Diabetes, hypercholesterolemia, and hypertension

d. Diabetes, hypercholesterolemia, and hypertension Controllable risk factors include hypertension, hypercholesterolemia, obesity, lack of exercise, smoking, diabetes, stress, alcohol abuse, and use of contraceptives. Uncontrollable risk factors for coronary artery disease include gender, age, and heredity.

The nurse is assessing a male client 24 hours following a cholecystectomy. The nurse noted that the T tube has drained 750 mL of green-brown drainage since the surgery. Which nursing intervention is appropriate? a. Clamp the T tube b. Irrigate the T tube c. Notify the physician d. Document the findings

d. Document the findings Following cholecystectomy, drainage from the T tube is initially bloody and then turns to a greenish-brown color. The drainage is measured as output. The amount of expected drainage will range from 500 to 1000 mL/day. The nurse would document the output.

A client reports sharp pain in the calf of the left leg when walking to the bathroom. A nurse notes that the leg has a confined area of warmth, tenderness, and redness. Which nursing action is most appropriate for this client? a. Encouraging the client to ambulate to increase circulation. b. Instructing the client to massage the affected area to relieve the tenderness. c. Applying cold packs to the affected area to decrease inflammation. d. Elevating the affected extremity to promote venous blood flow.

d. Elevating the affected extremity to promote venous blood flow.

Hypokalemia can cause which of the following symptoms to occur? a. Increased release of insulin b. Production of concentrated urine c. Decreased sensitivity to digitalis d. Excessive thirst

d. Excessive thirst

Polyethylene glycol-electrlyte solution (GoLYTELY) is prescribed for the female client scheduled for a colonoscopy. The client begins to experience diarrhea following administration of the solution. What action by the nurse is appropriate? a. Start an IV infusion b. Administer an enema c. Cancel the diagnostic test d. Explain that diarrhea is expected

d. Explain that diarrhea is expected The solution GoLYTELY is a bowel evacuant used to prepare a client for a colonoscopy by cleansing the bowel. The solution is expected to cause a mild diarrhea and will clear the bowel in 4 to 5 hours. Options A, B, and C are inappropriate actions.

Which of the following is a factor that increases blood urea nitrogen (BUN)? a. Hypothermia b. Decreased protein intake c. Overhydration d. GI bleeding

d. GI bleeding

Which of these agents is a major contributing factor in the promotion of peptic ulcer disorder? a. Candida albicans b. Staphylococcus infection c. Streptococcus infection d. Helicobacter pylori infection

d. Helicobacter pylori infection Recurrence of peptic ulcers is related to Helicobacter pylori, use of NSAIDs, smoking, and continued acid hypersecretion.

white male, age 43, with a tentative diagnosis of infective endocarditis is admitted to an acute care facility. His medical history reveals diabetes mellitus, hypertension, and pernicious anemia; he underwent an appendectomy 20 years ago and an aortic valve replacement 2 years ago. Which history finding is a major risk factor for infective endocarditis? a. Race b. Age c. History of diabetes mellitus d. History of aortic valve replacement

d. History of aortic valve replacement A heart valve prosthesis, such as an aortic valve replacement, is a major risk factor for infective endocarditis. Other risk factors include a history of heart disease (especially mitral valve prolapse), chronic debilitating disease, I.V. drug abuse, and immunosuppression. Although race, age, and a history of diabetes mellitus may predispose a person to cardiovascular disease, they aren't major risk factors for infective endocarditis.

Patients diagnosed with esophageal varices are at risk for hemorrhagic shock. Which is a sign of potential hypovolemia? a. Polyuria b. Warm moist skin c. Bradycardia d. Hypotension

d. Hypotension Signs of potential hypovolemia include cool, clammy skin, tachycarida, decreased blood pressure, and decreased urine output.

Which of the following would be a potential cause for respiratory acidosis? a. Diarrhea b. Vomiting c. Hyperventilation d. Hypoventilation

d. Hypoventilation

A client with chest pain, dyspnea, and an irregular heartbeat comes to the emergency department. An electrocardiogram shows a heart rate of 110 beats/minute (sinus tachycardia) with frequent premature ventricular contractions. Shortly after admission, the client has ventricular tachycardia and becomes unresponsive. After successful resuscitation, the client is taken to the intensive care unit (ICU). Which nursing diagnosis is appropriate at this time? a. Deficient knowledge (disease process) related to interventions used to treat acute illness b. Impaired physical mobility related to complete bed rest c. Social isolation related to restricted visiting hours in the ICU d. Ineffective tissue perfusion (cardiopulmonary) related to arrhythmia

d. Ineffective tissue perfusion (cardiopulmonary) related to arrhythmia The client suffered a lethal arrhythmia, requiring immediate resuscitation. This arrhythmia was caused by ineffective perfusion to the heart. Therefore, the client should have the nursing diagnosis Ineffective tissue perfusion (cardiopulmonary). Client teaching should be limited to clear, concise explanations that reduce anxiety and promote cooperation. An anxious client has difficulty learning, so the knowledge deficit would continue despite attempts at teaching. Impaired physical mobility and Social isolation are necessitated by the client's critical condition; therefore, they are considered therapeutic, not problems warranting nursing diagnoses.

In a client with chronic bronchitis, which sign would lead the nurse to suspect right-sided heart failure? a. Cyanosis of the lips b. Bilateral crackles c. Productive cough d. Leg edema

d. Leg edema Right-sided heart failure is characterized by signs of circulatory congestion, such as leg edema, neck vein distention, and hepatomegaly. Left-sided heart failure is characterized by circumoral cyanosis, crackles, and a productive cough.

While palpating a female client's right upper quadrant (RUQ), the nurse would expect to find which of the following structures? a. Sigmoid colon b. Appendix c. Spleen d. Liver

d. Liver The RUQ contains the liver, gallbladder, duodenum, head of the pancreas, hepatic flexure of the colon, portions of the ascending and transverse colon, and a portion of the right kidney. The sigmoid colon is located in the left lower quadrant; the appendix, in the right lower quadrant; and the spleen, in the left upper quadrant.

A client with a history of I.V. drug abuse is admitted to the medical-surgical unit for evaluation for infective endocarditis. Nursing assessment is most likely to reveal that this client has: a. retrosternal pain that worsens during supine positioning. b. pulsus paradoxus. c. a scratchy pericardial friction rub. d. Osler's nodes and splinter hemorrhages.

d. Osler's nodes and splinter hemorrhages. Infective endocarditis occurs when an infectious agent enters the bloodstream, such as from I.V. drug abuse or during an invasive procedure or dental work. Typical assessment findings in clients with this disease include Osler's nodes (red, painful nodules on the fingers and toes), splinter hemorrhages, fever, diaphoresis, joint pain, weakness, abdominal pain, a new or altered heart murmur, and Janeway's lesions (small, hemorrhagic areas on the fingers, toes, ears, and nose). The other options are common findings in clients with pericarditis, not infective endocarditis.

Malnutrition occurs as a result of part of: a. The aging process b. Low income c. Infirmity d. Overnutrition

d. Overnutrition

The calcium level of the blood is regulated by which mechanism? a. Thyroid hormone (TH) b. Androgens c. Adrenal gland d. Parathyroid hormone (PTH)

d. Parathyroid hormone (PTH)

A nurse is reviewing laboratory results and notes that a client's serum sodium level is 150 mEq/L. The nurse reports the serum sodium level to the physician and the physician prescribes dietary instructions based on the sodium level. Which food item does the nurse instruct the client to avoid? a. Peas b. Nuts c. Cauliflower d. Processed oat cereals

d. Processed oat cereals

A nurse is inserting a nasogastric tube in an adult male client. During the procedure, the client begins to cough and has difficulty breathing. Which of the following is the appropriate nursing action? a. Quickly insert the tube b. Notify the physician immediately c. Remove the tube and reinsert when the respiratory distress subsides d. Pull back on the tube and wait until the respiratory distress subsides

d. Pull back on the tube and wait until the respiratory distress subsides During the insertion of a nasogastric tube, if the client experiences difficulty breathing or any respiratory distress, withdraw the tube slightly, stop the tube advancement, and wait until the distress subsides. Options B and C are unnecessary. Quickly inserting the tube is not an appropriate action because, in this situation, it may be likely that the tube has entered the bronchus.

A client with emphysema has the following blood gas values: PCO2 35, PO2 78, and pH 7.40. The nurse's action should be to: a. Give O2 at 1L immediately for hypoxia. b. Take no action, these values are normal for the condition. c. Tell the client to breathe slower. d. Question the report and call the physician.

d. Question the report and call the physician.

A nurse is caring for a client with diabetic ketoacidosis and documents that the client is experiencing Kussmaul's respirations. Based on this documentation, which of the following did the nurse observe? a. Respirations that cease for several seconds. b. Respirations that are regular but abnormally slow. c. Respirations that are labored and increased in depth and rate. d. Respirations that are abnormally deep, regular, and increased in rate.

d. Respirations that are abnormally deep, regular, and increased in rate.

A client's ABGs are: pH 7.34, PaCO2 57 mm Hg, 22 HCO3. What is the correct interpretation? a. Metabolic alkalosis b. Respiratory alkalosis c. Metabolic acidosis d. Respiratory acidosis

d. Respiratory acidosis

A client who is found unresponsive has arterial blood gases drawn and the results indicate the following: pH is 7.12, PCO2 is 90 mm Hg, and HCO3- is 22 mEq/L. The nurse interprets the results as indicating which condition? a. Metabolic acidosis with compensation b. Respiratory acidosis with compensation c. Metabolic acidosis without compensation d. Respiratory acidosis without compensation

d. Respiratory acidosis without compensation

The adaptations of a client with complete heart block would most likely include: a. Nausea and vertigo b. Flushing and slurred speech c. Cephalalgia and blurred vision d. Syncope and low ventricular rate

d. Syncope and low ventricular rate In complete atrioventricular block, the ventricles take over the pacemaker function in the heart but at a much slower rate than that of the SA node. As a result there is decreased cerebral circulation, causing syncope.

A nurse reviews the electrolyte results of an assigned client and notes that the potassium level is 5.4 mEq/L. Which of the following would the nurse expect to note on the electrocardiogram as result of the laboratory value? a. ST depression b. Inverted T wave c. Prominent U wave d. Tall peaked T waves

d. Tall peaked T waves

Which of the following diagnostic tools is most commonly used to determine the location of myocardial damage? a. Cardiac catherization b. Cardiac enzymes c. Echocardiogram d. Electrocardiogram (ECG)

d. The ECG is the quickest, most accurate, and most widely used tool to determine the location of myocardial infarction. Cardiac enzymes are used to diagnose MI but can't determine the location. An echocardiogram is used most widely to view myocardial wall function after an MI has been diagnosed. Cardiac catherization is an invasive study for determining coronary artery disease and may also indicate the location of myocardial damage, but the study may not be performed immediately.

When assessing an ECG, the nurse knows that the P-R interval represents the time it takes for the: a. Impulse to begin atrial contraction b. Impulse to transverse the atria to the AV node c. SA node to discharge the impulse to begin atrial depolarization d. Impulse to travel to the ventricles

d. The P-R interval is measured on the ECG strip from the beginning of the P wave to the beginning of the QRS complex. It is the time it takes for the impulse to travel to the ventricle.

IV heparin therapy is ordered for a client. While implementing this order, a nurse ensures that which of the following medications is available on the nursing unit? a. Vitamin K b. Aminocaporic acid c. Potassium chloride d. Protamine sulfate

d. The antidote to heparin is protamine sulfate and should be readily available for use if excessive bleeding or hemorrhage should occur. Vitamin K is an antidote for warfarin.

The nurse prepares the client for insertion of a pulmonary artery catheter (Swan-Ganz catheter). The nurse teaches the client that the catheter will be inserted to provide information about: a. Stroke volume b. Cardiac output c. Venous pressure d. Left ventricular functioning

d. The catheter is placed in the pulmonary artery. Information regarding left ventricular function is obtained when the catheter balloon is inflated.

When preparing a client for discharge after surgery for a CABG, the nurse should teach the client that there will be: a. No further drainage from the incisions after hospitalizations b. A mild fever and extreme fatigue for several weeks after surgery c. Little incisional pain and tenderness after 3 to 4 weeks after surgery d. Some increase in edema in the leg used for the donor graft when activity increases

d. The client is up more at home, so dependent edema usually increases. Serosanguinous drainage may persist after discharge.

Which of the following types of pain is most characteristic of angina? a. Knifelike b. Sharp c. Shooting d. Tightness

d. The pain of angina usually ranges from a vague feeling of tightness to heavy, intense pain. Pain impulses originate in the most visceral muscles and may move to such areas as the chest, neck, and arms.

A client is scheduled for a cardiac catheterization using a radiopaque dye. Which of the following assessments is most critical before the procedure? a. Intake and output b. Baseline peripheral pulse rates c. Height and weight d. Allergy to iodine or shellfish

d. This procedure requires an informed consent because it involves injection of a radiopaque dye into the blood vessel. The risk of allergic reaction and possible anaphylaxis is serious and must be assessed before the procedure.

Which enzyme aids in the digestion of protein? a. Amylase b. Lipase c. Pepsin d. Trypsin

d. Trypsin Trypsin is an enzyme that aids in the digestion of protein. Lipase is an enzyme that aids in the digestion of fats. Pepsin is a gastric enzyme that aids in the digestion of fats. Amylase is an enzyme that aids in the digestion of starch.

Which of the following medications are used to decrease portal pressure, halting bleeding or esophageal varices? a. Sprironaclactone (Aldactone) b. Nitroglycerin c. Cimetidine (Tagamet) d. Vasopressin (Pitressin)

d. Vasopressin (Pitressin) Vasopressin may be the intitial therapy for esophageal varices, because it produces constriction of the sphlanchnic arterial bed and decreases portal hypertension. Nitroglycerin has been used to precvent the side effects of Vasopressin. Aldactone and Tagamet do not decrease portal hypertension.

Treatment for hypertension is important in the prevention of heart failure. A nurse tells a client that the greatest decrease in blood pressure is most likely to occur with: a. Increasing physical activity. b. Reducing sodium intake. c. Limiting alcohol intake. d. Weight loss.

d. Weight loss.

The nurse expects that a client with mitral stenosis would demonstrate symptoms associated with congestion in the: a. Aorta b. Right atrium c. Superior vena cava d. Pulmonary circulation

d. When mitral stenosis is present, the left atrium has difficulty emptying its contents into the left ventricle. Hence, because there is no valve to prevent backward flow into the pulmonary vein, the pulmonary circulation is under pressure.

A client with a bundle branch block is on a cardiac monitor. The nurse should expect to observe: a. Sagging ST segments b. Absence of P wave configurations c. Inverted T waves following each QRS complex d. Widening of QRS complexes to 0.12 second or greater.

d. Widening of QRS complexes to 0.12 second or greater. Bundle branch block interferes with the conduction of impulses from the AV node to the ventricle supplied by the affected bundle. Conduction through the ventricles is delayed, as evidenced by a widened QRS complex.

A 53-year-old client is about to undergo cardiac catheterization for which he signed an informed consent. As the nurse enters the room to administer sedation for the procedure, the client states, "I'm really worried about having this open heart surgery." Based on this statement, how should the nurse proceed? a. Medicate the client and document his comment. b. Medicate the client and notify the physician about the comment. c. Explain that cardiac catheterization doesn't involve open heart surgery, and then medicate the client. d. Withhold the medication and notify the physician immediately.

d. Withhold the medication and notify the physician immediately. The nurse should withhold the medication and notify the physician that the client doesn't understand the procedure. The physician then has the obligation to explain the procedure better to the client and determine whether or not the client understands. If the client doesn't understand, he can't give a true informed consent. If the medication is administered before the physician explains the procedure, the sedation may interfere with the client's ability to clearly understand the procedure. The nurse can't just medicate the client and document her finding; she must notify the physician.

Nurse Hannah is teaching a group of middle-aged men about peptic ulcers. When discussing risk factors for peptic ulcers, the nurse should mention: a. a sedentary lifestyle and smoking. b. a history of hemorrhoids and smoking. c. alcohol abuse and a history of acute renal failure. d. alcohol abuse and smoking.

d. alcohol abuse and smoking. Risk factors for peptic (gastric and duodenal) ulcers include alcohol abuse, smoking, and stress. A sedentary lifestyle and a history of hemorrhoids aren't risk factors for peptic ulcers. Chronic renal failure, not acute renal failure, is associated with duodenal ulcers.

Following coronary artery bypass grafting, a client begins having chest "fullness" and anxiety. The nurse suspects cardiac tamponade and prints a lead II electrocardiograph (ECG) strip for interpretation. In looking at the strip, the change in the QRS complex that would most support her suspicion is: a. narrowing complex. b. widening complex. c. amplitude increase. d. amplitude decrease.

d. amplitude decrease. Fluid surrounding the heart such as in cardiac tamponade, suppresses the amplitude of the QRS complexes on an ECG. Narrowing and widening complexes as well as an amplitude increase aren't what is expected on the ECG of an individual with cardiac tamponade.

A female client with hepatitis C develops liver failure and GI hemorrhage. The blood products that would most likely bring about hemostasis in the client are: a. whole blood and albumin. b. platelets and packed red blood cells. c. fresh frozen plasma and whole blood. d. cryoprecipitate and fresh frozen plasma.

d. cryoprecipitate and fresh frozen plasma. The liver is vital in the synthesis of clotting factors, so when it's diseased or dysfunctional, as in hepatitis C, bleeding occurs. Treatment consists of administering blood products that aid clotting. These include fresh frozen plasma containing fibrinogen and cryoprecipitate, which have most of the clotting factors. Although administering whole blood, albumin, and packed cells will contribute to hemostasis, those products aren't specifically used to treat hemostasis. Platelets are helpful, but the best answer is cryoprecipitate and fresh frozen plasma.

A client is admitted to the acute care facility for treatment of heart failure. The nurse expects the physician to prescribe which drug? a. prednisone (Orasone) b. hydroxychloroquine sulfate (Plaquenil Sulfate) c. lidocaine (Xylocaine) d. furosemide (Lasix)

d. furosemide (Lasix) To maintain fluid balance — crucial for a client with heart failure — the physician typically prescribes a diuretic, such as furosemide; vasodilating agents; and drugs that increase contractility, such as digitalis glycosides. Prednisone, a corticosteroid, and hydroxychloroquine, an antimalarial agent, aren't indicated for heart failure. Lidocaine would be used only if the client also had ventricular ectopy.

Early signs of hypervolemia include a. a decrease in blood pressure. b. moist breath sounds. c. thirst. d. increased breathing effort and weight gain.

d. increased breathing effort and weight gain.

Patients diagnosed with hypervolemia should avoid sweet or dry food because a. it obstructs water elimination. b. it can cause dehydration. c. it can lead to weight gain. d. it can increase the patient's desire to consume fluid.

d. it can increase the patient's desire to consume fluid.

A nurse caring for a group of clients reviews the electrolyte laboratory results and notes a potassium level of 5.5 mEq/L on the client's laboratory report. The nurse understands that which client is at highest risk for the development of a potassium value at this level? a. the client with colitis b. the client with Cushing's syndrome c. the client who has been overusing laxatives d. the client who has sustained a traumatic burn

d. the client who has sustained a traumatic burn

A client with second-degree atrioventricular heart block is admitted to the coronary care unit. The nurse closely monitors the heart rate and rhythm. When interpreting the client's electrocardiogram (ECG) strip, the nurse knows that the QRS complex represents: a. atrial repolarization. b. ventricular repolarization. c. atrial depolarization. d. ventricular depolarization.

d. ventricular depolarization. The QRS complex on the ECG strip represents ventricular depolarization. Atrial repolarization usually occurs at the same time as ventricular depolarization and can't be distinguished on the ECG. The T wave represents ventricular repolarization. The P wave represents atrial depolarization.

During preparation for bowel surgery, a male client receives an antibiotic to reduce intestinal bacteria. Antibiotic therapy may interfere with synthesis of which vitamin and may lead to hypoprothrombinemia? a. vitamin A b. vitamin D c. vitamin E d. vitamin K

d. vitamin K Intestinal bacteria synthesize such nutritional substances as vitamin K, thiamine, riboflavin, vitamin B12, folic acid, biotin, and nicotinic acid. Therefore, antibiotic therapy may interfere with synthesis of these substances, including vitamin K. Intestinal bacteria don't synthesize vitamins A, D, or E.

Treatment for superficial phlebitis

d/c IV, warm compresses, and elevation

Sources of Vitamin B3/Niacin

dairy, poultry, fish, lean meat, nuts, eggs

Respiratory compensation for alkalosis

decrease RR to retain CO2

Ulcer Diet

decrease mechanical irritants, decrease thermal irritants, decrease chemical irritants

Causes of diarrhea

decreased fluid absorption - laxative abuse, mucosal damage (Chron's, radiation, colitis, ischemic bowel dis.) increased fluid secretion - infectious bacteria endotoxins, antibiotics, foods w/ sorbitol, hormonal, adenoma of pancreas motility disturbances - IBS, gastrectomy

LABS FVE

decreased hematocrit, decreased BUN, decreased serum sodium, decreased urine specific gravity

Venturi mask

delivers a specified, controlled amount of oxygen consistently and accurately

Lipohypertrophy

development of fibrous fatty masses at the injection site and is caused by repeated use of an injection site - Instruct the client to avoid injecting insulin into affected sites. and about the importance of rotating insulin injection at one anatomical site

Causes of metabolic acidosis

diabetic ketoacidosis, lactic adicosis, starvation, alcoholism, renal failure, shock, severe diarrhea, GI fistula

Non pharmacologic treatments for Hypertension

dietary, exercise, stress management, cessation of smoking

Nitrates (CAD/Angina)

dilate coronary arteries - decrease preload and afterload - side effects: headache, hypotension, flushing 1 sublingual q5min as needed - max 3 doses

Calcium channel blockers (CAD/Angina)

dilate coronary arteries and reduce vasospasm

Mitral valve insufficiency can lead to...

dilation and hypertrophy of left atrium and left ventricle - can cause Afib and CHF

Clinical manifestations of metabolic acidosis

drowsiness/coma, decreased BP, bradycardia, nasea/vomiting, diarrhea, abdominal pain, deep rapid respirations, headache

Integumentary assessment findings for FVD

dry skin, poor skin turgor, dry mouth

xerostomia

dryness of the mouth

Stimulant Cathartic Laxatives

ducolax, cadcara, caster oil, ex lax, sse

Hemolytic Jaundice

due to increased breakdown of RBCs - blood transfusion reactions, anemia

Aortic stenosis - signs and symptoms

dyspnea on exertion, angina, syncope

Mitral valve insufficiency - signs and symptoms

dyspnea on exertion, orthopnea

Aortic insufficiency - signs and symptoms

dyspnea on exertion, orthopnea, palpitations, angina

Risk factors for peripheral vascular disease

elevated lipid levels, cigarette smoking, diabetes, hypertension, gender, age

Modifiable risk factors CAD

elevated lipid levels, hypertension, smoking, sedentary lifestyle, obesity, stress, diabetes, increased homocysteine levels

Myocardial injury is indicated on EKG by...

elevation or depression of ST segment, T wave inversion, or deep Q waves

Light Diet

eliminates gas forming food

Complications of Bronchitis

emphysema, cor pulmonale, polycthemia

Functions of Insulin

enables glucose to be transported across cell membrane converts glucose into glycogen for storage in liver and muscles helps excess glucose be converted to fat prevents protein breakdown for energy

Pericardial sac

encases the heart and protects from trauma and infection 2 layers - parietal (outer, tough/fibrous) and visceral (inner, thin)

Mixed urinary incontinence

encompasses several types of urinary incontinence, is involuntary leakage associated with urgency and also with exertion, effort, sneezing, or coughing

3 Layers of the heart

endocardium (inner), myocardium (muscle), pericardium (outer) - double layer filled with fluid

Procedures used for management of esophageal varices

endoscopic sclerotherapy esophageal balloon tamponade portacaval shunt TIPS - transjugular intrahepatic portosystemic shunt

Interventions for Esophageal Balloon Tamponade

ensure inflation of 2 balloons used to secure tube aspiration/irrigation port is used for saline lavage balloon is deflated q 8-12 hours semi-fowler's position label ports and attach to suction monitor for airway obstruction - if obstruction then cut tube

How does the sympathetic nervous system influence blood pressure?

epinepherine and adrenaline increase blood pressure

Minerals

essential as catalysts in biochemical reactions

Mitral stenosis can lead to...

eventually increased pressure in left atrium causes dilation and eventual CHF - also causes Afib

Pleural Effusion

excess pleural fluid - sx of something else

Anasarca

excessive accumulation of fluid in interstitial space throughout the body

Hydrostatic pressure

exerted on walls of blood vesssles

Hallmark for Asthma

expiratory wheeze

Signs and Symptoms of Hypokalemia

fatigue, anorexia, nausea/vomiting, muscle weakness, paresthesia, flat T waves

Mitral valve prolapse - signs and symptoms

fatigue, atypical chest pain, palpitations, dizziness, tachycardia

Clear Liquid Diet

fluid that requires minimal digestive action - used preop and before diagnostic tests to eliminate undigested food in GI tract

Signs and symptoms of hypermagnesemia

flushing, hypotension, drowsiness, decreased RR, decreased DTRs, cardiac changes

GI Stiumulants

for GERD stimulate motlity of upper GI tract and increase rate of gastric emptying without stimulating secretions ie. Urecholine, Reglan (increases les pressure), Prostigmin

H2 Receptor Antagonist

for GERD and PUD (tidine) suppress secretion of gastric acid avoid alcohol, NSAIDs, chocolate, caffeine, peppermint ie Tagamet, Zantac, Pepcid, Axid $$ - cigarette smoking interferes with drug action

Antacids

for GERD and PUD - can be magnesium or aluminum based Taken on regular schedule several times a day React with gastric acid to produce salts that lower acidity Taken 1 - 3 hours after meals and at bedtime Allow 1 hour between antacids and other meds

Carafate

for PUD mucosal healing agent taken 1 hr before meals antacids interfere with absorption

Proton Pump Inhibitors

for ulcer disease (prazole) taken before meals do not crush or chew ie. Prevacid, Protonix (can be IV), Nexium, Prilosec, Aciphex

Vitamin B12 (function)

formation of RBCs and maintenance of CNS - requires intrinsic factor to be absorbed

Primary Hypertension

hypertension with no known cause - risk factors

Respiratory alkalosis is always due to

hyperventalation

Which electrolyte imbalance causes flattened T waves on EKG?

hypokalemia

Water intoxication can cause which electrolyte imbalance?

hyponatremia

Tonicity of half-normal saline solution

hypotonic

Complications of pancreatitis

hypovolemic shock pseudocyst abscess pleural effusion atelectasis pneumonia tetany

Apraxia

inability to construct simple figures

Respiratory compensation for acidosis

increase RR to get rid of CO2

Superficial Phlebitis

inflammation of vein often related to IV, typically upper extremities

NPH

intermediate acting insulin

Subjective assessment for PVD

intermittent claudication, pain

Types of extracellular fluid

intravascular, interstitial, transcellular

Balloon valvuloplasty

invasive non surgical procedure - catheter is placed in femoral vein to valve and balloon is inflated to enlarge orifice - monitor for bleeding from insertion site - monitor for signs of emboli - monitor cardiac rhythm, heart sounds, and cardiac output

strangulated hernia

irreducible with obstruction to blood supply - surgical emergency

Lactated Ringer's solution is

isotonic

Normal saline (0.9% NaCl) is

isotonic

Acetylcysteine must be used cautiously in a client with asthma because:

it may induce bronchospasm

Treatment for Raynaud's phenomenon

keep hands warm with gloves, stop smoking, calcium channel blockers, nitrates

Patient Ed for arterial bypass surgery

keep incision clean and dry, frequent position change, medications, professional foot care, risk factor modification, surgical follow up

Pernicious Anemia

lack of intrinsic factor - inadequate absorption of Vitamin B12

Exercise stress test

leads are used - physician is present - patient walks on treadmill - EKG and vital signs are monitored - patient should avoid alcohol, smoking, caffeine

landmarks to auscultate left lower lobe anteriorly

left anterior axillary line, between 5th and 6th intercostal spaces

Clinical manifestations of respiratory acidosis

lethargy/coma, decreased BP, rapid shallow breathing, hypoventilation, hypoxia, headache, seizures, cardiac changes

Neuromuscular assessment finding for FVD

lethargy/coma, fever, muscle weakness

Clinical manifestation of respiratory alkalosis

lethargy/confusion, tachycardia, dysrhythmias, nausea/vomiting, epigastric pain, numbness

3 Levels of diabetic education

level 1 - blood glucose monitoring, insulin admin, s&s of hypoglycemia and hyperglycemia level 2 - maintaining blood glucose level, diet management, sick day guidelines level 3 - effects exercise, adjusting insulin and lifestyle, stress management

Diabetic Ketoacidosis

life-threatening complication of type 1 diabetes mellitus that develops when a severe insulin deficiency occurs - manifested by hyperglycemia, dehydration, ketosis, and acidosis - sudden onset

MAP

mean arterial pressure

Trental

medication that increases flexibility of RBCs

Manifestations of hyponatremia

mental confusion, muscle weakness, anorexia, restlessness, elevated body temperature, tachycardia, nausea, vomiting, and personality changes - convulsions or coma can occur is deficit is severe

Clients experiencing nausea and vomiting would most likely present with which acid-base disorder?

metabolic alkalosis

Eventually fluid congestion in lungs leads to

moist breath sounds.

Interventions for Acid Base Imbalance

monitor ABGs, I&O, vital signs, all system assessment with emphasis on respiratory and cardiac

Whole Grains

more nutritious than refined grains - contain entire grain kernel (including germ and bran) ie. whole wheat, bulgur, oatmeal, brown rice, whole cornmeal

Collateral circulation

more than one artery supplying a muscle with blood, develops when chronic ischemia occurs to meet the metabolic demands; therefore an occlusion of a coronary artery in a younger individual is more likely to be lethal than in an older individual

Raynaud's Phenomenon

mostly female 15-45 y/o - effects hands - spasm of vessel related to cold weather or physical/emotional stress - hands turn blue and then white - when spasm resolves hands turn red

Burger's disease

mostly male - thromboangitis obliterans - small/medium peripheral arteries become inflamed, spastic, and thrombotic - effects legs/feet - often smokers < 40 y/o and asian/eastern european - leads to tissue ulceration and gangrene

Causes of fecal incontinence

motor - muscle contraction sensory - dementia, stroke, spinal cord injury, degenerative disease

Symptoms and cause of intermittent claudication

muscle cramping and/or weakness on exercise - b/c not enough O2 getting to limb causing nerve damage

Low serum calcium levels produces what neuromuscular symptom?

muscle spasms/tetany

Signs and Symptoms of Hyperkalemia

muscle weakness, cardiac dysrhythmias, tall T waves on EKG

Coronary Artery Disease (definition)

narrowing or obstruction of one or more coronary arteries as a result of atherosclerosis causing decreased perfusion of myocardial tissue and inadequate myocardial oxygen supply leading to hypertension, angina, dysrhythmias, MI, heart failure, and death

Assessment findings for intestinal obstruction

nausea abdominal distention obstipation high pitched bowel sounds temp > 100 SBO projectile vomiting pain relieved with vomiting LBO persistent colicky pain no vomiting orange brown stool

Signs and Symptoms of Hyponatremia

nausea/vomiting, poor skin turgor, dry mucosa, weakness/lethargy/confusion, muscle cramps, twitching, seizures

Causes of non-mechanical intestinal obstruction

neuromuscular vascular disorders (emboli to mesenteric artery) post abdominal surgery inflammatory response (pancreatitis, appendicitis) electrolyte imbalance spinal fracture

Signs and symptoms of hypomagnesemia

neuromuscular irritability, depression, disorientation

Vitamin A Deficiency

night blindness, keratolmalacia, & mucous membrane infection

Patient teaching after prostate surgery

no intercourse or heavy lifting x 6 weeks avoid sitting/walking for prolonged periods of time drink at least 1-2 L of fluid daily urinate q 2-3 hrs kegels incontinent briefs for dribbling yearly rectal exams

Janeway lesions

nontender hemorrhagic lesions - fingers, toes, nose, earlobes - associated with endocarditis

PR interval

normal is .12 - .20 seconds - represents

Full compensation ABB disorder

normal pH, both CO2 and bicarb either high or low

Epitaxis

nose bleed

Colostomy irrigation

obtain colostomy irrigation set (cone tip) expel air from tubing position client near toilet instil 500-1,000 mL luke warm water hold irrigation bag 18 - 24 inches above the stoma (shoulder height) lubricate cone and insert into stoma 5 - 10 min in 35 -45 min out stop irrigation if sever cramping

Pulmonary embolism

occlusion of pulmonary vasculature by foreign substance - blocks blood flow distal to occlusion - can result in sudden death - lung is ventilated not perfused

Community Acquired Pneumonia (CAP)

occurs in community setting or within first 48 hours of hospital admission

Stable angina

occurs with activities involving exertion or emotional stress - stable pattern of onset, duration, and intensity

Intermediate acting insulin

onset: 2 - 4 hours peak: 4 - 12 hours duration: 12 - 18 hours *often used in combination with short acting insulin

Short acting insulin (regular)

onset: 30 minutes peak: 2 - 3 hours duration: 3 - 6 hours * may be administered IV (DKA)

Long acting insulin

onset: 6 - 10 hours peak: varies duration: 18 - 24 hours * do not mix with other insulins * take at same time every day

Sodium plays a role in

osmotic pressure, ABB, nerve impulse transmission

Signs and Symptoms of Hypophosphatemia

paresthesia, muscle weakness/pain, mental changes, resp failure

hemothorax

partial or complete collapse of the lung due to blood accumulating in the pleural space, may occur after surgery or trauma

PTT

partial thromboplastin time - assesses efficacy of heparin NL = 35 seconds

Expiration

passive, inspiratory muscles relax, diaphragm rises and rib descend, lungs recoil, intrapulmonary pressure increases, air flows out

Cardiac tamponade

pericardial effusion - compresses heart restricting ventricular filling - cardiav output drops

Cardiac tamponade - treatment

pericardiocentesis - pericardial window for recurrent effusion

Surgical interventions for ascites

peritoneovenous shunt La Veen/Denver shunt for continuous reinfusion or ascitic fluid into the venous system

Complications of diverticulitis

peritonitis abscess & fistula formation bowel obstruction urethral obstruction lower GI bleed

Integumentary assessment findings for FVE

pitting edema, pale cool skin

Antidiuretic hormone is secreted by the

pituitary gland

Complications of Pneumonia

pleurisy, atelectasis, pulmonary edema, respiratory failure,

Insulin lowers serum level of which electrolyte?

potassium

All clients exposed to persons with tuberculosis should receive

prophylactic isoniazid in daily doses of 300 mg for 6 months to 1 year to avoid the deleterious effects of the latent mycobacterium

Vitamin E (function)

preserves integrity of RBCs - prevents vitamin a from being brokendown - antioxidant

Functions of the liver

produces bile eliminates bilirubin metabolizes hormones and drugs metabolizes proteins, carbohydrates, & fats synthesizes plasma proteins synthesizes clotting factors stores vitamins and minerals

Anabolism

production of more complex chemical substances by synthesis of nutrients

Troponin

protein elevated when myocardial damage

Kwashiorkor

protein energy malnutrition - low protein to energy ratio - commonly occurs in children just after weaning when their diets do not have as much protein as found in mother's milk - protein plays a crucial role in maintaining fluid volume in blood vessels - edema develops b/c water does not get pulled back into circulatory system and accumulates in interstitial spaces

Marasmus

protein energy malnutrition - too few calories and insufficient protein - chronic wasting away of fat, muscle, and other tissues

PT

prothrombin time - assesses efficacy of coumadin NL = 11 seconds

Aortic insuffciency/regurgitation

related to Marfan's syndrome and bacterial endocarditis - backflow of blood from aorta to left ventricle during diastole leads to dilation of ventricle - usually asymptomatic for many years

Mitral insufficiency/regurgitation

related to RHD - backflow of blood into left atrium during systole because valve does not fully close

Aortic stenosis

related to RHD - tissue thickens and narrows opening - left ventricular obstruction during systole leads to hypertrophy of ventricle

Mitral stenosis

related to RHD - tissue thickens and narrows the valve opening - valve cannot open fully - prevents blood flow from left atrium to left ventricle

Mitral valve prolapse

related to endocarditis and RHD - present in healthy individuals - can lead to mitral valve insufficiency - 5 times risk of bacterial endocarditis

Antispasmodics

relax smooth muscle of GI tract ie. Bentyl

Alkaline phosphatase

released during liver damage or biliary obstruction

Metabolic acidosis is most commonly due to

renal failure

Causes of hyperphosphatemia

renal failure, excess intake

Causes of Hypermagnesemia

renal failure, excessive replacement

Causes of hypocalcemia

renal failure, hypoparathyroidism, malabsorption, pancreatitis

What type of sound is expected of percussion over normal lung tissue?

resonance

Respiratory acidosis is most commonly due to

respiratory issues, CO2 retention

Atrial flutter

saw tooth shaped flutter waves on EKG - artrial rate 250 - 300 - AV node selectively conducts impulse to ventricles - ventricular rate can be regular or variable

Complications of Chron's disease

scar tissue with narrowing & SBO fistulas between bowel and urinary tract bowel perforation/peritonitis fat malabsorption deficiencies (A,D,E,K) gluten intolerance cholelitiasis & nephrolithiasis (due to oxalate absorption)

Nephrosclerosis

sclerosis of the small arteries and arterioles of the kidney usually caused by vascular changes due to malignant hypertension and arteriosclerosis treated with antihypertensives

Causes of metabolic alkalosis

severe vomiting, excess gastric secretions, diuretics, potassium deficit, excess sodium bicarbonate intake

EKG and MI

shows either ST elevation, T wave inversion, or non-ST elevation - sometimes there is an abnormal q wave - ST and T waves return to normal within hours to days post MI - abnormal q waves are usually permanent

Steroids can lead to the retention of which electrolyte?

sodium

Pleural Space

space between visceral and parietal sacs - air tight - contains ~20ccs of pleural fluid

Diagnostic tests for PE

spiral CT, V/Q (ventilation perfusion scan), pulmonary angiogram, ABGs, d-dimer

Virchow's Triad

stasis of bood flow, vessel injury, hypercoagulability

Counterregulatory hormones

stimulate glycogen release - glucagon, epinephrine, growth hormone, cortisol, somastatin

Phamacologic stress test

stimulates sympathetic nervous system - NPO - avoid smoking and caffeine for 1hr prior

Predisposing factors for gastric ulcer

stress smoking the use of corticosteroids, NSAIDs, and alcohol hx of gastritis family hx of gastric ulcers infection with H. Pylori

Blood flow through the heart

superior/inferior vena cava --> right atrium --> tricuspid valve --> right ventricle --> pulmonary artery --> lungs --> pulmonary veins --> left atrium --> mitral/bicuspid valve --> left ventricle -- aorta (ascending/descending)

Kock ileostomy (continent ileostomy)

surgical intervention for ulcerative colitis An intra-abdominal pouch that stores the feces and is constructed from the terminal ileum. The pouch is connected to the stoma with a nipple-like valve constructed from a portion of the ileum ; the stoma is flush with the skin. A catheter is used to empty the pouch, and a small dressing or adhesive bandage is worn over the stoma between emptyings.

Ileoanal reservoir

surgical intervention for ulcerative colitis Creation of an ileoanal reservoir is a two-stage procedure that involves the excision of the rectal mucosa , an abdominal colectomy, construction of a reservoir to the anal canal, and a temporary loop ileostomy. The ileostomy is closed in 3 to 4 months after the capacity of the reservoir is increased and has had time to heal.

Total proctocolectomy with permanent ileostomy

surgical intervention for ulcerative colitis The procedure is curative and involves the removal of the entire colon. The end of the terminal ileum forms the stoma, which is located in the right lower quadrant.

Factor hepaticus

sweet odor on breath

Signs and symptoms of hyperphosphatemia

sx due to hypocalcemia, short term tetany

Factors influencing blood pressure

sympathetic nervous system, renal system, endocrine system

Hallmarks of Atelectasis

tachypnea, dyspnea, hypoxemia

The most important measure in preventing asthma attacks is...

taking prescribed medications on time

Nephrostomy tube

temporary - used to preserve renal function inserted into pelvis of kidney

Bubbling in the second chamber of a Pleur-evac system signifies...

that air is moving from the collection chamber to the water seal chamber

Signs and symptoms of DVT

unilateral leg pain or calf tenderness, edema, warmth, homan's sign

Unstable angina

unpredictable - increases in occurrence, duration, and severity over time - may not be relieved with nitroglycerine and may require hospitalization

Nursing care of nephrostomy tube

urine output is measured separately from each tube q 1-2 hrs ensure tube does not get kinked, clamped, layed on etc. do not irrigate without order - 5-10cc sterile saline - strict aseptic technique daily weights BUN/creatinine

Magnesium plays a role in

use of ATP, carbohydrate metabolism, protein synthesis, nucleic acid synthesis, muscle contraction, clotting

Atropine sulfate

used for bradycardia and first-degree AV blocks

Causes for sinus bradycardia

vagal stimulation digitalis toxicity (>2.4) Increased ICP MI trained athletes severe pain beta blockers hypothermia

Complications of constipation

valsalva maneuver & syncope divertulosis obstipation bowel perforation

Nitrates

vasodilator

Varicose veins

veins become dilated and tortuous, become large and bulge, valves stretch and become incompetent

Lidocaine (cardiac)

ventricular arrhythmias, specifically PVCs

What is the antidote for a warfarin overdose?

vitamin K1 (phytonadione)

Anecdote for coumadin

vitamin k

Metabolic alkalosis is most often caused by

vomiting, diuretic therapy or nasogastric suction

Signs and symptoms of superficial phlebitis

warmth, swelling, redness, and pain

Interventions for Hyponatremia

water restriction, IV and oral sodium replacement

Early signs of hypervolemia

weight gain, elevated BP, increased breathing effort

Signs and Symptoms of Achalasia

∙ dysphagia ∙ halitosis (foul smelling breath) ∙ pyrosis (heartburn) ∙ sour tasting food and liquids ∙ chest pain . coughing

Treatment for Achalasia

∙ esophageal dilation ∙ esophagomyotomy ∙ anticholinergics - Pro-Banthine ∙ eat semisoft food ∙ sleep with head of the bed elevated to prevent aspiration

Diagnostics for Anorexia Nervosa

∙ iron deficiency anemia ∙ ↑ BUN ∙ ↓ K - muscle weakness cardiac arrhythmias renal failure ∙ body wasting ∙ signs of malnutrition

Surgeries for Obesity

∙ jaw wiring ∙ liposuction ∙ lipectomy ∙ vertical banded gastroplasty ∙ Roux-en-Y bypass

Signs and Symptoms of Bulimia

∙ may have normal weight or weight may fluctuate with binging and purging ∙ abuse laxatives, diuretics, exercise, or diet medications ∙ Russell's sign ∙ swollen salivary glands and dental enamel decay

Treatment for Anorexia Nervosa

∙ psychiatric care ∙ nutritional support and counseling ∙ nutritional supplements ∙ underlying psychological problem must be addressed with individual and family counseling ∙ support groups

Signs and Symptoms of Anorexia Nervosa

∙ weight loss ∙ deliberate self starvation ∙ intense fear of gaining weight ∙ refusal to eat ∙ hair loss ∙ sensitivity to cold ∙ compulsive exercise ∙ amennorhea ∙ dry skin ∙ constipation


Ensembles d'études connexes

Networking Study Guide Test 2 (IN THE HOOD) (GONE WRONG??)

View Set

Westward Expansion Unit Study Guide

View Set

JOUR 310 Midterm - Grammar Portion

View Set

Match the words or phrase and the definitions.

View Set